You are on page 1of 233

Paulius Kantautas

Lukas Melninkas
Pijus Simonaitis
Paulius arka
Vaidotas Juronis
v2.0
Matematikos knyga
Except where otherwise noted, this work is licensed under
http://creativecommons.org/licenses/by-sa/3.0/
Copyright Paulius Kantautas, Lukas Melninkas, Pijus Simonaitis, Paulius arka 2010; Pijus
Simonaitis, Vaidotas Juronis 2011, Some Rights Reserved.
Except where otherwise noted, this work is licensed under Creative Commons Attribution Sha-
reAlike 3.0. You are free:
to Share to copy, distribute and transmit the work,
to Remix to adapt the work.
Under the following conditions:
Attribution. You must attribute the work in the manner specied by the author or
licensor (but not in any way that suggests that they endorse you or your use of the work).
Share alike. If you alter, transform, or build upon this work, you may distribute the
resulting work only under the same, similar or a compatible license.
With the understanding that:
Any of the above conditions can be waived if you get permission from the copyright
holder.
In no way are any of the following rights aected by the license:
- Your fair dealing or fair use rights;
- Authors moral rights;
- Rights other persons may have either in the work itself or in how the work is used,
such as publicity or privacy rights.
TURINYS
Apie knyg . . . . . . . . . . . . . . . . . . . . . . . . . . . . . . . . . . . . . 1
1 Skaii teorija 2
1.1 Dalumas . . . . . . . . . . . . . . . . . . . . . . . . . . . . . . . . . . . . 2
1.2 Lyginiai . . . . . . . . . . . . . . . . . . . . . . . . . . . . . . . . . . . . 10
1.3 Oilerio teorema . . . . . . . . . . . . . . . . . . . . . . . . . . . . . . . . 14
1.4 Kin liekan teorema . . . . . . . . . . . . . . . . . . . . . . . . . . . . . 19
1.5 Liekan grup . . . . . . . . . . . . . . . . . . . . . . . . . . . . . . . . . 22
1.6 Kvadratins liekanos . . . . . . . . . . . . . . . . . . . . . . . . . . . . . 28
1.7 Diofantins lygtys . . . . . . . . . . . . . . . . . . . . . . . . . . . . . . . 35
1.7.1 Dvi lygties puss . . . . . . . . . . . . . . . . . . . . . . . . . . . 35
2 Algebra 42
2.1 Nelygybs . . . . . . . . . . . . . . . . . . . . . . . . . . . . . . . . . . . 42
2.1.1 Pirmieji ingsniai . . . . . . . . . . . . . . . . . . . . . . . . . . . 45
2.1.2 Vidurki nelygybs . . . . . . . . . . . . . . . . . . . . . . . . . . 50
2.1.3 Cauchy-Schwarz nelygyb . . . . . . . . . . . . . . . . . . . . . . 61
2.1.4 Specialios technikos . . . . . . . . . . . . . . . . . . . . . . . . . 66
2.1.5 Drakon puota . . . . . . . . . . . . . . . . . . . . . . . . . . . . 74
2.2 Funkcins lygtys . . . . . . . . . . . . . . . . . . . . . . . . . . . . . . . 76
2.2.1 sistatykime x = 0 . . . . . . . . . . . . . . . . . . . . . . . . . . 76
2.2.2 Funkcij tipai . . . . . . . . . . . . . . . . . . . . . . . . . . . . . 81
2.2.3 Cauchy funkcin lygtis . . . . . . . . . . . . . . . . . . . . . . . . 88
3 Kombinatorika 92
3.1 Matematiniai aidimai . . . . . . . . . . . . . . . . . . . . . . . . . . . . 92
3.1.1 Strategija . . . . . . . . . . . . . . . . . . . . . . . . . . . . . . . 92
3.1.2 aidimas NIM . . . . . . . . . . . . . . . . . . . . . . . . . . . . 101
4 Geometrija 110
4.1 anga . . . . . . . . . . . . . . . . . . . . . . . . . . . . . . . . . . . . . 110
4.2 Udaviniai apilimui . . . . . . . . . . . . . . . . . . . . . . . . . . . . . 113
TURINYS TURINYS
4.3 Panaieji trikampiai ir brinio papildymai . . . . . . . . . . . . . . . . . 119
4.4 Apskritimai . . . . . . . . . . . . . . . . . . . . . . . . . . . . . . . . . . 125
4.5 Plotai . . . . . . . . . . . . . . . . . . . . . . . . . . . . . . . . . . . . . 135
4.6 Apibrtins gros . . . . . . . . . . . . . . . . . . . . . . . . . . . . . 138
4.7 Vienareikmiki udaviniai . . . . . . . . . . . . . . . . . . . . . . . . . . 143
4.8 Geometrins nelygybs . . . . . . . . . . . . . . . . . . . . . . . . . . . . 147
5 Sprendimai 151
Literatra 228
iv
Apie knyg
Matematikos knyga - tai knyga skirta matematika besidomintiems moksleiviams ir mok-
sleivms. Jos turinys yra gerokai nutols nuo sutinkamo mokykloje ir, pagal matema-
tikos olimpiad tradicij, orientuotas keturias matematikos sritis: skaii teorij, al-
gebr, kombinatorik ir geometrij. Turinys pateiktas naudojant prast matematin
kalb, tad prie teorem, rodym ir matematini paymjim nepratusiems gali pri-
reikti iek tiek daugiau atkaklumo ir mokytojo(-os) pagalbos.
Prie knygos krimo prisidjo keletas moni, kuriuos nortume paminti:
Benas Baanskas ir Gintautas Miliauskas rad ir itais krv klaid.
Gabriel Bakyt pirmojo leidimo redaktor.
ymantas Darbnas vienas i knygos idjos autori ir kvepj.
Albertas Zineviius padjs rayti ir apipavidalinti knyg.
Taip pat nortume paminti Nacionalin moksleivi akademij, kuri nekart kvp ir
paskatino tsti knygos krim.
Ai jums!
*
Kaip jau pastebjote i pirmj puslapi, i knyga yra ileista pagal Creative Commons
licenzij. Tai reikia, kad kartu su knyga js gaunate gerokai daugiau laisvs, nei pras-
tai, ir mes tikims, kad ta laisve js drsiai naudosits.
Kartu tai iek tiek paaikina, kodl ileidiama nebaigta knyga. Rayti po skyri ar
skyrel yra daug paprasiau ir efektyviau, nei i karto griebtis sunkiai kandamo umojo.
Atviras formatas neriboja nei autori skaiiaus, nei raymo trukms, tad jei vilgtel-
jus turin jums galvoje rikiuojasi trkstamo skyrelio tekstas, galbt metas ssti prie
klaviatros?
1 SKYRIUS
SKAII TEORIJA
Skaii teorija yra senas tradicijas turinti matematikos aka, nagrinjanti udavinius,
susijusius su skaiiais ir j dalumu. iame skyriuje supaindinsime su paiomis pag-
rindinmis svokomis ir panagrinsime dalel klasikins teorijos liekan grupes ir
kvadratinius simbolius.
1.1 Dalumas
Su skaii dalumo savoka jau greiiausiai esate pastami, tad pradioje keletas api-
brim pasitikslinimui. Turkite omenyje, kad visur bus kalbama apie natraliuosius
arba sveikuosius skaiius.
Apibrimas. Skaiius n dalijasi i skaiiaus a, jei egzistuoja toks skaiius b, kad
n = a b. Skaiius a dalo n (ymsime a[n) jei n dalijasi i a.
Apibrimas. Skaiius, i kurio dalijasi n, vadinamas n dalikliu. Skaiius, kuris dalijasi
i n, vadinamas n kartotiniu.
Apibrimas. Skaiius, kuris dalijasi tik i vieneto ir i savs, vadinamas pirminiu.
Sveikj skaii dalyba tenkina kelet savybi. sitikinkite, kad suprantate kiek-
vien, ir mintyse pabandykite sugalvoti po pavyzd:
Jei x[a ir x[b, tai x[a +b, x[a b ir x[ab;
Jei x[y ir y[z, tai x[z;
Jei x[a ir y[b, tai xy[ab.
1.1. Dalumas Skaii teorija
Skaidymas dauginamaisiais
Viena i pagrindini sveikj skaii savybi, susijusi su dalumu, yra vienareikmis
skaidymasis dauginamaisiais. Ja mes remsims ir naudosims labai danai, nors jos ir
nerodysime (rodymas yra kiek ilgokas ir vargu, ar tinkamas paiai painties su skaii
teorija pradiai).
Teiginys. Kiekvien skaii n galima vieninteliu bdu iskaidyti pirminiais daugina-
maisiais:
n = p

1
1
p

2
2
p

k
k
.
Maus skaiius skaidyti pirminiais dauginamaisiais nesunku tiesiog i eils tikri-
name pirminius skaiius ir skaiiuojame, kiek kart i j galima padalinti. Pavyzdiui,
120 = 2 2 2 3 5 = 2
3
3 5.
inodami, kaip skaiius isiskaido, galime nemaai apie j pasakyti. Pavyzdiui,
galime nurodyti jo daliklius:
Teiginys. Jei skaiius n dalijasi i skaiiaus a ir
n = p

1
1
p

2
2
p

k
k
,
tai tuomet
a = p

1
1
p

2
2
p

k
k
ir

i

i
su visais i = 1, . . . , k.
rodymas. Jei n dalijasi i a, tai tuomet egzistuoja toks b, kad n = ab. Iskaid a daugi-
namaisiais gauname, kad n skaidin turi eiti visi pirminiai kaip ir a su nemaesniais
laipsni rodikliais.
Panagrinkime skaii 12. Jis isiskaido kaip 2
2
3
1
. Pagal k tik rodyt teigin jo
dalikliais turt bti 2
2
3
1
,2
1
3
1
, 2
0
3
1
, 2
2
3
0
, 2
1
3
0
ir 2
0
3
0
. Sudaugin matome,
kad gavome skaiius 12, 6, 3, 4, 2 ir 1, kurie i ties yra visi 12 dalikliai. Tad nordami
rasti duoto skaiiaus dalikl turime paimti kakoki dal jo skaidinio. is pastebjimas
leidia nesunkiai suskaiiuoti, kiek i viso dalikli skaiius turi:
Teiginys. Skaiius n = p

1
1
p

2
2
p

k
k
turi (
1
+ 1)(
2
+ 1) (
n
+ 1) dalikli.
rodymas. Kiekvienas n daliklis bus uraomas kaip p

1
1
p

2
2
p

k
k
, kur
i

i
su visais
i = 1, . . . , k. Skirtingus daliklius gausime imdami skirtingus pirmini skaii laipsnius.
Parinkti
1
galime
1
+ 1 bdais (nepamirkime nulio!), parinkti
2
galime
2
+ 1
bdais ir taip toliau. Pagal kombinatorin daugybos taisykl i viso galsime sudaryti
(
1
+ 1)(
2
+ 1) (
n
+ 1) skirting laipsni rinkini, todl tiek bus ir skirting
dalikli.
3
1.1. Dalumas Skaii teorija
Didiausiasis bendras daliklis
Prisiminkime didiausiojo bendro daliklio ir maiausiojo bendro kartotinio svokas.
Apibrimas. Dviej ar daugiau skaii didiausiuoju bendru dalikliu (dbd) vadinsime
didiausi skaii, i kurio visi duotieji dalinasi.
Apibrimas. Dviej ar daugiau skaii maiausiuoju bendru kartotiniu (mbk) vadin-
sime maiausi skaii, kuris dalijasi i vis duotj.
Apibrimas. Du skaiius, kuri didiausiasis bendras daliklis yra lygus 1, vadinsime
tarpusavyje pirminiais.
Pavyzdiui, didiausias skaiius, i kurio dalijasi ir 15 ir 25, yra 5, o maiausias
skaiius, kuris dalijasi i 2, 3, 4, 5 ir 6, yra 60. Tad dbd(15, 25) = 5 ir mbk(2, 3, 4, 5, 6) =
60. Iekant didiausiojo bendro daliklio ir maiausiojo bendro kartotinio labai praveria
skaidymas dauginamaisiais. siirkite:
Nordami rasti didiausi skaii 2
4
3
1
ir 2
1
3
1
5
1
bendr dalikl turime paimti
didiausi manom skaidinio dal, priklausani abiems skaiiams. iuo atveju tai bt
2
1
3
1
.
Nordami rasti maiausi skaii 2
4
3
1
ir 2
1
3
1
5
1
bendr kartotin, turime paimti
maiausi manom skaidin, kur "tilpt" abiej skaii skaidiniai. iuo atveju tai
bt 2
4
3
1
5
1
.
Kitas bdas, ar bent jau pagrindin idja, kuri praveria iekant didiausio bendro
daliklio, yra Euklido algoritmas. Jis remiasi svarbia ir naudinga lygybe:
Teiginys.
dbd(a, b) = dbd(a b, b).
rodymas. Tegu dbd(a, b) = d. Tuomet d[a ir d[b, o kartu d[(a b). Vadinasi, dbd(a
b, b) bus nemaesnis nei d.
I kitos puss tegu dbd(a b, b) = d

. Kartut d

[(a b) ir d

[b, o tuo paiu d

[a,
nes a = (a b) +b. Vadinasi, dbd(a, b) bus nemaesnis nei d

.
Kadangi gavome d d

ir d

d, tai vadinasi d = d

, t.y. dbd(a, b) = dbd(a


b, b).
Pavyzdys. Pasinaudodami rodyta lygybe raskime dbd(14, 6), dbd(2
100
+1, 2
100
1) ir
dbd((p +q)
2
, p), kur p ir q pirminiai skaiiai.
Sprendimas. Didiausio bendro daliklio iekosime atimdami i didesnio skaiiaus ma-
esn:
dbd(14, 6) = dbd(8, 6) = dbd(2, 6) = dbd(2, 4) = dbd(2, 2) = 2.
dbd(2
100
+ 1, 2
100
1) = dbd(2, 2
100
1) = 1.
dbd((p +q)
2
, p) = dbd(p(p + 2q) +q
2
, p) = dbd(q
2
, p) = 1.

4
1.1. Dalumas Skaii teorija
Euklido algoritmas. Rasime dbd(a, b). Nemaindami bendrumo tarkime, kad a > b.
Tuomet a uraomas kaip
a = bq +r, kur dalybos liekana tenkina 0 < r < b. Analogikai
b = rq
1
+r
1
, kur 0 < r
1
< r,
r = r
1
q
2
+r
2
, kur 0 < r
2
< r
1
,
. . .
r
k2
= r
k1
q
k
+r
k
, kur 0 < r
k
< r
k1
,
r
k1
= r
k
q
k+1
.
I r > r
1
> ... > r
k
seka, kad kakada gausime dalybos liekan lygi 0. Tuomet,
kadangi
dbd(a, b) = dbd(b, r) = = dbd(r
k2
, r
k1
) = dbd(r
k1
, r
k
),
tai paskutinioji nenulin liekana r
k
ir bus didiausias bendrasis daliklis.
Paties Euklido algoritmo taip skrupulingai, kaip jis suformuluotas, netaikysime
daniausiai, kaip pavyzdyje, pakaks kelet kart pasinaudoti lygybe dbd(a, b) =
dbd(a, b a). Taiau urame j ne be reikalo labai svarbi bus jo ivada:
Ivada. Jei dbd(a, b) = d, tai egzistuoja tokie x, y Z, kad ax +by = d.
rodymas. I priepaskutins Euklido algoritmo lygybs galime ireikti r
k
per r
k1
ir
r
k2
. I dar ankstesns galima ireikti r
k1
per r
k2
ir r
k3
. stat pirmj iraik
gausime r
k
iraik per r
k2
ir r
k3
. Taip toliau vis tsdami gausime r
k
iraik per a,
b, t.y. rasime x, y, tenkinanius ax +by = dbd(a, b).
Pirminiai skaiiai
Jau pirmuosiuose puslapiuose galima atkreipt dmes tai, kok didel vaidmen skaii
teorijoje vaidina pirminiai skaiiai. Kadangi kiekvien sveikj skaii vieninteliu bdu
galima urayti kaip j sandaug, tai neretai jie yra vaizdiai vadinami sveikj skaii
atomais. rodykime vien i graiausi ir elegantikiausi matematikos teorem:
Teorema. Pirmini skaii yra be galo daug.
rodymas. Tarkime prieingai, kad pirmini skaii yra baigtinis skaiius. Sudauginki-
me juos visus ir pridkime vienet: p
1
p
2
p
n
+ 1. is skaiius nesidalija i n vieno
pirminio p
1
, . . . , p
n
, todl pats yra pirminis. Gavome nauj pirmin prietara.
Kartais tenka patikrinti, ar duotas skaiius yra pirminis, ar ne. Tam reikia patikrinti
visus potencialius jo daliklius. Truput pagalvoj, galime rasti sutrumpinim:
Teiginys. Jei skaiius n nesidalija i jokio pirminio skaiiaus, maesnio (arba lygaus)
u

n, tai jis pirminis.
rodymas. Ities, jei skaiius n turi dalikl a, tai turi ir dalikl
n
a
, bet tuomet arba
a

n, arba
a
n

n, vadinas,i n turs dalikl (o kartu ir pirmin dalikl), maesn u

n.
Pavyzdiui, norint patikrinti, ar 101 yra pirminis, utenka ibandyti 2, 3, 5 ir 7.
Kadangi n i vieno nesidalija, tai 101 yra pirminis.
5
1.1. Dalumas Skaii teorija
Dalumo poymiai
Ura skaii deimtainje sistemoje a
1
a
2
. . . a
n
, i jo skaitmen galime sprsti, ar jis
dalijasi i kai kuri ma skaii, ar ne. Naudingiausi dalumo poymiai yra ie:
Skaiius a
1
a
2
. . . a
n
dalijasi i 2, jei i 2 dalijasi jo paskutinis skaitmuo a
n
.
Skaiius a
1
a
2
. . . a
n
dalijasi i 3, jei i 3 dalijasi jo skaitmen suma a
1
+ +a
n
.
Skaiius a
1
a
2
. . . a
n
dalijasi i 4, jei i 4 dalijasi jo dviej skaitmen galn a
n1
a
n
.
Skaiius a
1
a
2
. . . a
n
dalijasi i 5, jei i 5 dalijasi jo paskutinis skaitmuo a
n
.
Skaiius a
1
a
2
. . . a
n
dalijasi i 8, jei i 8 dalijasi jo trij skaitmen galn a
n2
a
n1
a
n
.
Skaiius a
1
a
2
. . . a
n
dalijasi i 9, jei i 9 dalijasi jo skaitmen suma a
1
+ +a
n
.
Skaiius a
1
a
2
. . . a
n
dalijasi i 11, jei i 11 dalijasi jo alternuojanti skaitmen suma
a
1
a
2
+a
3
a
4
+ a
n
.
Vis dalumo poymi rodymai bus imtyti po pirmus du skyrelius, o kol kas
svarbiau juos siminti ir imokti atpainti.
Pavyzdiai
1 Pavyzdys. rodykite, kad su visais natraliaisiais n, n
2
+n dalijasi i 2.
Sprendimas. Jei n dalijasi i 2, tai ir n
2
dalijasi i dviej. Dviej skaii, besidalijani
i dviej (lygini), suma taip pat dalinsis i dviej.
Jei n nesidalija i 2, tai ir n
2
nesidalija i dviej. Dviej skaii, nesidalijani i
dviej (nelygini) suma dalijasi i dviej.
Vadinasi, tikrai, bet kuriuo atveju, n
2
+n dalinsis i dviej.
2 Pavyzdys. rodykite, kad jei n[5a + 3b ir n[3a + 2b, tai n[a ir n[b.
Sprendimas. Pastebkime, kad a galime ireikti kaip 2(5a + 3b) 3(3a + 2b), o b kaip
5(3a + 2b) 3(5a + 3b). Abu skirtumai i n dalijasi, todl dalinsis ir a, ir b.
3 Pavyzdys. raykite vaigdui vietoje tokius skaitmenis, kad skaiius 15 15 da-
lytsi i 99.
Sprendimas. Duotas skaiius dalinsis i 99 tada ir tik tada, kai dalinsis i 9 ir i 11. Jei
vietoje vaigdui raysime x ir y, tai pagal dalumo poymius gausime, kad 12+x+y
turi dalintis i 9, ir x y 8 turi dalintis i 11. Abi slygas tenkina x = 6, y = 9.
Pastaba. Teiginys, kad n dalinasi i ab tada ir tik tada, kai n dalinasi i a ir n dalinasi
i b, yra teisingas, tik kai a ir b yra tarpusavyje pirminiai.
4 Pavyzdys. rodykite dalumo i 3 poym.
6
1.1. Dalumas Skaii teorija
Sprendimas. Skaiius, uraytas deimtaine iraika kaip a
1
a
2
. . . a
n
, yra lygus a
1

10
n1
+a
2
10
n2
+ +a
n1
10 +a
n
. Pastebkime, kad visi deimties laipsniai yra
vienetu didesni u skaii, besidalijant i trij:
a
1
10
n1
+a
2
10
n2
+ +a
n1
10 +a
n
=
a
1
99 . . . 9
. .
n1
+a
2
99 . . . 9
. .
n2
+ +a
n1
9 +a
1
+a
2
+ +a
n1
+a
n
.
Matome, kad skaiius nuo savo skaitmen sumos skiriasi per 3 kartotin, todl arba abu
dalijasi i trij, arba abu nesidalija.
5 Pavyzdys. [IMO 1959] rodykite, kad trupmena
21n+4
14n+3
yra nesuprastinama su visomis
natraliosiomis n reikmmis.
Sprendimas. Trupmena bus nesuprastinama, jei didiausias skaitiklio ir vardiklio bend-
ras daliklis bus lygus vienam. Pasinaudoj dbd(a, b) = dbd(a, b a) gauname:
dbd(21n + 4, 14n + 3) = dbd(7n + 1, 14n + 3) = dbd(7n + 1, 1) = 1.

Udaviniai
1. Duota, kad n[3a ir n[12a + 5b. rodykite, kad n[10b. S
2. Duota, kad n[3a + 7b ir n[2a + 5b. rodykite, kad n[a ir n[b. S
3. Ar teisingos ios "dalumo savybs": S
a) Jei x[a +b, tai x[a ir x[b,
b) Jei x[a b, tai x[a arba x[b,
c) Jei x[a ir y[a, tai xy[a ?
4. rodykite, kad bet kaip sudliojus devynis skaitmenis 1, 2,..., 9, gautas devyn- S
enklis skaiius dalinsis i 9.
5. rodykite, kad skaiius abba dalijasi i 11. S
6. raykite vaigdutes vietoje tok skaitmen, kad skaiius 12345 dalytsi i: a) S
9; b) 8; c) 11.
7. Duota, kad skaiius a + 4b dalijasi i 13. rodykite, kad ir 10a +b dalijasi i 13. S
8. Raskite visus pirminius skaiius i intervalo [180, 200]. S
9. Su kuriomis natraliosiomis n reikmmis skaiius n
2
+ 5n + 6 pirminis? S
10. Duota, kad n[a +b. rodykite, kad n[a
3
+ 2a +b
3
+ 2b. S
11. Kokius skaiius galime ireikti suma 8x + 5y, kur x, y Z ? S
7
1.1. Dalumas Skaii teorija
12. Ar skaiius, kurio skaitmen suma lygi 5, gali dalintis i 11? S
13. rodykite, kad skaiius turi nelygin dalikli skaii tada ir tik tada, kai jis yra S
sveikojo skaiiaus kvadratas.
14. Duota, kad trupmena
a
b
yra suprastinama. Ar trupmena
ab
a+b
btinai yra su- S
prastinama? Ir atvirkiai, jei inoma, kad trupmena
ab
a+b
yra suprastinama, ar
trupmena
a
b
btinai yra suprastinama?
15. rodykite, kad mbk(a, b) dbd(a, b) = a b. S
16. Duota, kad 11[3x + 7y ir 11[2x + 5y. rodykite, kad 121[x
2
+ 3y
2
. S
17. rodykite, kad skaiiaus, kuris dalijasi i 99, skaitmen suma yra ne maesn u S
18.
18. Raskite bent vien n, kad intervale [n, n+10] nebt n vieno pirminio skaiiaus. S
19. Duotas 100-enklis skaiius a, kuris dalijasi i 9. inome, kad b yra a skaitmen S
suma, c yra b skaitmen suma, d yra c skaitmen suma. Kam lygus skaiius d?
20. Nurodykite kok nors skaiiaus n = 27
28
+4 dalikl skirting nuo 1 ir paties n.
1
S
21. rodykite, kad jei p pirminis, tai
_
p
k
_
=
p!
k!(pk)!
dalijasi i p su visais 1 k p1. S
22. Kiek yra sveikj skaii 1 n 100, kad S
a)dbd(n
2
+ 1, n + 1) > 1
b) dbd(n
2
+ 1, n + 2) > 1?
23. [Pan African 2001] Raskite visus sveikuosius n, su kuriais skaiius
n
3
+3
n
2
+7
yra S
sveikasis.
24. rodykite, kad 11 1
. .
3
n
dalijasi i 3
n
. S
25. [LitKo 2002] Raskite maiausi natralj skaii, kurio pus yra sveikojo skai- S
iaus kvadratas, tredalis yra sveikojo skaiiaus kubas, o penktadalis yra sveikojo
skaiiaus penktasis laipsnis.
26. Raskite maiausi sveikj skaii turint 75 daliklius ir besidalijant i 75. S
27. Su kuriomis neneigiamomis n reikmmis vienu metu 2n+1 ir 3n+1 yra pilnieji S
kvadratai ir 5n + 3 yra pirminis?
28. Samprotaudami panaiai kaip rodyme, kad pirmini skaii yra be galo daug, S
rodykite, kad pirmini skaii pavidalo 4k + 3 yra be galo daug.
29. Paymkime f(n) vienaenkl skaii, kur gauname vis daugindami n skaitme- S
nis. Pavyzdiui f(27) = f(14) = 4. Raskite visus n, su kuriais f(n) = 1.
30. [Ireland 2007] Raskite visus pirminius skaiius p ir q, tenkinanius p[q + 6 ir S
q[p + 7.
1
Lietuvos 5-6 klasi moksleivi matematikos olimpiada, 2005m.
8
1.1. Dalumas Skaii teorija
31. [IMO 2002] Tegu n 2 natralusis skaiius, kurio dalikliai yra 1 = d
1
< d
2
< S
d
k
= n. rodykite, kad d
1
d
2
+ d
2
d
3
+ + d
k1
d
k
yra visuomet maesnis u n
2
,
ir raskite, kada jis yra n
2
daliklis.
9
1.2. Lyginiai Skaii teorija
1.2 Lyginiai
Lyginia

i yra nepakeiiamas rankis sprendiant udavinius apie sveikj skaii daliji-


msi ir liekanas.
Apibrimas. Jei m[a b, tai sakysime, kad "a lygsta b moduliu m", ir ymsime
a b (mod m).
Pavyzdiui:
2 5 (mod 3), 100 0 (mod 20), 3 2 (mod 5).
Norint skmingai naudotis lyginiais prireiks keleto pastebjim:
a b (mod m) tada ir tik tada, kai a ir b duoda vienodas liekanas dalijami i m,
a b (mod m) tada ir tik tada, kai egzistuoja toks k Z, kad a = b +km,
jei a b (mod m) ir b c (mod m), tai a c (mod m).
Pirmasis teiginys leidia intuityviai interpretuoti lyginius a lygsta b moduliu m
reikia, kad a ir b duoda tas paias liekanas dalijami i m. inoma, kad tokiu atvju a
ir b skirtumas dalijasi i m, kas yra kitu bdu urayta antrjame teiginyje. Naudojant
i interpretacij, akivaizdiu tampa ir treias teiginys: jei a duoda toki pai liekan
kaip b, o b toki pai, kaip c, tai a ir c liekanos taip pat sutaps.
Kaip ir prastini lygi atveju, lyginius galima sudti, dauginti ir atsargiai dalinti:
Teiginys.
jei a b (mod m) ir a

(mod m), tai a +a

b +b

(mod m);
jei a b (mod m) ir a

(mod m), tai aa

bb

(mod m);
jei ac bc (mod m) ir dbd(m, c) = 1, tai a b (mod m).
rodymas. rodykime visus tris naudodamiesi apibrimu:
Jei m[a b ir m[a

, tai m[(a b) + (a

) m[(a +a

) (b +b

).
Jei m[a b ir m[a

, tai m[(a b)a

ir m[(a

)b m[(a b)a

+(a

)b
m[aa

bb

.
Jei m[ac bc, t.y. m[(a b)c ir m tarpusavyje pirminis su c, tai m[a b.
Naudodamiesi iomis savybmis galime pertvarkyti sudtingus reikinius.
Pavyzdys. Raskime, koki liekan duoda 25
5
+ 36
6
dalijamas i 11.
Kadangi 25 3 (mod 11), tai 25
5
3
5
(mod 11) (sudauginame lygyb ja paia
5 kartus, t.y. keliame abi puses penktuoju laipsniu). Toliau 3
5
= 9 9 3, o 9
2 (mod 11), todl
3
5
(2) (2) 3 1 (mod 11).
Analogikai
36
6
3
6
3
5
3 3 (mod 11).
Sudj gauname, kad dalindami 25
5
+ 36
6
i 11 gauname liekan 4.
10
1.2. Lyginiai Skaii teorija
Dalumo poymiai dar kart
rodykime dalumo poym i 11. Pastebkime, kad 10 1 (mod 11). Pakelkime abi
lygybs puses n-tuoju laipsniu:
10
n
(1)
n
(mod 11).
Iskleid skaii deimtaine iraika, gauname:
a
1
a
2
. . . a
n
= 10
n1
a
1
+ + 10a
n1
+a
n
(1)
n1
a
1
+ a
n1
+a
n
(mod 11).
rodykime dalumo poym i 8. Kadangi 2[10, tai, kai n 3, teisinga 8[10
n
(t.y.
10
n
0 (mod 8)). Pasinaudoj tuo gauname:
a
1
a
2
. . . a
n
= 10
n1
a
1
+ + 10a
n1
+a
n
100a
n2
+ 10a
n1
+a
n
a
n2
a
n1
a
n
(mod 8).
Skaii laipsni liekanos
Sveikj skaii laipsniai, o ypa kvadratai ir kubai, yra labai danai sutinkami skaii
teorijos udaviniuose. Sveikj skaii laipsni liekanos turi domi struktr, kuri
gana plaiai nagrinsime vliau, taiau susipainti galime jau dabar. Pradkime nuo
paties paprasiausio pavyzdio:
Pavyzdys. Sveikojo skaiiaus kvadrat dalindami i 3 niekada negausime liekanos 2.
Imkime bet kok sveikj skaii a. Galimi trys variantai:
a 0 (mod 3) arba a 1 (mod 3), arba a 2 (mod 3).
Pakl a kvadratu atitinkamai gausime
a
2
0 (mod 3) arba a
2
1 (mod 3), arba a
2
4 1 (mod 3),
t.y. liekanos 2 niekada negausime.
Lygiai taip pat nagrindami atvejus galime susidoroti su visais nedideliais laipsniais
ir moduliais.
Pavyzdys. Kokias liekanas galime gauti dalindami a
4
i 5, jei a bet koks sveikasis
skaiius?
Nagrinkime penkis variantus:
a 0 (mod 5) a
4
0 (mod 5),
a 1 (mod 5) a
4
1 (mod 5),
a 2 (mod 5) a
4
16 1 (mod 5),
a 3 (mod 5) a
4
(2)
4
1 (mod 5),
a 4 (mod 5) a
4
(1)
4
1 (mod 5).
Gavome, kad galime gauti tik liekanas 0 arba 1.
11
1.2. Lyginiai Skaii teorija
Pavyzdiai
1 Pavyzdys. Raskite, koki liekan gauname dalindami 2
1000
i 11.
Sprendimas. Liekan rasime dviem bdais, kurie abu yra pamokantys. Pirma, paban-
dykime kuo greiiau suskaiiuoti didelius dvejeto laipsnius vis daugindami lygybes:
2
4
5 (mod 11)
2
8
5
2
3 (mod 11)
2
24
3
3
5 (mod 11)
2
48
5
2
3 (mod 11)
2
1000
(2
48
)
10
(2
48
)
10
(2
4
)
10
3
10
3
10
5
10
45
10
1
10
1 (mod 11).
Arba kelkime laipsniais po vien ir iekokime dsningum:
2
1
2 (mod 11), 2
8
7 2 3 (mod 11),
2
2
4 (mod 11), 2
9
3 2 6 (mod 11),
2
3
8 (mod 11), 2
10
6 2 1 (mod 11),
2
4
5 (mod 11), 2
11
1 2 2 (mod 11),
2
5
5 2 10 (mod 11), 2
12
2 2 4 (mod 11),
2
6
10 2 9 (mod 11), 2
13
4 2 8 (mod 11),
2
7
9 2 7 (mod 11),
Matome, kad liekanos pradeda kartotis kas deimt, vadinasi, tkstantojo laipsnio
bus tokia pat kaip ir deimtojo, t.y. lygi 1.
2 Pavyzdys. rodykite, kad n
3
n dalijasi i 6 su visomis sveikosiomis n reikmmis.
Sprendimas. Vl isprskime dviem bdais. Pirmasis gudrus: pastebkime, kad n
3
n
isiskaido kaip (n 1)n(n + 1). I trij paeiliui einani sveikj skaii bent vienas
dalijasi i trij ir bent vienas dalijasi i dviej, vadinasi, j sandauga dalijasi i 6.
Antrasis universalus: skaiius n dalijamas i 6 gali duoti liekanas 0, 1, . . . 5. Pa-
tikrinkime kiekvien i j:
n 0 (mod 6) n
3
n 0 0 0 (mod 6),
n 1 (mod 6) n
3
n 1 1 0 (mod 6),
n 2 (mod 6) n
3
n 8 2 0 (mod 6),
n 3 (mod 6) n
3
n 27 3 0 (mod 6),
n 4 2 (mod 6) n
3
n 8 (2) 0 (mod 6),
n 5 1 (mod 6) n
3
n 1 (1) 0 (mod 6).

12
1.2. Lyginiai Skaii teorija
Udaviniai
1. Raskite liekanas, gaunamas dalijant S
a) 1 + 11 + 111 + 1111 + 11111 i 9,
b) 555 777 + 666 888 i 9,
c) 3
99
i 2,3,4,5,6 ir 7,
d) 7
777
i 10.
2. rodykite, kad ab +cd ad +bc (mod a c). S
3. Kokias liekanas galime gauti dalindami sveikojo skaiiaus kvadrat i 4? S
4. rodykite, kad 30[n
5
n. S
5. rodykite, kad jei 3[a
2
+b
2
, tai 3[a ir 3[b. S
6. rodykite, kad jei 7[a
2
+b
2
, tai 7[a ir 7[b. S
7. rodykite, kad nelyginio skaiiaus kvadratas duoda liekan 1 dalijamas i 8. S
8. rodykite, kad 6[a +b +c tada ir tik tada, kai 6[a
3
+b
3
+c
3
S
9. rodykite, kad jei skaiius a nesidalija i 2 ir i 3, tai a
2
1 (mod 24). S
10. rodykite, kad dviej nelygini skaii kvadrat suma negali bti kvadaratas. S
11. Su kuriomis n reikmmis 120[(n
5
n)? S
12. Raskite visus pirminius p ir q tenkinanius lygyb p
2
2q
2
= 1. S
13. rodykite, kad n
2
+ 3n + 5 nesidalija i 121 su visomis n reikmmis. S
14. [LitKo 2002] rodykite, kad 10
n
+ 45n 1 dalijasi i 27. S
15. rodykite, kad skaiiaus ir jo skaitmen sumos dalybos i 9 liekanos sutampa. S
16. rodykite, kad jei a b (mod n), tai dbd(a, n) = dbd(b, n). S
17. [LitKo 2003] Raskite visas natralisias n reikmes, su kuriomis reikinys 36
n
+ S
24
n
7
n
5
n
dalijasi i 899 be liekanos.
18. rodykite, kad jei p pirminis, tai (a +b)
p
a
p
+b
p
(mod p). S
19. Tegu q daugianaris su sveikaisiais koecientais. rodykite, kad bet kokiems S
sveikiesiems x ir y teisinga q(x) q(x +y) (mod y).
20. [LitMo 1988] Kiek skaitmen turi skaiius 1010 101, jeigu jis dalijasi i 9999? S
21. Raskite visus pirminius skaiius p, su kuriais 11 + p
2
turi ne daugiau nei 11 S
dalikli.
13
1.3. Oilerio teorema Skaii teorija
1.3 Oilerio teorema
Praeitame skyrelyje iekodami skaiiaus 2
1000
dalybos i 11 liekanos pastebjome, kad
keldami dvejet laipsniais 2
1
, 2
2
, 2
3
, . . . kakada gauname liekan 1, ir liekanos prade-
da kartotis. Pasirodo, is pastebjimas tinka daugumai skaii. Oilerio teorema kaip
tik tai ir rodo bei apibdina kartojimosi period. Jos atskiras atvjis yra maoji Fer-
ma (tariama Ferma) teorema, kurioje apsiribojama pirminiais moduliais. Nuo jos ir
pradkime.
Maoji Ferma teorema
Teorema. Tegu p pirminis skaiius, o a bet koks sveikasis, nesidalijantis i p. Tuomet
a
p1
1 (mod p).
rodymas. Uraykime visas skirtingas dalybos i p liekanas iskyrus 0:
1, 2, 3, . . . , p 2, p 1.
Padauginkime kiekvien i j i a:
1 a, 2 a, 3 a, . . . , (p 2) a, (p 1) a.
Parodysime, kad gautojo skaii rinkinio dalybos i p liekanos yra taip pat visos skir-
tingos ir be 0, t.y. tokios paios kaip pirmojo, tik, galbt, sumaiyta tvarka. Kad tarp
j nra 0 pamatyti nesunku, o kad jos visos skirtingos, rodysime prietaros bdu: jei
koki nors dviej skaii k a ir j a bt vienodos, tai j skirtumas dalintsi i p.
Taiau j skirtumas lygus a(k j) ir dalintis i p negali, nes a i p nesidalija pagal
slyg, o k j yra u p maesnis.
Vadinasi, kadangi abiej rinkini dalybos i p liekan aibs sutampa, tai j skaiius
sudaugin gausime po t pai liekan:
1 2 (p 1) a 1 a 2 a (p 1) (mod p)
(p 1)! a
p1
(p 1)! (mod p).
Kadangi dbd((p 1)!, p) = 1, tai galime padalinti:
a
p1
1 (mod p).
Pastaba. Maj Ferma teorem galima perrayti kaip a
p
a (mod p). i lygyb
kartais yra patogesn, nes galioja ir liekanai 0.
Naudojantis maja Ferma teorema iekoti sveikj skaii laipsni liekan moduliu
pirminio skaiiaus tampa visai paprasta:
Pavyzdys. Raskite, koki liekan gausime dalindami 7
727
i 17.
Pagal maj Ferma teorem 7
16
1 (mod 17). Kadangi 727 = 720+7 = 16 45+7,
tai
7
727
(7
16
)
45
7
7
7
7
(mod 17).
Likus 7
7
suskaiiuojame rankomis:
7
7
49
3
7 (2)
3
7 12 (mod 17).
14
1.3. Oilerio teorema Skaii teorija
Oilerio funkcija
rodindami maj teorem ne be reikalo atskyrme liekan 0 skaii besidalijant
i p keldami laipsniais tikrai niekada negausime liekanos 1 moduliu p. Nagrinjant
dalyb i sudtinio skaiiaus toki skaii atsiranda daugiau. Pavyzdiui, moduliu 6
nei dvejeto, nei trejeto, nei ketverto laipsniai niekada neduos liekanos 1. Tokius skaiius
atmesime ir nagrinsime tik tuos, su kuriais liekan 1 gauti galime. Kaip pamatysime
Oilerio teoremos rodyme, mums tinkantys skaiiai moduliu n bus tarpusavyje pirminiai
su n. Oilerio funkcija kaip tik ir ymi, kiek toki skaii yra.
Apibrimas. (n) ymi kiek yra skaii nedidesni nei n ir tarpusavyje pirmini su
n, t.y.
(n) = #a[1 a < n, dbd(a, n) = 1.
Nedideliems skaiiams reikm suskaiiuoti nesunku. Pavyzdiui (6) = 2, nes
vieninteliai skaiiai tarpusavyje pirminiai ir ne didesni nei 6 yra 1 ir 5. Bendru atveju
skaiiuoti galima naudojantis formule.
Teiginys.
(p

1
1
p

2
2
p

k
k
) = (p

1
1
p

1
1
1
)(p

2
2
p

2
1
2
)(p

k
k
p

k
1
k
).
rodymas. Suskaiiuokime, kiek yra skaii, kurie nra tarpusavyje pirminiai su duotuo-
ju. Paymj n = p

1
1
p

2
2
p

k
k
gausime, kad skaii, ne didesni nei n ir besidalijani
i p
1
yra
n
p
1
, besidalijani i p
2
yra
n
p
2
, , besidalijani i p
k
yra
n
p
k
. Jei sudsime
n
p
1
+ +
n
p
k
,
tai skaii` us, kurie dalijasi bent i dviej pirmini, bsime skaiiav per daug kart,
todl turime atimti:
n
p
1
+ +
n
p
k

n
p
1
p
2

n
p
k1
p
k
.
Taiau kart, skaiius, kurie dalijasi bent i trij pirmini, bsime skaiiav per
maai kart, todl turime pridti:
n
p
1
+ +
n
p
k

n
p
1
p
2

n
p
k1
p
k
+
n
p
1
p
2
p
3
+ +
n
p
n2
p
n1
p
n
.
Taip tsdami galiausiai suskaiiuosime, kiek yra skaii ne tarpusavyje pirmini su n.
Atm gaut rezultat i n rasime (n):
(n) = n (
n
p
1
+ +
n
p
k

n
p
1
p
2

n
p
k1
p
k
+ + (1)
k1
n
p
1
p
k
)
= n(1
1
p
1
)(1
1
p
2
) (1
1
p
k
)
= (p

1
1
p

1
1
1
)(p

2
2
p

2
1
2
)(p

k
k
p

k
1
k
).
15
1.3. Oilerio teorema Skaii teorija
Oilerio teorema
Teorema. Tegu n natralusis skaiius, o a sveikasis ir tarpusavyje pirminis su n.
Tuomet
a
(n)
1 (mod n).
rodymas. Uraykime visas skirtingas dalybos i n liekanas tarpusavyje pirmines su n:
r
1
, r
2
, . . . , r
(n)
.
Padauginkime kiekvien i j i a:
r
1
a, r
2
a, . . . , r
(n)
a.
Parodysime, kad gautojo skaii rinkinio dalybos i n liekanos yra taip pat visos skir-
tingos ir tarpusavyje pirmins su n, t.y. tokios paios kaip pirmojo rinkinio, tik, galbt,
sumaiyta tvarka. Kad jos visos tarpusavyje pirmins su n seka, i to, kad ir r
i
ir a
yra tarpusavyje pirminiai su n. Kad jos visos skirtingos, rodysime prietaros bdu: jei
koki nors dviej skaii r
k
a ir r
j
a dalybos liekanos bt vienodos, tai j skirtumas
dalintsi i n. Taiau j skirtumas lygus a(r
k
r
j
) ir dalintis i n negali, nes a yra
tarpusavyje pirminis su n, o r
k
r
j
yra u n maesnis.
Vadinasi, kadangi abiej rinkini dalybos i n liekan aibs sutampa, tai j skaiius
sudaugin gausime po t pai liekan:
r
1
r
2
r
(n)
a
(n)
r
1
r
2
r
(n)
(mod m).
Kadangi dbd(r
1
r
(n)
, p) = 1, tai galime padalinti:
a
(n)
1 (mod n).
Pavyzdiai
1 Pavyzdys. Raskite paskutin skaiiaus 13
13
skaitmen
Sprendimas. Paskutinis skaiiaus skaitmuo yra toks pat, kaip ir dalybos i 10 liekana.
Kadangi 13 ir 10 yra tarpusavyje pirminiai, tai galime pasinaudoti Oilerio teorema.
Raskime (10):
(10) = (2 5) = (2
1
2
0
)(5
1
5
0
) = 4.
Tuomet pagal Oilerio teorem 13
4
1 (mod 10), todl
13
13
= 13
12
13 13 3 (mod 10).

2 Pavyzdys. Raskite paskutin skaiiaus 13


13
13
skaitmen.
16
1.3. Oilerio teorema Skaii teorija
Sprendimas. Kadangi pagal praeit pavyzd 13
4
1 (mod 10), tai reikia rasti, koki
liekan gausime dalindami laipsn 13
13
i 4. T padaryti visai nesunku 13
13
1
13

1 (mod 4). Gavome


13
13
13
13
1
3 (mod 10).

3 Pavyzdys. Raskite du paskutiniuosius skaiiaus 133333


13333
1333
133
13
skaitmenis
Sprendimas. Paskutiniai du skaiiaus skaitmenys yra tokie patys, kaip ir dalybos i
100 liekana. Kadangi 100 ir 133333 yra tarpusavyje pirminiai, tai galime taikyti Oilerio
teorem. Raskime (100):
(100) = (2
2
5
2
) = (2
2
2
1
)(5
2
5) = 40.
Nordami rasti laipsnio 13333
1333
133
13
liekan moduliu 40, dar kart taikykime Oi-
lerio teorem. Randame (40) = 16.
Nordami rasti laipsnio 1333
133
13
liekan moduliu 16, dar kart taikykime Oilerio
teorem. Randame (16) = 8.
Nordami rasti laipsnio 133
13
liekan moduliu 8, dar kart (pagaliau paskutinj)
taikykime Olerio teorem. Kadangi (8) = 4, tai
133
13
133
1
5 (mod 8),
tada
1333
133
13
1333
5
5
5
5 (mod 16),
tada
13333
1333
133
13
13333
5
13
5
13 (mod 40),
tada
133333
13333
1333
133
13
133333
13
33
13
33 (11)
6
33 21
3
13 (mod 100).
Gavome, kad 133333
13333
1333
133
13
paskutiniai du skaitmenys yra 13.
Udaviniai
1. Raskite, koki liekan gausime dalindami 3
33
i 13, 7
77
i 17, 9
99
i 19. S
2. Raskite 11
11
11
dalybos i 15 liekan. S
3. Kodl keldami laipsniais skaiius, kurie nra tarpusavyje pirminiai su n, niekada S
negausime liekanos 1 moduliu n?
4. Tegu p, q skirtingi pirminiai. rodykite, kad pq[n
pq
n
p
n
q
+n su visais n N. S
5. Tegu a,b,c sveikieji skaiiai ir a+b +c = 0. Ar gali a
47
+b
47
+c
47
bti pirminis? S
17
1.3. Oilerio teorema Skaii teorija
6. rodykite, kad kiekvienam pirminiam p, iskyrus 2 ir 5, egzistuoja be galo daug S
pavidalo 11 . . . 11 skaii, besidalijani i p.
7. [LitKo 2002] rodykite, kad egzistuoja be galo daug toki natralij skaii n, S
kad 2003
n
1 dalijasi i n be liekanos.
8. [Bulgaria Winter Competition 2009] Ant lentos uraytas natralusis skaiius. S
Prie jo deins galime prirayti bet kok skaitmen iskyrus 9. rodykite, kad kaip
beraintume, ilgainiui gausime sudtin skaii.
9. [CWMO 2008] Tegu a
1
, a
2
, . . . , a
m
natralieji skaiiai, m 2. rodykite, kad S
egzistuoja be galo daug natralij n, su kuriais a
1
1
n
+ a
2
2
n
+ + a
m
m
n
yra
sudtinis.
10. [CMO 2008] Raskite visas funkcijas f : N N, visiems pirminiams p ir natra- S
liesiems n tenkinanias
f(n)
p
n (mod f(p)).
11. [IMO 2005] Raskite sveikuosius skaiius, kurie yra tarpusavyje pirminiai su visais S
sekos a
n
= 2
n
+ 3
n
+ 6
n
1 nariais.
18
1.4. Kin liekan teorema Skaii teorija
1.4 Kin liekan teorema
Raskime skaiiaus 2
100
dalybos i 10 liekan. Oilerio teoremos naudoti negalime, nes 2
ir 10 nra tarpusavyje pirminiai. Ieitis yra udavin iskaidyti dvi dalis rasti liekan
moduliu 2 ir moduliu 5 atskirai. Tai padaryti nesunku pagal Oilerio teorem
2
100
1 (mod 5),
ir, akivaizdiai,
2
100
0 (mod 2).
Kaip sujungti gaut informacij? Jei usiraysime 2
100
= 10k + r, kur r yra iekoma
dalybos liekana, tai gausime, jog r turi tenkinti du lyginius vienu metu:
_
r 1 (mod 5)
r 0 (mod 2)
Tarp skaii nuo 0 iki 9 toks yra tik vienas 6. Jis ir bus iekoma liekana.
Kin liekan teorema yra io samprotavimo apibendrinimas:
Teorema (Kin liekan teorema). Tegu n = m
1
m
2
m
k
, kur visi m
i
yra paporiui
tarpusavyje pirminiai. Visiems sveikiesiems r
1
, r
2
, . . . r
k
lygini sistema
_

_
r r
1
(mod m
1
)
r r
2
(mod m
2
)
.
.
.
r r
k
(mod m
k
)
turi vienintl sprendin intervale [0, n 1].
rodymas. Pirmiausia rodykime, kad bent vien sprendin turi paprastesn lygini sis-
tema:
_

_
r 1 (mod m
1
)
r 0 (mod m
2
)
.
.
.
r 0 (mod m
k
)
Ities, kadangi m
1
ir m
2
m
3
m
k
yra tarpusavyje pirminiai, t.y. j didiausias
bendras daliklis yra lygus 1, tai pagal Euklido algoritmo ivad egzistuoja tokie sveikieji
x ir y, kad xm
1
+ym
2
m
3
m
k
= 1. Skaiius ym
2
m
3
m
k
kaip tik ir bus sprendinys.
Paymkime j e
1
.
Isprend analogikas sistemas, kur liekana 1 atitiks vis kit m
i
gausime k skaii
e
1
, e
2
, . . . , e
k
. Nesunku sitikinti, kad sudaugin paporiui e
1
r
1
+e
2
r
2
+ e
k
r
k
gausime
pradins sistemos sprendin.
Parodysime, kad visi sistemos sprendiniai skiriasi per n kartotin. Tarkime, kad
turime du sistemos sprendinius r ir r

. Jie duoda vienodas liekanas dalijami i vis m


i
,
todl m
1
[(r r

), m
2
[(r r

), . . . , m
k
[(r r

). Kadangi visi m
i
yra paporiui tarpusavyje
pirminiai, tai gauname, kad n[(r r

).
Galiausiai pastebkime, kad jei prie vieno sprendinio pridsime ar atimsime n, gau-
sime kit sprendin. Tai ir rodo, kad bus lygiai vienas sprendinys intervale [0, n1].
19
1.4. Kin liekan teorema Skaii teorija
Pavyzdiai
1 Pavyzdys. Isprskite lygini sistemas:
_

_
r 2 (mod 3),
r 2 (mod 5),
r 2 (mod 7);
ir
_

_
r 1 (mod 2),
r 2 (mod 3),
r 3 (mod 5).
Sprendimas. Nors kin liekan teoremos rodymas konstruktyvus (t.y. jo metu yra pa-
rodoma, kaip gauti sprendinius), retai kada jis praveria sprendiant konkrei sistem.
Daniausiai efektyviau pabandyti tiesiog atspti sprendin, arba sprsti lygtis po vien
ir iekoti bendr sprendini. T ir padarysime.
Geriau siirjus pirmj sistem turt bti nesunku i karto atspti, kad jos
sprendiniu bus r = 3 5 7 + 2 arba tiesiog r = 2.
Antroji sistema kiek sudtingesn. I lygties r 3 (mod 5) inome, kad sprendinio
paskutinis skaitmuo bus 3 arba 8. Taiau pastarasis netinka, nes r 1 (mod 2). Lieka
i skaii, kuri paskutinis skaitmuo 3 rasti tenkinant lygt r 2 (mod 3). Patikrin
kelet variant randame r = 23.
2 Pavyzdys. Isprskite lygini sistem:
_

_
2r 1 (mod 3),
3r 2 (mod 5),
4r 3 (mod 7).
Sprendimas. Pertvarkykime lygtis. Pastebkime, kad 2r 1 (mod 3) tada ir tik tada,
kai 4r 2 (mod 3), nes dbd(2, 3) = 1. Kadangi 4r r (mod 3), tai vietoje buvusios
pirmosios lygties gauname ekvivaleni r 2 (mod 3). Analogikai i 2 padaugin ir
likusias gausime sistem
_

_
r 2 (mod 3),
r 4 (mod 5),
r 6 (mod 7).
Nesunku atspti, kad r = 3 5 7 1 (arba tiesiog r = 1) yra ios sistemos
sprendinys.

3 Pavyzdys. rodykite, kad egzistuoja deimt paeiliui einani natralij skaii,


besidalinani i deimtj pirmini skaii laipsni.
Sprendimas. Isirinkime bet kokius deimt pirmini skaii p
1
, p
2
, . . . , p
10
. rodysime,
kad egzistuoja toks natralusis r, kad p
10
1
[r, p
10
2
[r+1, , p
10
10
[r+9, tuomet r, r+1, . . . , r+9
ir bus iekomi paeiliu einantys skaiiai. Taiau perra slygas kaip
_

_
r 0 (mod p
10
1
)
r 1 (mod p
10
2
)
. . .
r 9 (mod p
10
10
)
20
1.4. Kin liekan teorema Skaii teorija
matome, kad toks r egzistuos pagal Kin liekan teorem.
Udaviniai
1. Isprskite lygini sistemas: S
_

_
r 0 (mod 5),
r 4 (mod 7),
r 3 (mod 11);
ir
_

_
3r 1 (mod 5),
3r 1 (mod 7),
3r 1 (mod 11).
2. Koki liekan gausime dalindami skaii 123456789101112 . . . 20082009 i 450? S
3. [Ireland 2000] Raskite maiausij natralj a, kad 5x
13
+13x
5
+9ax dalintsi S
i 65 su visomis natraliomis x reikmmis.
4. Ar egzistuoja toks natralusis a, kad skaiiai a, 2a, 3a, . . . , 1997a bt natralij S
skaii laipsniai?
5. [USAMO 2008] rodykite, kad kiekvienam n N egzistuoja tokie didesni u S
vienet tarpusavyje pirminiai skaiiai k
1
, k
2
, . . . , k
n
, kad k
1
k
2
k
n
1 isiskaido
kaip dviej paeiliui einani natralij skaii sandauga.
6. [France TST 2003] Sveikj skaii gardels ploktumoje tak vadinsime ne- S
matomu, jei j ir koordinai pradios tak jungianiai atkarpai priklauso dar
bent vienas gardels takas. rodykite, kad kiekvienam n atsiras kvadratas, kurio
kratins ilgis n ir kurio visi viduje esantys takai yra nematomi.
21
1.5. Liekan grup Skaii teorija
1.5 Liekan grup
iame skyrelyje kalbsime apie liekanas atsiedami jas nuo konkrei skaii. Sakydami,
pavyzdiui, sudaugin liekanas a ir b moduliu n gausime liekan c tursime omenyje,
kad sudaugin bet kok skaii, duodant liekan a su bet kokiu skaiiumi duodant
liekan b gausime skaii, duodant liekan c.
Nagrinkime liekanas moduliu n, tarpusavyje pirmines su n. J daugyba pasiymi
keturiomis savybmis:
udarumas - Sudaugin bet kurias dvi, vl gausime liekan, tarpusavyje pirmin su
n;
vienetinis elementas - Egzistuoja tokia liekana, btent 1, i kurios dauginant kitas
liekanas jos nepakinta;
atvirktinis elementas - Kiekvienai liekanai egzistuoja jai atvirktin liekana, t.y.
tokia, kad padaugin i jos gauname 1;
asociatyvumas - Kiekvienoms liekanoms a, b, c yra teisinga lygyb a(bc) = (ab)c.
Pirmosios dvi savybs labai lengvai patikrinamos. rodykime treij. Jei a ir n
tarpusavyje pirminiai, tai pagal Euklido algoritmo ivad, egzistuoja sveikieji skaiiai
x ir y tenkinantys lygyb ax +ny = 1. Tuomet x ir bus atvirtin a liekana, nes ax
1 (mod n). Ketvirtoji savyb, atrodanti kiek neprastai, galioja visiems sveikiesiems
skaiiams, todl galioja ir liekanoms.
Abstrakiojoje algebroje aib su joje apibrta operacija, tenkinania ias keturias
savybes, vadinama grupe, todl kartais mes pagrstai naudosime termin liekan grup,
turdami omenyje liekanas moduliu n, tarpusavyje pirmines su n.
Liekanos eil
Nagrinkime liekan moduliu n grup.
Apibrimas. Liekanos a eile vadinsime maiausi natralj laipsn s, su kuriuo a
s

1 (mod n).
Apibrimas. Liekan grups eile vadinsime liekan grups element skaii.
Naudodamiesi iais terminais galime performuluoti Oilerio teorem:
Teorema. Liekanos eil dalo grups eil.
rodymas. Grups eil yra lygi liekan, tarpusavyje pirmini su n, skaiiui, t.y (n).
I Oilerio teoremos inome, kad bet kuriai liekanai a yra teisinga a
(n)
1 (mod n).
Tegu s yra a eil ir tarkime, kad s nedalo (n). Tada dalindami (n) i s gausime
(n) = qs +r, kur 0 < r < s. Taiau tuomet
1 a
(n)
a
qs+r
a
r
.
Gavome, kad egzistuoja maesnis laipsnis u s, kuriuo pakl liekan a gauname 1.
Prietara.
22
1.5. Liekan grup Skaii teorija
Panagrinkime konkret atvej. Liekan moduliu 7 grup sudaro eios liekanos
1, 2, 3, 4, 5, 6. Vadinasi, kiekvieno elemento eil turi bti ei daliklis. Patikrinkime:
1
1
1 eil 1;
2
1
2, 2
2
4, 2
3
1 eil 3;
3
1
3, 3
2
2, 3
3
6, 3
4
4, 3
5
5, 3
6
1 eil 6;
4
1
4, 4
2
2, 4
3
1 eil 3;
5
1
5, 5
2
4, 5
3
6, 5
4
2, 5
5
3, 5
6
1 eil 6;
6
1
6, 6
2
1 eil 2.
Ciklin grup moduliu p
Apibrimas. Liekan grup, kurios visas liekanas galime urayti kaip kakurios
vienos liekanos g laipsnius, vadinsime cikline grupe. Liekan g vadinsime liekan grups
generatoriumi.
Kaip matme keliomis eilutmis aukiau, visas liekanas moduliu 7 tarpusavyje pi-
mines su 7 galima urayti kaip trejeto (ir kaip penketo) laipsnius, tad i grup yra
ciklin ir ji turi du generatorius 3 ir 5. Tai galioja ir bendresniu atveju:
Teorema. Liekan grup moduliu pirminio skaiiaus p yra ciklin.
rodym iskaidysime atskiras dalis. Pirma, rodysime, kad grup yra ciklin,
jei egzistuoja liekana, kurios eil sutampa su grups eile. Antra, grups eil iskai-
dysime dauginamaisiais p 1 = q

1
1
q

2
2
q

k
k
ir rodysime, kad egzistuoja elementai
g
1
, g
2
, , g
k
, kuri eils yra atitinkamai q

1
1
, q

2
2
, , q

k
k
. Treia, rodysime, kad san-
daugos g
1
g
2
g
k
eil yra lygi grups eilei.
Teiginys (Pirma dalis). Jei egzistuoja liekana, kurios eil yra lygi liekan grups eilei,
tai jos laipsniais galime urayti visas grups liekanas.
rodymas. Tarkime, kad egzistuoja liekana g, kurios eil lygi grups eilei p1. Kelkime
j laipsniais g
1
, g
2
, . . . , g
p1
. Jokie du i j negali bti lygs. Ities, jei gautume, kad
g
i
g
j
(i > j), tai i to sekt g
ij
1, ko bti negali, nes i j < p 1. Kadangi visi
laipsniai yra skirtingi ir j yra tiek, kiek grups liekan, tai ios dvi aibs sutampa.
Liekan, panaiai kaip ir realj skaii, vadinsime daugianario aknimi, jei sta-
t j gauname nulin liekan. Toliau einaniuose teiginiuose tursime omenyje, kad
nagrinjamos liekanos yra moduliu pirminio skaiiaus p.
Teiginys. n tojo laipsnio daugianaris turi ne daugiau kaip n akn.
rodymas. rodykime naudodami indukcij. Pirmojo laipsnio daugianaris x a turi tik
vien akn a. Tarkime, kad n1 laipsnio daugianaris turi ne daugiau kaip n1 akn.
Nagrinkime n tojo laipsnio daugianar p(x). Jei jis neturi n vienos aknies, tai
teiginys teisingas. Jei turi akn a, tai galime j iskaidyti p(x) = (x a)q(x), kur q(x)
yra n 1 laipsnio daugianaris. Kadangi daugianario p(x) aknis turi bti arba a, arba
daugianario q(x) aknimi, tai pagal indukcij p(x) turs ne daugiau nei n 1 + 1 = n
akn.
23
1.5. Liekan grup Skaii teorija
Teiginys. Daugianaris x
p1
1 turi lygiai p 1 akn.
rodymas. Pagal Oilerio teorem, jo aknimis yra visos liekanos.
Teiginys. Daugianaris x
d
1, kur d[p 1 turi lygiai d akn.
rodymas. Iskaidykime daugianar x
p1
1 dauginamaisiais:
x
p1
= (x
d
1)(x
p1d
+x
p12d
+ +x
d
+ 1).
Kadangi kairje pusje esantis daugianaris turi p1 akn, tai tiek pat akn turi turti
ir deinje pusje esantis daugianaris. Jei x
d
1 turt maiau nei d akn, tai deinje
pusje esantis daugianaris turt maiau nei d + (p 1 d) akn.
Teiginys (Antra dalis). Tegu p 1 isiskaido kaip p 1 = q

1
1
q

2
2
q

k
k
. Kiekvienam
i egzistuoja liekana, kurios eil yra q

i
1
.
rodymas. Liekanos eil bus lygi q

i
i
, jei ji bus aknis daugianario x
q

i
i
1 , bet nebus
aknis daugianario x
q

i
1
i
1. Kadangi pirmasis daugianaris turi daugiau akn nei
antrasis, tai q

i
i
eils liekana egzistuoja.
Teiginys (Treia dalis). liekan sandaugos g
1
g
2
g
k
eil yra lygi p 1.
rodymas. Sandaugos g
1
g
2
g
k
eil dalo grups eil, todl j galime urayti q

1
1
q

2
2
q

k
k
.
Jei ji nra lygi grups eilei, tai bent vienas i
i
yra maesnis u
i
. Paprastumo dlei
tarkime, kad tai
1
. Pakl g
1
g
2
g
k
didesniu nei eil laipsniu q

1
1
q

2
2
q

k
k
, gausime
1 (g
1
g
2
g
k
)
q

1
1
q

2
2
q

k
k
g
q

1
1
q

2
2
q

k
k
1
,
ko bti negali, nes g
1
eil yra q

1
1
ir ji nedalo q

1
1
q

2
2
q

k
k
.
Teisingas yra ir kiek bendresnis teiginys liekan grups yra ciklins moduliu bet
kokio pirminio skaiiaus laipsnio (p
n
) ir moduliu bet kokio pirminio skaiiaus laipsnio,
padauginto i dviej (2p
n
). i teigini rodym nepateiksime, nes jie gantinai ilgi ir
sudtingi.
Pavyzdiai
1 Pavyzdys. Tegu liekanos a eil moduliu pirminio p yra 2k. Tuomet a
k
1 (mod p).
Sprendimas. Pastebkime, kad (a
k
)
2
1 (mod p), bet a
k
, 1 (mod n). Kadangi
daugianaris x
2
1 turi lygiai dvi aknis 1 ir 1, tai a
k
turi bti lygus antrajai.
2 Pavyzdys. rodykite Wilson teorem: jei p pirminis, tai
(p 1)! 1 (mod p).
24
1.5. Liekan grup Skaii teorija
Sprendimas. Pirmasis rodymas. Daugianario x
p1
1 aknimis yra visos liekanos
moduliu p iskyrus 0, todl galime iskaidyti
x
p1
1 (x 1)(x 2) (x (p 1)).
Lieka statyti x = 0.
Antrasis rodymas. Kiekviena liekana turi sau atvirktin, su kuria sudauginta lygi
1. Suporavus liekanas su j atvirktinmis, liks tos, kurios yra paios sau atvirktins.
Jos tenkina x
2
1 (mod p) ir yra tik dvi 1 ir 1, ir j sandauga lygi 1.
Treiasis rodymas. Kadangi liekan moduliu p grup yra ciklin, tai visas liekanas
galime urayti kaip generatoriaus g laipsnius g
1
, g
2
, g
p1
. J sandauga yra lygi
g
p(p1)/2
. Kadangi p(p1)/2 nesidalija i (p1), tai g
p(p1)/2
, 1 (mod p). Kita vertus
(g
p(p1)/2
)
2
1 (mod p),
o daugianaris x
2
1 turi tik dvi aknis 1 ir 1, todl g
p(p1)/2
turi bti lygus antrjai.

3 Pavyzdys. Jei dbd(a, b) = 1, tai


dbd(a
n
b
n
, a
m
b
m
) = a
dbd(n,m)
b
dbd(n,m)
.
Sprendimas. Akivaizdu, kad a
dbd(n,m)
b
dbd(n,m)
[ dbd(a
n
b
n
, a
m
b
m
). rodykime
kit pus. Paymkime d = dbd(a
n
b
n
, a
m
b
m
). Tuomet
a
n
b
n
(mod d) (ab
1
)
n
1 (mod d)
(atvirktin b liekana moduliu d egzistuos, nes dbd(a, b) = 1 = dbd(b, d) = 1).
Analogikai ir
(ab
1
)
m
1 (mod d).
Pasinaudoj Euklido algoritmo ivada ir ireik dbd(n, m) = xm + yn gauname, kad
ir (ab
1
)
dbd(n,m)
1 (mod d), t.y. d[a
dbd(n,m)
b
dbd(n,m)
.
4 Pavyzdys. rodykite, kad 6
p2
+ 3
p2
+ 2
p2
1 dalijasi i p, kur p > 3 pirminis.
Sprendimas. Pateiksime kiek kitok io performuluoto IMO 2005 udavinio sprendim,
nei skyrelyje Oilerio teorema:
Pagal maj Ferma teorem, gauname kad
6
p1
3
p1
2
p1
1 (mod p),
vadinasi,
6
p2
+ 3
p2
+ 2
p2
1 6
1
+ 3
1
+ 2
1
1
1
6
+
1
3
+
1
2
1 0 (mod p).
Kiek pagrstai susumavome liekanas lyg prastas trupmenas? Pasirodo, visikai
pagrstai. prasta trupmen suma
x
a
+
y
b
=
ay+bx
ab
yra ne kas kita, kaip kitaip urayta
lygyb
xa
1
+yb
1
= (ay +bx)a
1
b
1
,
kuri, akivaizdu (pakanka atskliausti), yra teisinga ir liekanoms.
25
1.5. Liekan grup Skaii teorija
Udaviniai
1. Raskite liekan grups moduliu 11 generatorius. S
2. rodykite, kad liekanos ir jos atvirktins liekanos eils sutampa. S
3. Kodl visos liekanos moduliu sudtinio skaiiaus nesudaro grups? S
4. Teiginys, kad n-tojo laipsnio daugianaris turi ne daugiau nei n akn nra tei- S
singas liekanoms moduliu sudtinio skaiiaus. Pavyzdiui, moduliu 6 daugianaris
x
2
+x turi keturias aknis 0, 2, 3 ir 5. Kuri teiginio rodymo dalis tampa neteisinga
sudtiniams skaiiams?
5. Tegu p nelyginis pirminis skaiius. rodykite, kad a yra liekan mod p genera- S
torius tada ir tik tada, kai a
p1
q
, 1 (mod p) su visais pirminiais p 1 dalikliais
q.
6. rodykite, kad liekan grup moduliu pirminio p turi (p 1) generatori. S
7. Parodykite, kad 2 liekan grups moduliu 29 generatorius. Parod isprskite S
lygtis:
a) x
7
1 (mod 29)
b) x
6
+x
5
+... +x + 1 0 (mod 29)
8. rodykite, kad vis grups moduliu pirminio p generatori sandauga yra lygi S
(1)
(p1)
.
9. Isprskite lygt x
17
1 mod 19. S
10. rodykite, kad 1
k
+ 2
k
+ ... + (p 1)
k
0 (mod p), jei p 1 nedalo k ir S
1 (mod p), jei p 1 dalo k.
11. Tegu p = 2
n
+1, n 2 pirminis skaiius. rodykite, kad 3 yra grups moduliu S
p generatorius:
a) Tegu g vienas i grups moduliu p generatori. Parodykite, kad visi nelyginiai
g laipsniai taip pat bus generatoriais.
b) Parodykite, kad jei 3 nra generatorius, tai 3 a
2
(mod p) su kakokiu a.
c) Tegu 2u a 1 (mod p). Parodykite, kad u yra treios eils elementas.
d) Gaukite prietar.
12. rodykite, kad jei a yra treios eils elementas moduliu pirminio p > 3, tai a +1 S
yra etos eils elementas mod p.
13. rodykite, kad liekan grup moduliu pq, kur p ir q skirtingi pirminiai skaiiai, S
nra ciklin.
14. rodykite, kad n nedalo 2
n
1, kur n > 1 natralusis skaiius. S
15. [Ireland 1992] rodykite, kad vis natralij skaii, maesni u n ir tarpusa- S
vyje pirmini su n, kub suma dalijasi i n.
26
1.5. Liekan grup Skaii teorija
16. Raskite visus daugianarius q(x) su sveikais koecientais, tenkinanius q(n)[2
n
1 S
su visais n N.
17. Raskite visus pirminius p ir q, su kuriais pq[2
p
+ 2
q
. S
18. [Russia 2009] Tegu x, y sveikieji skaiiai ir 2 x, y, 100. rodykite, kad S
egzistuoja toks n N, su kuriuo x
2
n
+y
2
n
nra pirminis.
19. [INAMO 2009] rodykite, kad kiekvieniems tarpusavyje pirminiams a ir b egzis- S
tuoja tokie natralieji m ir n, kad a[m, b[n, bet a n, b m, ir tenkinantys
m[n
2
+n ir n[m
2
+m.
20. [India TST] Tegu n 2 natralusis skaiius ir n[3
n
+ 4
n
. rodykite, kad 7[n. S
21. [Hong Kong TST 2009] rodykite, kad lygtis x
37
y
3
+ 11 (mod p) turi spren- S
dini su visais pirminiais p 100.
27
1.6. Kvadratins liekanos Skaii teorija
1.6 Kvadratins liekanos
iame skyrelyje apvelgsime teorij, apibdinani, kokias liekanas galime gauti dalin-
dami sveikj skaii kvadratus i pirmini skaii.
Apibrimas. Liekanas moduliu pirminio skaiiaus p, kurias galime gauti dalinda-
mi sveikj skaii kvadratus i p, vadinsime kvadratinmis, o tas, kuri negalime,
nekvadratinmis. Nulin liekan laikysime iskirtine. Kvadratinms ir nekvadratinms
liekanoms ymti naudosime Leandro simbol:
_
a
p
_
=
_

_
1, jei a yra kvadratin liekana moduliu p,
1, jei a nra kvadratin liekana moduliu p,
0, jei a 0 (mod p).
Pairkime, kaip tai atrodo konkreiu atveju:
Pavyzdys. Raskime visas kvadratines liekanas moduliu 7.
Sprendimas. Pakelkime visas liekanas moduliu 7 kvadratu:
1
2
1 (mod 7), 2
2
4 (mod 7), 3
2
2 (mod 7),
4
2
2 (mod 7), 5
2
4 (mod 7), 6
2
1 (mod 7).
Gavome, kad kvadratins liekanos yra 1, 2 ir 4, o nekvadratins 3, 5 ir 6. Naudodami
Leandro simbol tai galime urayti taip:
_
1
7
_
= 1,
_
2
7
_
= 1,
_
4
7
_
= 1,
_
3
7
_
= 1,
_
5
7
_
= 1,
_
6
7
_
= 1.
_
0
7
_
= 0.

Kvadratini liekan struktra


rodysime kelet teigini, kurie pads labiau suprasti kvadratini liekan struktr.
Teiginys. Tegu g liekan grups moduliu p generatorius. Tuomet visos kvadrati-
ns liekanos bus uraomos kaip lyginiai g laipsniai, o nekvadratins liekanos kaip
nelyginiai.
rodymas. Pastebkime, kad pats generatorius nra kvadratin liekana. Ities, jei g
t
2
(mod p), tai g
(p1)/2
t
p1
1 (mod p) prietara. Lyginiai generatoriaus laipsniai
bus kvadratins liekanos, nes g
2k
(g
k
)
2
(mod p), o nelyginiai nebus, nes i g
2k+1

t
2
(mod p) sekt g (tg
k
)
2
(mod p), kas reikt, kad generatorius g yra kvadratin
liekana.
28
1.6. Kvadratins liekanos Skaii teorija
Taigi galime naudotis tam tikra prasme analogiku sveikiesiems skaiiams kvad-
ratikumo" kriterijumi liekana yra kvadratin tada ir tik tada, kai ji yra lyginis
generatoriaus laipsnis. I to seka, kad dviej kvadratini arba dviej nekvadratini
liekan sandauga yra kvadratin liekana, o vienos kvadratins ir vienos nekvadratins
nekvadratin. Tai galime urayti kaip:
Teiginys.
_
a
p
__
b
p
_
=
_
ab
p
_
.
Pastebkime, kad i lygyb galioja ir tuo atvju, kai a ar b dalijasi i p. Treiasis
teiginys leidia nustatyti, ar liekana kvadratin, ar ne, pairjus jos (p 1)/2 laipsn:
Teiginys. a
(p1)/2

_
a
p
_
(mod p).
rodymas. Jei a yra kvadratin liekana, tai a t
2
(mod p) ir
a
(p1)/2
t
p1
1 (mod p).
Jei a nra kvadratin liekana tai ji yra nelyginis generatoriaus laipsnis, t.y. a g
N
,
taiau tuomet
a
(p1)/2
g
N(p1)/2
g
(p1)/2
, 1 (mod p).
Kadangi iuo atveju a
(p1)/2
nelygsta vienam, o jos kvadratas lygsta vienam, tai a
(p1)/2
lygsta 1.
I io teiginio seka labai svarbi ir naudinga ivada:
Ivada. 1 yra kvadratin liekana moduliu p tada ir tik tada, kai p 1 (mod 4), t.y.
_
1
p
_
= (1)
p1
2
.
rodymas. Utenka statyti a = 1 praeito teiginio lygyb.
Kvadratinio apveriamumo teorema
rodysime centrin io skyrelio teorem. Ji pati yra labai naudinga sprendiant udavi-
nius, taiau rodymas yra ilgokas, tad nesikrimskite, jei nepavyks jo i karto veikti.
Teorema (Kvadratinio apveriamumo teorema). Tegu p ir q nelyginiai pirminiai
skaiiai. Tuomet
_
q
p
__
p
q
_
= (1)
p1
2
q1
2
.
rodymas. Paimkime bet koki liekan a moduliu p ir dauginkime j i i = 1, 2, . . . , (p
1)/2. Kiekvienai sandaugai uraykime lygyb
a i = p q
i
+r
i
taip, kad liekana r
i
bt tarp (p 1)/2 ir (p 1)/2, o ne tarp 1 ir p 1 kaip prasta.
Neigiam liekan skaii paymkime
a
ir sudauginkime lygybes moduliu p. Gausime
a
(p1)/2
(p1)/2

i=1
i (1)

a
(p1)/2

i=1
[r
i
[ (mod p).
29
1.6. Kvadratins liekanos Skaii teorija
Pastebkime, kad joki dviej liekan moduliai negali bti vienodi, nes gautume
r
i
= r
j
a i a j (mod p) p[a(i j),
ko negali bti, nes a i p nesidalija, ir p < i j < p. Kadangi liekanos [r
i
[ yra
skirtingos ir tarp 1 ir (p 1)/2, tai jos tegali bti lygios 1, . . . , (p 1)/2, i ko seka, kad

(p1)/2
i=1
i =

(p1)/2
i=1
[r
i
[. Tuomet, suprastin lygyb, gauname
_
a
p
_
a
(p1)/2
(1)

a
(mod p),
arba, stat a = q,
_
q
p
_
(1)

q
(mod p).
Nordami rasti, ar
q
yra lyginis, ar nelyginis, pradioje uraytas lygybes perray-
kime moduliu 2. Vietoje
q i = p q
i
+r
i
gausime
i q
i
+[r
i
[ (mod 2),
nes p, q nelyginiai ir r
i
r
i
(mod 2). Dalmuo q
i
, jei liekana r
i
buvo teigiama, yra
lygus
qi
p
|, o jei liekana buvo neigiama
qi
p
| +1. Tad susumav visas lygybes gausime
(p1)/2

i=1
i +
(p1)/2

i=1

qi
p
| +
(p1)/2

i=1
[r
i
[ (mod 2).
Samprotaudami kaip ir praeit kart gauname, kad
(p1)/2

i=1
i =
(p1)/2

i=1
[r
i
[,
todl suprastin randame

q

(p1)/2

i=1

qi
p
| (mod 2),
ir tuomet
_
q
p
_
(1)

(p1)/2
i=1

qi
p

.
Analogikai, kadangi q taip pat yra pirminis, gausime
_
p
q
_
(1)

(q1)/2
i=1

pi
q

.
Sudauginkime iraikas:
_
q
p
__
p
q
_
(1)

(p1)/2
i=1

qi
p
+

(q1)/2
i=1

pi
q

.
Lieka rodyti, kad
(p1)/2

i=1

qi
p
| +
(q1)/2

i=1

pi
q
| =
p 1
2
q 1
2
.
30
1.6. Kvadratins liekanos Skaii teorija
Tuo sitikinti nesunku pirmoji suma atitinka sveikuosius staiakampio takus (r.
brin), esanius po tiese y =
q
p
x, o antroji atitinka sveikuosius staiakampio takus
esanius vir tiess.
Kvadratinio apveriamumo teorema galioja tik nelyginiams pirminiams. Dvejet
reikia nagrinti atskirai:
Teorema. Liekana 2 yra kvadratin moduliu p tada ir tik tada, kai p 1 (mod 8),
t.y.
_
2
p
_
= (1)
p
2
1
8
.
rodymas. Pasinaudosime kvadratinio apveriamumo teoremos rodymo metu gauta ly-
gybe (dar vadinama Gauss lema) atveju a = 2:
_
2
p
_
(1)

2
(mod p).
Pasirodo, iuo atveju galima suskaiiuoti tiksli
2
reikm, priklausomai nuo pirminio
p dalybos i 8 liekanos. Nagrinkime 4 atvejus:
p = 8k + 1 iuo atveju bus i viso 4k liekan, padauginus jas i dviej, 2k bus nedidesns nei
4k ir 2k bus didesns. Vadinasi, bus lygus 2k ir
_
2
p
_
= 1.
p = 8k + 3 iuo atveju bus i viso 4k +1 liekana, padauginus jas i dviej, 2k bus nedidesns
nei 4k + 1 ir 2k + 1 bus didesns. Vadinasi bus lygus 2k + 1 ir
_
2
p
_
= 1.
p = 8k + 5 iuo atveju bus i viso 4k + 2 liekanos, padauginus jas i dviej, 2k + 1 bus ne
didesns nei 4k + 2, ir 2k + 1 bus didesns. Vadinasi, bus lygus 2k + 1 ir
_
2
p
_
= 1.
p = 8k + 7 iuo atveju bus i viso 4k + 3 liekanos, padauginus jas i dviej, 2k + 1 bus ne
didesns nei 4k+3 ir 2k+2 bus didesns. Vadinasi, bus lygus 2k+2 ir
_
2
p
_
= 1.
31
1.6. Kvadratins liekanos Skaii teorija
Kvadratinio apveriamumo teorema taip vadinasi ne be reikalo. Jos dka uuot
skaiiavus
_
p
q
_
, galima skaiiuoti
_
q
p
_
. Geriau siirjus teoremos formuluot pasidaro
aiku, kad i dviej Leandro simboli reikms sutaps, jei bent vienas i p, q duos
dalybos liekan 1 moduliu 4, ir bus skirtingos, jei abiej pirmini dalybos liekanos
moduliu 4 bus lygios 3. Pairkime, kaip tai atrodo praktikai.
Pavyzdiai
1 Pavyzdys. Raskite, ar 23 yra kvadratin liekana moduliu 37.
Sprendimas. Abu duoti skaiiai yra nelyginiai pirminiai, todl galime taikyti kvadratinio
apveriamumo teorem. Kadangi 37 1 (mod 4), tai gausime
_
23
37
__
37
23
_
= 1 =
_
23
37
_
=
_
37
23
_
.
Pagrindin nauda, kuri gauname i apvertimo, yra ta, kad dabar skaitiklis yra didesnis
u vardikl, tad galime j redukuoti moduliu. Kadangi 37 14 (mod 23), tai gausime
_
37
23
_
=
_
14
23
_
.
Redukav, pritaikome lygyb
_
ab
p
_
=
_
a
p
_ _
b
p
_
:
_
14
23
_
=
_
2
23
__
7
23
_
.
Pirmj i dauginamj galime i karto suskaiiuoti. Kadangi 23 1 (mod 8), tai
_
2
23
_
= 1. Antrj vl apversime ( kart gausime, kad apverstasis yra prieingo enklo,
nes 7 23 3 (mod 4)):
_
7
23
_
=
_
23
7
_
=
_
2
7
_
.
Kadangi 7 1 (mod 8), tai gauname, kad
_
2
7
_
= 1, vadinasi, visk sujung gausime
_
23
37
_
= 1, t.y. 23 nra kvadratin liekana moduliu 37.
2 Pavyzdys. Raskite, ar 41 yra kvadratin liekana moduliu 61.
Sprendimas. Darysime t pat, k ir praeitame pavyzdyje:
_
41
61
_
=
_
61
41
_
=
_
20
41
_
=
_
2
41
_
2
_
5
41
_
=
_
41
5
_
=
_
1
5
_
= 1.

3 Pavyzdys. Raskite, moduliu kuri pirmini, 3 yra kvadratin liekana.


32
1.6. Kvadratins liekanos Skaii teorija
Sprendimas. Iekosime, kada
_
3
p
_
yra lygus vienetui. Taikysime kvadratinio apveria-
mumo teorem. Kadangi 3 3 (mod 4), tai
_
3
p
_
= (1)
p1
4
_
p
3
_
.
Sandauga bus lygi 1, kai abu daugikliai lygus 1 arba 1. Kadangi
_
p
3
_
lygus 1, kai
p 1 (mod 3) ir 1, kai p 2 (mod 3), tai gausime, kad mums tinka pirminiai
skaiiai, tenkinantys sistemas:
_
p 1 (mod 4)
p 1 (mod 3)
ir
_
p 1 (mod 4)
p 1 (mod 3)
Tokiais bus pirminiai p 1 (mod 12).
4 Pavyzdys. Kokie pirminiai skaiiai gali bti daugianario x
2
+5 dalikliais? (Skaii
vadiname daugianario q(x) dalikliu, jei su kuria nors x reikme q(x) i jo dalijasi.)
Sprendimas. Jei p yra daugianario x
2
+5, tai su kakokia x reikme bus tesinga lygyb
x
2
+ 5 0 (mod p) x
2
5 (mod p),
t.y. 5 turs bti kvadratin liekana moduliu p. Tar, kad p ,= 5, skaiiuojame:
_
5
p
_
=
_
1
p
__
5
p
_
= (1)
p1
2
_
p
5
_
.
Sandauga bus lygi 1, jei p 1 (mod 4) ir p 1 (mod 5), arba jei p 1 (mod 4)
ir p 2 (mod 5). Isprend lygini sistemas, randame, kad tiks p 1 (mod 20),
p 3 (mod 20), p 7 (mod 20), p 9 (mod 20) bei atskiras atvejis p = 5.
Udaviniai
1. Raskite
_
79
101
_
. S
2. rodykite, kad jei pirminis p > 3 dalo skaii a
2
+12, tai tuomet p 2 (mod 3). S
3. rodykite, kad i vis nelygi nuliui liekan moduliu pirminio p, pus yra kvad- S
ratins ir pus nekvadratins.
4. Nustatykite, moduliu kuri pirmini, 6 yra kvadratin liekana. S
5. [LitMo 1987]Skaiius N lygus pirmj n 2 pirmini skaii sandaugai. rody- S
kite, kad nei vienas i skaii N 1 ir N +1 nra natraliojo skaiiaus kvadratas.
6. rodykite, kad, kaip ir prastai, kvadratin lygtis ax
2
+bx+c (mod p), a , 0, p ,= 2 S
turs sprendini tada ir tik tada, kai diskriminantas b
2
4ac bus kvadratin liekana
(skaitant nul) moduliu p.
7. [Brazil 2003] Raskite maiausi pirmin, kuris dalo daugianar n
2
+ 5n + 23. S
33
1.6. Kvadratins liekanos Skaii teorija
8. rodykite, kad jei pirminis p 3 (mod 4), tai i p[a
2
+b
2
seka p
2
[a
2
+b
2
. S
9. Tegu pirminis p = 4n+1. rodykite, kad visi n dalikliai yra kvadratins liekanos S
moduliu p.
10. rodykite, kad kvadratini liekan sandauga lygsta 1 moduliu p, kai p S
3 (mod 4), ir 1, kai p 1 (mod 4).
11. rodykite, kad 1
2
3
2
5
2
(p 2)
2
(1)
(p+1)/2
(mod p). S
12. Pasinaudoj lygybe x
4
+4 = ((x+1)
2
+1)((x1)
2
+1) parodykite, kad 4 bus S
bikvadratin liekana mod p tada ir tik tada, kai p 1 mod 4. (a yra bikvadratin
liekana, jei egzistuoja sprendinys x
4
a mod p)
13. rodykite, kad pirminiai daugianario x
4
x
2
+ 1 dalikliai lygsta 1 mod 12. S
14. rodykite, kad visi pirminiai yra daugianario x
6
11x
4
+ 36x
2
36 dalikliai. S
15. Tegu pirminis p 3 mod 4, ir q = 2p + 1 taip pat pirminis. rodykite, kad S
q[2
p
1.
16. inoma, kad jei pirminis p 1 mod 4, tai jis uraomas kaip dviej kvadrat S
suma p = a
2
+b
2
. Tegu a nelyginis dmuo. rodykite:
a)
_
a
p
_
= 1;
b)
_
a+b
p
_
= (1)
(a+b)
2
1
8
;
c) (a +b)
2
2ab (mod p);
d) (a +b)
p1
2
(2ab)
p1
4
(mod p).
Tegu f toks, kad b af (mod p). rodykite, kad f
2
1 mod p ir kad 2
p1
4

f
ab/2
(mod p).
rodykite, kad 2 yra bikvadratin liekana moduliu p tada ir tik tada, kai p ura-
omas kaip A
2
+ 64B
2
.
17. [JBMO 2007] rodykite, kad jei p yra pirminis, tai A = 7p + 3
p
4 nra pilnas S
kvadratas.
18. [Kazakhstan 2004] Raskite visus pirminius p, su kuriais lygtis x
2
+y
2
= 2003+pz S
turi sveikj sprendini.
19. [Vietnam 2004] Tegu S(n) skaiiaus n skaitmen suma. Raskite maiausi ga- S
lim S(m) reikm, jei m dalijasi i 2003.
34
1.7. Diofantins lygtys Skaii teorija
1.7 Diofantins lygtys
Lygtys yra vadinamos diofantinmis, kai yra iekoma j sveikj sprendini. iame
skyrelyje apvelgsime kelet metod padedaniu jas sprsti. Atkreipsime dmes, kad,
skirtingai nuo prast lygi, sprsti daniausiai yra bandyti rodyti, kad lygtis spren-
dini neturi, arba jei ir turi, tai labai specinius.
1.7.1 Dvi lygties puss
Pradsime nuo trij pagrindini princip, besiremiani labai bendru pastebjimu:
Lygybs abi puss yra vienodo dydio, vienodai skaidomos dauginamaisiais ir duoda
vienodas liekanas dalijamos i natralij skaii.
Dydis
Pradkime nuo pavyzdi. Isprsime tris paprastas lygtis.
Pavyzdys. Raskite lygties x
2
= x + 2 sveikuosius sprendinius.
Sprendimas. i lygtis yra kvadratin, ir j galima isprsti prastai, taiau minutlei
t pamirkime ir pabandykime pasinaudoti tuo, kad kairioji pus beveik visada yra
didesn u deinij. vertinkime - kai x 3, tai x
2
3x x + 6 > x + 2, o
kai x 2, tai x
2
> 0 x + 2, tad vienintliai sveikieji skaiiai, kuri negaljome
atmesti samprotaudami apie skirtingus lygties pusi dydius, yra 1, 0, 1 ir 2. Lieka
tik patikrinti, kurie i j tinka, ir rasti, kad lygties sprendiniai yra 1 ir 2.
Pavyzdys. Raskite lygties x
2
+y
2
= 100 sveikuosius sprendinius.
Sprendimas. Sveikj skaii kvadratai yra visuomet neneigiami ir auga palyginti spar-
iai. ios lygties atveju, kaip tik tuo ir pasinaudosime - jei x arba y yra moduliu didesni
u 10, tai kairioji pus tampa didesn u 100. Atkreip dmes tai, kad jei (x, y) yra
sprendinys tai ir (x, y) yra sprendinys gauname, kad utenka patikrinti x reikmes
nuo 0 iki 10. Tai padaryti nesunku - randame, kad sprendiniai bus (0, 10), (6, 8), (8, 6),
(10, 0) su visomis skirtingomis enkl kombinacijomis.
Pavyzdys. Raskite lygties xy = x +y sveikuosius sprendinius.
Sprendimas. Dviej sveikj skaii sandauga beveik visada yra didesn u sum.
Pasinaudosime tuo, taiau pirmiausia atmeskime neigiamus atvejus. Aiku, kad abu ir
x, ir y, negali bti neigiami, nes tuomet sandauga bus teigiama, o suma neigiama. Negali
bti ir vienas neigiamas, vienas teigiamas, pvz. x > 0, y < 0, nes tuomet xy y < y+x.
Tad iekokime sprendini, kuriuose x 0 ir y 0 ir taip pat neprarasdami bendrumo
tarkime, kad y yra nemaesnis nei x. Parodysime, kad x negali bti didesnis u 2. I
ties, jei x 3, tai xy 3y > y +x. Vadinasi x gali gyti tik reikmes 0, 1 ir 2. Patikrin
randame sprendinius (0, 0) ir (2, 2).
35
1.7. Diofantins lygtys Skaii teorija
Bandant vertinti reikini dydius, natraliai praveria algebrins nelygybs ir su-
pratimas apie funkcij didjim, argumentui artjant begalyb (pavyzdiui, didesnio
laipsnio daugianaris nuo kakurios reikms visuomet gis didesnes reikmes u maes-
nio laipsnio daugianar). Puiki ir paprasta i idj iliustracija - 1988 met Lietuvos
matematikos olimpiados udavinys:
Pavyzdys. [LitMo 1988] Isprskite natraliaisiais skaiiais lygt 3x
2
+2y
2
= 4xy+2x.
Sprendimas. Parodysime, kad kairioji pus beveik visada yra didesn u deinij. I
ties - pagal aritmetinio-geometrinio vidurkio nelygyb 2x
2
+2y
2
4xy, ir x
2
> 2x, kai
x > 2. Vadinasi, lieka patikrinti tik dvi reikmes - x = 1 ir x = 2. Tinka tik antroji,
randame sprendin (2, 2).
Daniausiai, kaip ir turi bti olimpiadiniuose udaviniuose, lygybs pusi dydi
skirtumo idja bna umaskuota ir reikia akylumo norint j irti. Pavyzdiui:
Pavyzdys. [LitKo 2009] Raskite lygties (a
2
9b
2
)
2
33b = 16 sveikuosius neneigiamus
sprendinius.
Sprendimas. is udavinys organizatoriams greiiausiai pasirod kiek sunkokas, todl
olimpiadoje buvo suformuluotas kaip dviej dali, pirmoji i kuri pra rodyti, kad
visi sprendiniai tenkina nelygyb [a3b[ 1. rodyti tai labai paprasta, taiau irti
uuomin gerokai sunkiau. Paprasiausia tai padaryti turbt bt iskaidant pirmj
dmen dauginamaisiais: (a
2
9b
2
)
2
= (a 3b)
2
(a + 3b)
2
, tuomet
(a 3b)
2
(a + 3b)
2
(a + 3b)
2
9b
2
.
Vadinasi, kairioji lygties pus yra ne maesn nei 9b
2
33b = (9b 33)b, bet io
reikinio reikm yra didesn u 16 su visomis b reikmmis virijaniomis 4, vadinasi
lieka patikrinti vos kelet reikmi.
Taiau atidkime sprendim al ir dar kart pavelkime lygt, bandydami
kiek kitaip vertinti kairiosios puss dyd. Prieastis, dl kurios (a
2
9b
2
)
2
yra beveik
visada daug didesnis u 33b yra ta, kad skirtumas tarp kvadrat yra pakankamai didelis.
Ities, jei a
2
nra lygus 9b
2
, tai ariausiai (tuomet skirtumas maiausias) jis gali bti tik
tuomet, kai yra artimiausiai esantis kvadratas. O artimiausias kvadratas yra (3b 1)
2
,
bet net tuomet skirtumas visvien yra 6b 1, o (6b 1)
2
33b yra didesnis u 16
su visomis b reikmmis didesnmis u 1! Lieka vos du atvejai, i kuri gauname po
sprendin: (4, 0) ir (4, 1).
Viena (labai svarbi!) i samprotavimo apie dyd variacij -terpimo tarp kvadrat
triukas. Norint parodyti, kad sveikasis skaiius nra kvadratas, utenka parodyti, kad
jis yra tarp dviej gretim kvadrat ir n vienam i j nelygus. i strategija tinka,
inoma, ir auktesniems laipsniams.
Pavyzdys. Raskite lygties y
2
= x
2
+x + 1 sveikuosius sprendinius.
Sprendimas. Kairioji lygties pus yra kvadratas, o deinioji beveik visada nra, nes
x
2
< x
2
+ x + 1 < (x + 1)
2
(arba (x + 1)
2
< x
2
+ x + 1 < x
2
, jei x neigiamas).
Vienintels x reikms, su kuriomis ios nelygybs nra teisingos yra x = 1 ir x = 0,
gauname sprendinius (1, 1) ir (0, 1).
36
1.7. Diofantins lygtys Skaii teorija
Liekanos
Nagrinjant lygt moduliu pasirinkto skaiiaus, apribojim, kad abi lygybs puss turi
duoti vienod liekan moduliu to skaiiaus, danai galima perkelti apribojim ieko-
miems sprendiniams. Kartais tas apribojimas bna pakankamas, kad galtume visikai
isprsti lygt, bet daniau jis tampa pagalbine informacija, kuri tampa naudinga sujun-
gus j su kitomis idjomis. Pradkime nuo paprasiausi atvej, kai nagrinjant lygt
moduliu tinkamai parinkto skaiiaus ji isisprendiama iki galo.
Pavyzdys. Raskite lygties x
2
= 3y 1 sveikuosius sprendinius.
Sprendimas. Nagrinkime i lygt moduliu 3. Norint, kad (x, y) bt sprendinys,
abiej lygybs pusi dalybos liekana i 3 turi bti vienoda. Deins puss dalybos
liekana bus 1, o kairs, priklausomai nuo x, arba 0, arba 1. Gavome, kad su jokiais
(x, y) jos nesutaps, todl lygtis sprendini neturi.
Pavyzdys. Raskite lygties x
2
= 2
n
1 sveikuosius sprendinius.
Sprendimas. Nagrinkime lygt moduliu 4. Dein pus, kad n > 1, lygsta 1 moduliu
4, o kair 0 arba 1. Kadangi liekanos nesutampa, tai lieka tik atvejai n 1, kuriuos
patikrin (n negali bti neigiamas, nes tuomet 2
n
nebt sveikasis) randame sprendinius
n = 1, x = 1 ir n = 0, x = 0.
Pavyzdys. Raskite lygties 2 +x
2
+x
3
= 6
n
sveikuosius sprendinius
Sprendimas. Nagrinkime lygt moduliu 3 arba moduliu 5, arba moduliu 7. Visais
trimis atvejais lengva sitikinti, kad abi puss duoda skirtingas liekanas.
Kaip jau usiminme, lygties nagrinjimas moduliu (arba sprendimas moduliu) da-
niausiai yra tik dalis sprendimo. Pavyzdiui:
Pavyzdys. [Lietuvos TST 2009] Raskite lygties x
3
+x
2
= 16+2
y
natraliuosius spren-
dinius.
Sprendimas. Nagrinkime lygt moduliu 7. Kairioji pus gali gyti liekanas 0, 1, 2, 3, 5,
o deinioji 3, 4 ir 6. Vienintl bendra liekana yra 3, ir ji gyjama kai y dalijasi i 3.
Panaudokime gaut informacij - paymkime y = 3a ir perraykime lygt kaip
(2
a
)
3
= x
3
+x
2
16.
Lieka pastebti, kad galime pritaikyti terpimo tarp kub dj: su visais x > 4 turime
x
3
< x
3
+x
2
16 < (x + 1)
3
,
vadinasi, lieka patikrinti tik keturias x reikmes. Randame vienintel sprendin (4, 6).

Pavyzdys. [MEMO 2009, Aivaras Novikas] Raskite lygties 2


x
+2009 = 3
y
5
z
neneigia-
mus sveikuosius sprendinius.
37
1.7. Diofantins lygtys Skaii teorija
Sprendimas. Pirmiausia sitikinkime, kad x negali bti maesnis u 3. Ities - staius
reikmes 0, 1, 2 kairiojoje pusje gauname 2010, 2011, 2013 ir n vienas i i skaii
neisiskaido tik trejeto ir penketo laipsnius. Tad tarkime, kad x 3. rodysime, kad
visi trys x, y, z turi bti lyginiai.
x - Jei y > 0, tai nagrinkime lygt moduliu 3 gausime (1)
x
1 0, vadinasi x
lyginis. Jei y = 0, tai z > 0, tuomet nagrinkime lygt moduliu 5. Gausime
2
x
1 0, vadinasi x dalijasi i 4, t.y. yra lyginis.
y - nagrinkime lygt moduliu 4. Kadangi x > 2, tai gausime 1 (1)
y
, vadinasi y
lyginis.
z - nagrinkime lygt moduliu 8. Kadangi x > 2 ir y lyginis, tai gausime 1 5
z
,
vadinasi z lyginis.
Paymj x = 2a, y = 2b, z = 2c galime lygt pertvarkyti
2009 = (3
b
5
c
2
a
)(3
b
5
c
+ 2
a
).
Kadangi 2009 isiskaido kaip 7
2
41, tai dviej dauginamj sandaug galime j iskai-
dyti tik trim bdais: 12009, 7287 ir 4149. Vienintelis iskaidymas, kurio dauginamieji
skiriasi per dvejeto laipsn yra 41 49, i kur randame vienintl sprendin (4, 4, 2).
Lygties sprendim moduliu visuomet verta prisiminti sprendiant diofantines lyg-
tis ir ypa tas, kuriose i pirmo vilgsnio nesimato joki silpn viet. Neretai verta
sprsti lygt moduliu nedideli skaii (pvz. 2, 3, 4, 5, 7, 8, 9) ir akylai stebti gaunam
informacij. Taip pat visuomet verta gerai siirti lygt, kartais koecientai ar dideli
laipsniai gali pasueruoti skaii, moduliu kurio pavyks ipeti k nors vertingo.
Pastebsime, kad sprendimas moduliu danai bna skmingas, jei viena arba abi
lygties puss gyja nedaug liekan moduliu nagrinjamo skaiiaus. Kiek liekan gyja
reikiniai pavidalo x
k
(kur x kintamasis) kartais padeda vertinti liekan grupi teorija.
Prisiminkime, kad liekan grups moduliu pirminio p eil yra p1, o moduliu sudtinio
n yra (n). Jei p 1 (ar (n)) ir k didiausias bendras daliklis yra didelis, tai tuomet
x
k
gis nedaug reikmi moduliu p (ar n). Konkreiau:
p = 3 - liekan grups eil 2 - x
2
(ir kiti lyginiai laipsniai) gys 2 liekanas i 3
n = 4 - liekan grups eil 2 - x
2
gis 2 liekanas i 4
p = 5 - liekan grups eil 4 - x
4
gis 2 liekanas i 5
p = 7 - liekan grups eil 6 - x
6
gis 2, x
3
gis 3 liekanas i 7
n = 8 - liekan grups eil 4 - x
4
gis 2, x
2
gis 3 liekanas i 8
n = 9 - liekan grups eil 6 - x
6
gis 2, x
3
gis 3 liekanas i 9
p = 11 - liekan grups eil 10 - x
10
gis 2, x
5
gis 3 liekanas i 11
irint i io tako, 1998 met Balkan Matematikos Olimpiados udavinys atrodo
labai paprastas:
38
1.7. Diofantins lygtys Skaii teorija
Pavyzdys. [BMO 1998] Parodykite, kad lygtis x
2
+ 4 = y
5
neturi sveikj sprendini.
Sprendimas. Atkreipkime dmes y
5
. is reikinys gis nedaug reikmi moduliu 11, o
tiksliau, kadangi y
10
0, 1 (mod 11), tai y
5
0, 1, 1 (mod 11). Tad sprskime lygt
moduliu 11 - x
2
gys reikmes 0, 1, 4, 9, 5, 3, todl kairioji pus gis reikmes 4, 5, 8, 2, 9, 7.
Nei viena i j nra lygi 0, 1 ar 1, vaidinasi lygtis sprendini neturi.
Skaidymasis
Vl pradiai pateiksime por paprast pavyzdi.
Pavyzdys. Raskite visus sveikuosius lygties xy = x +y sprendinius.
Sprendimas. Vienas i btin gdi norint skmingai taikyti skaidymosi idjas yra
skaidymas dauginamaisiais. Pavelkime du skirtingus ios jau matytos lygties per-
tvarkymus: (x 1)(y 1) = 1 ir x(y 1) = y. Pirmuoju atveju lygtis i karto isprsta
- jei dviej sveikj skaii sandauga lygi 1, tai jie arba abu lygs 1, arba 1. Antrasis
iskaidymas yra i pirmo vilgsnio prastesnis, bet domesnis: kadangi y 1 ir y yra
tarpusavyje pirminiai, tai bet koks y 1 daliklis dalins kair lygybs pus, bet nedalins
deins. Vadinasi y 1 negali turti joki dalikli, todl yra lygus 1 arba 1. Gauname
sprendinius (0, 0) ir (2, 2).
Pavyzdys. Raskite visus sveikuosius lygties x
2
= 2
n
+ 1 sprendinius.
Sprendimas. Pastebkime, kad jei (x, n) yra sprendinys, tai ir (x, n) bus sprendinys,
tad iekokime tik teigiam x.
Iskaidykime dauginamaisiais: (x1)(x+1) = 2
n
. Deinioji pus yra dvejeto laips-
nis, todl kairiosios puss abu dauginamieji taip pat turi bti dvejeto laipsniai. Taiau
vienintliai dvejeto laipsniai, tarp kuri skirtumas yra du (o btent toks skirtumas yra
tarp daugikli), yra 2 ir 4, vadinasi x = 3, n = 3.
Alternatyviai galima samprotauti taip: kadangi didiausias x 1 ir x + 1 bendras
daliklis yra nedidesnis u 2, tai vienas i dauginamj dalinsis daugiausia tik i 2
1
,
vadinasi, bus lygus 2 arba 1, vadinasi x lygus 0, 1, 2 arba 3. I j tinka tik x = 3.
Kaip jau buvo matyti praeitos dalies pavyzdyje i MEMO 2009 olimpiados, ne vi-
suomet i karto pavyksta iskaidyti lygt dauginamaisiais - kartais pirmiausia reikia
gauti papildomos informacijos apie iekomus sprendinius. Taip pat ne visuomet aiku,
k daryti iskaidius. Bendros strategijos greiiausiai nra, bet visuomet verta atkreipti
dmes dauginamj bendrus daliklius. Danai pastebjus, kad jie j neturi (arba jie
labai riboti) galima pasistmti priek.
Pavyzdys. [IMO 2006] Raskite lygties 1 + 2
x
+ 2
2x+1
= y
2
sveikuosius sprendinius.
Pirmiausia pastebkime, kad x negali bti maesnis u 1, nes tuomet kair pus
nebus sveikasis skaiius. Patikrin x reikmes nuo 1 iki 2 randame vienintel sprendin
(0, 2), tad tarkime, kad x 3 ir y > 0 (i sprendinio (x, y) gausime ir sprendin
(x, y)).
39
1.7. Diofantins lygtys Skaii teorija
Iskaidykime dauginamaisiais:
2
x
(2
x+1
+ 1) = (y 1)(y + 1).
Kadangi dbd(y 1, y + 1) 2, o sandauga (y 1)(y + 1) dalijasi i 2
x
, tai bent
vienas i dauginamj dalinisis i 2
x1
. Atkreipkite dmes, kad y 1 negali bti daug
kart didesnis u 2
x1
, nes tuomet deinje pus bus didesn u kairij. Lieka visk
tvarkingai pabaigti. Nagrinkime du atvejus:
2
x1
[y 1 - paymj y 1 = a2
x1
ir stat lygt gausime 2
x
+ 2
2x+1
= a2
x1
(a2
x1
+ 2)
arba 1 + 8 2
x2
= a
2
2
x2
+a. Aiku, kad a < 3, bet a = 1 ir a = 2 netinka.
2
x1
[y + 1 - paymj y +1 = a2
x1
ir stat lygt gausime 1 +8 2
x2
= a
2
2
x2
a. Aiku,
kad a < 4, patikrin maesnes reikmes randame, kad tinka a = 3, tuomet x = 4
ir y = 23.
Vadinasi, visi lygties sprendiniai yra (0, 2) ir (4, 23).
Pavyzdys. [BMO 2009] Raskite lygties 3
x
5
y
= z
2
sveikuosius teigiamus sprendinius.
Sprendimas. Sprskime lygt moduliu 4. Kair pus lygsta (1)
x
1, o dein 0 arba
1. Norint, kad jos bt lygios x turi bti lyginis. Paymj x = 2a gausime
5
y
= (z 3
a
)(z + 3
a
).
Kadangi dbd(z 3
a
, z + 3
a
)[2 3
a
, tai vienas i dauginamj nesidalins i 5, vadinasi,
bus lygus 1. Taiau z +3
a
> 3, todl lieka vienintelis variantas z 3
a
= 1 - gauname
lygt
5
y
= 2 3
a
1.
Pastebkime, kad a = 1, y = 1 yra sprendinys. Jei a > 1, tai sprsdami moduliu 9
gausime 5
y
1, todl y dalijasi i 3. Taiau tuomet 5
y
+ 1 dalinsis i 7, o 2 3
a
nesidalins. Radome, kad lygtis turi vienintel sprendin (2, 1, 2).
Retais atvejais pavyksta panaudoti elegantikas idjas apie kai kuri reikini pir-
minius daliklius. Pavyzdiui, i kvadratini liekan skyrelio inome, kad x
2
+a negali
turti pirminio daliklio, su kuriuo
_
a
p
_
= 1, taip pat kaip ir dviej kvadrat suma
negali dalintis i pirminio skaiiaus, lygstanio 3 moduliu 4, nelyginio laipsnio.
Pavyzdys. [IMO Longlist 1984] rodykite, kad lygtis 4mnmn = x
2
neturi sveikj
sprendini.
Sprendimas. Iskaidykime dauginamaisiais:
(4m1)(4n 1) = 4x
2
+ 1.
Kairje pusje esantys dauginamieji lygsta 3 moduliu 4, vadinasi dalijasi bent i vieno
pirminio p, kuris irgi lygsta 3 moduliu 4. Taiau dein pus tokio pirminio daliklio
turti negali, nes tuomet (2x)
2
1 (mod p), ko negali bti, nes 1 yra kvadratin
liekana tik moduliu pirmini, kurie lygsta 1 moduliu 4.
40
1.7. Diofantins lygtys Skaii teorija
Udaviniai
1. Raskite lygties x
2
= 200 + 9y sveikuosius sprendinius. S
2. Raskite lygties x
2
= 100 +y
2
sveikuosius sprendinius. S
3. Raskite lygties x
2
+y
2
= 4z + 3 sveikuosius sprendinius. S
4. Raskite lygties x
2
+ 2x = 4y + 2 sveikuosius sprendinius. S
5. Raskite lygties x
2
+y
2
= 2x + 3y + 4 sveikuosius sprendinius. S
6. [LitMo 1987] Nurodykite natralij skaii, didesni u 100, trejet (x, y, z), S
tenkinant lygyb x
2
+yz
2
xy xz
2
= 1987.
7. Raskite lygties 2
x
= 3
y
+ 1 sveikuosius sprendinius. S
8. Raskite lygties 2
x
= 3
y
1 sveikuosius sprendinius. S
9. [LitMo 1988] Isprskite natraliaisiais skaiiais lygt x
2
+ (x +y)
2
= (x + 9)
2
. S
10. [LitMo 1989] Isprskite lygt x
2y
= 2
z
1 natraliaisiais skaiiais. S
11. [LitMo 1989] Isprskite sveikaisiais skaiiais lygt 2x
2
y
2
+y
2
6x
2
12 = 0. S
12. [LitMo 1989] Isprskite natraliaisiais skaiiais lygt 13x
2
+ 17y
2
= 1989
2
. S
13. [IMO Longlist 1972] Raskite visus sveikuosius lygties 1 +x +x
2
+x
3
+x
4
= y
4
S
sprendinius.
14. [IMO Longlist 1977] Raskite visus sveikuosius lygties 7a +14b = 5a
2
+5ab +5b
2
S
sprendinius.
15. [LitMo 1986] Isprskite lygt x
y
= y
xy
natraliaisiais skaiiais. S
16. [LitMo 1987] Isprskite lygt 6!x! = y! natraliaisiais skaiiais. S
17. [LitKo 2007] Raskite visus sveikj skaii x, y, z ir t ketvertus (x, y, z, t) tenki- S
nanius lygt x
2
+y
2
+z
2
+t
2
= 3(x +y +z +t).
18. Raskite visus natraliuosius lygties x
3
y
3
= xy + 61 sprendinius. S
19. [JBMO 2009] Raskite lygties 2
a
3
b
+ 9 = c
2
natraliuosius sprendinius. S
20. Raskite lygties 3
x
2
y
= 1 natraliuosius sprendinius. S
21. Raskite lygties x
2
+ 3 = 12y
3
16y + 1 sveikuosius sprendinius. S
41
2 SKYRIUS
ALGEBRA
2.1 Nelygybs
iame skyrelyje daugiausia to, k veiksime su nelygybmis, sudarys bandymai jas ro-
dyti. Nelygybi rodinjimas bus pagrindin veikla, o j rodymas bus aukiausia
siekiamyb ir didiausia vertyb. Skaitytojui, susipainusiam tik su mokykliniu nelygy-
bi kursu, tai gali atrodyti ne tik naujai, bet keistai ar baisiai. Nuo ko pradti, norint
rodyti? Nelygybi rodinjimo losoja remiasi vos keliais paprastais principais.
Bandydami rodyti, naudosims nelygybmis-teoremomis, su kuriomis susipainsime
iame skyriuje ir inosime, kad jos tikrai tikrai galioja. ios teoremos - tarsi laiptai,
kuriais lipame i kairs nelygybs puss dein. Jei turime rodyti A B, o pagal
teorem T, turime A B, tai mes rodme nelygyb vienu uoliu, kas nebuvo labai
domu. Tik nuo sprendjo priklauso, kiek ir koki uoli reiks atlikti norint pasiekti
rezultat. Kadangi daniausiai tenka lipti daugiau nei vienu laipteliu, reikt suinoti,
kaip tai daroma.
Tarkime, norime rodyti A C. Tegu, remiantis teorema X, tikrai tikrai galioja
A B. Jei pasisteng gausime, kad, anot teoremos Y , B C, tai tada A C, k
ir reikjo rodyti. Bet jeigu netyia pagal teorem Z tikrai galioja B C, tai reik
ne tai, kad rodoma nelygyb yra neteisinga, bet kad teoremos X laiptelis buvo per
status. Pagalvokite: jei i tako A stipriai nusileidiate tak B, bet pamatote,
kad C - aukiau u B, niekaip negalsite pasakyti kuris i A ir C yra aukiau, nes
nelygybs gali nurodyti tik, ar kakas yra daugiau/maiau u kak, bet ne kiek stipriai.
Kitaip tariant, inome tik tiek, kad jei mes tik leidoms, tai esame emiau, o jeigu tik
kilome - tai aukiau, na o jeigu kaip liftu vainjomes tai auktyn tai emyn, tai jau
niekas nebesupaisys, kokiame auktyje esame. Tai yra pagrindinis nelygybi rodinjimo
principas, taiau yra kelios plaiai naudojamos jo formos.
Nelygyb visada galime ekvivaleniai pertvarkyti (ekvivaleniai reikia, kad jei at-
likome tam tikrus pertvarkymus ir i nelygybs X gavome nelygyb Y , tai atlikdami
logikus atvirkius pertvarkymus, i Y galime vl gauti X) ir tada naudoti/rodinti
pertvarkytj. Tie pertvarkymai gali bti labai vairs: prie abiej nelygybs pusi
2.1. Nelygybs Algebra
galime pridti po konstant, padauginti i jos, pakelti laipsniu, logaritmuoti ir antiloga-
ritmuoti. Visada reikia bti atsargiems: kai kada ne visi ie veiksmai yra galimi. Taip
pat prisiminkite, kad nelygyb dauginant i neigiamos konstantos ar keliant neigiamu
laipsniu, nelygybs enklas apsiveria, t.y.: i virsta , o i > < ir atvirkiai.
Dvi teisingas nelygybes galime visada sudti, o jei jos abi teigiamos, ir sudauginti.
Taigi, jei turime A C ir B D, tai rodme A + B C + D, o jei A, B, C ir D
teigiami, tai ir A B C D. Pastebkime, kad nei dalinti, nei atimti nelygybi vienos
i kitos negalime. Netikintiems: imkime dvi teisingas nelygybes 8 4 ir 8 3. Nei
atm, nei padalin teisingos nelygybs negausime.
Lygybs atvejis yra viena subtiliausi negriet nelygybi dali. Naudojant teo-
remas privalu stebti, ar vis dar manoma pasiekti lygyb. Jei pasidaro nemanoma,
tai udavinio isprsti greiiausiai nepavyks. Kaip suinoti lygybs atvej? Daniausiai
reikia tiesiog atspti, kas neretai yra gana paprasta. Atsivelgimas lygybs atvej leis
sutaupyti laiko ir aklai nenaudoti iai pasmerkt strategij. Lygybs atvejis naudo-
jamas dar ir ekstremum iekojimui.
Ekstremumas - maiausia arba didiausia funkcijos reikm duotame intervale. Pro-
fesionalai ekstremum iekojimui naudoja ivestines ir Lagrano daugiklius. Skaitytojus
raginame susipainti su ia stabios galios technika, jei to padaryti dar nespjote. iame
skyriuje ekstremum iekojimui naudosime alternatyv bd - nelygybes. Ne paslap-
tis, kad remiantis klasikinmis nelygybmis, ciklini ar simetrini reikini nuo keli
kintamj ekstemum iekojimas yra daug paprastesnis ir, danai, greitesnis. Rei-
kinio minimumo ar maksimumo iekojimas nelygybmis remiasi dviem elementariais
ingsniais:
1. Randame reikinio maksimali ar minimali rib, tai yra, u k jis yra tikrai
ne didesnis ar ne maesnis. Pavyzdiui, jei gautume, kad funkcija F C, kur
C - kokia tai konstanta, tai su jokiais funkcijos parametrais negalime gauti F
reikms, maesns u C. Gali pasirodyti, kad tai reikt, jog C yra vienas
funkcijos ekstremum - minimumas, bet taip nebtinai yra, todl privaloma engti
antrj ingsn.
2. Rad galim reikm, privalu patikrinti, ar ji pasiekiama. Ji bus gyjama lygybs
atveju, taigi, i pritaikyt nelygybi lygybs atvej turime atsekti, kokios turi
bti kintamj reikms.
Jei antrojo ingsnio ipildyti nepavyksta, tai reikia, kad pirmasis ingsnis atliktas
neteisingai. Daniausia klaida - panaudot nelygybi lygybs atvej praradimas, kai ie
neegzistuoja arba netenkina reikinio apibrimo srities. Na, o jeigu pavyko atlikti abu
veiksmus, js skmingai radote funkcijos ekstremum. Tokios uduoties atsakymas for-
muluojamas vardijant ne tik rast reikm, bet ir parametr, su kuriais tai pasiekiama,
reikmes.
Nelygybs yra itin plati ir labai vairi matematikos aka. iame skyriuje supaindin-
sime su pagrindinmis sprendimo technikomis, triukais. Nemanoma mintinai imokti
vis nelygybi, taiau galima imokti suprasti pagrindines tendencijas ir greiiau su-
rasti idj, padsiani atlikti uduot. Idjoms gyvendinti reikalingi rankiai. Jais ir
taps vairios nelygybs-teoremos, metodai, pavyzdi, udavini rezultatai. Tai pads
isprsti didij dal udavini, kurie pasirodys ne tik skyreliuose Udaviniai, bet ir
43
2.1. Nelygybs Algebra
olimpiadose. Nesitikime, kad skaitytojas pajgs pats isprsti visus pateiktus udavi-
nius, juk kai kurie j - tikri algebros briliantai, taiau pastangos nenueis perniek. Bkite
drss!
44
2.1. Nelygybs Algebra
2.1.1 Pirmieji ingsniai
Beveik visos klasikins nelygybs remiasi faktu, kad realaus skaiiaus kvadratas yra ne-
maesnis u nul. Taiau suvesti bet koki nelygyb kvadrat, padaugint i teigiam
skaii, sum daniausiai bna ma maiausiai lyktu. Todl gausyb talenting pa-
saulio matematik per amius sunkiai dirbo, kurdami vis spdingesnius ir galingesnius
rankius, kuriems paklsta net paios sudtingiausios problemos. i ranki veikimo
principai reikalauja dmesio, o j supratimas leis juos naudoti itin efektyviai ir suma-
niai. iame skyrelyje ir pradsime nuo pai pamat: nagrinsime, k galime pasiekti
i tokio nekaltai atrodanio fakto kaip:
Teorema. Jei x R, tai x
2
0. Lygyb galios tada ir tik tada, kai x = 0.
Kai kurios teoremos bus rodytos, bet tik ne i. inoma, tai labai svarbi nelygyb
ir sunku sivaizduoti nelygyb, kuri ja nesiremt, bet rodymas yra toks paprastas, kad
ymiai daugiau prasms yra vaistyti popieri ir laik nekant apie jos akivaizdum
negu i tikrj j rodyti. rodymas remiasi tokiais gerai inomais teiginiais kaip Mano
draugo draugas yra mano draugas ir Mano prieo prieas yra mano draugas. Pravartu
inoti, kad Mano daugo prieas yra mano prieas ir Mano prieo draugas yra mano
prieas, nors paskutinieji du nelygybs rodyti ir nepadeda.
Kadangi pavyzdiai kalba geriau u bet koki nelygybi sprendimo ir rodymo teo-
rij, tai ir judkime prie j.
Pavyzdiai
1 Pavyzdys. Jei 0 a, b R, tai:
a +b 2

ab.
Lygyb galios tada ir tik tada, kai a = b.
rodymas. Pertvarkykime nelygyb (matematik kalba nekant, nelygyb yra ekviva-
lenti) (

b)
2
0. I akivaizdios ankstesns teoremos seka, kad gauta nelygyb yra
teisinga. tai kaip vienu uoliu isprendme pirmaj nelygybi skyriaus udavin.
Dauguma io skyrelio udavini, kaip ir pirmasis, bus paremti reikini pertvarky-
mais kvadrat sum. Be to, nepamirime naudotis gautais udavini ir pavyzdi
rezultatais, kurie ymiai supaprastins sprendimus.
2 Pavyzdys. Raskite S = 2a
2
+ 9c
2
+ 5b
2
+ 2ab 8bc 8ac 2a + 4c + 2 minimum,
kai a, b, c R.
Sprendimas. Pertvarkome reikin: S = (a +b 2c)
2
+ (2c a + 1)
2
+ (2b c)
2
+ 1
1. Spjamas minimumas yra 1, belieka patikrinti, ar jis pasiekiamas. Tai atliekame
sprsdami lygi sistem:
_

_
a +b 2c = 0;
2c a + 1 = 0;
2b c = 0.

_
a = 3;
b = 1;
c = 2.
Vadinasi, minimali S reikm lygi 1, ir ji gaunama, kai a = 3, b = 1, c = 2.
45
2.1. Nelygybs Algebra
Pastaba. Galima ir kitaip sugrupuoti duoto rekinio narius, taiau tuomet gauta lygi
sistema neturs vis reikiam sprendini, arba ie bus netinkami.
3 Pavyzdys. rodykite, kad su teigiamais realiaisiais skaiiais x ir y galioja nelygyb
1
x
+
1
y

4
x +y
.
rodymas. Padauginame nelygyb i xy(x +y). Gausime nelygyb xy +y
2
+x
2
+xy
4xy, kuri yra ekvivalenti (x y)
2
0, kas yra akivaizdu.
Pastaba. Beveik visos miniatirins dviej kintamj nelygybs gali bti lengvai nu-
lautos naudojant brutalios jgos taktik, k mes ir padarme paskutiniame nag-
rintame pavyzdyje. inoma, tokia taktika gali nulauti ir daug masyvesnes keli
kintamj nelygybes, taiau taip sprsti nra taip malonu ir greita, kaip iekant teisin-
go kokios nors teoremos pritaikymo bdo. Tai pamatysime kitame pavyzdyje:
4 Pavyzdys. Tegu a, b, c bus teigiami realieji skaiiai. rodykite, kad
ab
c
+
ac
b
+
bc
a
a +b +c.
rodymas. Pagal nelygyb a +b 2

ab, gauname
ab
c
+
ac
b
2a.
Taip pat
ab
c
+
bc
a
2b,
ir
ac
b
+
bc
a
2c.
Sudj ias nelygybes gausime norim rezultat.
Danai tenka rodinti griozdikas nelygybes, kur daugyb kart tenka perrainti
ilgus ir vienus kitus panaius reikinius. Matematikai, bdami nepataisomi tinginiai
yra sugalvoj kelet ymjim, kurie sumaina sugadinamo popieriaus kiek ir padeda
sistemingai pateikti reikiam informacij. Susipainkime su ciklinmis ir simetrinmis
sumomis bei sandaugomis:
Apibrimas. Tegu A
0
= a
1
, a
2
, ..., a
n
. Tuomet

cyc
f(A
0
) = f(a
1
, a
2
, ..., a
n
) +f(a
2
, a
3
, ..., a
n
, a
1
)+
+f(a
3
, a
4
, ..., a
n
, a
1
, a
2
) +... +f(a
n
, a
1
, ..., a
n1
).
Taigi, ciklin suma - tai suma, kur sumuojamos funkcijos argumentai yra perstu-
miami per vien pozicij n kart. Pavyzdiui:

cyc
a
2
+a
b
=
a
2
+a
b
+
b
2
+b
c
+
c
2
+c
a
;

cyc
0 a +b
4
c
3
d
=
b
4
c
3
d
+
c
4
d
3
a
+
d
4
a
3
b
+
a
4
b
3
c
.
46
2.1. Nelygybs Algebra
Apibrimas.

sym
f(A
0
) = f(A
1
) +f(A
2
) +... +f(A
n!
).
ia A
i
- visos aibs A
0
nari perstatos. Kadangi perstat yra n!, tai tiek dmen ir
gausime.
Kitaip sakant, simetrin suma - tai funkcij suma, kur funkcij argumentai yra visos
j perstatos. Pavyzdiui:

sym
a
2
+b
c
3
=
a
2
+b
c
3
+
b
2
+a
c
3
+
c
2
+b
a
3
+
b
2
+c
a
3
+
a
2
+c
b
3
+
c
2
+a
b
3
.
Analogikai apibriamos ir sandaugos

cyc
f(A
0
) bei

sym
f(A
0
).
5 Pavyzdys (L.M.). Tegu a, b, c bus tokie realieji skaiiai, kad abc = 1/2. rodykite,
kad

cyc
1
a
2
+b
2
a +b +c.
rodymas. Pagal nelygyb a
2
+b
2
2ab gauname (a
2
+b
2
)
1
(2ab)
1
. Taigi

cyc
1
a
2
+b
2

cyc
c
2abc
=
a +b +c
2abc
= a +b +c.
6 Pavyzdys. Tegu a, b, c - tokie teigiami skaiiai, kad a
2
+ b
2
+ c
2
= 3. rodykite
nelygyb a
3
(b +c) +b
3
(a +c) +c
3
(a +b) 6.
rodymas. Kadangi a
2
+b
2
+c
2
= 3, duotoji nelygyb ekvivalenti
a
3
(b +c) +b
3
(a +c) +c
3
(a +b)
2
3
(a
2
+b
2
+c
2
)
2
2

cyc
a
4
+ 4

cyc
a
2
b
2
3

cyc
ab(a
2
+b
2
)

cyc
a
4
+b
4
+ 4a
2
b
2
3ab(a
2
+b
2
) 0

cyc
a
4
4a
3
b + 6a
2
b
2
4ab
3
+b
4
+ab(a
2
+b
2
) 2ab ab 0

cyc
(a b)
4
+

cyc
ab(a b)
2
0,
kas yra akivaizdu.
47
2.1. Nelygybs Algebra
Udaviniai
1. rodykite, kad jei x, y yra teigiami realieji skaiiai, tai galioja x
3
+y
3
xy(x+y). S
2. rodykite, kad visiems realiesiems a, b, c galioja nelygyb a
2
+b
2
+c
2
ab+ac+bc. S
3. rodykite, kad visiems realiesiems a, b, c, d galioja nelygyb a
2
+ b
2
+ c
2
+ d
2
S
ab +ac +ad. Kada galios lygyb?
4. rodykite, kad visiems realiesiems teigiamiems a, b, c galioja nelygyb a
3
+b
3
+ S
c
3
3abc.
5. Duoti realieji a, b, x, y, kur x, y > 0. rodykite, kad galioja
a
2
x
+
b
2
y

(a+b)
2
x+y
. S
Kada galios lygyb? Kaip galtume praplsti (apibendrinti) i nelygyb?
6. rodykite, kad visiems teigiamiems realiesiems x, y galioja nelygyb
1
x
+
1
y
S
2
_
2
x
2
+y
2
.
7. Duoti tokie teigiami realieji a, b, c, kad ab +bc +ac = 1. rodykite, kad 10a
2
+ S
10b
2
+c
2
4.
8. Tegu a, b, c - teigiami realieji skaiiai, tokie, kad a
2
+ b
2
+ c
2
= 1. Raskite S
minimum
S =
a
2
b
2
c
2
+
b
2
c
2
a
2
+
c
2
a
2
b
2
.
9. Raskite minimali reikinio = 5a
2
+ 6b
2
+ 5c
2
+ 2ac 4a + 4c reikm, kai S
a, b, c - realieji skaiiai.
10. rodykite, kad jei x ir y yra realieji i intervalo (0, 1), tai galioja nelygyb S
1
1 x
2
+
1
1 y
2

2
1 xy
.
11. Tegu a, b, c - tokie teigiami realieji, kurie tenkina
1
a
+
1
b
+
1
c
a+b+c. rodykite, S
kad a +b +c 3abc.
12. [LitKo 2006 (slyga mauml modikuota)] Tegu S
E = 5(x
2
+y
2
+z
2
) + 6(xy +yz +zx) 4(13x + 15y + 16z) + .
Raskite minimali E reikm, kai x, y, z yra realieji skaiiai, o - js mgsta-
miausias realusis skaiius.
13. [LitMo 1987] rodykite, kad teigiamiems realiesiems a, b, c galioja S
a
3
a
2
+ab +b
2
+
b
3
b
2
+bc +c
2
+
c
3
c
2
+ac +a
2

a +b +c
3
.
14. [USAMO 1998] rodykite, kad teigiamiems realiesiems a, b, c galioja S
1
a
3
+b
3
+abc
+
1
b
3
+c
3
+abc
+
1
c
3
+a
3
+abc

1
abc
.
48
2.1. Nelygybs Algebra
15. [IMO 1996 Shortlist] Tegu a, b, c bus tokie teigiami realieji, kad abc = 1. rody- S
kite, kad
ab
a
5
+b
5
+ab
+
bc
b
5
+c
5
+bc
+
ca
c
5
+a
5
+ca
1.
16. [IMO 2005] Duota, kad a, b, c - realieji, tokie, kad abc 1. rodykite, kad galioja S
nelygyb
a
5
a
2
a
5
+b
2
+c
2
+
b
5
b
2
b
5
+a
2
+c
2
+
c
5
c
2
c
5
+a
2
+b
2
0.
17. [Vascile Cartoaje] rodykite, kad realiesiems a, b, c galioja S
(a
2
+b
2
+c
2
)
2
3(a
3
b +b
3
c +c
3
a).
49
2.1. Nelygybs Algebra
2.1.2 Vidurki nelygybs
Apibrimas. Duoti teigiami realieji x
1
, x
2
, ..., x
n
. Laipsnio r vidurkis yra ymimas
M
r
(x) ir apibriamas
M
r
(x) =
_
x
r
1
+x
r
2
+... +x
r
n
n
_
1/r
.
M
1
(x
1
, x
2
, ..., x
n
) yra ymimas A(x
1
, x
2
, ..., x
n
) ir vadinamas aritmetiniu vidurkiu.
M
2
(x
1
, x
2
, ..., x
n
) yra ymimas S(x
1
, x
2
, ..., x
n
) ir vadinamas kvadratiniu vidurkiu.
M
1
(x
1
, x
2
, ..., x
n
) yra ymimas H(x
1
, x
2
, ..., x
n
) ir vadinamas harmoniniu vidur-
kiu.
Nors i pateiktos iraikos sunku apibrti M
0
, yra inoma, kad kai r 0, tai
M
r
(x) G(x
1
, x
2
, ..., x
n
) = (x
1
x
2
...x
n
)
1/n
, kas yra vadinama geometriniu vidur-
kiu.
Teorema (Bendroji vidurki nelygyb). Jei x = (x
1
, x
2
, ..., x
n
) yra realij teigiam
skaii aib, tai su r s galios nelygyb M
s
(x) M
r
(x). Lygyb bus pasiekiama tada
ir tik tada, kai x
1
= x
2
= ... = x
n
.
Nelygyb yra rodoma su Hlderio nelygybe, su kuria skaitytoj supaindinsime
gerokai vliau.
Daniausiai naudojamos vidurki nelygybs yra atskiri bendrosios teoremos atvejai.
Teorema (AM-GM nelygyb). Jei x
1
, x
2
, ..., x
n
yra teigiami sveikieji, tai galioja
x
1
+x
2
+... +x
n
n

n

x
1
x
2
...x
n
.
rodymas. Yra beveik 40 ios nelygybs rodymo bd. ia pateiksime vieno didiausio
vis laik matematikos korifjaus pranczo Augustin-Louis Cauchy rodym.
Kai n = 1 ir n = 2, nelygyb teisinga. rodysime, kad jei nelygyb teisinga su n, tai
ji teisinga su 2n:
x
1
+x
2
+... +x
2n
2n
=
1
2
_
x
1
+x
2
+... +x
n
n
+
x
n+1
+x
n+2
+... +x
2n
n
_

1
2
_
n

x
1
x
2
...x
n
+
n

x
n+1
x
n+2
...x
2n
_

1
2
2
_
n

x
1
x
2
...x
2n
=
2n

x
1
x
2
...x
2n
Taigi, nelygyb yra teisinga, kai n - dvejeto laipsnis. Jei n nra dvejeto laipsnis, tai
btinai rasime tok m > n, kuris yra dvejeto laipsnis. Tegu tada - t n skaii
aritmetinis vidurkis. Tuomet
=
x
1
+x
2
+... +x
n
n
=
x
1
+x
2
+... +x
n
+ (mn)
m

m
_
x
1
x
2
... x
n

(mn)

n

a
1
a
2
... a
n
.
50
2.1. Nelygybs Algebra
Tai ubaigia rodym. Taigi, nelygyb yra teisinga visiems n, o lygybs atvejis galios
tada ir tik tada, kai x
1
= x
2
= ... = x
n
.
Pastaba. Kitos plaiai naudojamos ios nelygybs formos:
x
1
+x
2
+... +x
n
n
n

x
1
x
2
...x
n
;
x
1
x
2
...x
n

_
x
1
+x
2
+...+x
n
n
_
n
.
Teorema (SM-AM nelygyb). Jei x
1
, x
2
, ..., x
n
yra teigiami realieji, tai galioja

x
2
1
+x
2
2
+... +x
2
n
n

x
1
+x
2
+... +x
n
n
.
Lygyb galios tada ir tik tada, kai x
1
= x
2
= ... = x
n
.
rodymas. Kaip lem naudosime ankstesnio skyrelio penkto udavinio rezultat.
Lema. Jei x
1
, x
2
, ..., x
n
ir y
1
, y
2
, ..., y
n
- teigiami realieji, tai galioja nelygyb
x
2
1
y
1
+
x
2
2
y
2
+... +
x
2
n
y
n

(x
1
+x
2
+... +x
n
)
2
y
1
+y
2
+... +y
n
.
Jei taikysime lem su y
1
= y
2
= ... = y
n
= n, tai ir gausime norim nelygyb.
Lygybs atvejis bus tada ir tik tada, kai x
1
= x
2
= ... = x
n
.
I i dviej teorem seka treioji.
Teorema (SM-GM nelygyb). Jei x
1
, x
2
, ..., x
n
yra teigiami realieji, tai galioja

x
2
1
+x
2
2
+... +x
2
n
n

n

x
1
x
2
...x
n
.
Lygyb galios tada ir tik tada, kai x
1
= x
2
= ... = x
n
.
Danai naujai ivest nelygybi teisingum reikia tikrinti, juk nenorime bandyti
rodyti neteising. Yra inoma Muirheado nelygyb, apibendrinanti AM-GM nelygyb,
kuri yra daniausiai taikoma i vidurki nelygybi. iai naujajai nelygybei vesime
kelet apibrim ir ymjim.
Apibrimas. ymsime
T[a
1
, a
2
, ..., a
n
] =

sym
f(x
1
, x
2
, ..., x
n
),
kai f(x
1
, x
2
, ..., x
n
) = x
a
1
1
x
a
2
2
... x
a
n
n
, o a
1
, a
2
, ..., a
n
ir x
1
, x
2
, ..., x
n
- teigiami realieji
skaiiai.
Apibrimas. Sakysime, kad seka A = a
1
, a
2
, ..., a
n
maoruoja (angl. majorize)
sek B = b
1
, b
2
, ..., b
n
(ymsime A ~ B), jeigu tenkinamos trys slygos:
a
1
+a
2
+a
3
+.. +a
n
= b
1
+b
2
+b
3
+... +b
n
;
a
1
a
2
a
3
... a
n
0 ir b
1
b
2
b
3
... b
n
0;
51
2.1. Nelygybs Algebra
a
1
+a
2
+... +a
i
b
1
+b
2
+... +b
i
, su visais 0 < i < n.
Jau galime formuluoti teorem:
Teorema (Muirhead). Jei A = a
1
, a
2
, ..., a
n
ir B = b
1
, b
2
, ..., b
n
yra teigiam rea-
lij skaii sekos, ir A ~ B, tai galioja nelygyb
T[A] T[B].
Lygyb galios tada, kai sekos A ir B yra identikos.
Pastaba. Nors Muirheado nelygyb turi normal teoremos status, ji nra pripas-
tama kaip dalis ocialaus olimpiados udavinio sprendimo. Ji daniausiai naudojama
nustatyti, ar naujai gaut nelygyb galime rodyti tinkamai pritaik AM-GM nelygyb.
Pats AM-GM nelygybs taikymas yra grynai technin problema, kuri atskirais atvejais
yra lengvai isprendiama.
Iliustruokime naujas inias keliais pavyzdiais.
Pavyzdys. Tarkime, sprendme sprendme kok labai dom udavin ir gavome, kad
lieka rodyti
a
6
b
2
c +a
6
c
2
b +b
6
c
2
a +b
6
a
2
c +c
6
a
2
b +c
6
b
2
a
a
5
b
4
+a
5
c
4
+b
5
c
4
+b
5
a
4
+c
5
a
4
+c
5
b
4
.
Vienintel mintis, kuri auna galv, pamaius toki nelygyb yra: Blogai. Paste-
bkime, kad kair nelygybs pus yra, taip sakant, T[6, 2, 1], o dein - T[4, 5, 0]. ios
dvi laipsni sekos nemaoruoja. Vadinasi, Muirheado nelygybs taikyti negalima. Tai
reik, kad AM-GM nelygyb yra per silpna, o tai byloja, kad problema yra pakanka-
mai sudtinga, jei i nelygyb yra apskritai teisinga. Jei gautume, kad su kuriuo nors
kintamj rinkiniu nelygyb yra neteisinga, teks sugrti prie pradins nelygybs.
Pavyzdys. Jei turtume panai anktesnio pavyzdio, bet vos kitoki nelygyb
a
6
b
2
c +a
6
c
2
b +b
6
c
2
a +b
6
a
2
c +c
6
a
2
b +c
6
b
2
a
a
5
b
3
c +a
5
c
3
b +b
5
c
3
a +b
5
a
3
c +c
5
a
3
b +c
5
b
3
a,
tai matytume, kad kairs puss seka T[6, 2, 1] maoruoja deins T[5, 3, 1] puss sek
ir nelygyb yra teisinga. Pilnam rodymui trksta tik tinkamos AM-GM nelygybs
formos. tai kaip galime j konstruoti: matome, kad deinje maiausias laipsnis yra
1, kaip ir kairje. Taigi, nordami gauti nar, pavyzdiui, b
5
a
3
c, i kairs puss galime
naudoti tik narius a
6
b
2
c ir b
6
a
2
c, nes visi kiti prie c duos laipsn, didesn u 1. Tebnie
prie i dali esantys koecientai atitinkamai k ir l. Pagal AM-GM:
ka
6
b
2
c +lb
6
a
2
c (k +l)
k+l

a
6k+2l
b
6l+2k
c
k+l
.
Tuomet, kadangi turime gauti nar b
5
a
3
c, sprsime lygi sistem:
_
6k + 2l = 3(k +l)
6l + 2k = 5(k +l)
l = 3k.
Ir tikrai, kai l = 1, o k = 3, pagal AM-GM bus:
a
6
b
2
c + 3b
6
a
2
c 4
4

b
20
a
12
c
4
= 4b
5
a
3
c.
52
2.1. Nelygybs Algebra
Pritaik nelygyb simetrinms sumoms ir gausime tai, k reikjo rodyti:
4

sym
a
6
b
2
c =

sym
a
6
b
2
c + 3b
6
a
2
c 4

sym
b
5
a
3
c.
Kai kurie skyrelyje pateikti udaviniai yra tiesiogins Muirheado nelygybs ivados.
Tikims, kad skaitytojui bus drsiau juos sprsti inant, kad nelygybs tikrai galioja.
Vidurki nelygybs yra neatsiejamos nuo homogenikumo svokos, tad pats laikas
su ja susipainti. Kad geriau suvoktume, kaip atpainti homogenik nelygyb, susipa-
insime su funkcijos laipsnio svoka.
Tegu f(a
1
, a
2
, a
3
, ..., a
n
) bus tiesiog funkcija nuo kintamj a
1
, a
2
, a
3
, ..., a
n
.
Jei turima funkcija yra vienanaris, t.y.: f(a
1
, a
2
, a
3
, ..., a
n
) = a

1
1
a

2
2
a

3
3
...a

n
n
, tai
vienanario laipsnis bus deg f(a
1
, a
2
, a
3
, ..., a
n
) =
1
+
2
+
3
+... +
n
.
Pavyzdys. 3-io laipsnio vienanariai yra a
3
, b
2
c,
a
4
d
,
a
7
bc
3
.
Suddami ar atimdami vienanarius, gausime vis naujas funkcijas, kuri laipsnius
galsime nustatyti pasinaudodami keliomis taisyklmis. Tegu f(A
i
), g(A
i
), h(A
i
) - funk-
cijos, kur A
i
- kokia nors realij kintamj aib.
Jei turime f(A
1
) ,= 0 ir f(A
1
) = g(A
2
) h(A
3
), o deg g(A
2
) = deg h(A
2
), tai
deg f(A
1
) = deg g(A
2
) = deg h(A
3
).
Jei f(A
1
) = g(A
2
) h(A
3
), tai deg f(A
1
) = deg g(A
2
) + deg h(A
3
).
Pavyzdys. Tokias taisykles ir j derinius taikydami galsime skaiiuoti kai kuri funk-
cij laipsnius:
a
3
b+a
laipsnis bus 2,
3

a
2
c
2
5

a
7
b
7
+c
7
laipsnis bus
2
3

7
5
=
11
15
. Dmesio! Tokios
funkcijos kaip f(a, b) =
a
3
a
2
b
laipsnio skaiiuoti negalime.
Funkcij (nelygyb) galsime vadinti homogenine, jei j galima pertvarkyti pavi-
dal h(A) =

f
i
(A
j
) ir vis funkcij f
i
(A
j
) laipsniai lygs.
Homogenikumo ocialus apibrimas:
Apibrimas. Jei h(A)yra funkcija nuo kintamj aibs A = a
1
, a
2
, . . . , a
n
, tai h yra
homogenin funkcija tada ir tik tada, kai h(ta
1
, ta
2
, ta
3
, ..., ta
n
) = t
n
h(a
1
, a
2
, a
3
, ..., a
n
),
kur t - bet koks teigiamas skaiius.
Jei yra duota nehomogenin nelygyb, taiau taip pat yra duota ir papildoma slyga,
danai naudinga nelygyb pertvarkyti taip, kad ji tapt homogenine, na o tada, nely-
gyb turs bti teisinga net tada, kai kintamieji netenkins duotos papildomos slygos.
T jau esame atlik kelis kartus, net ir neinodami homogenikumo svokos.
Paymtina, kad visos vidurki nelygybs galioja tik su teigiamais realiaisiais skai-
iais, o lygybs atvejis pasiekiamas, kai visi kintamieji lygs. Nors tai nra labai sudtin-
gas dalykas, danai svarbu atkreipti dmes, kad jis bt ilaikomas, ypa sprendiant
nehomogenines nelygybes ar iekant funkcij ekstremum.
Nors daugiausiai naudosime AM-GM nelygyb, kartais praveria ir kitos vidurki
nelygybs. Pavyzdiai iliustruos, kad nelygyb galime taikyti tiek pereinant nuo aritme-
tinio vidurkio prie geometrinio, tiek atvirkiai. Pirmuosiuose nagrinsime, kas vyksta,
53
2.1. Nelygybs Algebra
kai apibrimo sritis yra ribota arba yra konkreti slyga, neleidianti tiesiogiai taikyti
nelygybs. Toliau pateikiami pavyzdiai atspindi neretai pasitaikanius atvejus, kada
tiesioginis, aklas AM-GM nelygybs taikymas neduoda jokios naudos, o reikia su-
galvoti kaip pertvarkyti duot nelygyb, ar prisidti ir atsiimti papildom reikini,
kad taikoma AM-GM nelygyb padt pasiekti norim rezultat. Pateiksime ir kelet
nehomogenini nelygybi, kurioms isprsti reiks itin daug fantazijos.
Pavyzdiai
7 Pavyzdys. Duotas realus a 3. Raskite S = a +
1
a
minimum.
Dana klaida. Pagal AM-GM nelygyb, S = a +
1
a
2
_
a
1
a
= 2 Min S = 2.
Paaikinimas. Jei S minimumas yra 2, tai tada a =
a
1
= 1, kas prietarauja duotai
slygai, kad a 3.
Sprendimo iekojimas. Pastebime, kad S > a 3. Spjame, kad minimumas bus
pasiekiamas, kai a = 3. Tuomet
1
a
=
1
3
=
a
9
.
Sprendimas. S = a+
1
a
=
1
a
+
a
9
+
8a
9
2
_
1
a

a
9
+
83
9
=
10
3
. Minimumas bus pasiekiamas,
kai
1
a
=
a
9
a = 3.
Pastaba. Teisingo sprendimo paslaptis iame udavinyje, kaip ir kituose panaiuose io
skyrelio udaviniuose, yra teisingo lygybs atvejo atspjimas.
8 Pavyzdys (Macedonia 1999). Realieji teigiami a, b, c tenkina a
2
+b
2
+c
2
= 1. Raskite
minimum T = a +c +b +
1
abc
.
Dana klaida. Pagal AM-GM nelygyb T 4
4
_
a b c
1
abc
= 4. T minimumas yra
4.
Paaikinimas. Jei T minimumas yra 4, tai tada a = b = c =
1
abc
= 1, kas prietarauja
duotai slygai.
Sprendimo iekojimas. Kadangi T yra simetrin, minimumas greiiausiai bus pasiekia-
mas, kai a = b = c =
1

3
.
Sprendimas.
T = a +b +c +
1
9abc
+
8
9abc
4
4
_
a b c
1
9abc
+
8
9abc
(AM-GM nelygyb)

3
+
8
9
_
_
a
2
+b
2
+c
2
3
_
3
(SM-GM nelygyb)
=
4

3
+
8

3
= 4

3.

9 Pavyzdys. Duota a, b, c - teigiami realieji skaiiai, tokie, kad a +b +c


3
2
. Raskite
minimum
S =
_
a
2
+
1
b
2
+
_
b
2
+
1
c
2
+
_
c
2
+
1
a
2
.
54
2.1. Nelygybs Algebra
Dana klaida. Pagal AM-GM nelygyb S 3
3
_
_
a
2
+
1
b
2

_
b
2
+
1
c
2

_
c
2
+
1
a
2
3
6
_
_
2
_
a
2

1
b
2
_ _
2
_
b
2

1
c
2
_ _
2
_
c
2

1
a
2
_
= 3
6

8 = 3

2 Min S = 3

2.
Paaikinimas. Jei S minimumas yra 3

2, tai tada a = b = c =
a
1
=
b
1
=
c
1
= 1, kas
prietarauja duotai slygai.
Sprendimo iekojimas. Kadangi S yra ciklin a, b, c iraika, labai tiktina, kad minimu-
mas bus pasiekiamas, kai a = b = c =
1
2
. Tuomet a
2
= b
2
= c
2
=
1
4
=
1
16a
2
=
1
16b
2
=
1
16c
2
.
Sprendimas. Visur taikome AM-GM nelygyb:
S =

cyc
_
a
2
+
1
16b
2
+... +
1
16b
2

cyc

_
17
17

a
2
16
16
b
32

17

cyc
17
_
a
16
8
b
16

17
_
_
3
3

cyc
17
_
a
16
8
b
16
_
_
= 3

17
17
_
1
16
8
a
5
b
5
c
5
=
3

17
2
17
_
(2a 2b 2c)
5

17
2
17
_
_
2a+2b+2c
3
_
15

3

17
2
.
Minimumas yra
3

17
2
, pasiekiamas, kai a = b = c = 1/2.
10 Pavyzdys. Tegu a, b, c - teigiami realieji, tokie, kad a + b + c = 3. Raskite S =
3
_
a(b + 2c) +
3
_
b(c + 2a) +
3
_
c(a + 2b) maksimum.
Sprendimas. Taikome AM-GM, taiau prieinga puse:
S =

cyc
3
_
a(b + 2c) =

cyc
1
3

9

3
_
3a(b + 2c) 3

1
3

cyc
3a + (b + 2c) + 3
3
=
1
3

9

6(a +b +c) + 9
3
= 3
3

3.
Vadinasi, maksimumas yra 3
3

3 ir pasiekiamas, kai a = b = c = 1.
11 Pavyzdys. rodykite, kad kai n - natralusis skaiius didesnis u 1, galioja
n

1 +
n

n
n
+
n

1
n

n
n
< 2.
rodymas. Taikome AM-GM. Akivaizdu, kad lygybs atvejis negalios, tad nelygyb bus
grieta.
+
_

_
n
_
1 +
n

n
n
<
1
n

__
1 +
n

n
n
_
+n 1
_
= 1 +
n

n
n
2
n
_
1
n

n
n
<
1
n

__
1
n

n
n
_
+n 1
_
= 1
n

n
n
2
Sudj gausime tai, k reikjo rodyti.
55
2.1. Nelygybs Algebra
12 Pavyzdys. rodykite, kad teigiami realieji a, b, c tenkina nelygyb
a
3
b
2
+
b
3
c
2
+
c
3
a
2
a +b +c.
Sprendimas. Pagal AM-GM nelygyb:
a
3
b
2
+b +b 3
3

a
3
b
2
b b = 3a,
b
3
c
2
+c +c 3
3

b
3
c
2
c c = 3b,
c
3
a
2
+a +a 3
3

c
3
a
2
a a = 3c.
Sudj ias nelygybes gausime tai, k ir reikjo rodyti.
13 Pavyzdys. rodykite, kad teigiamiems realiesiems a, b, c galioja
a
5
b
3
+
b
5
c
3
+
c
5
a
3

a
4
b
2
+
b
4
c
2
+
c
4
a
2
.
rodymas. Pagal AM-GM nelygyb:
a
5
b
3
+
a
5
b
3
+
a
5
b
3
+
a
5
b
3
+b
2
5
5

_
a
5
b
3
_
4
b
2
= 5
a
4
b
2
;
b
5
c
3
+
b
5
c
3
+
b
5
c
3
+
b
5
c
3
+c
2
5
5

_
b
5
c
3
_
4
c
2
= 5
b
4
c
2
;
c
5
a
3
+
c
5
a
3
+
c
5
a
3
+
c
5
a
3
+a
2
5
5

_
c
5
a
3
_
4
a
2
= 5
c
4
a
2
.
Sudj gausime
4
_
a
5
b
3
+
b
5
c
3
+
c
5
a
3
_
+a
2
+b
2
+c
2
5
_
a
4
b
2
+
b
4
c
2
+
c
4
a
2
_
. (1)
Taip pat:
a
4
b
2
+b
2
2

a
4
b
2
b
2
= 2a
2
;
b
4
c
2
+c
2
2

b
4
c
2
c
2
= 2b
2
;
c
4
a
2
+a
2
2

c
4
a
2
a
2
= 2c
2
.
Sudj gausime
a
4
b
2
+
b
4
c
2
+
c
4
a
2
a
2
+b
2
+c
2
. (2)
Sudj nelygybes (1) ir (2) gausime tai, k ir reikjo rodyti.
56
2.1. Nelygybs Algebra
14 Pavyzdys (Nesbitto nelygyb). rodykite, kad teigiamiems realiesiems a, b, c galioja
a
b +c
+
b
a +c
+
c
a +b

3
2
.
Pastaba. Matematikos profesionalai danai rungiasi, kuris ino daugiau ios nelygybs
rodymo bd. Kvalikacinis raundas - 4. Kol kas pateiksime tik vien. Nesbitto
nelygyb yra dalinis Shapiro nelygybs atvejis.
rodymas. Tegu S =
a
b+c
+
b
a+c
+
c
a+b
, A =
b
b+c
+
c
a+c
+
a
a+b
, B =
c
b+c
+
a
a+c
+
b
a+b
. Tada
pagal AM-GM
A+S =
a +b
b +c
+
b +c
a +c
+
c +a
a +b
3
3

a +b
b +c

b +c
a +c

c +a
a +b
= 3;
B +S =
a +c
b +c
+
b +a
a +c
+
c +b
a +b
3
3

a +c
b +c

b +a
a +c

c +b
a +b
= 3; be to,
A+B = 3.
S =
A+S+B+SAB
2

3+33
2
=
3
2
.
15 Pavyzdys. rodykite, kad tokiems realiesiems teigiamiems a, b, c, kur a +b +c = 3,
galioja
a
3
(a +b)(a +c)
+
b
3
(b +c)(b +a)
+
c
3
(c +a)(c +b)

3
4
.
rodymas. Duot nelygyb veriame homogenine naudodami duot slyg ir pertvarko-
me:

1
a +b +c

cyc
_
a
3
(a +b)(a +c)
+
a +b
8
+
a +c
8

a +b
8

a +c
8
_

1
4
.
Sprendiame naudodami AM-GM nelygyb:
KAIR PUS
1
a +b +c

cyc
_
3
3

a
3
(a +b)(a +c)

a +b
8

a +c
8
_

1
2
=
1
a +b +c

cyc
3a
4

1
2
=
3(a +b +c)
4(a +b +c)

1
2
=
1
4
.
16 Pavyzdys. rodykite, kad jei a, b, c - tokie teigiami realieji skaiiai, kad a +b +c =
3abc, tai
1
a
3
+
1
b
3
+
1
c
3
3.
rodymas. a +b +c = 3abc
1
ab
+
1
bc
+
1
ca
= 3. Pagal AM-GM gausime
2
_
1
a
3
+
1
b
3
+
1
c
3
_
+ 3 =

cyc
_
1
a
3
+
1
b
3
+ 1
_
3
_
1
ab
+
1
bc
+
1
ca
_
= 9.
57
2.1. Nelygybs Algebra
17 Pavyzdys. Duoti a, b, c yra teigiami realieji skaiiai. rodykite, kad
a
b
+

b
c
+
3
_
c
a
>
5
2
.
rodymas. Pertvarkome ir naudojame AM-GM nelygyb:
S =
a
b
+
1
2

b
c
+
1
2

b
c
+
1
3
3
_
c
a
+
1
3
3
_
c
a
+
1
3
3
_
c
a
> 6
6

_
a
b

_
_
1
2

b
c
_
_
2
_
1
3
3
_
c
a
_
3
=
6
6

108
>
5
2
.
Udaviniai
1. Tegu a, b - teigiami realieji, tokie, kad a+b 1. Raskite S = ab+
1
ab
minimum. S
2. Tegu a, b, c - teigiami realieji, tokie, kad a +b +c
3
2
. Raskite S = a +b +c + S
1
a
+
1
b
+
1
c
minimum.
3. Tegu a, b, c - teigiami realieji, tokie, kad a + b + c = 1. Raskite maksimali S
S =
3

a +b +
3

b +c +
3

a +c reikm.
4. Tegu a, b, c - teigiami realieji, tokie, kad a 2, b 6, c 12. Raskite didiausi S
galim reikm, kuri gyja
=
bc

a 2 +ac
3

b 6 +ab
4

c 12
abc
.
5. rodykite, kad natraliesiems n galioja S
I =
_
2 + 1
2
+
3
_
3 + 1
3
+... +
n
_
n + 1
n
< n.
6. rodykite, kad teigiamiems realiesiems a, b, c galioja S
a
3
b
2
+
b
3
c
2
+
c
3
a
2

a
2
b
+
b
2
c
+
c
2
a
.
7. [Mircea Lascu, Gazeta Matematic a] Tegu a, b, c tokie teigiami realieji skaiiai, S
kad abc = 1. rodykite nelygyb
b +c

a
+
c +a

b
+
a +b

a +

b +

c + 3.
58
2.1. Nelygybs Algebra
8. rodykite, kad teigiamiems realiesiems a, b, c galioja S
a
2
b
2
+
b
2
c
2
+
c
2
a
2

a
3
b
3
+
b
3
c
3
+
c
3
a
3
.
9. rodykite, kad teigiamiems realiesiems a, b, c galioja S
a
2
b
5
+
b
2
c
5
+
c
2
a
5

1
a
3
+
1
b
3
+
1
c
3
.
10. Tegu realieji teigiami a, b, c tenkina a + b + c = 1. rodykite, kad jiems galioja S
(1 +a)(1 +b)(1 +c) 8(1 a)(1 b)(1 c).
11. [APMO 1998] rodykite, kad teigiamiems realiesiems a, b, c galioja S
_
1 +
a
b
__
1 +
b
c
__
1 +
c
a
_
2 +
2(a +b +c)
3

abc
.
12. Duoti teigiami realieji a, b, c, d. Raskite minimali reikinio reikm: S
S =
_
1 +
2a
3b
__
1 +
2b
3c
__
1 +
2c
3d
__
1 +
2d
3a
_
.
13. Duoti teigiami realieji a, b, c tokie, kad a +b +c = 3. rodykite, kad S
a
3
b(2c +a)
+
b
3
c(2a +b)
+
c
3
a(2b +c)
1.
14. Duoti teigiami realieji a, b, c tokie, kad ab +bc +ac = 1. rodykite, kad S
1
a(a +b)
+
1
b(b +c)
+
1
c(c +a)

9
2
.
15. [Romania Junior Balkan TST 2008] Duoti teigiami realieji skaiiai tenkina ab + S
bc +ac = 3. rodykite, kad jiems galioja nelygyb
1
1 +a
2
(b +c)
+
1
1 +b
2
(a +c)
+
1
1 +c
2
(a +b)

1
abc
.
16. [France Pre-MO 2005] rodykite, kad jei a, b, c - tokie teigiami realieji, kad a
2
+ S
b
2
+c
2
= 3, tai galioja
ab
c
+
bc
a
+
ca
b
3.
17. [Walther Janous, Crux Mathematicorum] rodykite, kad su teigiamais realiaisiais S
x, y, z galioja nelygyb
x
x +
_
(x +y)(x +z)
+
y
y +
_
(y +z)(x +y)
+
z
z +
_
(x +z)(y +z)
1.
59
2.1. Nelygybs Algebra
18. [Russia 2002] Tegu x, y, z - teigiami realieji skaiiai, kurie tenkina x+y +z = 3. S
rodykite nelygyb

x +

y +

z xy +xz +yz.
19. [IMO 1998 Shortlist] Tegu a, b, c bus tokie teigiami realieji skaiiai, kad abc = 1. S
rodykite, kad
a
3
(1 +b)(1 +c)
+
b
3
(1 +c)(1 +a)
+
c
3
(1 +a)(1 +b)

3
4
.
20. Duoti teigiami realieji a, b, c tokie, kad a
_
b
c
+b
_
c
a
+c
_
a
b
= 3. rodykite, kad S
a
6
b
3
+
b
6
c
3
+
c
6
a
3
3.
21. [IMO 1990 Shortlist] Realieji a, b, c, d tenkina ab +bc +cd +da = 1. rodykite, S
kad jie tenkins ir
a
3
b +c +d
+
b
3
c +d +a
+
c
3
d +a +b
+
d
3
a +b +c

1
3
.
22. [Tran Phuong] rodykite, kad su visais teigiamais realiaisiais a, b, c galioja S
bc
a
2
+
ca
b
2
+
ab
c
2
+abc
a
7
b
2
c
2
+
b
7
c
2
a
2
+
c
7
a
2
b
2
+
1
a
2
b
2
c
2
.
60
2.1. Nelygybs Algebra
2.1.3 Cauchy-Schwarz nelygyb
Cauchy-Schwarz nelygyb yra viena daniausiai taikom ir labiausiai nauding olim-
piad udavini sprendimuose.
Teorema (Cauchy-Schwarz nelygyb). Tegu (a
1
, a
2
, a
3
, . . . , a
n
) ir (b
1
, b
2
, b
3
, . . . , b
n
) bus
realij skaii sekos. Tuomet galios nelygyb
(a
2
1
+a
2
2
+a
2
3
+. . . +a
2
n
)(b
2
1
+b
2
2
+b
2
3
+. . . +b
2
n
) (a
1
b
1
+a
2
b
2
+a
3
b
3
+. . . +a
n
b
n
)
2
.
Lygyb galios tada ir tik tada, kai
a
1
b
1
=
a
2
b
2
= . . . =
a
n
b
n
.
Pateiksime kelet populiariausi nelygybs rodym.
Pirmas rodymas. Pasinaudosime Lagrange (Lagrano) tapatybe, kuri padeda nelygyb
rodyti ikart:
(a
2
1
+a
2
2
+a
2
3
+. . . +a
2
n
)(b
2
1
+b
2
2
+b
2
3
+. . . +b
2
n
) (a
1
b
1
+a
2
b
2
+a
3
b
3
+. . . +a
n
b
n
)
2
=

1i<jn
(a
i
b
j
a
j
b
i
)
2
.
Antras rodymas. Tegu (a
1
, a
2
, . . . , a
n
) ir (b
1
, b
2
, . . . , b
n
) bus realij skaii sekos. Im-
kime funkcij
f(x) = (a
1
x b
1
)
2
+ (a
2
x b
2
)
2
+. . . + (a
n
x b
n
)
2
.
Pastebkime, kad f(x) 0, vadinasi f(x) diskriminantas D 0. Kita vertus,
f(x) = (a
2
1
+a
2
2
+. . . +a
2
n
)x
2
2(a
1
b
1
+a
2
b
2
+. . . +a
n
b
n
)x + (b
2
1
+b
2
2
+. . . +b
2
n
).
Tada
D = 4(a
1
b
1
+a
2
b
2
+. . . +a
n
b
n
)
2
4(a
2
1
+a
2
2
+. . . +a
2
n
)(b
2
1
+b
2
2
+. . . +b
2
n
) 0
(a
2
1
+a
2
2
+. . . +a
2
n
)(b
2
1
+b
2
2
+. . . +b
2
n
) (a
1
b
1
+a
2
b
2
+. . . +a
n
b
n
)
2
.
Treias rodymas. Pagal nelygyb x
2
+y
2
2xy:
a
2
i
a
2
1
+a
2
2
+a
2
3
+. . . +a
2
n
+
b
2
i
b
2
1
+b
2
2
+b
2
3
+. . . +b
2
n

2a
i
b
i
_
(a
2
1
+a
2
2
+a
2
3
+. . . +a
2
n
)(b
2
1
+b
2
2
+b
2
3
+. . . +b
2
n
)
.
Sudj visus dmenis su visais i, kai 1 i n, gausime tai, k ir reikjo rodyti.
Ketvirtas rodymas. Prisiminkime nelygybi skyrelio Pirmieji ingsniai udavin nr.
5. Gaunama nelygyb yra vadinama Cauchy-Schwarz (CS) nelygybs Engel forma.
61
2.1. Nelygybs Algebra
Teorema (CS - Engel forma). Jei (a
1
, a
2
, . . . , a
n
) ir (b
1
, b
2
, . . . , b
n
) yra realij skaii
sekos, kai visi b
i
> 0, tai galioja
a
2
1
b
1
+
a
2
2
b
2
+. . . +
a
2
n
b
n

(a
1
+a
2
+. . . +a
n
)
2
b
1
+b
2
+. . . +b
n
.
Lygyb galios tada ir tik tada, kai
a
1
b
1
=
a
2
b
2
= . . . =
a
n
b
n
.
I esms tai ir yra Cauchy-Schwarz nelygyb, tiksliau, kitokia jos forma: belieka
visiems i statyti a
i
a
i
b
i
ir b
i
b
2
i
ir gausime standartin iraik, kuri bus teisinga
su visais realiaisiais (a
1
, a
2
, . . . , a
n
) ir (b
1
, b
2
, . . . , b
n
).
Kitos Cauchy-Schwarz nelygybs formos:
a
2
1
+a
2
2
+. . . +a
2
n

(a
1
b
1
+a
2
b
2
+...+a
n
b
n
)
2
b
2
1
+b
2
2
+...+b
2
n
;

_
(a
2
1
+a
2
2
+... +a
2
n
)(b
2
1
+b
2
2
+... +b
2
n
) a
1
b
1
+a
2
b
2
+. . . +a
n
b
n
;
Kai a
1
, a
2
, . . . , a
n
ir b
1
, b
2
, . . . , b
n
teigiami:
(a
1
+a
2
+... +a
n
)(b
1
+b
2
+... +b
n
) (

a
1
b
1
+

a
2
b
2
+... +

a
n
b
n
)
2
;
a
1
+a
2
+... +a
n

(

a
1
b
1
+

a
2
b
2
+...+

a
n
b
n
)
2
b
1
+b
2
+...+b
n
;

_
(a
1
+a
2
+... +a
n
)(b
1
+b
2
+... +b
n
)

a
1
b
1
+

a
2
b
2
+. . . +

a
n
b
n
.
Sunku net pasakyti, ar naudingesn Engel forma, ar pati Cauchy-Schwarz nelygyb.
Daniausiai, jas taikant gaunamas tas pats rezultatas. Svarbu atkreipti dmes, kad
skiriasi Cauchy-Schwarz ir Engel formos nelygybi apibrimo sritys: pirmoji galioja su
visais realiaisiais, o antroji reikalauja, kad trupmen vardikliai bt teigiami. Nepaisant
i skirtum, ios dvi nelygybs yra vadinamos vienu vardu - Cauchy-Schwarz nelygybe.
Sprendiant i lygybs atvejo, jei AM-GM nelygyb sumaina reikin iki lygi kin-
tamj, kuomet Cauchy-Schwarz lygybs atvejis pasiekiamas tada, kai kintamieji yra
proporcingi, galime sakyti, kad Cauchy-Schwarz nelygyb yra lankstesn ir bendresn.
Daugel ankstesni pavyzdi ir udavini galima padaryti ir naudojant Cauhcy-
Schwarz nelygyb. Skaitytoj raginame paiam pabandyti tai atlikti. Mes engsime prie
pavydi, kuriuose matysis, kaip vairiai galime pritaikyti Cauchy-Schwarz nelygyb,
gaudami netiktinus rezultatus.
Pavyzdiai
18 Pavyzdys (Baltic Way 2008). rodykite, kad jei realieji a, b, c tenkina a
2
+b
2
+c
2
= 3,
tai galioja
a
2
2 +b +c
2
+
b
2
2 +c +a
2
+
c
2
2 +a +b
2

(a +b +c)
2
12
.
Kada galios lygyb?
62
2.1. Nelygybs Algebra
rodymas. Pastebkime, kad 2 +b > 0, nes b
2
3. Taip pat bus 2 +a > 0 ir 2 +c > 0.
Tuomet pagal Cauchy-Schwarz nelygyb:
KAIR PUS
(a +b +c)
2
6 +a
2
+b
2
+c
2
+a +b +c
.
Taigi, belieka rodyti a +b +c 3 2a +2b +2c 3 +a
2
+b
2
+c
2
(a 1)
2
+(b
1)
2
+ (c 1)
2
0, kas yra akivaizdu. Lygyb galios, kai a = b = c = 1.
19 Pavyzdys. Duoti teigiami realieji a b c d tenkina a +b +c +d = 1. Raskite
maiausi reikinio Z = 4a
2
+ 3b
2
+ 2c
2
+d
2
reikm.
Sprendimas. Pastebkime, kad a
1
4
, a+b
1
2
, a+b +c
3
4
, a+b +c +d = 1. Sudj
gausime 4a + 3b + 2c +d
10
4
. Pagal Cauchy-Schwarz nelygyb:
Z = 4a
2
+ 3b
2
+ 2c
2
+d
2

(4a + 3b + 2c +d)
2
10

5
8
.
Minimumas bus
5
8
. Jis pasiekiamas, kai a = b = c = d =
1
4
.
20 Pavyzdys (Pham Kim Hung). rodykite, kad teigiami realieji a, b, c tenkina
a
2
bc
2a
2
+b
2
+c
2
+
b
2
ac
2b
2
+a
2
+c
2
+
c
2
ab
2c
2
+a
2
+b
2
0.
rodymas. Jei nelygyb padauginsime i -2 ir prie kairs puss trupmen pridsime po
1, o deinje pridsime 3, tai gausime ekvivaleni nelygyb:
(b +c)
2
2a
2
+b
2
+c
2
+
(a +c)
2
2b
2
+a
2
+c
2
+
(a +b)
2
2c
2
+a
2
+b
2
3. (1)
Pagal Cauchy-Schwarz nelygyb:
KAIR PUS (1)

cyc
b
2
b
2
+a
2
+

cyc
c
2
c
2
+a
2
=

cyc
b
2
+a
2
b
2
+a
2
= 3
21 Pavyzdys (Nesbitto nelygyb). Jei a, b, c - teigiami realieji skaiiai, tai galioja
nelygyb
a
b +c
+
b
a +c
+
c
a +b

3
2
.
rodymas. Naudosime Cauchy-Schwarz nelygyb:
KAIR PUS =
a
2
ab +ac
+
b
2
ab +bc
+
c
2
ac +bc

(a +b +c)
2
2(ab +bc +ac)

3(ab +bc +ac)
2(ab +bc +ac)
=
3
2
.
63
2.1. Nelygybs Algebra
22 Pavyzdys (Iran 1998). Skaiiai x, y, z tokie, kad x 1, y 1, z 1, ir
1
x
+
1
y
+
1
z
=
2. rodykite, kad galios nelygyb

x +y +z

x 1 +
_
y 1 +

z 1.
rodymas. Pertvarkykime duot slyg:
1
x
+
1
y
+
1
z
= 2
x1
x
+
y1
y
+
z1
z
= 1.
Taikysime Cauchy-Schwarz nelygyb:
x +y +z = (x +y +z)(
x 1
x
+
y 1
y
+
z 1
z
) (

x 1 +
_
y 1 +

z 1)
2
,
k ir reikjo rodyti.
Udaviniai
1. Deimt teigiam realij skaii tenkina a
1
+a
2
+... +a
10
= 1 ir a
1
a
2
+a
3
S
a
4
+a
5
+a
6
a
7
+a
8
+a
9
+a
10
. Raskite reikinio Z = a
2
1
+a
2
2
+... +a
2
10
maiausi
pasiekiam reikm.
2. Teigiami realieji a, b, c, d tenkina nelygybes a 1, a + b 5, a + b + c 14 ir S
a +b +c +d 30. rodykite, kad galioja nelygyb

a +

b +

c +

d 10.
3. [IMO 1995] Teigiami realieji a, b, c yra tokie, kad abc = 1. rodykite, kad teisinga S
nelygyb
1
a
3
(b +c)
+
1
b
3
(a +c)
+
1
c
3
(a +b)

3
2
.
4. rodykite, kad teigiamiems realiesiems x
1
, x
2
, ..., x
n
galioja nelygyb S
_
x
1
(3x
2
+x
3
) +
_
x
2
(3x
3
+x
4
) +... +
_
x
n
(3x
1
+x
2
) 2(x
1
+x
2
+... +x
n
).
5. [Darij Grinberg] rodykite, kad teigiamiems realiesiems a, b, c galioja S
a
(b +c)
2
+
b
(a +c)
2
+
c
(a +b)
2

9
4(a +b +c)
.
6. Tegu a, b, x, y, z bus teigiami realieji skaiiai. Parodykite, kad S
x
ay +bz
+
y
az +bx
+
z
ax +by

3
a +b
.
7. rodykite, kad teigiamiems realiesiems skaiiams, tokiems, kad a + b + c = 3, S
galioja nelygyb
a
1 +b
2
c
+
b
1 +c
2
a
+
c
1 +a
2
b

3
2
.
8. Parodykite, kad teigiamiems realiesiems a
1
, a
2
, . . . , a
n
ir b
1
, b
2
, . . . , b
n
galioja S
nelygyb
_
a
2
1
+b
2
1
+. . . +
_
a
2
n
+b
2
n

_
(a
1
+a
2
+. . . +a
n
)
2
+ (b
1
+b
2
+. . . +b
n
)
2
.
64
2.1. Nelygybs Algebra
9. [JBMO 2002 Shortlist] rodykite, kad jei teigiami realieji skaiiai tenkina abc = S
2, tai galios nelygyb
a
3
+b
3
+c
3
a

b +c +b

a +c +c

a +b.
10. [Walther Janous, Crux Mathematicorum] Tegu x, y ir z bus teigiami realieji. S
rodykite, kad galios
x
x +
_
(x +y)(x +z)
+
y
y +
_
(y +x)(y +z)
+
z
z +
_
(z +x)(z +y)
1.
11. rodykite, kad teigiamiems realiesiems skaiiams a, b, c, d, e, f galioja nelygyb S
a
b +c
+
b
c +d
+
c
d +e
+
d
e +f
+
e
f +a
+
f
a +b
3.
12. [Ukraine 2001] rodykite, kad teigiamiems realiesiems a, b, c, x, y, z, kai x+y+z = S
1, galioja nelygyb
ax +by +cz + 2
_
(xy +xz +yz)(ab +bc +ac) a +b +c.
13. [Japan TST 2004] Tegu a, b, c - tokie teigiami realieji skaiiai, kuri suma lygi S
1. rodykite, kad galios nelygyb
1 +a
1 a
+
1 +b
1 b
+
1 +c
1 c

2a
b
+
2b
c
+
2c
a
.
14. [Iran TST 2009] Duoti teigiami realieji a, b, c, kuri suma lygi 3. rodykite, kad S
1
2 +a
2
+b
2
+
1
2 +c
2
+a
2
+
1
2 +b
2
+c
2

3
4
.
15. [Komal Magazine] rodykite, kad teigiamiems realiesiems a, b, c galioja S
(a
2
+ 2)(b
2
+ 2)(c
2
+ 2) 3(a +b +c)
2
.
65
2.1. Nelygybs Algebra
2.1.4 Specialios technikos
iame skyrelyje susipainsime su keliomis populiariomis gudrybmis, kurios gali labai
pagelbti udavini sprendime. Sprendim varikliukais liks mums jau gerai inomos
nelygybs, tokios kaip AM-GM ir Cauchy-Schwarz. Pagrindin gudryb - keitiniai. Jei
skaitytojas abejoja j galingumu, tegu pabando pateiktas nelygybes ispsti alterna-
tyviu bdu. Dalis pavyzdi ir udavini yra susij su geometrija, taiau algebrinse
nelygybse utenka ir elementari ini.
Homogenizacija ir Normalizacija
Homogenizacija - tai nehomogenins nelygybs vertimas homogenine, daniausiai tam
naudojant duot papildom slyga. Iki iol mes nieko nebijodami drsiai homogeni-
zuodavome nelygybes ir bd nematme, taiau neretai taip primityviai homogenizuoti
nehomogenin nelygyb yra bjauroka ir visikai nenaudinga. Todl iame skyrelyje su-
sipainsime su specialiais homogenizuojaniais keitiniais, kurie duos gerokai daugiau
naudos.
i keitini esm yra inaudoti papildom slyg taip, kad visi kintamieji tapt
nulinio laipsnio, o ir tuomet visa nelygyb taps nulinio laipsnio. Kiekvienai duotai
slygai galime sugalvoti atitinkam keitini.
Duota abc = k
3
. Geras keitinys bt a =
kx
y
, b =
ky
z
, c =
kz
x
. Visada galime
sugalvoti spdingesn: a =
kx
3
y
z
4
ir t.t.
Duota a + b + c = k. Bene vienintelis naudingas keitinys bt a =
xk
x+y+z
, b =
yk
x+y+z
, c =
zk
x+y+z
, taiau neribokime savo fantazijos: a =
kx(x+2y)
(x+y+z)
2
, b =
ky(y+2z)
(x+y+z)
2
,
c =
kz(z+2x)
(x+y+z)
2
ir pan..
Duota ab + bc + ac = k. Kintamuosius galime keisti poromis: bc =
xk
x+y+z
,
ac =
yk
x+y+z
, ab =
zk
x+y+z
.
inoma, kai turime daugiau kintamj, reiks sugalvoti analogik keitini, taiau
nereikt persistengti - danai tokie keitiniai tik subjauroja nelygyb ir ji tampa tik
dar labiau komplikuota.
23 Pavyzdys. Tegu a, b, c - tokie teigiami skaiiai, kad abc = 1. rodykite nelygyb
1
a
2
+a + 1
+
1
b
2
+b + 1
+
1
c
2
+c + 1
1.
Sprendimas. Pakeiskime a =
yz
x
2
, b =
xz
y
2
, c =
xy
z
2
. Nelygyb tampa:

cyc
1
y
2
z
2
x
4
+
yz
x
2
+ 1
1

cyc
x
4
y
2
z
2
+x
2
yz +x
4
1.
66
2.1. Nelygybs Algebra
O pagal Cauchy-Schwarz ir AM-GM nelygybes:

cyc
x
4
y
2
z
2
+x
2
yz +x
4

(x
2
+y
2
+z
2
)
2
x
4
+y
4
+z
4
+

cyc
x
2
y
2
+

cyc
yzx
2

(x
2
+y
2
+z
2
)
2
x
4
+y
4
+z
4
+

cyc
x
2
y
2
+

cyc
1
2
(x
2
y
2
+x
2
z
2
)
=
(x
2
+y
2
+z
2
)
2
x
4
+y
4
+z
4
+ 2

cyc
x
2
y
2
= 1.

Normalizacija yra tarsi prieingas dalykas homogenizacijai. Tegu N(a


1
, a
2
, ..., a
n
)
0 - homogenin nelygyb. Pagal homogenikumo apibrim, pakeit a
i
= tx
i
visiems
i, kur t - teigiamas skaiius gausime, N(a
1
, a
2
, ..., a
n
) = t
n
N(x
1
, x
2
, ..., x
n
). Vadinasi,
liks rodyti N(x
1
, x
2
, ..., x
n
) 0, kur visi x
i
yra proporcingai norimai stipriai padid-
j/sumaj. Tai reikia, kad naujos kintamj aibs savybs (suma, sandauga, kvad-
rat suma, ir pan.) yra pasikeit. Niekas nedraudia juos mainti tiek, kad j suma,
sandauga ar dar kokia aibs savyb bt lygi konkreiam, ms pasirinktam dydiui.
Pavyzdiui, jei norime rodyti homogenin nelygyb nuo trij teigiam kintamj
f(a, b, c) 0, nemaindami bendrumo galime tarti, kad ab + bc + ac = 3. Tuomet,
naudodami AM-GM ir kitas nelygybes galime nustatyti kit kintamj aibs savybi
ribas: 3 = ab + bc + ac 3
3

a
2
b
2
c
2
abc 1, 3 = ab + bc + ac a
2
+ b
2
+ c
2
,
9 = 3(ab +bc +ac) (a +b +c)
2
a +b +c 3.
24 Pavyzdys (Nesbitto nelygyb). rodykite, kad teigiamiems skaiiams galioja
a
b +c
+
b
a +c
+
c
a +b

3
2
.
Sprendimas. Nelygyb yra homogenin. Nemaindami bendrumo tariame, kad a +b +
c = 1. inome, kad
ab +bc +ac
1
3
(a +b +c)
2
=
1
3
.
Tuomet
3
2
= 3
9
2

1
3
3
9
2
(ab +bc +ac).
Reikia, liks rodyti
a
b +c
+
b
a +c
+
c
a +b
3
9
2
(ab +bc +ac)
arba

cyc
a
b +c
+
9a(b +c)
4
3.
Na o pagal AM-GM nelygyb:

cyc
a
b +c
+
9a(b +c)
4

cyc
2

a 9a(b +c)
4(b +c)
= 3

cyc
a = 3.
67
2.1. Nelygybs Algebra

25 Pavyzdys. rodykite, kad teigiamiems realiesiems a, b, c galioja nelygyb

ab +bc +ac
3

3

(a +b)(b +c)(a +c)


8
.
Sprendimas. Nelygyb homogenin, tad neprarasdami bendrumo tariame, kad ab +
bc +ac = 3. Tada KAIR PUS = 1. Be to, pagal AM-GM nelygyb galime nesunkiai
rasti, kad a +b +c 3 ir abc 1. inodami tapatyb, nelygyb pertvarkome:
(a +b)(b +c)(a +c) = (a +b +c)(ab +bc +ac) abc = 3(a +b +c) abc 8.
Tuomet DEIN PUS 1 = KAIR PUS, k ir reikjo rodyti.
Algebriniai ir trigonometriniai keitiniai
Visi kiti nei anksiau aprayti algebriniai keitiniai yra grynas fantazijos reikalas. B-
dami itin paprasti, jie danai labai stipriai palengvina darb.
26 Pavyzdys (Nguyen Van Thach). Tegu a, b, c - teigiami realieji skaiiai. rodykite,
kad jiems galioja nelygyb
a
3
a
3
+b
3
+abc
+
b
3
b
3
+c
3
+abc
+
c
3
c
3
+a
3
+abc
1.
Sprendimas. Pakeiskime
b
a
= x,
c
b
= y,
a
c
= z ir pastebkime, kad tada xyz = 1.
Tuomet:
a
3
a
3
+b
3
+abc
=
1
1 +
_
b
a
_
3
+
b
a

c
a
=
1
1 +x
3
+
x
z
=
xyz
xyz +x
3
+x
2
y
=
yz
yz +x
2
+xy
.
Pagal Cauchy-Schwarz nelygyb:

cyc
yz
yz +x
2
+xy

(xy +xz +yz)
2

cyc
yz(yz +x
2
+xy)
.
Taigi, lieka rodyti
(xy +yz +xz)
2

cyc
yz(yz +x
2
+xy).
Nepabijoj reikinio iskleisti matysime, kad tai yra tapatyb.
27 Pavyzdys (St. Petersburg 2009). Duotiems teigiamiems realiesiems skaiiams ga-
lioja sryis a +b +c = ab +bc +ac. rodykite, kad jiems galioja nelygyb
a +b +c + 1 4.
68
2.1. Nelygybs Algebra
rodymas pagal Mathias Tejs Knudsen. Jei a + b < 1, tai a + b + c = ab + bc + ca =
c(a + b) + ab < c + (a + b)(a + b) < c + a + b ir gauname prietar. Taigi a + b 1 ir
analogikai b + c 1 bei a + c 1. veskime keitin a = x +
1
2
, b = y +
1
2
, c = z +
1
2
,
tuomet duota slyga taps ab+bc+ac =
3
4
. Ankiau gautas rezultatas bus ekvivalentus
x + y 0, x + z 0, y + z 0, vadinasi ne daugiau kaip vienas i skaii x, y, z yra
neigiamas. Pakeitus, pagrindin nelygyb pavirsta
8xyz 1.
Jei vienas i x, y, z yra neigiamas, nelygyb akivaizdi, o jei visi teigiami - pagal AM-GM
nelygyb:
3
4
= xy +yz +xz 3
3
_
x
2
y
2
z
2
8xyz 1.

Pastaba. Keitinys a =
1
x
, b =
1
y
, c =
1
z
iuo atveju irgi labai padt, nes tuomet duota
slyga nepasikeist, o pagrindin nelygyb gyt kitoki, galbt, patogesn form, bet
tai jau visai kitas sprendimas.
Uuominos trigonometrinius keitinius gali bti labai vairios: slyga, jog kintamie-
ji yra intervale [0, 1], arba konstrukcija

1 x
2
sueruoja apie sinusus, kosinusus, o
algebrin konstrukcija

1 +x
2
- tipinis tangeto ar kotangento taikymo atvejis, kadangi
1

1+tan
2
x
= [ cos x[ ir
1

1+cot
2
x
= [ sin x[.
28 Pavyzdys (Latvia 2002). Teigiami realieji skaiiai a, b, c, d tenkina
1
1 +a
4
+
1
1 +b
4
+
1
1 +c
4
+
1
1 +d
4
= 1.
rodykite, kad tada teisinga yra nelygyb abcd 3.
Sprendimas. Pakeiskime a
2
= tan A, b
2
= tan B, c
2
= tan C, d
2
= tan D, kur
A, B, C, D (0,

2
). inodami, kad tan
2
+ 1 =
1
cos
2

, pertvarkome duot slyg

cos
2
A+ cos
2
B + cos
2
C + cos
2
D = 1.
Pagrindin nelygyb tampa
tan A tan B tan C tan D 9.
Pagal AM-GM nelygyb:
_

_
sin
2
A = 1 cos
2
A = cos
2
B + cos
2
C + cos
2
D 3
3

cos
2
Bcos
2
C cos
2
D
sin
2
B = 1 cos
2
B = cos
2
C + cos
2
D + cos
2
A 3
3

cos
2
C cos
2
Dcos
2
A
sin
2
C = 1 cos
2
C = cos
2
D + cos
2
A+ cos
2
B 3
3

cos
2
Dcos
2
Acos
2
B
sin
2
D = 1 cos
2
D = cos
2
A+ cos
2
B + cos
2
C 3
3

cos
2
Acos
2
Bcos
2
C
Visk sudaugin gausime reikiam rezultat.
69
2.1. Nelygybs Algebra
Poktai su trikampiu
Danai, ypa rimtesnse olimpiadose, yra mgiami udaviniai, susiejantys kelias ma-
tematikos disciplinas. iame maame skyrelyje nagrinsime algebros ir geometrijos
jungin: nelygybes trikampio kratinms.
Pagrindinis dalykas, naudingas inoti rodinjant nelygyb trikampio kratinms,
yra trikampio nelygyb: bet kuri dviej kratini ilgi suma yra didesn u likusiosios
ilg.
29 Pavyzdys (Pham Kim Hung). Duoto trikampio kratini ilgiai yra a, b, c. rodykite,
kad galioja
1

a +b c
+
1

b +c a
+
1

a +c b

9
ab +bc +ac
,
kai trikampio perimetras 3.
Pirmas rodymas. Paymekime x =

a +b c, y =

b +c a, z=

a +c b, tuomet
x
2
+y
2
+z
2
= 3, o nelygyb pavirs
1
x
+
1
y
+
1
z

36
(x
2
+y
2
+z
2
)
2
+x
2
y
2
+x
2
z
2
+y
2
z
2
. (sitikinkite!)
Kas yra ekvivalentu
(xy +xz +yz)(9 +x
2
y
2
+x
2
z
2
+y
2
z
2
) 36xyz.
Pagal trikampio nelygyb, gauname, kad x, y, z - teigiami skaiiai, taigi, jiems galime
taikyti AM-GM nelygyb. I tikrj: sudauginus
xy +xz +yz 3
3
_
x
2
y
2
z
2
ir
9 +x
2
y
2
+x
2
z
2
+y
2
z
2
12
12
_
x
4
y
4
z
4
gausime reikiam rezultat.
Ypa fantastikas yra Ravi keitinys: inome, kad trikamp ABC brus apskritim,
kuris kratines AB, BC ir AC lieia atitinkamai takuose X, Y ir Z, gausime AX =
AZ = p, BX = BY = r ir CY = CZ = s. Tuomet AB = p + r, BC = r + s ir
AC = p + s. Akivaizdu, kad p, r, s - teigiami dydiai. Toks keitinys atria sprendjui
rankas nuo trikampio ir leidia dirbti su bet kokiais teigiamais skaiiais.
Antras rodymas. Atlikime Ravi keitin: a = p + r, b = r + s, c = p + s. Tursime
p +r +s =
3
2
. Pagrindin nelygyb taps:
1

2p
+
1

2r
+
1

2s

9
(p +r +s)
2
+pr +rs +ps
.
Tai yra ekvivalentu
(
9
4
+pr +rs +ps)(

ps +

pr +

ps) 9

prs.
70
2.1. Nelygybs Algebra
Pagal AM-GM nelygyb:
9
4
+pr +rs +ps 12
12
_
1
4
9
p
2
r
2
s
2
ir

ps +

pr +

ps 3
3

prs.
ias dvi sudauginame ir gauname tai, k ir reikjo rodyti.
Cauchy Reverse Technique
Tok spding pavadinim gali turti nebent koks nors labai sudtingas ir niekam ne-
reikalingas matematinis metodas. Taip jau atsitiko, kad btent itaip yra vadinamas
itin paprastas ir tuo genialus nelygybi sprendimo bdas.
Kai turime nelygyb, ir mums tiesiog nieti rankas pritaikyti AM-GM nelygyb, bet
to padaryti negalime, nes nelygybs enklas yra prieingas, atliekame paprast triuk:
I trupmenos ikeliame sveikj dal, kuri yra didesn u pradin trupmen. Tada prie
naujo trupmeninio likuio gausime minus ir galsime ilieti savo energij ir pykt pri-
taikydami AM-GM nelygyb. Nematant, kaip tai vyksta i tikrj, pagal aprayma tai
atrodo visikai nesuprantama, tad pereikime prie pavyzdi, kurie spalvingai iliustruos
mint.
30 Pavyzdys. rodykite, kad su teigiamais realiaisiais skaiiais teisinga nelygyb
a
4
a
3
+ 2b
3
+
b
4
b
3
+ 2c
3
+
c
4
c
3
+ 2d
3
+
d
4
d
3
+ 2d
3

a +b +c +d
3
.
Sprendimas. Pertvarkykime kairs puss dmenis, kad jie tapt apversti ir ikart
taikykime AM-GM nelygyb:

cyc
a
4
a
3
+ 2b
3
=

cyc
a
4
+ 2ab
3
2ab
3
a
3
+ 2b
3
= a +b +c +d

cyc
2ab
3
a
3
+ 2b
3
a +b +c +d

cyc
2ab
3
3
3

a
3
b
6
= a +b +c +d
2
3
(a +b +c +d)
=
a +b +c +d
3
.

31 Pavyzdys. rodykite, kad teigiamiems realiesiems a, b, c, kur a +b +c = 3, galioja


nelygyb
1
1 + 2b
2
c
+
1
1 + 2c
2
a
+
1
1 + 2a
2
b
1.
71
2.1. Nelygybs Algebra
rodymas. Partvarkome ir du kartus taikome AM-GM nelygyb (stebuklinga, kad gali-
me taikyti AM-GM nelygyb toje paioje nelygybje ir majania, ir didjania puse):

cyc
1
1 + 2b
c
=

cyc
1 + 2b
2
c 2b
2
c
1 + 2b
2
c
3

cyc
2b
2
c
3
3

b
4
c
2
= 3

cyc
2
3

b
2
c
3
3

cyc
2(2b +c)
9
= 3
2 3(a +b +c)
9
= 1.
Udaviniai
1. [Romania Junior TST 2003] rodykite, kad teigiami realieji skaiiai, tenkinantys S
abc = 1, taip pat tenkina ir
1 +
3
a +b +c

6
ab +bc +ac
.
2. [Clock-Tower School Junior Competition 2009] Teigiami realieji skaiiai a, b, c S
tenkina abc = 8. rodykite, kad jiems taip pat galioja nelygyb
a 2
a + 1
+
b 2
b + 1
+
c 2
c + 1
0.
3. [Zhautykov Olympiad 2008] rodykite, kad teigiamiems realiesiems skaiiams, S
kurie tenkina abc = 1, galioja nelygyb
1
b(a +b)
+
1
c(b +c)
+
1
a(a +c)

3
2
.
4. rodykite nelygyb, kuri galioja su teigiamais realiaisiais a, b, c, d: S
a
b
2
+c
2
+d
2
+
b
c
2
+d
2
+a
2
+
c
d
2
+a
2
+b
2
+
d
a
2
+b
2
+c
2

3

3
2

1

a
2
+b
2
+c
2
+d
2
.
5. [USAMO 2003] rodykite nelygyb, kuri teisinga su teigiamais realiaisiais a, b, c: S
(2a +b +c)
2
2a
2
+ (b +c)
2
+
(2b +a +c)
2
2b
2
+ (a +c)
2
+
(2c +a +b)
2
c
2
+ (a +b)
2
8.
6. [Korea 1998] Teigiami realieji skaiiai tenkina sry x+y +z = xyz. rodykite, S
kad jiems galioja nelygyb
1

1 +x
2
+
1
_
1 +y
2
+
1

1 +z
2

3
2
.
7. [Crux Mathematicorum] Parodykite, kad teigiamiems realiesiems skaiiams, ku- S
rie tenkina abcde = 1, galioja
72
2.1. Nelygybs Algebra
a+abc
1+ab+abcd
+
b+bcd
1+bc+bcde
+
c+cde
1+cd+cdea
+
d+dea
1+de+deab
+
e+eab
1+ea+eabc

10
3
.
8. [George Tsintifas, Crux Mathematicorum] rodykite nelygyb teigiamiems rea- S
liesiems skaiiams:
(a +b)
3
(b +c)
3
(c +d)
3
(d +a)
3
16a
2
b
2
c
2
d
2
(a +b +c +d)
4
.
9. [Romania Junior TST 2002] Skaiiai a, b, c priklauso intervalui [0, 1]. rodykite, S
kad jiems galioja nelygyb

abc +
_
(1 a)(1 b)(1 c) < 1.
10. [IMO 1983]. rodykite, kad trikampio kratins a, b, c tenkina nelygyb S
a
2
b(a b) +b
2
c(b c) +c
2
a(c a) 0.
11. [Samin Riasat] rodykite, kad trikampio kratins a, b, c tenkina nelygyb S
a
3a b +c
+
b
3b c +a
+
c
3c a +b
1.
12. rodykite, kad trikampio kratins tenkina nelygyb S
_
3
_

ab +

bc +

ac
_

a +b c +

a +c b +

b +c a.
13. [Bulgaria TST 2003] Duoti teigiami realieji skaiiai a, b, c tenkina a +b +c = 3. S
rodykite, kad jiems teisinga nelygyb
a
1 +b
2
+
b
1 +c
2
+
c
1 +a
2

3
2
.
14. [Pham Kim Hung] Duoti tokie teigiami skaiiai a, b, c, d, kad a +b +c +d = 4. S
rodykite, kad jie tenkina nelygyb
a
1 +b
2
c
+
b
1 +c
2
a
+
c
1 +d
2
a
+
d
1 +a
2
b
2.
15. Duoti n teigiam skaii a
1
, a
2
, a
3
, . . . , a
n
, kuri kvadrat suma lygi n. rodykite, S
kad jiems galioja nelygyb
1
a
3
1
+ 2
+
1
a
3
2
+ 2
+
1
a
3
3
+ 2
+. . . +
1
a
3
n
+ 2

n
3
.
16. Turime skaiius a, b, c, kuri suma lygi 3. rodykite, kad jiems taip pat galios S
nelygyb
a + 1
b
2
+ 1
+
b + 1
c
2
+ 1
+
c + 1
a
2
+ 1
4.
17. [Pham Kim Hung] Parodykite, kad teigiamiems realiesiems skaiiams a, b, c, ku- S
rie tenkina a
2
+b
2
+c
2
= 1, galioja
1
2 a
+
1
2 b
+
1
2 c
3.
18. Tegu a, b, c bus tokie teigiami skaiiai, kad a+b+c = 1. Parodykite, kad teisinga S
a
2
a + 2b
3
+
b
2
b + 2c
3
+
c
2
c + 2a
3
1.
73
2.1. Nelygybs Algebra
2.1.5 Drakon puota
I tamsiausi keri, tolimiausi ukampi susirinko jos ir jie pasirodyti vieni kitiems.
Ne jgos, o savo aiaruojaios ivaizdos parodyti, emocijomis pasidalinti atvyko. Kiek-
vienas sveias laukiamas, kiekvieno istorija ypatinga. Ir suksis jie valso ritme iki ryto,
kol giedoriai gaidiai paskelbs puotos pabaig. Kai drakonai atsisveikin pakils skry-
diui namo, liks ia j leten spaudai, nag dryiai ir neatsargi kosteljim apdegint
uuolaid likuiai, bylojantys apie i spding padar egzistavim. Kas ino, galbt
kada nors kas nors gals regti nors vien j dvikovoje su piktu burtininku, kada degs
em, uvirs vandenynai, o dangus apsitrauks ledu.
iame skyrelyje skaitytoj supaindinsime su dar keliomis nelygybsmis, kurios u-
davini sprendimuose pasitaiko iskirtinai retai. Ne dl to, kad ios nelygybs yra
silpnos ar neuniversalios, prieingai: dl to, kad sunki udavini yra gerokai maiau
nei lengvj. Tai bus tik paintinis skyrelis, siekiantis parodyti artimiausias fantastikai
teormas-nelygybes, todl nepateiksime nei pavyzdi, nei udavini, tik kelet taikymo
komentar ir leisime skaitytojui pasinerti sav vaizduot.
Teorema (Hlder). Tegu a
11
, . . . , a
n1
, . . ., a
1k
, . . . , a
nk
bus k skaiius aibi, kur
kiekviena j turi po n teigiam element, o p
1
, . . . , p
k
bus teigiam skaii aib, kurios
vis element suma lygi 1. Tuomet
(a
11
+. . . +a
n1
)
p
1
. . . (a
1k
+. . . +a
nk
)
p
k
a
p
1
11
. . . a
p
k
1k
+. . . +a
p
1
n1
. . . a
p
k
nk
,
arba
k

j=1
_
n

i=1
a
ij
_
p
j

i=1
_
_
k

j=1
a
p
j
ij
_
_
.
Komentarai ir taikymas. Daniausiai yra taikoma forma, kai visi p
j
yra lygs, taiau
spdingiausiai nelygyb dirba, kai jie yra skirtingi. Pastebkime, kad kai k = 2, o
p
1
= p
2
=
1
2
, gauname Cauchy-Schwarz nelygyb, o ir visa Hlder nelygybs forma yra
tarsi Cauchy-Schwarz nelygybs apibendrinimas.
Teorema (Chebyshev). Jei turime aibes a
1
a
2
. . . a
n
ir b
1
b
2
. . . b
n
, tai
a
1
b
1
+a
2
b
2
+...+a
n
b
n
n

a
1
+a
2
+...+a
n
n

b
1
+b
2
+...+b
n
n

a
1
b
n
+a
2
b
n1
+...+a
n1
b
2
+a
n
b
1
n
.
Teorema (Minkowski). Jei a
1
, a
2
, . . . , a
n
ir b
1
, b
2
, . . . , b
n
teigiam skaii sekos ir
p 1, tai
_
n

i=1
(x
i
+y
i
)
p
_1
p

_
n

i=1
x
p
i
_1
p
+
_
n

i=1
y
p
i
_1
p
.
Teorema (Schur). Tarkime, kad a, b, c - neneigiami skaiiai, o r > 0. Tada
a
r
(a b)(a c) +b
r
(b a)(b c) +c
r
(c a)(c b) 0.
Lygyb galios tada ir tik tada, kai a = b = c arba du i j lygs, o treiasis lygus 0.
Komentarai ir taikymas. Kai r = 1, gausime a
3
+ b
3
+ c
3
+ 3abc a
2
(b + c) + b
2
(a +
c) +c
2
(a +b), kas yra viena daniausi Schuro nelygybs taikymo form.
74
2.1. Nelygybs Algebra
Teorema (Perstat nelygyb). Turime aibes a
1
a
2
. . . a
n
ir b
1
b
2
. . . b
n
.
Tada kiekvienai aibs 1, 2, . . . , n perstatai galios
a
1
b
1
+. . . +a
n
b
n
a
1
b
(1)
+. . . +a
n
b
(n)
a
n
b
1
+a
n1
b
2
+. . . +a
1
b
n
.
Lygybs pirmu ir antru atveju galios atitinkamai tada, kai aibs perstata bus grietai
majanti ir grietai didejanti.
Komentarai ir taikymas. rodinjant ciklines ar simetrines nelygybes, visada galima
nemainant bendrumo apsibrti, kokie yra kintamj sryiai tarpusavyje (pvz. jei
yra ciklin nelygyb nuo a, b, c, tai galime sakyti, kad a b c ar panaiai). Tai leis
suformuoti reikiamas nemajanias sekas, kurioms galiot perstat nelygyb.
Teorema (Jensen). Tegu f : A R bus ikila (angl. convex) funkcija. Tada bet ko-
kiems x
1
, x
2
, . . . , x
n
A ir neneigiamiems w
1
, w
2
, . . . , w
n
, kuri suma teigiama, galios
w
1
f(x
1
) +. . . +w
n
f(x
n
) (w
1
+. . . +w
n
)f
_
w
1
x
1
+. . . +w
n
x
n
w
1
+. . . +w
n
_
.
Kai f yra igaubta (angl. concave), galioja atvirkia nelygyb.
Komentarai ir taikymas. Funkcija intervale yra ikila, jei jos antros eils ivestin tame
intervale yra ne maiau u 0, arba igaubta, jei ne daugiau u 0. Na o praktikai t
galima pamatyti funkcijos grake: ikilos funkcijos grakas tame intervale savo forma
bus panaus funkcijos y = x
2
grak, o igaubtos - funkcijos y = x
2
grak.
Teoremos idj galime suformuluoti taip: ikilos funkcijos reikmi vidurkis yra ne
maesnis u funkcijos nuo argument vidurkio reikm. Igaubtai funkcijai, inoma,
atvirkiai. Taikant i nelygyb, daniausiai w
1
= w
2
= . . . = w
n
= 1.
75
2.2. Funkcins lygtys Algebra
2.2 Funkcins lygtys
Dauguma suprantame, k reikia isprsti lygt. Tai yra rasti visus uraytos lygy-
bs sprendinius ir rodyti, kad daugiau j nra. Isprsti funkcin lygt reikia beveik
t pat - rasti visas funkcijas, tenkinanias lygyb ir rodyti, kad daugiau toki nra.
Daugumos funkcini lygi sprendimai turi pana pobd - manipuliuojama duota
lygtimi siekiant gauti kuo daugiau apribojim tiktiniems sprendiniams. Skms at-
veju, apribojim pakanka ir galima nusakyti sprendini aib (kartais ji bna tuia)
bei patikrinti, kad ities visos rastos funkcijos yra sprendiniai. Pirmjame skyrelyje su-
paindinsime su paia pagrindine sprendimo idja - ksuot reikmi statymu vietoje
kintamj funkcinje lygtyje. Antrjame parodysime, kaip i duotos lygties gauti in-
formacijos apie funkcijos tip (pvz. lyginum, monotonikum, injektyvum), bei kaip
j pritaikyti. Treijame isprsime ymij Cauchy funkcin lygt ir panaudosime j
sprsdami sudtingesnius udavinius.
2.2.1 sistatykime x = 0
Nieko nelaukdami usiraykime pirmj funkcin lygt:
1 Pavyzdys. Raskite visas funkcijas f : R R, tenkinanias lygt
f(x +y) = f(x)
su visais realiaisiais x ir y, bei lygyb f(0) = 0.
ios funkcins lygties slyga susideda i keturi dali. Apvelkime jas:
- Iekom funkcij apibrimo ir reikmi sritys. iuo atveju duota f : R R, t.y.
sprendini reikia iekoti tarp vis funkcij apibrt realiuosiuose skaiiuose ir su
realiomis reikmmis.
- Lygtis, kuri turi tenkinti iekomos funkcijos.
- Lygtyje dalyvaujani kintamj kitimo sritys. iuo atveju duota, kad lygt funk-
cijos turi tenkinti su visomis realiomis x ir y reikmmis
- Papildomos slygos. iuo atveju duota, kad reikia iekoti tik t lygties sprendini,
kurie papildomai tenkina f(0) = 0.
Sprendimas. sistatykime x = 0, gausime f(y) = f(0) = 0, t.y. f(y) = 0 su visais
y R. Patikrin gauname, kad sprendinys tinka.
Sprendimas trumpesnis u slyg, tad su nespjusiais pastebti, kaip jis pralek,
pasiirkime sultint kartojim. Slygoje duota, kad iekomos funkcijos turi tenkin-
ti lygt f(x + y) = f(x) su visomis realiosiomis x ir y reikmmis. Vadinasi, turs
tenkinti lygt ir kai vienam i kintamj parinksime konkrei reikm, k paprastai
vardijome kaip statykime x = 0. Toliau, inodami, kad iekomos funkcijos turi su
visomis realiomis y reikmmis tenkinti lygt f(y) = f(0), bei kad iekomos funkcijos
turi tenkinti papildom slyg f(0) = 0, darome ivad, kad iekomos funkcijos turi su
76
2.2. Funkcins lygtys Algebra
visomis realiosiomis y reikmmis tenkinti f(y) = 0. Taiau i slyga yra tokia stipri,
kad ji nurodo vien vienintel funkcij! Lieka patikrinti, ar ji yra sprendinys. Kadangi
visuose realiuose takuose ji gyja reikm 0, tai stat j lygt gausime akivaizdiai
teising lygyb 0 = 0. Lygtis isprsta.
sistatyti vietoje vieno ar keli kintamoj nul yra daniausiai pasitaikanti funkcini
lygi sprendimo idja, nuo kurios neretai verta pradti sprsti nematyt lygt. Taiau
reikia turti omenyje, kad retai kada vien io triuko uteks, tad svarbu turti ir kit
ginkl. Pavyzdiui:
2 Pavyzdys. Raskite visas funkcijas f : R R, kurios tenkina lygt
f(x +y) = f(x
2
+y
2
)
su visais realiaisiais x ir y.
Sprendimas. sistatykime x = y, gausime f(2y) = f(2y
2
). sistatykime x = y,
gausime f(0) = f(2y
2
). Abi lygybs turi galioti su visomis realiosiomis y reikmmis,
tad galime jas sujungti: f(2y) = f(0). Lieka siirjus konstatuoti, kad iekomos
funkcijos visuose realiuose takuose gis t pai reikm kaip ir take 0. Toki funkcij
be galo daug, ir jos prastai uraomos f(x) = c, kur c - bet kuris i realij skaii (dar
vadinamas konstanta). Lieka patikrinti, ar visos tokios funkcijos tinka. stat gausime
c = c, vadinasi tinka.
Naudodami ias paprastas nulio ir x = y sistatymo idjas isprskime dar kelet
lygi. Atkreipsime dmes tai, kad labai svarbi dalis yra teisingai interpretuoti gaut
po sistatymo lygyb. Kartais ji bna bevert, o kartais sujungus su kakuo papildomu
galima gauti k nors naudingo. Sunkesniuose udaviniuose tai ne visuomet pavyksta,
tad verta apsiarvuoti kantrybe ir bandyti sistatyti vairias kintamj reikmi kom-
binacijas.
3 Pavyzdys. [LitKo 2008] Raskite visas tokias realisias funkcijas f, kad f(x)f(y)
f(xy) = x +y su visomis realij skaii x ir y poromis.
Sprendimas. sistatykime x = 0 ir y = 0. Gausime f(0)
2
= f(0), t.y. f(0) = 0 arba
f(0) = 1. Panagrinkime abu atvejus:
f(0) = 0 - sistatykime pradin lygt x = 0, gausime 0 = y. i lygyb jokiai funkcijai
negalioja su visomis realiomis y reikmmis, todl atvej atmetame.
f(0) = 1 - sistatykime pradin lygti x = 0, gausime f(y) = y +1. Patikrin matome,
kad i funkcija tinka: (x + 1)(y + 1) (xy + 1) = x +y.
Gavome, kad funkcija f(x) = x + 1 bus vienintlis sprendinys.
4 Pavyzdys. Raskite visas funkcijas f : R R
0
1
tenkinanias lygyb
f(2u) = f(u +v)f(v u) +f(u v)f(u v)
su visomis realiomis u ir v reikmmis.
1
R
0
ymsime visus neneigiamus realiuosius, o R
+
visus teigiamus realiuosius skaiius.
77
2.2. Funkcins lygtys Algebra
Sprendimas. sistatykime u = 0 ir v = 0. Gausime f(0) = 2f(0)
2
, t.y. f(0) = 0 arba
f(0) =
1
2
. Panagrinkime abu atvejus:
f(0) = 0 - sistatykime u = 0, gausime 0 = f(v)
2
+f(v)
2
. i lygt tenkina vienintl
funkcija - f(v) = 0.
f(0) =
1
2
- Vien kart sistatykime u = 0, kit u = v, gausime dvi lygtis:
1
2
= f(v)
2
+
f(v)
2
ir f(2v) = f(2v). I j seka, kad
1
2
= f(v)
2
+f(v)
2
= 2f(v)
2
ir, kadangi
iekome funkcij gyjani tik neneigiamas reikmes, f(v) =
1
2
.
Patikrin matome, kad abi rastos funkcijos f(v) = 0 ir f(v) =
1
2
tinka.
5 Pavyzdys. Raskite visas funkcijas f : R R, kurios su visais realiaisiais x ir y
tenkina f(x +f(y)) = x +f(f(y)) ir f(2004) = 2005.
Sprendimas. statykime y = 0, gausime f(x + f(0)) = x + f(f(0)). siirjus gaut
lygyb tampa aiku, kad j tenkina tiktai funkcijos f(x) = x + c, kur c - konstanta.
Pridjus papildom slyga lieka vienintel funkcija f(x) = x+1, kuri ir yra sprendinys.

Pasiaikinkime kiek isamiau, kaip i lygybs f(x+f(0)) = x+f(f(0)) gauti f(x) =


x + c. Paimkime bet kuri funkcij, kuri tenkina pirmj lygt. Kad ir kokia ji bt,
f(0) ir f(f(0)) bus konrets skaiiai, nepriklausantys nuo x. Patogumo dlei pakeiskime
t = x + f(0), tuomet gausime, kad su visomis realiosiomis t reikmmis f(t) = t +
f(f(0)) f(0). Lieka tik konkret skaii f(f(0)) f(0) paymti c. i nekintani
reikini paymjimo idja yra gana dana, tad verta j sidmti.
Udaviniai
1. Raskite visas funkcijas f : R R, kurios su visais realiaisiais x, y tenkina S
f(x +y) +f(x y) = 2x
2
+ 2y
2
.
2. Raskite visas funkcijas f : R R, kurios su visais realiaisiais x, y tenkina S
f(x) +f(x +y) = y + 2.
3. Raskite visas funkcijas f : R R, kurios su visais realiaisiais x, y tenkina S
f(x) = (x y)f((x y + 1)x) +f(y).
4. Raskite visas funkcijas f : R R, kurios su visais realiaisiais x ir y tenkina S
yf(x) = xf(y).
5. Raskite visas funkcijas f : R R su visais realiaisiais x ir y tenkinanias lygt S
f(x +f(y)) = f(x) +yf(x).
78
2.2. Funkcins lygtys Algebra
6. Raskite visas funkcijas f : R R su visais x R tenkinanias lygt S
xf(x) +f(x) + 1 = 0.
7. Raskite visas funkcijas f : R R su visais x R, x ,= 0, 1 tenkinanias lygt S
f(x) +f(
x 1
x
) = 1 x.
8. Raskite visas funkcijas f : R R, kurios su visais realiaisiais x, t, z tenkina S
(x +t)f(z) = f(xz) +f(tz).
9. [LitMo 2000, Pan African 2003] Raskite funkcijas f : R R su visomis realio- S
siomis x,y reikmmis tenkinanias lygt
(x +y)(f(x) f(y)) = f(x
2
) f(y
2
).
10. Raskite visas funkcijas f : R R, kurios su visais realiaisiais x, t tenkina S
f(x)f(t) = f(x) +f(t) +xt 1.
11. [LitMo 1994] Ar egzistuoja bent viena funkcija f : R R tenkinanti lygt S
f(f(x)) = x
3
?
12. Raskite visas funkcijas f : R R, kurios su visais realiaisiais x, y tenkina S
f(f(x y)) = f(x) f(y) f(x)f(y) xy.
13. Raskite visas funkcijas f : R R, kurios su visais realiaisiais x, y tenkina S
(x y)
2
f(x +y) = (x +y)
2
f(x y).
14. Raskite visas funkcijas f : R R, kurios su visais realiaisiais x, y tenkina S
f(x +f(y)) = f(f(x)) +y.
15. Raskite visas funkcijas f : R R, kurios tenkina f(1) = 1 ir su visais realiaisiais S
x, y
f(x +y) = 3
y
f(x) + 2
x
f(y)
16. [LitMo 2008] Funkcija f(x) apibrta teigiamiems skaiiams, gyja teigiamsias S
reikmes ir su visais teigiamais x, y tenkina lygyb
f(x)f(y) = f(xy) +f(
x
y
).
a.) Nurodykite bent tris tokias funkcijas.
b.) rodykite, kad f(x) 2, f(1) = 2.
c.) rodykite, kad jei f(x) tenkina slyg, tai j tenkina ir funkcija f
2
(x) 2.
79
2.2. Funkcins lygtys Algebra
17. Raskite visas funkcijas f : R
+
R
+
su visais teigiamais x ir y tenkinanias S
lygt
f(xy) = f(x +y).
18. Raskite visas funkcijas f : R R, kurios su visais realiaisiais x ir y, nelygiais S
nuliui, tenkina
f(x +y) = f(1/x + 1/y).
19. [Brazil 1993] Raskite bent vien funkcij f : R R su kiekvienu x R tenki- S
nani
f(0) = 0 ir f(2x + 1) = 3f(x) + 5.
20. Raskite visas funkcijas f : R R, kurios su visais realiaisiais x, y tenkina S
f(x
3
) f(y
3
) = (x
2
+xy +y
2
)(f(x) f(y))
21. Raskite visas funkcijas f : R R tenkinanias lygyb f(x)
2
= 1 su kiekvienu S
x R .
80
2.2. Funkcins lygtys Algebra
2.2.2 Funkcij tipai
ioje uduotyje panagrinsime varius funkcij tipus, sutinkamus sprendiant funkci-
nes lygtis. Greiiausiai jau yra tek girdti, kas yra lygin, nelygin, monotonin ar
periodin funkcija, tad per daug nesiplsdami prisiminkime tikslius apibrimus.
Apibrimas. Funkcij f : A B, kur aib A simetrin nulio atvilgiu, vadinsime
lygine, jei x A teisinga f(x) = f(x), ir nelygine, jei x A teisinga f(x) =
f(x).
Apibrimas. Funkcij f : A B vadinsime periodine, jei egzistuoja toks a A, kad
f(a +x) = f(x) x A.
Apibrimas. Funkcij f : A B vadinsime monotonine, jei ji yra arba nedidjanti,
arba nemajanti, t.y. arba f(x) f(y) su visais x > y (x, y A), arba f(x) f(y)
su visais x > y (x, y A).
Atkreipsime dmes, kad didjanti funkcija daniausiai reikia nemajanti (ir at-
virkiai majanti - nedidjanti), todl yra vartojami terminai grietai didjanti ir
grietai majanti, norint pabrti, jog funkcija negali bti pastovi.
Vos prisimin, lyginum, nelyginum, periodikum ir monotonikum i karto pa-
liksime nuoalyje ir pereisime prie injektyvi ir surjektyvi funkcij nagrinjimo. Vargu
ar suklysime teigdami, kad ios dvi svokos yra centrins sprendiant kiek sudtingesnes
olimpiadose sutinkamas funkcines lygtis, tad joms skirsime labai daug dmesio.
Injektyvumas ir surjektyvumas
Funkcij vadinsime injektyvia, jei ji kiekvien reikm gyja tik vien kart. Danai
sutinkamos injektyvios funkcijos yra tiess f(x) = ax + b, a ,= 0 (ypa f(x) = x ir
f(x) = x), bet nesunku rasti ir daugiau pavyzdi: f(x) = x
3
, f(x) =
1
x
, f(x) = e
x
.
Elementariausias neinjektyvios funkcijos pavyzdys - f(x) = x
2
. I ties - ji, pavyzdiui,
reikm 1 gyja du kartus: f(1) = f(1) = 1.
Atkreipsime dmes, kad nagrinjant injektyvum yra svarbi apibrimo sritis. Pa-
vyzdiui, nors f(x) = x
2
ir nra injektyvi visoje realij tiesje, ji tokia tampa apribojus
apibrimo srit iki neneigiam skaii.
Pateiksime formal apibrim, kur, kaip pamatysime, labai patogu tiesiogiai tai-
kyti sprendiant funkcines lygtis:
Apibrimas. Funkcij f : A B vadinsime injektyvia, jei visiems a, b A teisinga
f(a) = f(b) a = b.
Antroji svoka - surjektyvumas - apibdina funkcijas, kurios gyja visas savo reik-
mi srities reikmes. Jei nagrinsime funkcijas f : R R, tai surjektyviomis bus tos
paios tiess f(x) = ax+b, a ,= 0, arba, pavyzdiui, visi nelyginio laipsnio daugianariai.
Nesurjektyvios bus pavyzdiui f(x) = x
2
ir f(x) = e
x
, nes negyja neigiam reikmi.
Vlgi, apribojus reikmi srit nesurjektyvi funkcija gali tapti surjektyvia, tad visada
reikia aikiai suprasti, kas tiksliai yra apibrimo ir kas yra reikmi sritis kiekvienu
atveju ir po kiekvieno pertvarkymo.
Formalus surjektyvumo apibrimas:
81
2.2. Funkcins lygtys Algebra
Apibrimas. Funkcij f : A B vadinsime surjektyvia, jei kiekvienam b B
egzistuoja toks a A, kad f(a) = b.
Funkcij, kuri yra ir injektyvi, ir surjektyvi, vadinsime bijektyvia. Bijektyvi funkcija,
arba tiesiog bijekcija, kiekvienam apibrimo srities elementui priskiria unikal reikmi
srities element, ir kiekvienas reikmi srities elementas yra priskirtas. Kaip jau inote
(arba jei ne, tai nesunku suvokti), bijektyvi funkcija turi atvirktin.
Panaudojimas
Panagrinkime kelet situacij darydami prielaid, kad iekoma funkcija yra injektyvi
arba surjektyvi.
Pavyzdys. Raskite visas funkcijas f : R R, tenkinanias lygt
f(f(x)) = f(x).
i lygtis turi be galo daug sprendini, kuri struktra kiek komplikuota. Galite
pabandyti juos rasti.
Kas pasikeist, jei inotume, kad iekoma funkcija yra injektyvi? Pairkime - jei
funkcija injektyvi, tai i f(a) = f(b) seka, kad a = b su visais a, b. iuo atveju vietoje
a stovi f(x), o vietoje b stovi x, todl i f(f(x)) = f(x) sekt f(x) = x su visais x -
lygtis isprsta!
Kas atsitikt, jei inotume, kad ms iekoma funkcija yra surjektyvi? Surjektyvi
funkcija gyja visas reikmi srities reikmes, iuo atveju visus realiuosius skaiius. Jei
f(x) galt bti bet koks realus skaiius, tai tuomet paymj f(x) = y gautume
f(y) = y su visais realiaisiais y - lygtis isprsta!
Pavyzdys. Raskite visas funkcijas f : R R, tenkinanias lygt
f(x +f(y)) = f(f(x) +y).
Jei inotume, kad funkcija yra injektyvi, i karto gautume f(x +f(y)) = f(f(x) +
y) x + f(y) = f(x) + y, o toki lygt jau sprsti mokame. Utenka sistatyti,
pavyzdiui, y = 0 ir gauti f(x) = x +c, kur c bet koks realus skaiius.
Pavyzdys. Raskite visas funkcijas f : R R, tenkinanias lygt
f(x +f(y)) = f(y
2
+x
2
f(y)) +xf(x).
Jei inotume, kad iekoma funkcija injektyvi, utekt sistatyti x = 0 ir i lygties
f(f(y)) = f(y
2
) gauti, kad f(y) = y
2
su visais y R. Patikrin pastebtume, kad i
funkcija netinka, vadinasi sprendini nebt.
Pavyzdys. Funkcija f : R R tenkina lygt f(f(x) +x) = x. Raskite f(0).
Jei inotume, kad f yra surjektyvi funkcija, tai reikt, kad egzistuoja toks a, kad
f(a) = 0 (kitaip sakant - nulis yra gyjamas). stat x = a, gautume f(f(a) +a) = a
f(0 +a) = a 0 = a, vadinasi a = 0, t.y. f(0) = 0.
udavin galima buvo isprsti ir kitaip - sistaius x = 0 bei x = f(0), taiau
idja, kuria pasinaudojome, yra daug bendresn ir neretai labai naudinga.
82
2.2. Funkcins lygtys Algebra
Gavimas
Pamaius, kad kartais injektyvumas ir surjektyvumas tikrai yra naudingi, kyla klausi-
mas, kaip gauti, jog iekomos funkcijos pasiymt itomis savybmis. Pabandykime
pasiaikinti.
Pavyzdys. Funkcija f : R R tenkina lygt f(f(x)) = x. rodykite, kad ji yra
injektyvi ir surjektyvi.
Injektyvumas. Mums reikia rodyti, kad jei f(a) = f(b), tai a = b. Pasirodo, tai
visai nesudtinga. Jei f(a) = f(b), tai ir f(f(a)) = f(f(b)) (funkcija nuo vienod
argument tikrai duoda vienodas reikmes), bet kadangi f(f(a)) = a ir f(f(b)) = b,
tai aiku, kad a = b.
Surjektyvumas. Mums reikia rodyti, kad kiekvienam a egzistuoja toks b, kad f(b) =
a. Bet pairkime lygt dar kart - jei statysime x = a gausime f(f(a)) = a, t.y. f
nuo kako lygu a, vadinasi reikm a yra gyjama. iuo atvju, inoma, iekomas b bus
lygus f(a), bet daniausiai mums jis nelabai domus - pakanka inoti, kad egzistuoja.
Pavyzdys. Funkcija f : R R tenkina lygt f(x +f(y)) = f(x) +y. rodykite, kad ji
yra injektyvi ir surjektyvi.
Injektyvumas. Mums reikia rodyti, kad jei f(a) = f(b), tai a = b. Pasinaudosime
laisvu kintamuoju y: perra lygt y = f(x + f(y)) f(x) ir vietoje y paeiliui stat a
ir b gauname, kad deiniosios puss bus vienodos (nes f(a) = f(b)), todl vienodomis
turs bti ir kairiosios.
Surjektyvumas. Mums rekia rodyti, kad funkcija gyja visas reikmes. Laisvas
kintamasis y ia taip pat pravers, nes jis gali gyti bet koki reikm. I pairos lyg
ir trukdo f(x), bet lengvai galime j apeiti - stat x = 0 gausime f(f(y)) = f(0) + y.
Kadangi f(0) yra skaiius, o y gyja visas reikmes, tai ir f(0) +y gyja visas reikmes.
I ia jau aiku, kad ir funkcija jas visas gis.
Pavyzdys. Funkcijos f, g : R R tenkina lygt f(g(x)) = x. rodykite, kad g yra
injektyvi, o f surjektyvi.
Jei dvi funkcijos vienoje lygtyje neigsdina, tai sprendimas akivaizdus. Injektyvu-
mas - jei g(a) = g(b), tai ir f(g(a)) = f(g(b)) a = b. Surjektyvumas dar paprastesnis,
mat kiekvienam a teisinga f(g(a)) = a, taigi f reikm a gyja.
Taigi, bendru atveju, strategija paprasta. Nordami rodyti iekomos funkcijos in-
jektyvum tariame, kad f(a) = f(b), ir statoms a ir b lygt, tikdamiesi kokiu nors
bdu gauti a = b. Nordami rodyti surjektyvum bandome gauti f nuo bet kokio argu-
mento lygi reikiniui, kuris gali gyti visas reikmes. Abi strategijos yra gana bendros
ir atskiru atveju jas pritaikyti gali bti gana sudtinga, tad nusiteikite pakovoti dl i
nauding funkcijos savybi.
Pavyzdiai
6 Pavyzdys. Raskite visas funkcijas f : R R su visomis x ir y reikmmis tenki-
nanias
f(f(x) +y) = 2x +f(f(y) x).
83
2.2. Funkcins lygtys Algebra
Sprendimas. statykime y = f(x), gausime, kad su visais x teisinga f(0) 2x =
f(f(y) x), vadinasi, funkcija surjektyvi. rodysime, kad funkcija yra ir injektyvi. Jei
ji tokia nra, tai egzistuoja tokie a, b, kad f(a) = f(b) ir a ,= b. statykime y = a ir
y = b, gausime
f(f(x) +a) = 2x +f(f(a) x),
f(f(x) +b) = 2x +f(f(b) x)
ir i ia
f(f(x) +a) = f(f(x) +b).
Kandangi funkcija surjektyvi, tai gauname
f(x +a) = f(x +b) f(x) = f(x + (b a)).
Paymj b a = r gauname, kad funkcija periodin su periodu r ,= 0. Taiau stat
x = y = r pradin lygt, gauname f(f(r)) = 2r+f(f(r)) r = 0, prietara. Gavome,
kad funkcija turi bti injektyvi, ir stat x = 0 gauname f(y) = y +c.
7 Pavyzdys. Raskite visas funkcijas f : R R su visais realiaisiais x, y tenkinanias
lygt
f(xy +f(x)) = xf(y) +f(x).
Sprendimas. Pabandykime rodyti, kad funkcija injektyvi. Tam pasinaudosime labai
elegantika idja - sukeisime vietomis kintamuosius:
f(xy +f(y)) = yf(x) +f(y).
Jei tarsime, kad f(x) = f(y), tai gautos ir pradins lygi kairiosios puss bus lygios,
vadinasi, turs bti lygios ir deiniosios:
xf(y) +f(x) = yf(x) +f(y) = f(y)(x 1) = f(x)(y 1).
I ia gauname, kad funkcija visas reikmes gyja po vien kart, iskyrus, galbt, nul
(nes jei f(x) ,= 0, tai f(x) = f(y) = x = y).
Natralus sprendimo tsinys, patyrinti, kas atsitinka, kai funkcija gyja nul keliuo-
se takuose, tad tarkime, kad f(x
0
) = 0 ir x
0
,= 0. sistatykime x = x
0
, y = 1, gausime
f(1) = 0. stat y = 1, gausime f(x+f(x)) = f(x). Jei kokiam nors take f(x) ,= 0, tai,
kaip jau inome, tame take funkcija yra injektyvi, bet tada x+f(x) = x = f(x) = 0
- prietara. Vadinasi, jei funkcija gyja reikm 0 ne tik nulyje, tai ji tapaiai lygi nuliui.
Liko inagrinti atvej, kai funkcija nul gyja tik nulyje. Tuomet inome, kad funk-
cija injektyvi. stat x = 0 gauname f(f(0)) = f(0) = f(0) = 0, stat y = 0
gauname f(f(x)) = f(x) = f(x) = x.
Udaviniai
1. rodykite, kad grietai didjanti funkcija yra injektyvi. Ar btinai bijektyvi S
funkcija turi bti monotonika?
84
2.2. Funkcins lygtys Algebra
2. Ar funkcija f : R R tenkinanti lygt f(x+y) = f(x
2
)+f(y
2
) su visais x, y R S
gali bti injektyvi?
3. rodykite, kad funkcija f : R R tenkinanti lygt f(x + y) = xf(y
2
) + yf(x
2
) S
yra nelygin.
4. Raskite visas lygines monotonines ir lygines injektyvias funkcijas. Raskite bent S
vien lygin surjektyvi funkcij.
5. inome, kad f : R
+
R
+
tenkina f(x) 1 su visais x R
+
ir f(x+y)f
2
(y) = S
f(x). rodykite, kad f didjanti. (R
+
ia ir toliau ymi teigiamus realiuosius)
6. Funkcija f : R R tenkina lygt f(xf(x)) = x ir yra surjektyvi. Raskite f(1). S
7. Raskite visas didjanias funkcijas f : R R tenkinanias lygt f(f(x)) = x. S
8. Tegu f : R R yra injektyvi ir su visais x tenkina S
f(x)f(1 x) = f(ax +b).
rodykite, kad a = 0, f(1 b) = 1 ir kad f nra surjektyvi.
9. Raskite visas grietai didjanias funkcijas f : R R su visais x, y R tenki- S
nanias lygyb
f(x +f(y)) = f(x +y) + 2005.
10. Raskite visas funkcijas f : R
+
R
+
su visais x, y R
+
tenkinanias lygyb S
(x +y)f(f(x)y) = x
2
f(f(x) +f(y)).
11. Raskite visas funkcijas f : R
+
R
+
su visais x, y R
+
tenkinanias lygyb S
(x +y)f(yf(x)) = x
2
(f(x) +f(y)).
12. Raskite visas funkcijas f, g : R R, kur g yra bijekcija ir kurios su visais S
realiaisiais x, y tenkina
f(g(x) +y) = g(f(y) +x).
13. Raskite visas funkcijas f : R R su visais x, y R tenkinanias lygyb S
f(x +y +f(xy)) = f(f(x +y)) +xy.
14. rodykite, kad nra funkcij f, g : R R su visais realiaisiais x tenkinani S
lygybes
g(f(x)) = x
3
ir f(g(x)) = x
2
.
15. Tegu funkcija f : R R su visais realiaisiais x tenkina lygt S
4f(f(x)) = 2f(x) +x
rodykite, kad f(x) = 0 tada ir tik tada, kai x = 0.
85
2.2. Funkcins lygtys Algebra
16. Raskite visas funkcijas f : R
+
R
+
su visais x, y R
+
tenkinanias lygyb S
f(x)f(yf(x)) = f(x +y).
17. Raskite visas funkcijas f : R
+
R
+
tenkinanias lygyb S
f(x +yf(x)) = f(x)f(y), x, y R
+
,
jei inome, jog egzistuoja tik baigtinis skaiius toki x R
+
, kad f(x) = 1.
18. Raskite visas funkcijas f : R R su visais realiaisiais x, y tenkinanias lygt S
f(y) +f(x +f(y)) = y +f(f(x) +f(f(y))).
19. Raskite visas funkcijas f : R R su visais realiaisiais x, y tenkinanias lygt S
f(f
2
(x) +f(y)) = xf(x) +y.
20. Raskite visas funkcijas f : R R su visais realiaisiais x, y tenkinanias lygt S
f(xf(x) +f(y)) = f
2
(x) +y.
21. Raskite visas funkcijas f, g, h : R R, kurios su visais realiaisiais x, y, z tenkina S
f(h(g(x)) +y) +g(z +f(y)) = h(y) +g(y +f(z)) +x.
22. Raskite visas funkcijas f : R R su visais realiaisiais x, y tenkinanias lygt S
f(x
2
+xy +f(y)) = f
2
(x) +xf(y) +y.
23. Raskite visas funkcijas f : R R su visais realiaisiais x, y tenkinanias lygt S
f(f(x) f(y)) = (x y)
2
f(x +y).
24. Raskite visas funkcijas f, g : R R, kurios su visais realiaisiais x, y tenkina S
f(xg(y + 1)) +y = xf(y) +f(x +g(y))
ir
f(0) +g(0) = 0.
25. Raskite visas funkcijas f : R R, kurios su visais realiaisiais x, y tenkina S
f(x
2
+f(y)) = y +xf(x).
26. [IMO 1992] Raskite visas funkcijas f : R R, kurios su visais realiaisiais x, y S
tenkina
f(x
2
+f(y)) = y +f
2
(x).
86
2.2. Funkcins lygtys Algebra
27. Raskite visas funkcijas f : R R su visomis x ir y reikmmis tenkinanias S
f(x +f(xy)) = f(x +f(x)f(y)) = f(x) +xf(y).
28. Raskite visas funkcijas f : R R, kurios su visais realiaisiais x, y tenkina S
f(xf(y)) +f(yf(x)) = 2xy.
29. *Raskite visas funkcijas f : R R, kurios su visais realiaisiais x, y tenkina
f(xf(y)) = (1 y)f(xy) +x
2
y
2
f(y).
30. *Raskite visas funkcijas f : R
+
R
+
, kurios su visais realiaisiais x, y tenkina
f(x +f(y)) = f(x +y) +f(y).
31. *[Japan 2008] Raskite visas funkcijas f : R R su visais realiaisiais x, y tenki-
nanias lygt
f(x +y)f(f(x) y) = xf(x) yf(y).
32. *Raskite visas funkcijas f : R R su visais realiaisiais x, y tenkinanias lygt
f(x +y +f(xy)) = xy +f(x +y).
33. *[Brazil 2006] Raskite visas funkcijas f : R R, kurios su visais realiaisiais x, y
tenkina
f(xf(y) +f(x)) = 2f(x) +xy.
34. *[Dan Barbilian 2005] Tegu f : R
+
R
+
yra nelygi konstantai funkcija su visais
x, y, z R
+
tenkinanti
f(x)f(yf(x))f(zf(x +y)) = f(x +y +z)
rodykite, kad f yra injektyvi ir raskite visas tokias funkcijas.
35. *Raskite visas funkcijas f : R
+
R
+
, kurios su visais teigiamais x, y tenkina
f
_
f(x)
yf(x) + 1
_
=
x
xf(y) + 1
.
87
2.2. Funkcins lygtys Algebra
2.2.3 Cauchy funkcin lygtis
Sprendiant sudtingas funkcines lygtis danai susiduriama su lygtimi:
f(x +y) = f(x) +f(y).
i lygtis vadinama Cauchy funkcine lygtimi. J nesudtinga isprsti jei iekosime
funkcij, kuri apibrimo sritis racionalieji skaiiai. T ir padarykime:
Teorema. Jei f : Q R tenkina lygt f(x+y) = f(x) +f(y) su visais racionaliaisiais
x ir y, tai f - tiesin, t.y. f(q) = kq visiems q Q, kur k R - konstanta.
rodymas. stat y = x, gausime f(2x) = 2f(x). stat y = 2x, gausime f(3x) = 3f(x).
Taip tsdami toliau, po nesudtingos indukcijos tursime
f(nx) = nf(x).
i lygyb stat x =
1
n
gausime
f(1)
n
= f(
1
n
). Tada, pradinje lygtyje imdami x =
1
n
,
o y =
1
n
,
2
n
,
3
n
ir t.t., vl po paprastos indukcijos ireikime:
f(
m
n
) = mf(
1
n
) = f(1)
m
n
,
kur m ir n - bet kokie natralieji skaiiai, vadinasi,
m
n
- bet koks teigiamas racionalusis.
Tada, paymj f(1) = k, gausime
f(q) = kq,
kur k - realioji konstanta, o q - bet koks teigiamas racionalusis. Kita vertus, pradinje
lygtyje pam y = 0 ir y = x, gausime f(0) = 0 ir f(x) = f(x) x R, taigi,
f(q) = kq bus lygties sprendinys ir neigiamiems racionaliesiems.
Deja, jei pradin salyg f : Q R pakeisime f : R R, tai Cauchy funkci-
n lygt isprsti pasidarys labai sudtinga. Racionaliesiems skaiiams ir toliau galios
f(q) = kq, tad bt visai natralu manyti, kad f(x) = kx visiem realiesiems x, ta-
iau rodyta, kad egzistuoja begalyb labai neelementari, netiesini sprendini. J
egzistencij priimsime be rodymo ir vilgtelsime labai svarbi i sprendini savyb:
Teorema. Tarkime, turime funkcij f : R R, kuri tenkina Cauchy funkcin lygt ir
f(q) = q visiems q Q, o kakokiam R : f() ,= . Duoti trys skaiiai x, y, r Q,
r > 0, x ,= y. Jei (x, y) paymsime apskritimo centro kordinates, o r - jo spindul, tai
nesvarbu, kokius x, y, r parinksime, tame apskritime visados galsime rasti funkcijos f
grako tak.
rodymas. Tarkime, kad f() = + , ,= 0. Paymkime =
yx

. Aiku, kad
manoma pasirinkti tok racional skaii b ,= 0, kad: [ b[ <
r
2||
, ir tok racional
skaii a, kad: [ a[ <
r
2|b|
. Paymkime X = x +b( a). Tada
f(X) = f(x +b( a))
= x +bf() bf(a)
= y +b( +) ba
= y +b( a) ( b).
88
2.2. Funkcins lygtys Algebra
Aiku, kad x r < X < x + r ir y r < f(X) < y + r, todl takas (X, f(X)) bus
ms apskritimo viduje.
Pastaba. Nors teorem rodme tik atveju, kai f(q) = q visiems q Q, nesunku siti-
kinti, kad teorema galios ir bendru atveju, kai f(q) = kq.
Jei sugebtume nupieti Cauchy lygties netiesinio sprendinio grak, tokio grako
tak galtume rasti, kur tik sugalvotume, visoje begalinje ploktumoje - ities labai
avu ir grau, bet taip pat aiku, kad rimtai sprendiant udavinius, geriau su iais
sprendiniais neprasidti. Jei turime funkcij i realij realiuosius ir lygtis susiveda
Cauchy lygt, reikia iekoti kakoki papildom slyg, kurios leist atmesti aviuo-
sius Cauchy lygties sprendinius.
Papildomos slygos
Naudodamiesi paskutinija teorema nesunkiai galime sugalvoti kelet slyg, leisiani
atmesti imantriuosius netiesinius sprendinius. Tarkime, f : R R - funkcija visiems
realiesiems tenkinanti Cauchy funkcin lygt. Tada:
Teorema. Jei egzistuoja intervalas (a, b), kuriame funkcija f aprta (t.y. f(x) > m
arba f(x) < M su visomis x (a, b) reikmmis, m, M - konstantos), tai f - tiesin.
rodymas. I tikrj, i antrosios teoremos seka, kad jei f - netiesin, tai ji gali bet
kuriame intervale gyti reikm i bet kokio ms norimo intervalo, vadinasi, jei f yra
apribota kakokiame intervale, tai ji gali bti tik tiesin.
Teorema. Jei egzistuoja intervalas, kuriame f yra monotonin, tai f - tiesin.
rodymas. Jei f - monotonin kakokiame intervale (jei intervalas neudaras, tai galime
paimti koki nors jo udar dal), tai tame intervale ji bus ir aprta - egzistuos jos
maksimumas arba minimumas, taigi, ji gali bti tik tiesin.
Teorema. Jei egzistuoja intervalas, kuriame f yra tolydi, tai f - tiesin.
rodymas. Jei f - tolydi kakokiame intervale (jei intervalas neudaras, tai galime paimti
koki nors jo udar dal), tai tame intervale ji ir aprta, taigi, ji gali bti tik tiesin.
Trys pastarosios teoremos - klasikiniai, gerai inomi faktai. Naudojant jas kokioje
nors rimtoje olimpiadoje rodyti j nebtina.
Pavyzdiai
8 Pavyzdys. Raskite visas funkcijas f : Q R, kurios su visomis racionalij skaii
poromis x ir y tenkina f(x +y) = f(x) +f(y) +xy(x +y).
Sprendimas. Pakeiskime - f(x) = g(x) +
x
3
3
. stat pradin lygt gausime g(x +y) =
g(x) +g(y), t.y. Cauchy funkcin lygt racionaliesiems skaiiams. Gauname g(x) = kx,
kur k - kakokia realioji konstanta, o tada f(x) = kx +
x
3
3
.
9 Pavyzdys. Raskite visas funkcijas f : R R, kurios su visomis realij skaii
poromis x ir y tenkina f(x+y) = f(x) +f(y), ir su visais x ,= 0 tenkina f(x)f(
1
x
) = 1.
89
2.2. Funkcins lygtys Algebra
Sprendimas. Turime Cauchy funkcin lygt realiesiems skaiiams, taigi, i duotosios
slygos f(x)f(
1
x
) = 1 reikia ipeti k nors naudingo. I ios slygos iplaukia, kad f(x)
ir f(
1
x
) yra vienodo enklo, t.y. abu neigiami arba teigiami. stat Cauchy lygti y =
1
x
gausime:
[f(x +
1
x
)[ = [f(x)[ +[f(
1
x
)[ 2
_
[f(x)[ [f(
1
x
)[ = 2.
Reikinys x +
1
x
, keiiant x, gauna bet koki reikm i intervalo [2, +), vadinasi
intervale [2, +) f(x) 2, arba f(x) 2. Gavome, kad funkcija iame intervale yra
savotikai aprta (negauna reikmi i intervalo (2, 2)), tad galime atmesti netiesi-
nius Cauchy lygties sprendinius. Belieka antr slyg statyti f(x) = kx. Gausime
k = 1 arba k = 1, ir, nesunku patikrinti, kad sprendiniai f(x) = x ir f(x) = x tiks.

10 Pavyzdys. Raskite visas funkcijas f : R R, kurios su visomis realij skaii


poromis x ir y tenkina f(xy) = f(x)f(y) ir f(x +y) = f(x) +f(y).
Sprendimas. Pirmoje lygtyje pakeit y = x gausime, kad f(x
2
) = f(x)
2
, vadinasi,
visiems neneigiamiems x, f(x) 0 ir intervale [0, +) funkcija yra aprta. Tada
f(x) = kx. Patikrin randame, kad tiks tik k = 1.
11 Pavyzdys. Raskite visas funkcijas f : R R, kurios su visomis realij skaii
poromis x ir y tenkina f(xy) = f(x)f(y) ir intervale (0, +) yra monotonikos.
Sprendimas. Statykime x = y = 0, gausime f(0) = 0, arba f(0) = 1. Jei f(0) = 1, tai
sistat x = 0 gausime f(x) = 1 visiems x, tad nagrinkime atvej, kai f(0) = 0.
Tarkime, kad egzistuoja z ,= 0, toks, kad f(z) = 0. Tada pradinje lygtyje pam
x =
x
z
ir y = z gausime f(y) = 0 visiems y.
Belieka inagrinti atvej, kai f(0) = 0 ir su jokia kita reikme funkcija nelygi nuliui.
Pradinje lygtyje stat y = x gausime, kad f(x
2
) = f(x)
2
, arba f(x) > 0, kai x > 0.
Vadinasi teigiamiems x, y galios:
ln f(xy) = ln f(x)f(y) = ln f(x) + ln f(y).
Paymj ln f(x) = g(x), gausime g(xy) = g(x) + g(y). Aiku, kad ir funkcija g yra
monotonika. Kintamieji x ir y teigiami, taigi galime pakeisti x = e
x
, y = e
y
. Gausime
g(e
x+y
) = g(e
x
) +g(e
y
).
Paymj dar kart h(x) = g(e
x
), gausime, kad h - monotonin ir jai galioja
h(x +y) = h(x) +h(y),
taigi h(x) = kx. Lieka grti atgal - g(e
x
) = kx, kur pakeit x = ln x, gausime g(x) =
k ln x = ln x
k
. Vadinasi, ln f(x) = ln x
k
, arba f(x) = x
k
, kur k - kakoks realusis, o x -
teigiamas.
Lieka rasti tik reikmes neigiamiems skaiiams. Statykime pagrindin lygt x =
y = 1, gausime f(1) = 1, arba f(1) = 1. Tuomet sistat lygt y = 1
90
2.2. Funkcins lygtys Algebra
gausime f(x) = f(x), arba f(x) = f(x). Pirmu atveju neigiamiems x gausime
f(x) = x[x[
k1
, antruoju: f(x) = [x[
k
. Taigi, visus sprendinius galime urayti taip:
f(x) = 0, f(x) =
_

_
x
k
, x > 0,
0, x = 0,
[x[
k
, x < 0;
f(x) =
_

_
x
k
, x > 0,
0, x = 0,
x[x[
k1
, x < 0.

Prie io pavyzdio galtume paminti dar dvi danai pasitaikanias paprastesnes,


vadinamsias "Cauchy tipo" lygtis - t(x + y) = t(x)t(y) ir z(xy) = z(x) + z(y). Turint
atitinkamus apribojimus (tolydumas, monotonikumas (aprtumas netiks, nes darant
ketinius jis dingsta)) j sprendiniai yra atitinkamai t(x) = a
x
ir z(x) = log
a
x, ir spren-
diamos jos analogikais keitiniais.
Udaviniai
1. Raskite visas funkcijas f : Q R, kurios su visomis racionalij skaii poromis S
x ir y tenkina f(x +y) = f(x) +f(y) + 2xy.
2. Raskite visas funkcijas f : Q R, kurios su visomis racionalij skaii poromis S
x ir y tenkina f(x +f(y)) = f(x + 1) +y.
3. Raskite visus tolydi funkcij f, g, h : R R trejetus, kurie su visomis realij S
skaii poromis x ir y tenkina f(x +y) = g(x) +h(y).
4. Raskite visas funkcijas f : R R, kurios su visomis realij skaii poromis x S
ir y tenkina f(xy) = f(x)f(y) f(x +y) + 1.
5. Raskite visas funkcijas f : R [0, +), kurios su visomis realij skaii S
poromis x ir y tenkina f(x
2
+y
2
) = f(x
2
y
2
) +f(2yx).
6. Raskite visas funkcijas f : R R, kurios su visomis realij skaii poromis x S
ir y ir 2 n N tenkina f(x
n
+f(y)) = f
n
(x) +y.
7. Raskite visus funkcij f, g : R R dvejetus tokius, kad: S
a) Jei x < y, f(x) < f(y).
b) Visoms realij poroms x ir y galioja f(xy) = g(y)f(x) +f(y).
8. Raskite visas funkcijas u : R R, kurioms egzistuoja tokia grietai monotonin S
funkcija f : R R, kad visoms realij poroms x ir y yra teisinga lygyb f(x +
y) = u(y)f(x) +f(y).
9. Raskite visas funkcijas f : (0, 1) R, kurios su visomis realij skaii poromis S
x ir y tenkina f(
x+y
1+xy
) =
f(x)f(y)
|1+xy|
ir yra tolydios.
91
3 SKYRIUS
KOMBINATORIKA
3.1 Matematiniai aidimai
Kairiajame apatiniame 55 lentos langelyje stovi ak. Arklys Dominykas ir Asiliukas
Dainius pakaitomis perklinja t ak kaimynin pagal kratin langel; pradeda
visada Dominykas. Pralaimjusiu Kiel Kamil garsiai paskelbia t, kuris perkelia ak
tok laukel, kuriame ta ak jau yra pabuvojusi. Ar gali kuris nors i j Arklys
Dominykas arba Asiliukas Dainius perklinti ak taip, kad jis visada laimt, nors
ir k bedaryt kitas aidjas ir kaip jis tada turt perklinti t ak?
1
3.1.1 Strategija
Jeigu asilas eina vir, tai ir arklys turi eiti vir, o jei negali, tai kur nors o-
n, taip prasideda, dja, nelabai skmingas bandymas aprayti strategij, kuria turt
vadovautis Dominykas, nordamas laimti.
Dviej aidj matematini aidim strategijos kaip tik ir bus pagrindin io skyre-
lio tema. Aptarsime dvi daniausiai pasitaikanias laimini bei pralaimini pozicij
radimo, bei simetrijos. Taip pat susipainsime su aidimais, kuriuose galime nustatyti
laimtoj net ir nenurodydami, kaip jis turt aisti. Aptardami strategijas ir nag-
rindami pavyzdius visuomet laikysime, kad abudu aidjai i paskutinij stengiasi
nepralaimti ir nedaro kvail jim.
Laiminios ir pralaiminios pozicijos
Tad pradkime nuo vieno i daniausiai pasitaikani bd, naudojamo norint nustatyti
aidimo laimtoj vis galim aidimo pozicij aibs padalinimo dvi dalis, vadinamas
laiminiosiomis ir pralaiminiosiomis pozicijomis. aidjas, bdamas laiminiojoje po-
zicijoje visuomet gali paeiti taip, kad varovas atsidurt pralaiminioje pozicijoje. is,
1
Lietuvos 5-6 klasi moksleivi matematikos olimpiada, 2011m.
3.1. Matematiniai aidimai Kombinatorika
savo ruotu, yra pasmerktas po bet kurio jimo pastatyti varov laiminij. Laimin-
iosioms pozijoms, inoma, turi priklausyti ir aidim pergale ubaigianios pozicijos,
ar bent jau (jei aidiama iki kol kuris nors aidjas nebegals padaryti jimo) jos turi
garantuoti, kad aidjas jim padaryti visuomet gals.
1 Pavyzdys. Ant stalo yra n akmenuk. aidjas gali nuimti bet kok akmenuk skaii
ne didesn u k. aidjai A ir B jimus atlieka pakaitomis, pradeda A. Laimi tas
aidjas, kuris nuim paskutin akmenuk. Kuris aidjas laims su atitinkamais n?
Sprendimas. Jei n < k + 1, tada laims A nuimdamas visus akmenukus, tad pozicijos
su tokiu akmenuk skaiiumi yra laiminios (pozicijos, kuriose pradjs aidim ir ais-
damas protingai tikrai laimsi). Jei n = k + 1, tai A negali nuimti vis akmenuk, B
savo jimu gals tai padaryti ir laims aidim, tai yra pralaiminti pozicija. Skaiius
k + 1 yra svarbus dl to, kad vieno jimo metu vis akmenuk paimti negali, o dviem
jimais tai visada galsi padaryti.
Pastebj tai, gauname, kad jei akmenuk skaiius n yra k+1 kartotinis, tai A savo
jimo metu negali pasiekti kito kartotinio, o B savuoju jau gals tai padaryti. Nulis
yra k + 1 kartotinis, tad B galiausiai j ir pasieks, taip laimdamas aidim.
Analogikai, jei akmenuk skaiius n nra k+1 kartotinis, tai artimiausi kartotin,
kaip ir visus likusius, pasieks A ir laims.
iame pavyzdyje visi galimi akmenuk kiekiai padalinami dvi grupes. Pirmojoje,
pralaimini pozicij grupje, yra k + 1 kartotiniai (1), antrojoje, laimini pozicij
grupje, lik skaiiai (2). I (2) visada galima patekti (1), o bet koks jimas i (1) veda
(2).
2 Pavyzdys. Ant stalo yra n akmenuk. aidjas gali paalinti 2
m
akmenuk, kur m
yra sveikasis neneigiamas skaiius. Kuris aidjas laims dabar?
Sprendimas. Jei n 1 (mod 3) arba n 2 (mod 3), tai A paalindamas atitinkamai
1 arba 2 akmenukus gaus skaii dal i trij, o antrasis aidjas, negaldamas atimti
trejeto kartotinio, gaus nedal i trij. Kadangi 0 yra dalus i trij, tai aidim laims
A. Jei n 0 (mod 3), aidim laimi B.
iame pavyzdyje vl visi galimi akmenuk kiekiai padalinami dvi grupes. Pirmojoje
grupje yra 3 kartotiniai (1), antrojoje lik skaiiai (2). I (2) visada galima patekti
(1), o bet koks jimas i (1) veda (2).
3 Pavyzdys. Ant stalo yra n akmenuk. aidjas gali paalinti bet kok pirmin skaii
arba vien akmenuk. Kaip aidimas vyks dabar?
Sprendimas. Jei n nra keturi kartotinis, tai laimi pirmasis aidjas, visuomet nuim-
damas tiek akmenuk, kad gaut keturi kartotin. Jei n yra keturi kartotinis, laimi
antrasis aidjas.
4 Pavyzdys. Ant stalo yra n akmenuk. aidjas gali paalinti p
n
akmenuk, kur p
bet koks pirminis, o n neneigiamas sveikasis skaiius. Kaip aidimas vyks dabar?
93
3.1. Matematiniai aidimai Kombinatorika
Sprendimas. 6 yra maiausias skaiius, kuris nra pirminio skaiiaus laipsnis. Jei n yra
nedalus i ei, tada A gali j padaryti tok ir taip usitikrinti, kad pats negaus ei
kartotinio. A laims aidim. Jei n yra ei kartotinis, panaiai aisdamas laimi B.

Jei aidjas A VISADA gali atlikti tok jim, po kurio B negali vienu jimu laimti
aidimo, tai B NIEKADA ir nelaims. Jei aidimas kada nors baigsis, tai pergal vs
A.
Simetrija
Antroji i danai pasitaikani strategij yra simetrija. Jei aidimo laukas turi simetri-
jos a ar centr, aidjas gali suskirstyti vis lauk simetrik jim poras. aidjui A
atlikus vien jim i ios poros, aidjui B tereikia atlikti antrj taip jis usitikrina,
kad po kiekvieno prieininko jimo jis gals atlikti dar bent vien jim. Pavyzdiui:
5 Pavyzdys. aidjai A ir B staiakampje lentelje 2 n paeiliui spalvina po vien
langel arba du bendr sien turinius langelius. Nuspalvinto langelio spalvinti nebegali-
ma. Pralaimi tas aidjas, kuris nebegali atlikti jimo. Nurodykite, kuris aidjas turs
laimini strategij su atitinkamais n.
Sprendimas. Kai n yra nelyginis, tai A pirmu jimu spalvina du centrinius langelius. ie
langeliai tampa lentos simetrijos aimi. Kiekvienas lentels langelis turi sau simetrik,
jie yra suskirstyti poras. Dabar po bet kurio B jimo A gals atlikti simetrik jim
centrini langeli atvilgiu. A aidjas niekada nepralaims. Kadangi langeli skaiius
baigtinis ir kiekvienu jimu sumaja, tad aidimas yra baigtinis. I i dviej teigini
seka, kad pirmasis aidjas turi laiminij strategij.
Kai n yra lyginis, tada, kad ir kok jim atlikt A, B gals atlikti simetrik jim
lentels centro atvilgiu. Kadangi aidimas baigtinis, B turs laiminij strategij.

6 Pavyzdys. aidimo erdv yra apvalus stalas. aidjai A ir B pakaitomis deda iden-
tikas monetas ant stalo. Monetos negali persidengti. Pralaimi aidjas, kuris nebegali
atlikti jimo. rodykite, kad aidim laims A.
Sprendimas. Pirmu jimu A deda monet taip, kad jos centras sutapt su stalo centru,
o vliau deda monetas simetrikai B padtoms centrins monetos atvilgiu. Kadangi
po kiekvieno A jimo laisva stalo vieta lieka simetrika centro atvilgiu, tai kaip beeit
B, A gals pakartoti jim simetrikai, tokiu bdu galiausiai laimdamas.
Pastaba. Atkreipsime dmes, kad udavin sprendiant neatsargiai galima isprsti
j taip: kadangi stalas simetrikas centro atvilgiu, tai kaip beit A, B gals paeiti si-
metrikai. Tai, inoma, netiesa, nes jei A ddamas monet udengia centr, B simetri-
kai monetos dti negali. Tad nors i pirmo vilgsnio aidimo erdv atrod simetrika,
ji tokia nebuvo (o A paddamas monet centr j toki padaro).
7 Pavyzdys. Apskritime paymta n tak, i eils sunumeruot skaiiais 1, 2, . . . , n.
is apskritimas yra aidimo A(n) erdv. Du aidjai P ir L paeiliui bria po styg,
94
3.1. Matematiniai aidimai Kombinatorika
jungiani du takus, kuri numeriai yra vienodo lyginumo. Pradeda P. Leidiama
jungti tik takus, kurie nra sujungti su n vienu kitu. Nubrtos stygos negali kirstis.
Pralaimi tas aidjas, kuris negali atlikti jimo. Kuris aidjas laimi su atitinkamais
n?
Sprendimas. Jeigu ikart nesimato, kaip sprsti udavin, pravartu pabandyti paprastes-
nius atvejus. Lengva suprasti, kad aidimus A(1) ir A(2) aidjas P pralaimi, aidimus
A(3) ir A(4) laimi. aidim A(5) laimi P sujungdamas 1 ir 3 takus.
Galime sivaizduoti, kad takai (nekeiiant j tarpusavio padties) yra idlioti tai-
syklingojo n-kampio virnse; tai aidimo eigai ir baigiai takos neturi.
Nagrinsime aidimus A(n), kai n = 4k. Parodysime, kad juos laimi P. Apskritimo
takai, priklausantys vienam skersmeniui, yra vadinami diametraliai prieingais. iuo
atveju vis diametraliai prieing tak lyginumas yra vienodas. Pirmo jimo metu
P tereikia sujungti bet kuriuos diametraliai prieingus takus. Nubrtas skersmuo
tampa apskritimo simetrijos aimi. Kiekvienas takas turi sau simetrik io skersmens
atvilgiu. Suskirst simetrikus takus poras pastebime, kad L negali brti stygos i
karto per du vienos poros takus, kitaip i kirst simetrijos a. kiekvien L nubrt
styg P atsako simetrika iai skersmens atvilgiu. Parodysime, kad jis visada gals tai
padaryti. P taktika garantuoja, kad po kiekvieno jo jimo arba abu poros takai yra
laisvi arba per abu eina po styg (1). Tarkime, kad L sujungia takus A ir B, jiems
simetriki atitnkamai yra C ir D (jie yra tikrai laisvi pagal (1)). Tarkime, kad P negali
sujungti tak C ir D, tada tarp j yra takas E ir styga CD kerta styg EF. Bet
jau yra nubrta styga, simetrika CF (1), o i kerta AB. Gavome prietar. aidim
laimi P.
Kai n = 4k+2, laimi L. Dabar diametraliai prieing tak lyginumas yra skirtingas.
L suskirsto diametraliai prieingus takus poras. Jei P bria styg per A ir B, tai
L atsako styga einania per diametraliai iems prieingus takus C ir D. P negali
brti stygos per abu poros takus, nes i lyginumas skiriasi. L strategija garantuoja,
kad po kiekvieno jo jimo arba abu poros takai yra panaudoti arba abu yra laisvi (1).
Tarkime, kad i strategija negarantuoja L pergals. P paskutiniu jimu bria styg per
A ir B, C ir D yra iems diametraliai prieingi ir jie abu yra laisvi pagal (1). Vadinasi
tarp j yra takas E, o styga EF kerta CD. Bet jau yra nubrta styga per tak
diametraliai prieing E (1) ir ji kerta ties AB. Prietara. P bus aidjas, kuriam
pirmajam pritrks jim. Laims L.
Kai n = 4k+1, laimi P. Savo pirmu jimu jis sujungia n ir n2. Kartu i tolimesnio
aidimo ikrinta takas n 1. Viso lieka 4k +1 3 = 4(k 1) +2 tak, o atvej jau
inagrinjome aukiau.
Kai n = 4k + 3, laimi P. Savo pirmuoju jimu jis sujungia 2k + 1 ir 2k + 3, kartu
i aidimo ikrinta 2k + 2. Lieka 4k tak, tarp kuri negalima nubrti n vieno
skersmens, tad aidiama kaip atveju su 4k + 2 tak.
8 Pavyzdys. (Leningradas 1989) Du aidjai A ir B aidia aidim ant 1010 lentos.
aidjas gali rayti plius arba minus tui lentels langel. Pradeda A. Jeigu po
aidjo jimo atsiranda trys i eils einantys langeliai (horizontaliai, vertikaliai arba
striai) su vienodais enklais, aidjas laimi. Ar kuris nors aidjas turi laiminij
strategij? Jei taip, tai kuris?
95
3.1. Matematiniai aidimai Kombinatorika
Sprendimas. B turi laiminij strategij. Jeigu jis gali vienu jimu laimti, tai jis ne-
sivarydamas tai padarys. Kitu atveju jis rao prieing enkl padtam A simetrik
langel centro atvilgiu. Nesunku sitikinti, kad taip aidiant A aidjas niekada ne-
gals laimti. Belieka rodyti, kad B tai gals padaryti visada. Nagrinkime centrin
kvadrat 4 4 po to, kai A centrin 2 2 ra antrj savo enkl. Dabar jame greta
yra rayti du vienodi enklai. Turdami omenyje, kad A negali laimti io aidimo,
nesunkiai galime parodyti, kad B visada laims. Pabandykite tai padaryti patys.
Netiesioginiai sprendimai (angl. non-constructive solutions)
Nagrintuose udaviniuose mes pateikme strategijas, kuriomis vadovaudamasis A ar-
ba B visada gals laimti aidim. Taiau kartais tai daryti yra visai nebtina. Jei
klausiama, ar aidjas A visada gali laimti, neprivalome nurodyti bdo, kaip A tai
gali padaryti. Utenka parodyti, kad A galiausiai pasieks pergal. Tokie sprendimai,
nesilantys algoritmo pergalei pasiekti, vadinami netiesioginiais sprendimais.
9 Pavyzdys. aidjai A ir B pakaitomis lentoje rao sveikuosius teigiamus skaiius
ne didesnius u p. Draudiama rayti skaiius, kurie dalija nors vien i jau urayt.
Pralaimi tas, kuris nebegali atlikti jimo. Kas laimi atveju p = 10? p = 1000?
Sprendimas. Abiem atvejais laimi A. Pirmuoju atveju udavin nesudtinga isprsti ir
tiesiogiai: A urao 6. Tada B gali rayti tik skaiius i por (4, 5), (10, 8), (9, 7) ir A
visada gali urayti antrj skaii i tos poros. Isprskime udavin netiesiogiai:
Pastebkime, kad vienas skaiius ia ypatingas. Tai yra 1. B niekada negali jo
parayti, tai gali atlikti tik A ir tik pirmuoju jimu. Nagrinkime tok aidim (1),
kuriame A pirmo jimo metu neparao vieneto. Jei iame aidime jis turi laiminij
strategij, tai ms darbas jau baigtas, tad tarkime, kad A tok aidim visada pralaimi.
Kas vyksta jei A pirmo jimo metu parao vienet (2)? Tada aidimas virsta (1), tik ia
jau B yra pirmasis aidjas ir jis visada aidim pralaimi, kitaip A jau bt laimjs.
Taigi A tikrai gali laimti (1) arba (2), kadangi jis pats pasirenka, kur aidim ais,
tai jis laims ir vis aidim.
10 Pavyzdys. aidiamas achamat aidimas, bet aidjai pakaitomis atlieka po du
jimus. Pradeda A. Ar jis iuose achmatuose gali garantuoti, kad niekada nepralai-
ms?
Sprendimas. Taip. Tarkime, kad B turi laiminij strategij. A pajuda pirmyn ir atgal
su irgu ir taip jis apsikeiia pozicijomis su B, dabar jau jis turi laiminij strategij.
Gavome prietar. Vadinasi A turi nepralaiminij strategij.
Pastaba. Atkreipkite dmes, kad is aidimas gali tstis be galo ilgai.
11 Pavyzdys. (aidimas CHOMP) aidjai A ir B lauo mn dydio okolado plytel
pakaitomis. aidjas pasirenka kur nors langel ir ilauia i plytels staiakamp,
kurio prieingos virns yra is langelis ir pradins plytels virutinis deinysis kampas
(staiakampio kratins lygiagreios plytels kratinms). Pralaimi tas aidjas, kuris
atsilauia apatin kairj kamp. Su kokiomis okolado plytelmis gali laimti B?
96
3.1. Matematiniai aidimai Kombinatorika
Sprendimas. B gals laimti tik atveju m = n = 1. Nagrinkime likusius atvejus. ia
ypatingas yra virutinis deinysis langelis. B niekada jo negaus, j A atlau pirmuoju
jimu. Tarkime, kad pirmasis aidjas, kad ir kaip aist, negali laimti. Jis pirmuoju
jimu atlauia virutin dein langel, B tada atlieka jim (), kuris, kaip tarme, atves
j pergal. Taiau akivaizdu, kad A savo pirmo jimo metu gali atlikti jim () ir
atsidurti laiminioje pozicijoje. Prietara. Vadinasi aidim visada laims A.
12 Pavyzdys. (Tournament of Towns 2005) Matelotas ir Kauntelotas nori isidalinti
25 monetas, kuri verts yra 1, 2, 3, . . . , 25 kapeikos. Kiekvienu jimu vienas aidjas
pasirenka monet, o kitas nusprendia, kuriam i j jinai atiteks. Pirmasis monet
renkasi, inoma, Matelotas, o kitus monet pasirinkimus atlieka tas, kuris tuo momentu
turi daugiau kapeik. Jei abu aidjai turi lygiai kapeik, sprendim atlieka tas, kuris
tai dar prie tai. Laimi tas, kuris gal gale turi daugiausiai kapeik. Kuris aidjas
turi laiminij strategij?
Sprendimas. Toki strategij turi Kauntelotas. Po pirmojo Mateloto pasilymo jis
gali atsisakyti monetos arba j paimti. Jei jis gali laimti pams monet, tai taip ir
padaro. O jeigu pams monet laimti negali, tai duoda j Matelotui ir po tokio jimo
Matelotas niekaip negali surinkti daugiau kapeik. Kauntelotas laimi.
Udaviniai
1. aidjai A ir B paeiliui lauia okolado plytel mn iilgai linij ir atsilaut dal S
suvalgo. Apatinis kairysis langelis yra unuodytas, j suvalgs aidjas pralaimi.
Su kokiomis m ir n reikmmis aidjas B turi laiminij strategij?
2. aliasis ir Purpurinsis pakaitomis deda alius ir purpurinius irgus ant laisv S
achmat lentos langeli, pradeda aliasis. Negalima irgo padti taip, kad j
kirst prieininko gra. Laimi tas, kuris atlieka paskutin jim. Kas laims?
3. Pradioje n = 2. A ir B pakaitomis prideda prie turimo skaiiaus n bet kok S
jo dalikl, kuris nra lygus n, ir prieininkui pateikia naujj n. Laimi tas, kuris
parao skaii ne maesn u 1990. Kas laims?
4. aidimas pradedamas skaiiumi 1. aidjai pakaitomis skaii daugina i na- S
trinio skaiiaus didesnio u 1, bet maesnio u 10 ir taip gauna nauj skaii.
Laimi tas, kuris primasis gauna skaii didesn u 1000. Ar kuris nors aidjas
turi laiminij strategij? Jei taip, kokia ji?
5. Duotas nelyginis natrinis n > 1. Ant lentos uraytas skaiius k = 2 du aidjai S
pakaitomis gali pakeisti k 2k arba k + 1. Pralaimi tas, kuris urao ant lentos
skaii didesn u n. Su kuriais n antrasis aidjas turi laiminij strategij?
6. Du aidjai pakaitomis spalvina po vien 4 4 lentels langel. Pralaimi tas, po S
kurio jimo lentelje atsiranda pilnai nuspalvintas kvadratas 2 2. Kuris aidjas
turi laiminij strategij?
97
3.1. Matematiniai aidimai Kombinatorika
7. Ant stalo guli 2002 kortos. Ant j surayti skaiiai nuo 1 iki 2002. Du aidjai S
paeiliui ima nuo stalo po kort ir slepia j kienje. Laimi tas aidjas, kurio vis
kort, esani kienje, sumos paskutinis skaitmuo yra didesnis u prieininko.
Ar kuris nors aidjas turi laiminij strategij? Jei taip, tai kokia ji?
8. P ir L sugalvoja po natrin skaii ir pateikia j atsitiktiniam asmeniui A. A S
geba suskaiiuoti i skaii sandaug bei sum ir urao iuos du skaiius ant
atskir korteli. Vien i korteli (P ir L neino kuri) A parodo vaikinams, o
kit magikai pradangina. Parodytoji kortel paenklinta simintinu skaiiu 2002.
P vilgteli skaii ir prisipasta, kad neino, kok skaii sugalvojo L. Tai
inodamas, L taip pat atsako, kad nenutuokia, koks yra P skaiius. Koks yra L
pasirinktas skaiius?
9. n n achmat lentos kairiajame apatiniame kampe guli akmenukas. A ir B S
jimus atlieka pakaitomis, pradeda A. aidjai gali pastumti akmenuk gretim
langel, kuris dar niekada nebuvo aplankytas. Laimi tas, kuris atlieka paskutin
jim.
1) Kas laimi su lyginiais n?
2) Kas laimi su nelyginiais n?
3) Kas laimi, jei aidimo pradioje akmenukas yra gretimame kampiniam lange-
lyje?
10. A padeda irg pasirinkt 8 8 lentos langel. Tada B atliek jim ir toliau S
jimai atliekami pakaitomis. Kiekviename langelyje irgas gali pabti tik vien
kart. Pralaimi tas, kuris nebegali atlikti jimo. Kas laimi?
11. Netiktai aidim vl aidia A ir B, A pradeda, jimai atliekami pakaitomis. S
Yra dvi krvels atitnkamai po p ir q akmenuk. jimo metu aidjas gali paimti
pasirinkt akmenuk i pasirinktos krvels, paimti po akmenuk i kiekvienos
krvels arba perkelti akmenuk i vienos krvels kit. Kas laimi su atitnkamais
p ir q?
12. aidim CHOMP, kurio taisykls nurodytos netiesiogini sprendim skyrelyje, S
visuomet (iskyrus atvej 1 1) laimi pirmasis aidjas, k mes rodme netiesio-
giai. Raskite tiesiogin io udavinio sprendim, t.y. sugalvokite strategij, kuri
pelnyt pirmajam aidjui pergal atvejais:
1) m = n.
2) m = 2, n bet koks natralusis skaiius.
13. Duotas trikampis pyragas, kurio plotas yra vienetas. A renkasi tak X trikam- S
pio ploktumoje. B pjauna tiese, einania per X. Kok didiausi plot B gali
atsipjauti?
14. Duotas daugianaris x
3
+. . . x
2
+. . . x+ = 0. A parao sveikj skaii, nelyg S
0, vietoj kurio nors tritakio. Tada B rao sveikj skaii ir daugianar sveikuoju
skaiiumi ubaigia A. rodykite, kad A gali aisti taip, kad visos trys daugianario
aknys bt sveikieji skaiiai.
98
3.1. Matematiniai aidimai Kombinatorika
15. Arklys Dominykas i skyrelio pradios nervinasi, kad perskaits vis teorij i S
isprends ger sauj udavini, vis dar negali sugavoti laimjimo prie Asiliuk
strategijos. Negi jis jos taip ir nesugalvos?
16. [All Russian Olympiad 1992] Krvelje yra N akmenuk. aidjas gali paimti k S
akmenuk, kur k dalina akmenuk skaii paimt prieininko jo paskutinio jimo
metu. Pirmu jimu pirmasis aidjas gali paimti kiek nori akmenuk, iskyrus 1
ir N. Laimi tas, kuris paima paskutin akmenuk. Su kokiu maiausiu N 1992
antrasis aidjas turi laiminij strategij?
17. Ploktumoje nubriami 1994 vektoriai. Du aidjai paeiliui renkasi vektorius S
ir juos sumuoja su jau turimais. Pralaimi tas, kuris gal lage turi trumpesn
vektori. Ar pirmasis aidjas turi nepralaimini strategij?
18. A ir B pakaitomis keiia tritakius x
10
+. . . x
9
+. . . x
8
+. . . x
7
+. . . x
6
+. . . x
5
+ S
. . . x
4
+. . . x
3
+. . . x
2
+. . . x + 1 = 0 realiuosius skaiius. Jei aidimo pabaigoje
daugianaris turi nors vien realij akn, laimi B. Ar gali B laimti?
19. [All Russian Olymnpiad 1994] aidjai A, B paeiliui atlieka jimus su irgu S
1994 1994 achmat lentoje. A atlieka horizontalius (pereina gretim eilut)
jimus, o B vertikalius. A pasirenka irgo pozicij ir atlieka pirm jim. irgas
negali atsidurti langelyje, kuriame jau yra buvs. Pralaimi tas, kuris nebegali
atlikti jimo. rodykite, kad A turi laiminij strategij.
20. [Tournament of Towns 2009] Du aidjai paeiliui spalvina po N tak ant ap- S
skritimo. Pirmojo spalva raudona, antrojo mlyna. Spalvinti to paties tako
negalima. aidimo pabaigoje gaunamas apskritimas padalintas 2N lank. Ran-
damas ilgiausias lankas, kurio abu galai nuspalvinti ta paia spalva. aidim laimi
ios spalvos savininkas. Ar kuris nors aidjas turi laiminij strategij su visais
N > 1?
21. [IMO shortlist 1991] A ir B aidia aidim. Kiekvienas urao po sveik tei- S
giam skaii ir duoda j teisjui. Teisjas lentoje urao du skaiius, vienas j
yra aidj skaii suma. Tada teisjas klausia A: Ar inai, kok skaii ura
B?" Jei A atsako neigiamai, tada teisjas to paties klausia B ir t.t. Tarkime, kad
A ir B yra baisiai protingi ir niekada nemeluoja. rodykite, kad is aidimas yra
baigtinis.
22. [IMO shortlist 2004] Duotas natrinis n > 1. Ant lentos uraytas skaiius S
k = 2, o du aidjai pakaitomis gali pakeisti k 2k arba k + 1. Pralaimi tas,
kuris urao ant lentos skaii didesn u n. Su kuriais n antrasis aidjas turi
laiminij strategij?
23. Yra dvi krvels, vienoj n, kitoj m akmenuk. Dar yra Julius su Gyiu. Jie S
gali nuimti ir suvalgyti norim skaii akmenuk i vienos krvels arba po lyg
skaii akmenuk i abiej krveli. Aiku, kad pradeda Gytis. Laimi tas, kuris
suvalgo paskutin akmenuk. Raskite pirmas 10 pralaimii pozicij. Raskite
rekursin sry tarp pralaimini pozicij ir j numeri (t.y. ireikite n-aj
pralaimii pozicij, per ankstesnes). Gal pavyks ireikti visas pralaiminias
pozicijas per j numerius?
99
3.1. Matematiniai aidimai Kombinatorika
24. [Kvant 1987] aidimo erdv yra begalin ploktuma. A savo jimu nuspalvina S
vien tak raudonai, o B k tak mlynai. A laimi, jei po jo jimo ploktumoje
atsiranda kvadratas, kurio kratins lygiagreios aims ir visos jo virns raudo-
nos. jimai atliekami pakaitomis. Ar A gali laimti, kai k = 1? k = 2? k js
mgstamiausias natralusis skaiius?
100
3.1. Matematiniai aidimai Kombinatorika
3.1.2 aidimas NIM
iame skyrelyje nagrinsime aidim NIM, kuris, kaip pamatysime vliau, tam tikra
prasme apima daugyb matematini aidim.
Apibrimas. NIM metu du aidjai pakaitomis ima akmenukus i n krveli. ai-
djas gali paimti norim skaii akmenuk i pasirinktos krvels. Jis turi paimti bent
vien, gali paiimti ir visus esanius toje krvelje. Pralaimi tas aidjas, kuris nebe-
gali atlikti jimo. aidimo pozicij ymsime Q = (x
1
, . . . , x
n
), kur x
i
yra akmenuk
skaiius atitinkamoje krvelje.
Pabandykime panagrinti paprasiausius NIM atvejus:
13 Pavyzdys. NIM aidimas su dviem lygiomis krvelmis. Kuris aidjas turi lai-
minij strategij?
Sprendimas. Antrasis aidjas visada gali atsakyti simetriku jimu ir taip besielgdamas
laimti aidim.
14 Pavyzdys. Kaip baigsis NIM aidimas su lyginiu lygi krveli skaiiumi? Nelygi-
niu?
Sprendimas. Lyginiu atveju antrasis aidjas gali suskirstyti visas krveles poras ir si-
vaizduoti, kad aidia vienu metu daug aidim po dvi lygias krveles. Tokius aidimus
jis moka laimti atlikdamas simetrikus jimus por viduje. ia aidim iskaidme
dalinius aidimus, kuriuos jau mokame sprsti, tai mums padjo isprsti sumin aidi-
m. iuo atveju mums pasisek, kad visus dalinius aidimus laimi tas pats aidjas, bet
iame skyrelyje pamatysite, kad taip galime sprsti ir kur kas sudtingesnius udavi-
nius. Jei turime nelygin krveli skaii pirmajam aidjui tereikia nuimti pasirinkt
krvel ir taip gauti anksiau inagrint aidim, kuriame jis jau bus antrasis aidjas
ir pagaliau diaugsis pergale.
15 Pavyzdys. NIM aidimas su trimis krvelmis, kuriose yra 3, 5 ir 7 akmenukai.
Kuris aidjas turi laiminij strategij?
Rugils Bendinskaits sprendimas. Pirmiausia, pralaiminti pozicija yra (x, x, 0), nes
tada, kad ir kiek imt aidjas, prieininkas gali paimti tiek pat i kitos krvels. Tada
laiminios pozicijos yra (x, x, y) ir (0, x, x + y), nes i j galime padaryti pralaimini
pozicij (x, x, 0). Toliau pralaiminti pozicija yra (1, 2, 3), nes kad ir k darytume, i jos
gauname tik laiminias pozicijas (0, 2, 3), (1, 1, 3), (1, 0, 3), (1, 2, 2), (1, 2, 1), (1, 2, 0). I
ia mes gauname, kad laiminios pozicijos yra (1, 2, 3 + x), (1, 2 + x, 3), (1 + x, 2, 3),
nes i j galima padaryti (1, 2, 3). Sekanti pralaiminti pozicija yra (1, 4, 5), nes i jos
gauname (0, 4, 5), (1, 3, 5), (1, 2, 5), (1, 1, 5), (1, 0, 5), (1, 4, 4), (1, 4, 3), (1, 4, 2), (1, 4, 1),
(1, 4, 0). I ios pozicijos taip pat galime ivesti laiminisias (1, 4, 5+x), (1, 4+x, 5), (1+
x, 4, 5). Toliau pralaiminti pozicija yra (2, 4, 6), i jos gauname (1, 4, 6), (0, 4, 6), (2, 3, 6),
(2, 2, 6), (2, 1, 6), (2, 0, 6), (2, 4, 5), (2, 4, 4), (2, 4, 3), (2, 4, 2), (2, 4, 1), (2, 4, 0). I jos
analogikai ivedame laiminisias pozicijas: (2, 4, 6+x), (2, 4+x, 6), (2+x, 4, 6). Kita
pralaiminti pozicija yra (3, 5, 6), i jos -> (2, 5, 6), (1, 5, 6), (0, 5, 6), (3, 4, 6), (3, 3, 6),
101
3.1. Matematiniai aidimai Kombinatorika
(3, 2, 6), (3, 1, 6), (3, 0, 6), (3, 5, 5), (3, 5, 4), (3, 5, 3), (3, 5, 2), (3, 5, 1), (3, 5, 0). I jos irgi
gauname laiminias pozicijas: (3, 5, 6 +x), (3, 5 +x, 6), (3 +x, 5, 6).
Taigi (3, 5, 7) irgi yra laiminti pozicija (3, 5, 6+x). Pirmas turi laiminij strategij.

Pirmasis aidjas A nori patikrinti ar gali laimti aidim. Jis isirao visus aidi-
mus, kuriuos gali pasiekti vieno jimo metu. Jei tarp i aidim yra toki, kuriuose
pradedantis aidjas pralaimi (juos pradeda aidjas B), tai A venia pergal. Kad
patikrintume iuos aidimus turime vl isirayti visus pasiekiamus i j vienu jimu
ir t.t. Jei tik aidimas ir galim jim skaiius visose situacijose yra baigtiniai, tai
procesas kakada baigsis ir isiaikinsime, kuris aidjas laimi pradin aidim.
Daniausiai patogiau yra pradti nuo aidimo pabaigos pozicijos ir nagrinti visas
pozicijas pasiekiamas i jos vienu atgaliniu jimu, taiau bet kuriuo atveju, tai ilgas ir
keblus procesas, kur itame skyrelyje mes supaprastinsime ir formalizuosime. Nordami
isprsti aidim NIM, mes apibrime pozicijos NIM sum, kuri mums suteiks vis
reikaling informacij apie aidimo pozicij. Isprend j, NIM sum apibendrinsime
iki NIM verts, kuri bus galima priskirti daugelio aidim (ne tik NIM) pozicijoms.
NIM suma ir NIM aidimo sprendimas
NIM sprendimo pagrindas yra vadinamasis NIM sumavimas. Uraykime kiekvienos
krvels akmenuk skaii dvejetaine sistema. Atlikdami paprast sumavim stulpeliu
simename, kiek deimi turime perneti kit eil, o NIM sumavimas yra sumavimas
be perneim sudedame atskirai kiekvien stulpel. NIM sumavimas ymimas simboliu
, o jo rezultas vadinamas pozicijos NIM suma.
Panagrinkime pavyzd su 21, 17 ir 15 akmenuk. ie skaiiai dvejetainje sistemoje
usirao kaip 10101
2
, 10001
2
ir 1111
2
= 01111
2
. Sumuokime:
10101
10001
01111
01011
Gavome, kad 21 17 15 = 01011
2
= 11. Sumuojant nesunku pastebti, kad
stulpelio suma bus lygi nuliui, jei tame stulpelyje yra lyginis skaiius vienet, ir vienetui,
jei vienet skaiius yra nelyginis.
Bet kuri pozicij, kurios NIM suma yra lygi 0 (t.y. dvejetain sumos iraika
sudaryta vien i nuli), vadinsime pozicija (). Kaip tuojau sitikinsime, ios pozicijos
vaidina labai svarb vaidmen.
Teiginys. I bet kokios pozicijos, kuri nra (), galime pereiti ().
rodymas. Imkime bet kuri pozicij, kuri nra () ir suraykime dvejetaines akmenuk
skaiiaus vertes stulpeliu, kaip pavyzdyje. Raskime kairiausi stulpel, kurio NIM sumos
vert yra lygi vienetui (toks stulpelis atsiras, nes nagrinjama pozicija nra ()). Imame
didiausi akmenuk kiek A, kurio dvejetainje iraikoje ioje pozicijoje yra vienetas
ir atliekame NIM sumavim visiems akmenuk kiekiams iskyrus A. Gauname sum
B. Nesunku suprasti, kad A B, sitikinkite tuo. Kadangi A B, tai galime i A
102
3.1. Matematiniai aidimai Kombinatorika
paimti tiek akmenuk, kad likt B, o tada vis krveli NIM suma bus lygi B+B = 0.
Atsidursime pozicijoje ().
Panagrinkime rodym jau matytu atveju 21, 17, 15. Kairiausiasis stulpelis, kurio
suma nelygi nuliui yra antrasis i kairs. Didiausias skaiius, kuris tame stulpelyje
turi vienet yra 15 = 01111
2
(skaiius A), o likusi skaii, 21 = 10101
2
ir 17 = 10001
NIM suma (reikm B) lygi 00100
2
= 4. Vadinasi, reikm 15 pakeit 4 (t.y. num
11 akmenuk) gausime pozicij, kurios NIM suma lygi nuliui. sitikinkime:
10101
10001
00100
00000
Teiginys. I pozicijos () negalime pereiti kit pozicij ().
rodymas. Norint tai atlikti reikt kiekvieno stulpelio vienet skaii pakeisti lyginiu
skaiiumi, o kadangi galime keisti tik vienos krvels akmenuk skaii, tai to tikrai
negalsim padaryti.
Teorema. NIM aidime pozicijos () yra pralaiminios, o visos kitos laiminios.
rodymas. Jei pirmasis aidjas pradeda aidim pozicijoje, kuri nra (), tai jis visuo-
met daro jim taip, kad antrajam tekt pozicija (). O k bedaryt antrasis, po jo
jimo pozicija nebus (), tad ir paskutinio akmenuko jis nepaims.
Jei pirmasis aidjas pradeda aidim pozicijoje (), laimini strategij analogikai
turi antrasis adjas.
Imokome atpainti laiminias ir pralaiminias NIM pozicijas, pabandykime panag-
rinti jau matyt NIM aidim:
16 Pavyzdys. NIM aidimas su trimis krvelmis, kuriose yra 3, 5 ir 7 akmenukai.
Kuris aidjas turi laiminij strategij?
Sprendimas. Jei Q yra NIM aidimo pozicija, tai N(Q) ymsime tos pozicijos NIM
sum. Kadangi N(3, 5, 7) = 1, tai laims pirmasis aidjas.
Pabandykime pairti, kaip is aidimas galt bti aidiamas. Kadangi N(3, 5, 7) =
1, tai pirmam aidjui tereikia pakeisti vienet skaii paskutiniame NIM sumavimo
stulpelyje, pavyzdiui nuimti vien akmenuk i bet kurios krvels (jos visos turi po
nelygin skaii akmenuk, tad paskutiniame stulpelyje visos turi po vienet). Tarkime
jis nuima nuo maiausios, tada N(2, 5, 7) = 0.
Kad ir kiek akmenuk nuimt antrasis aidjas, jis negals gauti NIM sumos lygios
0. Jis gals keisti akmenuk skaii tik vienoje krvelje, o po io pakeitimo bent
vienoje vietelje dvejetainje iraikoje vietoj 1 atsiras 0 (akmenuk skaiius sumaja),
ir tas pradings vienetukas sugadins nulin NIM sum. Tarkime antrasis aidjas ima
akmenukus i didiausios krvels ir pakeii pozicij N(2, 5, 1) = 6 = 110
2
.
Pirmasis aidjas, nordamas vl palikti antrj pozicijoje (), isirenka didiausi
krvel, kurios dvejetains iraikos treiojoje pozicijoje yra 1 tokia krvel yra su 5
akmenukais. N(2, 1) = 3, tad nuimame nuo 5 krvels 53 = 2 akmenukus ir gauname
N(2, 3, 1) = 0.
103
3.1. Matematiniai aidimai Kombinatorika
Tsiame proces, kol antrasis aidjas nebegals atlikti jimo. Matome, kad lai-
minti strategija nra unikali, pavyzdiui, pirmuoju jimu pirmasis aidjas galjo su-
mainti bet kuri krvel.
Normals baigtiniai aidimai ir NIM vert
Kaip ir adjome, NIM sum apibendrinsime platesnei klasei aidim, o konkreiau
normaliems ir baigtiniams. Apibrimai:
Apibrimas. Bealiu (angl. impartial ) aidimu vadinsime tok, kuriame aibs jim,
kuriuos gali atlikti abu aidjai, yra identikos. Tokiuose aidimuose aidjai skiriasi tik
jim atlikimo tvarka. NIM aidimas yra bealis, nes abu aidjai gali nuimti po kiek
nori akmenuk; achmatai nra bealis, nes aidjas negali judinti prieininko gr.
Apibrimas. Dviej aidj aidimas yra normalus (angl. normal ), jei jis yra bealis
ir laimi tas, kuris atliko paskutin jim. NIM yra normalus aidimas.
Apibrimas. Baigtiniu aidimu vadinsime tok, kuriame pabaigos pozicijai pasiekti
aidiant optimaliai uteks baigtinio jim skaiiaus ir aidjas kiekvieno jimo metu
renkasi i baigtinio skaiiaus galim jim.
Visi toliau teorijoje nagrinjami aidimai bus baigtiniai ia prasme. Yra aidim,
kurie aidiant optimaliai baigiasi lygiosiomis, jie nra baigtiniai. Nra aiku ar ach-
matai yra baigtinis aidimas, o NIM tikrai yra. Anksiau sprendme udavin, kur ant
stalo dedamos monetos, ia aidjas galjo monet padti begalybje vieteli (jei sta-
lo pavirius didesnis u monet), tai taip pat nebuvo baigtinis aidimas ia apraoma
prasme.
Apibrimas. NIM vert tai sveikasis neneigiamas skaiius p, priskiriamas normalaus
baigtinio aidimo pozicijai Q pagal toki taisykl jis yra lygus maiausiam neneigia-
mam sveikajam skaiiui, nepriskirtam jokiai pozicijai, kuri yra pasiekiama i Q vieno
jimo metu.
aidimo pabaigos NIM vert lygi 0, i jos jau negali pasiekti jokios kitos pozicijos.
Pozicijos Q NIM vert ymsime F(Q) (vadinama Sprague-Grundy funkcija) ir raysime
skliaustuose.
Nesunku sitikinti, kad NIM vert pasako, ar nagrinjama pozicija yra laiminti ar
pralaiminti. I ties, labai panaiai NIM sum, aidimo pabaigos pozicijos vert yra (0)
(nes i jos negalima patekti joki kit pozicij), i bet kurios pozicijos turinios vert
nelygi (0) galima pereiti pozicij su verte (0) (pagal apibrim), o i pozicijos su
verte (0) negalima pereiti pozicij su verte (0) (vlgi pagal apibrim). Tad aidjas,
pradedantis nenulin NIM vert turinioje pozicijoje, turi laimini strategij, arba
Teorema. Normalaus baigtinio aidimo pozicijos su NIM verte (0) yra pralaiminios,
o visos kitos laiminios.
Pasinaudodami tuo isprskime jau matyt nesudting pavyzd:
Pavyzdys. Ant stalo yra n akmenuk. aidjas gali nuimti bet kok akmenuk skaii
nedidesn u k. aidjai A ir B jimus atlieka pakaitomis, pradeda A. Laimi tas
aidjas, kuris nuima paskutin akmenuk. Kuris aidjas laims su atitinkamais n?
104
3.1. Matematiniai aidimai Kombinatorika
Sprendimas. Raskime vis pozicij NIM vertes. Pradkime nuo pradi pozicijos su
nuliu akmenuk vert (0), nes i jos negalima patekti niekur kitur. Pozicijos su vienu
akmenuku vert (1), nes i jos galima patekti tik vert (0) turini pozij. Pozicijos su
dviem akemenukais akmenukais vert (2) (jei tik k ,= 1), nes i jos galima patekti tik
vertes (0) ir (1) turinias pozicijas. Aiku, kad taip tsdami gauname, jog jei akmenuk
yra n, kur n k, tai pozicijos NIM vert bus lygi (n).
Jei akmenuk yra n = k + 1 tai negalima patekti pozicij, kurios NIM vert 0
(negalime nuimti vis akmenuk), tad ios pozicijos vert (0) (maiausias neneigiamas
sveikasis skaiius, kuris nra NIM vert jokios pozicijos, kuri galima pasiekti i turimos
vieno jimo metu).
Jei akmenuk yra n = k + 2, tai i jos galima patekti pozicij, kurios NIM vert
0, bet negalima patekti pozicij, kurios NIM vert 1, tad ios pozicijos vert 1. Taip
tsdami gauname, kad bendru atveju pozicijos su n akmenuk NIM vert bus lygi n
dalybos i k + 1 liekanai.
Tad, kaip jau ir tikjoms, jei A pradeda pozicijoje, kurios akmenuk skaiius dalijasi
i k + 1 (t.y. su NIM verte 0), tai jis pralaims. Kitu atveju A laimi.
Ir dar vien:
17 Pavyzdys. Ant stalo yra n akmenuk. aidejas gali nuimti 1, 3 arba 8 akmenukus.
Matemagikas ir B jimus atlieka pakaitomis, pradeda Matemagikas. Laimi tas aidejas,
kuris nuima paskutin akmenuk. Kuris aidejas laims su atitinkamais n?
Sprendimas. Akmenuk skaiiams 0, 1, 2, 3, 4, 5, 6, 7, 8, 9, 10 atitinkamai priskiriame
NIM vertes lygias 0, 1, 0, 1, 0, 1, 0, 1, 2, 3, 2 (sitikinkite!). Atlikdami i procedra
didesniems skaiiams pastebime, kad NIM verts kinta periodikai, periodo ilgis 11
(sitikinkite! Galite rodyti indukcikai). Nulines vertes turi visi akmenuk skaiiai,
kuri forma yra 11k, 11k + 2, 11k + 4 arba 11k + 6, kur k sveikasis neneigiamas. Jei
startin pozicija bus vienos i i form, Matemagikas pralaims, jei ne laims.
O dabar pamginkime isprsti iek tiek supaprastint NIM aidim naudodami ne
NIM sumas, bet NIM vertes.
18 Pavyzdys. NIM aidimas su dviem krvelmis, kuriose yra m ir n akmenuk. Kuris
aidjas turi laiminij strategij?
Beveik sprendimas Pradin pozicija yra (n, m), o galutin - (0, 0). I galutins pozicijos
negali patekti joki kit pozicij, tad tikrai negali patekti ir pozicij su NIM verte 0,
tad F(0, 0) = 0. Taip pat paprastai randame, kad F(0, 1) = 1, nes i ia galime nuimti
t vien akmenuk ir gauti pozicij, kurios NIM vert yra 0, o joki kit pozicij i ia
nepasieksime. I (0, 2) galime pasiekti (0, 1) ir (0, 0), kurioms jau priskirtos verts 0 ir
1, tad iai lieka vert 2. Analogikai tsdami, gauname, kad F(0, n) = n.
Kaip su (1, 1)? Galime pasiekti tik (1, 0) ir (0, 1), tad negalime pasiekti pozicijos su
NIM verte lygia 0, vadinasi 0 ir bus ios pozicijos vert.
Tsiame toliau, F(2, 1) = 3, nes galime pasiekti tik (1, 1), (0, 1) ir (0, 2), su vertmis
0, 1 ir 2.
Dar pasisteng randame, kad F(3, 1) = 2, F(2, 2) = 0, bet kuo toliau tuo NIM vertes
vis sunkiau priskirinti, pasiekiam pozicij skaiiai auga ir reikia atlikti vis daugiau
105
3.1. Matematiniai aidimai Kombinatorika
tikrinim. Tad nors ir galtume tsti ir galbt veikti udavin, kol kas pasiduodame.

Jau io udavinio metu pamatme, kad grynai mechanikai priskyrinti NIM vertes
nra paprasta. Taiau prisiminkime, kad NIM vert vadinome NIM sumos apibendri-
nimu. Ne be reikalo:
Teorema. NIM aidime pozicijos NIM suma yra tos pozicijos NIM vert.
rodymas. inome, kad aidimo pabaigos NIM suma ir NIM vert sutampa, tad jei
mums pavykt parodyti, kad i pozicijos Q, kuriai N(Q) = A, vieno jimo metu galime
patekti pozicijas, kuri NIM sumos bt visi maesni u A sveikieji neneigiami skaiiai,
bet negalime patekti pozicij, kurios NIM suma lygi A, tai parodytume, kad NIM suma
tenkina NIM verts apibrim.
Antroji dalis nesudtinga: jei N(Q) = A, tai negalime patekti kit pozicij su
NIM suma lygia A, nes kiekvieno stulpelio vienetuk skaii turtume pakeisti lyginiu
skaiiumi, o galime keisti tik vien eilut (viena krvel).
NIM sprendime parodme, kad jei A nelygu 0, tai i turimos pozicijos galime patekti
pozicij, kurios NIM suma lygi 0. Telieka parodyti, kad galime patekti ir pozicijas,
kuri NIM sumos bt visi maesni u A sveikieji teigiami skaiiai. Tam usiraome
A dvejetaine iraika. Joje yra kakiek vienet, nes A ,= 0. Tarkime, kad norime
gauti pozicij, kurios NIM suma bt B. Atsiras bent vienas toks stulpelis, kuriame
skaiiuje A yra vienetukas, o skaiiuje B jau nuliukas, nes A daugiau u B. Isirenkame
kairiausi toki stulpel. Nesunku sitikinti, kad visi A ir B skaitmenys kairiau io
stulpelio sutampa. Tada isirenkame toki krvel, kurios dvejetainje iraikoje iame
stulpelyje yra vienetukas. vienetuk pakeiiame nuliuku, tada visus deiniau esanius
skaitmenis galime pakeisti kaip norime, vistiek gausime maesn krvel negu turjome.
Nesunku suprasti, kad juos galsime pakeisti taip, kad gautume pozicijos NIM sum
lygi B.
Apsiginklav ia teorema galime lengvai baigti skaiiuoti praeito pavyzdio NIM ver-
tes, bet i naudos neperdaugiausia NIM ir taip mokjome sprsti. Kur kas spdingiau
i teorem panaudosime sujung su kita, apibdinani suminius aidimus:
Apibrimas. Dviej aidim H ir G suma H + G vadinsime tok aidim, kur abu
aidimai yra aidiami vienu metu. Tai yra, aidjas renkasi, kuriame aidime atlikti
jim ir j atlieka.
Teorema. Jei turime du bealius normalius baigtinius aidimus A ir B, kuri pozicij
Q
A
ir Q
B
NIM verts lygios F(Q
A
) = a ir F(Q
B
) = b, tai suminio aidimo A + B
pozicijos Q
A
+Q
B
vert yra lygi a b, arba
F(Q
A
+Q
B
) = F(Q
A
) F(Q
B
).
rodymas. Prisimin praeito rodymo schem (parodme, kad i pozicijos su verte A
galima patekti pocijas su visomis maesnmis u A vertmis, bet negalima patekti
kit pozicij su verte A), tarkime prieingai, kad kokiai nors pozicijai Q
A
+ Q
B
suma
F(Q
A
) F(Q
B
) netenkina NIM verts apibrimo. Tuomet turime dvi galimybes:
106
3.1. Matematiniai aidimai Kombinatorika
1. I suminio aidimo pozicijos Q
A
+ Q
B
su F(Q
A
+ Q
B
) = a b galime patekti
kit pozicij su veriu a b. Nemaindami bendrumo tariame, kad tam pasiekti
atliktume jim aidime A. Atlik jim patektume pozicij aidime A su NIM
verte c ir gautume ab = cb, o i to seka, kad a = c (sitikinkite!). Taiau a yra
aidimo A NIM vert, o pagal apibrim vieno jimo metu negalime i pozicijos
su NIM verte a pereiti kit pozicij su NIM verte a, tad gavome prietar.
2. I suminio aidimo su veriu a b negalime patekti aidim su veriu c, kur
c < ab. Taiau NIM aidime su dviem krvelmis po a ir b akmenuk egzistuoja
toks jimas kakurioje krvelje, kad gautos pozicijos NIM vert bt c. Nema-
indami bendrumo galime teigti, kad t jim reikia atlikti su krvele a nuimant
d akmenuk. Lieka pastebti, kad jei aidime A pereitume i pozicijos su NIM
verte lygia a pozicij su NIM verte lygia a d (o t visuomet galime padaryti
pagal NIM verts apibrim), tai suminio aidimo vertis a d b bt lygus c,
prietara.
Vadinasi suminio aidimo NIM vert yra dalini aidim NIM veri NIM suma.
is faktas leidia mums iarnoti aidimus. Vien aidim aisti kaip daug atskir
arba suplakti daug aidim vien krv. Svarbi informacija mums tra NIM vert,
tai viskas, kas apibdina beal aidim. Tiesa yra vienas "bet". Danai jau ir iskai-
dyt aidim NIM veri apskaiiavimas gali bti labai kebli problema arba iskaidyti
aidimo tiesiog nepavyksta. Tad ia aprayti rankiai teorikai turi isprsti kiekvien
beal normal baigtin aidim (NIM vertes visada galit apsiskaiiuoti, tereikia laiko ir
atidumo), bet deja labai sudtingiems aidimamas isprsti jums gali neutekti sauls
sistemos gyvavimo amiaus, tad gali tekti pasiplanuoti laik. Panagrinkime paskutinj
pavyzd:
19 Pavyzdys. aidimas pradedamas su keturiomis akmenuk krvelmis, kuri dydiai
3, 4, 5 ir 6. A ir B atlieka jimus pakaitomis. Galima atlikti du jimus:
1. Paimti vien akmenuk i krvels, jei joje po pamimo lieka ne maiau negu 2
akmenukai.
2. Paimti vis krvel i 3 arba 2 akmenuk.
Laimi tas, kuris atlieka paskutin jim. Kuris aidjas gali visada laimti?
Sprendimas. Iskaidykime aidim keturis suminius aidimus, po vien kiekvienai
krvelei. Tuomet tuose aidimuose krvels su 3, 4, 5 ir 6 akmenukais atitinkamai turs
NIM vertes lygias 2, 0, 1 ir 0. Pagal k tik rodyt teorem suminio aidimo pozicijos
NIM vert bus lygi 2 0 1 0 = 3, tad pirmasis aidjas laims.
Pastaba. Mes remiams faktu, kad NIM veri NIM suma yra lygi suminio aidimo
NIM vertei. Tai leidia sumin aidim iskaidyti 4 nesudtingus aidimus, isprsti
juos atskirai ir greitai visk sulipdyti atgal. is aidimas yra B5 udavinys i 1995
met Putnamo varyb. Platesn jo analiz galite rasti "The William Lowell Putnam
Mathematical Competition 19852000 Problems, Solutions, and Commentary".
107
3.1. Matematiniai aidimai Kombinatorika
Paskutiniai trichai
Jau anksiau galjome sivaizuoduoti beal aidim kaip grandin besikaitaliojani
laimini ir pralaimini pozicij. Jei tu pralaimi, tai gudriais jimais gali tik pratsti
savo kani. Dabar mes aidim sutraukme vien skaii, aidimo NIM vert. Ji ne
tik parodo, kuris aidjas laims aidim, bet leidia isiaikinti, kas laims suminius
aidimus, kuri sudedamoji nagrinjamas aidimas yra. Jei kas susidomjote ia tema,
tai galite tsti saviviet skaitindami J. H. Conway "On numbers and games" ir to
paties autoriaus su bendraygiais ileist veikal "Winning Ways for Your Mathematical
Plays"
Udaviniai
1. Merlinkas sugalvoja natrin skaii N > 1. Matekaralius nupieia N netui S
staiakampi (nebtinai lygi), sudaryt i vienetini langeli. Merlinkas i piei-
nli iburia analogikas okolado plyteles. Jis pirmasis atsilauia nuo pasirinktos
plytels okolado (lauia iilgai linij) ir j suvalgo arba suvalgo vis plytel. ji-
mai vyksta pakaitomis. Pralaimi tas aidjas, kuris nebegali atlikti jimo. Ar
Merlinkas turi laiminij strategij?
2. Ant stalo yra n akmenuk. aidjas gali paimti nedaugiau negu pus j. aidjai S
A ir B jimus atlieka pakaitomis, pradeda A. Laimi tas aidjas, kuris atlieka
paskutin jim. Kuris aidjas laims su atitinkamais n?
3. aidjai pakaitomis renkasi skaiius i aibs 1, 2, 3, 4, 5, 6, 7, 8, 9. Jei aidjas S
surenka tris skaiius, kuri suma lygi 15 - jis laimi. Kaip baigiasi aidimas, jei
aidiama optimaliai? is aidimas gali bti pakeistas gerai inom aidim A
taip, kad kiekvienas jmas turimame aidime turt vien ir vienintel atitinkam
jim A. Kas tai per aidimas A?
4. rodykite, kad jei turime n beali normali aidim A
1
, . . . , An, tai suminio S
aidimo bet kurios pozicijos NIM vert bus lygi vis aidim pozicij NIM veri
NIM sumai:
F(Q
A
1
+. . . +Q
A
n
) = F(Q
A
1
) F(Q
A
n
).
5. aidimo erdv yra n langeli ilgio juosta. aidjai pakaitomis spalvina po du S
gretimus langelius. aidia du aidjai, pralaimi tas, kuris nebegali atlikti jimo.
Kuris aidjas turi laiminij strategij su n = 9, n = 13, n = 15?
2
6. Turime staiakamp gretasien a b c, a, b, c N, kurio apatiniame kairia- S
me artimesniame aidjams kampe tupi ak. aidjai gali j stumtelti vir,
dein arba tolyn nuo savs (Ta treia trajektorija). Laimi tas, kuris nutpdo
ak prieingame gretasienio kampe. (Virutiniame deiniame tolimajame). Du
aidjai jimus atlieka pakaitomis. Kok sry turi tenkinti a, b, ir c, kad pirmasis
aidjas turt laiminij strategij?
7. [Misere NIM] aidiamas NIM, bet pralaimi tas, kuris atlieka paskutin jim. S
2
Tai Project Euler 306-tas udavinys. Tai gana domus projektas tiems, kurie nra super progra-
muotojai. Galite pabandyti isprsti ir vis udavin. 301-as udavinys nagrinja NIM, jei paiekosite,
rasite ir daugiau udavini tinkam iam skyreliui
108
3.1. Matematiniai aidimai Kombinatorika
Kaip aisti turint vien, dvi, n krveli?
109
4 SKYRIUS
GEOMETRIJA
4.1 anga
iame skyriuje mokysims sprsti geometrijos udavinius. Geometrija reikalauja kiek
kitokio mstymo nei algebra ar kombinatorika, ir dl to dalis olimpiadinink geometrijos
nelabai mgsta/moka ir geometrijos udaviniams sprsti renkasi algebrinius metodais -
kompleksinius skaiius ar trigonometrij. Deja, nemaos dalies udavini iais metodais
nemanoma isprsti, o bandant prarandama daug laiko. Todl skyriaus tikslas yra i-
mokyti mstyti geometrikai, lavinti pastabum, surasti trump sintetin" sprendim,
kur greiiausiai udavinio krjai turi kaip ocial. inoma, tai nereikia, kad visada
yra vienas geriausias sprendimas, o ir ne kiekvienas moksleivis turi gabum ar patir-
ties pastebti gerokai neakivaizdius dalykus. Tam geometrijos skyrelis sukurtas taip,
kad tikt mokytis tiek jaunesniesiems moksleiviams, kurie dar tik pradjo mokytis geo-
metrijos mokykloje, tiek vyresniems, norintiems imokti paprasiausi ir efektyviausi
gudrybi.
Geometrijos udavinius lengva suskirstyti pagal tem, todl udaviniai yra surinkti
vos i keletos olimpiad - daugiausia Peterburgo miesto ir Miest turnyro. Jie yra pa-
naaus sunkumo Lietuvos Respublikins olimpiados udavinius, ir todl lengvesni nei
kit skyri udaviniai. Jie nebtinai surikiuoti pagal sunkum, bet pirmieji daniausiai
lengvesni nei paskutiniai. Sunkiausieji gali bti kietas rieutlis net ir patyrusiems ve-
teranams, bet tikrai yra isprendiami. Autorius silo tiesiog sprsti i eils, o ustrigus
prie udavinio imti kit. Reikia paminti, kad skyreliai yra idstyti eils tvarka, t.y.
prie sprendiant udavinius reikt bent bti perskaiius, kas parayta ankstesniuose
skyreliuose.
Kai kurie pastebs, kad kitaip nei daugumoje kit geometrijos knyg, nemokyk-
lins" teorijos yra gana nedaug, o nauj teorem tik keletas. Taip yra dl to, kad
knyga skirta grynai ruotis olimpiadoms ir palikti tiktai praktin pritaikym turintys
faktai. O ir knyga dar bus papildoma ateityje. Vietoje teorem yra dtos gerai i-
nomos lemos, kurias dert imokti mintinai. Tai tiesiog naudingos gudrybs, kurias
panaudojus olimpiados metu reikia arba rodyti, arba tiesiog terpti sprendim.
4.1. anga Geometrija
Keletas patarim, kaip brti brinius
Geometrijos udaviniai neatsiejami nuo brini, todl mokti greitai ir graiai nubrti
reikiam brin yra nekanojamas ir nelengvai iugdomas gdis; nors kiek toliau kny-
goje vietomis rasite pastab, kaip tai padaryti, taiau bendri pastebjimai yra surayti
ia: pirma, jei duotas bet koks trikampis, keturkampis ar kitokia gra, tai visomis
igalmis stenkits kad brinyje tas trikampis nebt statusis, lygiaonis, ar neduok
Dieve, lygiakratis, o keturkampis nebt rombas, lygiagretainis ar trapecija. Slygai
tai neprietaraus, bet kartais trukdys sprsti, nes, pavyzdiui, lygiaoniame trikampyje
auktins ir pusiaukampins pagrindai brinyje sutaps arba bus labai arti ir maiysis,
arba jei nubrite ABCD lygiagretain, tai keturkampyje ABCD kratins AB ir CD
kirsis kur nors u js popieriaus lapo rib , ir todl negalsite paymti AB ir CD
sankirtos tako. Nubrti smail nelygiaon trikamp yra nemenkas ikis, nes visada
atsiras dvi kratins, nesiskirianios per daugiau nei 30 proc., ir ios kratins brinyje
bus panaaus ilgio. Paprast gantinai nelygiaon smail trikamp matote paveiksllyje
emiau.
A
B C
Taip pat brdami brinius nebijokite brti daug kart: jei nepavyko nubrti
graiai i pirmo karto, brkite i naujo, o ne bandykite pataisyti. Taip pat nebrkite
per mao brinio, nes gali pradti maiytis raids, kampai ir pan. Kiek gudus galima
ivis briniuose nepalikti raidi, o jas pridti tik isprendus ir uraant sprendim. Ir
svarbiausia, brkite tikslius brinius, t.y. jei slyga rao, kad AB = CD, tai stenkits,
kad brinyje bent jau panaiai bt. Jei reiks, brkite kad ir 10 skirting brini,
kol gausite tinkam. Viso ito reikia dl 2 prieasi: pirma, tai leidia pasitikrinti,
ar gerai supratote slyg: jei reikia rodyti kad AB | CD, o js visi briniai yra
labai tiksls ir visuose briniuose AB CD, tai tikriausiai kakur padarte klaid;
ir antra, tiksls briniai padeda isprsti udavinius, gali suteikti vag, pavyzdiui,
jei reikia rodyti, kad keturkampis ABCD yra brtinis, o visuose js briniuose
ABCD atrodo labai panaus kvadrat, tai galbt ABCD i tikrj yra kvadratas ir
rodyti, kad ABCD yra kvadratas yra lengviau. Taip pat nepamirkite pasiymti vis
udavinyje duot slyg. Paskutinis patarimas: jei jau pavyko nusibrti ger brin,
bet nekyla joki mini kaip isprsti udavin, tai pabandykite persibrti brin,
tiktai apverst ar kitu kampu ( arba, inoma, pasukti lap). Paprastai naujo tokio pat
brinio nusibrimas daug naudos neduoda, bet nusibrus pasukt ar apverst gali
kilti nauj idj.
111
4.1. anga Geometrija
Btini geometrijos pagrindai
Jeigu dar nesimokote vienuoliktoje klasje, tai mokykloje dar nesimokte viso mokykli-
ns geometrijos kurso. iaip iame geometrijos skyriuje tikimasi, kad inote/mokate j
vis, tad jeigu dar kako neinote i mokyklos kurso, geriausia bt nueiti bibliotek ir
pasiimti visus matematikos vadovlius iki 10 klass ir imokti vis geometrij - geomet-
rijos mokykloje yra labai nedaug ir ji gana lengva (stereometrijos mokytis nebtina).
Taiau jei esate atuntokas ar devintokas ir bijote, kad nesuprasite kosinus teoremos,
nenusiminkite - pirmuosiuose poskyriuose turt pakakti tiek geometrijos, kiek yra iki
9 klass, be to, skyreliuose svarbiausia teorija bus duota.
112
4.2. Udaviniai apilimui Geometrija
4.2 Udaviniai apilimui
iame skyrelyje sudti nesudtingi udaviniai. Beveik visi jie yra i septintok, atun-
tok ar devintok olimpiad, ir todl yra vieni lengviausi kokie gali bti olimpiadoje.
Beveik visi jie yra apie paprastus objektus - tieses, trikampius ir kampus. Priminsime
kelet pagrindini svok ir teigini, kurie nevisuomet yra akcentuojami mokykloje,
taiau danai sutinkami olimpiadose.
Danai pamirtamos savybs ir maiomos svokos
Apibrimas. Trikampio Pusiaukratin eina i trikampio virns prie t virn
esani kratin ir dalija j pusiau. Trikampio Pusiaukampin eina i trikampio vir-
ns prie t virn esani kratin ir dalija prie tos virns esant kamp pusiau.
Teiginys (Trikampio pusiaukampins savyb). Trikampio ABC pusiaukampin AD
dalija prieing kratin dvi atkarpas BD ir DC, kuri ilgi santykis yra lygus kit
dviej to trikampio kratini AB ir AC ilgi santykiui
BD
CD
=
AB
AC
.
rodymas. Nubrkime per tak C ties, lygiagrei atkarpai AB. Tegu i ties kerta
pusiaukampins AD tsin take E. Tada ABD = DCE, BAD = CED, todl
trikampiai ABD ir CDE panas pagal 3 kampus, be to, trikampis ACE yra lygiaonis,
todl
AB
BD
=
CE
DC
=
AC
DC
( prisiminkite, kad
a
b
=
c
d

a
c
=
b
d
), k ir reikjo rodyti.
A
C B D
E
Teiginys (Trikampio priekampio savyb). Trikampio ABC kampai tenkina lygyb A+
B = 180

C. Geometrikai tai reikia, kad kamp A ir B suma yra lygi ioriniam


kampui C (kitaip inomam kaip kampo C priekampiui). Paveiksllyje emiau dviej
ali kamp suma yra lygi raudonam kampui (kampo C priekampiui).
113
4.2. Udaviniai apilimui Geometrija
A
B C
Teiginys. Paprasta, bet danai naudojama trikampi panaumo savyb: jeigu trikam-
piai ABC ir A

tenkina BAC = B

ir
AB
A

=
AC
A

, tai tada trikampiai


ABC ir A

yra panas.
A

A
C B
Esmin panaij trikampi savyb: panaij trikampi atitinkam komponent
(turima omenyje tuos, kurie matuojami ilgio vienetais) santykis yra lygus t trikampi
panaumo koecientui, o atitinkami kampai tarp atitinkam tiesi ar atkarp pana-
iuose trikampiuose yra lygs. Pavyzdiui, tarkime, kad trikampiai ABC ir A

yra
panas ir
AB
A

=
BC
B

=
CA
C

= k.
Tada, pam takus D ir D

ant atitinkamai BC ir B

taip, kad
BD
DC
=
B

, gausime
BAD = B

bei
AD
A

= k. Taip pat i karto gauname, kad panaij trikampi


atitinkam auktini, pusiaukratini ir pusiaukampini ilgi santykis yra lygus pai
trikampi kratini ilgi santykiui (trikampi panaumo koecientui).
A
C B D
A

Apibrimas. Sakome, kad takas A yra simetrikas takui A

tako O atvilgiu,
jeigu takas O yra atkarpos AA

vidurio takas.
A
B A

O
114
4.2. Udaviniai apilimui Geometrija
Apibrimas. Takas A yra simetrikas takui A

tiess l atvilgiu, jeigu ties l


eina per atkarpos AA

vidurio tak ir yra jai statmena. Tokiu atveju ties l vadinama


simetrijos aimi.
A
B
C
A

l
Pagrindiniai sprendimo bdai
Joki ypating triuk su lengvais udavinias nra; pagrindinis sprendimo bdas yra
turbt sprendimas kampais - suymti visus svarbius kampus kintamaisias, paymti
lygius kampus, tada iekoti panai, vienod ar lygiaoni trikampi, taikyti mo-
kyklines savybes ir panaiai. Nepamirkite atidiai perskaityti slygos ir pirmiausia
paymti tai, kas duota slygoje.
Nerayta udavini Rask kamp" taisykl: jeigu udavinio slygoje nra nurodyta
joki specini detali, t.y nenurodyta jokie kamp dydiai ar kratini ilgiai, o slygoje
prao rasti kokio nors kampo dyd, tai tas kampas greiiausiai bus 30

kartotinis arba
45

. Kiek reiau kampas bna 15

kartotinis, o itin retais atvejais pasitaiko, kad tas


iekomas kampas yra 18

kartotinis. Tad vos pamaius tok udavin geriausia bt


nusibrti kuo tikslesn brin ir pairti, ar iekomas kampas panaus 30

, 60

arba
90

, ir greiiausiai tai ir bus atsakymas; kitu atveju reikia bandyti kitus 15

kartotinius.
Pavyzdys. Trikampyje ABC pusiaukampin ir pusiaukratin i virns A sutampa.
rodyti, kad ABC lygiaonis.
Sprendimas. Tegu M yra BC vidurio takas. Tada i pusiaukampins savybs
AB
AC
=
BM
MC
= 1, todl AB = AC.
Pavyzdys. Duota trapecija ABCD su pagrindais AD ir BC. rodyti, kad AD vidurio
takas, BC vidurio takas bei AC ir BD sankirtos takas guli vienoje tiesje.
Sprendimas. Tegu E yra BC vidurio takas, F yra AD vidurio takas, o Gyra trapecijos
striaini sankirtos takas. Tada nesunkiai i trij kamp poymio trikampiai BGC
ir AGD yra panas, todl mes galime taikyti savyb, pamint aukiau: abiejuose
iuose trikampiuose mes nubriame atitinkamas pusiaukratines GE ir GF, ir, kadangi
atitinkami kampai tarp atitinkam atkarp yra lygs (iuo atveju kampai tarp kratins
ir pusiaukratins), mes gauname, kad DGF = BGE ir todl takai F, G, E guli
vienoje tiesje.
115
4.2. Udaviniai apilimui Geometrija
A
B C
D
E
F
G
Pavyzdys. Duotas ikilasis keturkampis ABCD. striaini BD ir AC vidurio stat-
menys kerta kratin AD atitinkamai takuose X ir Y taip, kad X yra tarp A ir Y
Pasirod, kad BX | CY . rodyti, kad BD AC.
A
B
C
D
E
F
I
H
X Y
Sprendimas. Tegu takai H, E, I, F yra atitinkamai striaini sankirtos takas, striai-
ns BD vidurio takas, t vidurio statmen sankirtos takas bei striains AC vidurio
takas (r. paveiksll viruje). Mums reikia rodyti, kad BD AC. Tai bt tas
pats, kaip ir rodyti, kad HEIF yra staiakampis, o tai yra ekvivalentu XIY = 90

.
Pastebkime, kad trikampiai XBD ir ACY yra lygiaoniai. Tada trikampyje XIY
IXY + IY X = EXD + FY A =
BXD
2
+
CY A
2
=
CY D
2
+
CY A
2
= 90

, todl
XIY = 90

, k ir reikjo rodyti.
Pastebkime, kad sprsdami mes ne puolme tiesiai rodinti, kad BD AC, o
suradome ekvivalent teigin kur rodyti buvo lengviau. Taip mokti pastebti tokius
ekvivalenius faktus yra svarbu ne tik geometrijoje, bet ir kitose matematikos akose,
mat atsiranda pasirinkimo laisv - galima isirinkti, k bandyti rodyti. Nemaa dalis
olimpiadini geometrijos udavini yra dirbtinai pasunkinami tokiu principu.
Pavyzdys. Ant staiojo trikampio ABC ambins AB paimti takai M ir N tokie,
kad BC = BM ir AC = AN. rodyti, kad MCN = 45

C
A B
M N
116
4.2. Udaviniai apilimui Geometrija
Sprendimas. Pastebkime, kad trikampiai BCM ir NCA yra lygiaoniai. Tada
MCN = (BCN +NCM) + (NCM +MCA) (BCN +NCM +MCA)
= BCM +NCA90

=
180

CBM
2
+
180

CAN
2
90

= 45

,
ko ir reikjo.
Mes iame pavyzdyje pasinaudojome viena gerai inoma lema ir rodme j: jeigu
i tako O ieina atkarpos OA, OB, OC, OD taip, kaip parodyta paveksllyje emiau
(tokia paia tvarka), tai tada AOC + BOD = AOD + BOC. Tokiu atveju, jei
inome 3 i 4 ios tapatybs kamp, tai galime nesunkiai rasti ir ketvirtj.
A
B
C
D
O
Udaviniai
1. rodyti, kad keturkampio kratini vidurio takai yra lygiagretainio virns. S
2. Trikampyje ABC nubr pusiaukratin BD ir auktin BE. Pasirod, kad S
kampas B dabar padalintas tris lygias dalis. Rasti trikampio kampus.
3. Trikampyje ABC nubr pusiaukampines AD ir BE. Pasirod, kad BEA = S
BAE = ADC. Rasti trikampio kampus.
4. Trikampyje ABC ant BC yra takas D toks, kad DC = AC = AB ir AD = BD. S
Rasti trikampio kampus.
5. Trikampyje ABC BE ir CF yra auktins, o D yra BC vidurio takas. Jei S
DEF yra lygiakratis, rodykite, kad A = 60

6. Ant lygiagretainio ABCD kratins AB (arba ant jos tsinio) paimtas takas M S
toks, kad MAD = AMO, kur O - lygiagretainio striaini sankirtos takas.
rodyti, kad MD = MC.
7. Duotas ikilasis keturkampis ABCD toks, kad jo striaines statmenos ir kertasi S
take O, BC = AO. Takas F paimtas toks, kad CF CD ir CF = BO. rodyti,
kad ADF yra lygiaonis.
8. Duotas trikampis ABC su A = 60

. N yra AC vidurio statmens ir AB S


sankirta, o M yra AB vidurio statmens ir AC sankirta. rodyti, kad MN = BC.
117
4.2. Udaviniai apilimui Geometrija
9. Duotas trikampis ABC toks, kad kampo A pusiaukampin, kratins AB vi- S
durio statmuo ir auktin i tako B kertasi viename take. rodyti, kad kampo
A pusiaukampin, kratins AC vidurio statmuo ir auktin i tako C kertasi
viename take.
10. Ant trikampio ABC kratini AB, BC, CA atitinkamai paimti takai C

, A

, B

. S
inoma, kad AC

= B

C, CB

= A

B, BA

= C

A. ro-
dyti, kad A

, B

, C

- kratini vidurio takai.


11. Duotas trikampis, jo pusiaukampini sankirtos takas sujungtas su virnmis, S
ir taip gauti 3 maesni trikampiai. Vienas j panaus pradin. Rasti trikampio
kampus.
12. Ant trikampio ABC kratini AB, BC, CA atitinkamai paimti takai C
1
, A
1
, B
1
. S
Ar atkarp AA
1
, BB
1
, CC
1
vidurio takai gali bti vienoje tiesje?
13. Duotas kvadratas ABCD, jo viduje takas M. rodykite, kad trikampi ABM, S
BCM, CDM ir DAM pusiaukratini susikirtimo takai taip pat yra kvadrato
virnes.
14. Trikampyje dvi auktins yra ne trumpesns nei kratins kurias jos remiasi. S
Rasti trikampio kampus.
15. Duotas trikampis ABC su A = 60

, pusiaukratin CM ir auktin BN kertasi S


take K, CK = 6, KM = 1. Rasti trikampio ABC kampus.
16. Ant trikampio ABC kratini AB ir BC atitinkamai paimti takai D ir E tokie, S
kad
AD
DB
=
BE
EC
= 2 ir ACB = 2DEB. rodyti, kad ABC lygiaonis.
118
4.3. Panaieji trikampiai ir brinio papildymai Geometrija
4.3 Panaieji trikampiai ir brinio papildymai
Sprsdami praeito skyrelio udavinius ar skaitydami j sprendimus greiiausiai paste-
bjote, kad daugumoje j reikjo kak papildomai paymti - tak, ties ar atkarp,
ir tik tada sprendimas tapdavo poros eilui ilgio (pavyzdyje emiau matote ret i-
imt, kai utenka originalaus brinio). Sugebjimas pastebti, k pribrti yra turbt
svarbiausias raktas skmingam olimpiadini udavini sprendimui, nes beveik vis u-
davini sprendimai traukia kit objekt (tak ar tiesi), nei duota slygoje. iame
skyrelyje bus duota keletas paprast patarim, kurie kartais suteikia idj, k ir kur
pribrti.
Pavyzdys. Duota trapecija ABCD su pagrindais AD ir CB. striains kertasi take
E. Per E ivesta ties, lygiagreti pagrindams, kerta AB take F ir CD take G. rodyti,
kad GE = FE.
A
B C
D
E
F G
Sprendimas. Trikampiai BEF ir BAD, BEC ir AED, CEG ir CAD yra panas.
Taigi,
AD
FE
=
BD
BE
=
BE+ED
BE
= 1 +
ED
BE
= 1 +
EA
EC
=
AC
EC
=
AD
EG
, taigi EG = FE.
Panaieji ir vienodieji trikampiai
Geometrijos udavini briniuose pagal slyg visada reikia susiymti lygius kampus
ir lygias atkarpas. Taip yra dl to, kad tada nesunkiai galima pastebti panaiuosius
trikampius, o kaip vliau pamatysime, ir brtinius keturkampius bei kitokias gras.
Panaieji trikampiai yra vienas svarbiausi sprendimo bd olimpiadinje geometrijoje,
todl juos pastebti yra labai svarbu. Vis dlto, j kartais brinyje nebna ir pasimato
tik papildius brin. Todl danai brin papildyti reikia taip, kad atsirast panaij
trikampi. Tai gali atrodyti per daug abstraktus patarimas, bet yra keletas idj, kurios
danai pasiteisina:
Kad atsirast panaus trikampis, daniausiai tereikia nubrti tik viena atkarp.
Jei turime trikamp S, ir atrodo, kad bt naudinga turti kit trikamp, pana
S, tai brinyje verta iekoti kampo, kuris lygus vienam i S kamp (tam, inoma,
reikia bti susiymjus lygius kampus brinyje). Tada j galime panaudoti kaip
pagrind panaiojo trikampio statybai.
Pavyzdys. Smailiajame trikampyje ABC ant AC ir AB atitinkamai paimti ta-
kai K ir L taip, kad KL | BC ir KL = KC. Ant kratins BC paimtas takas
M taip, kad KMB = BAC. rodyti, kad KM = AL.
119
4.3. Panaieji trikampiai ir brinio papildymai Geometrija
A
B C
K L
M N
Sprendimas. Paimkime tak N (kit negu C) ant BC taip, kad KL = KC =
KN. Tada KCN yra lygiaonis, taigi LKA = BCA = KNM. Pagal slyg
KMN = LAK, taigi trikampiai KNM ir LKA yra vienodi pagal kratin ir
3 kampus, ir todl LA = KM.
Vienodus trikampius galime pribrti susirad ne tik vienodus kampus, bet ir
vienodas atkarpas, t.y. jei trikampis S turi kratin, lygi a, o brinyje yra kita
kratin lygi a, tai j galima pabandyti panaudoti trikampio, tokio paties kaip
ir S, pagrindui (galima sakyti, padarome trikampio kopij). Danai udavinio
slyga sueruoja, kur tai daryti.
Pavyzdys. Kvadrate ABCD ant kratini BC ir CD atitinkamai yra paimti
takai K ir M taip, kad AM yra kampo KAD pusiaukampin. rodyti, kad
AK = DM +BK.
A B
C D
K
M E
Sprendimas. Geriausia bt kaip nors panaudoti tai ko prao, t.y AK = DM +
BK. Trikampio KAB kratin AB yra tokio pat ilgio, kaip ir kitos kvadrato
kratins, tai prie vienos j galima perkelti trikamp KAB. Mes pasirenkame
kratin AD - taigi pastatome trikamp, tok pat, kaip ir KAB prie kratins
AD. Tada DM + BK = DM + ED = EM - tai jau nemaas pasiekimas, nes
radome atkarp, kurios ilgis yra DM +BK. Taigi reikia rodyti, kad EM = AK,
arba, kad EM = AE. Tam tereikia rodyti, kad AEM yra lygiaonis, kas beveik
akivaizdu: AMD = MAB = MAE, ko ir reikjo.
120
4.3. Panaieji trikampiai ir brinio papildymai Geometrija
Jeigu neturite joki idj, k reikt pribrti, tai tada bandykite ivesti daugyb statme-
n ir tada iekoti panai trikampi ir sudarinti lygybi grandinles, iekoti panai
staij trikampi. Tokiu atveju sprendim urayti bna sunkiau ir jis bna ilgesnis,
taiau nereikia splioti, k pribrinti ir kaip.
X+Y = Z
Kartais pasitaiko, kad udavinio slygoje duota, kad kakuri dviej atkarp ilgi,
tarkime a ir b, suma yra lygi kakokios treios atkarpos ilgiui c. Panaiai pasitaiko,
kad kak tokio reikia rodyti, kaip pavyzdyje aukiau. Norint panaudoti toki i
pairos keistok slyg daniausiai tenka veikti taip: arba ant atkarpos, kurios ilgis c,
paymti vien i tak, dalinani j atkarpas ilgi a ir b, ir tada bandyti panaudoti
t tak, arba pratsti vien i trumpesnij atkarp tiek, kad gautume atkarp, kurios
ilgis lygus c. Tuomet pratsimo ilgis bus lygus B. Taip brinyje atsiras nauja pora
lygi atkarp. Kartais vien to neutenka: atkarp, kuri ilgis yra a, b arba c gali bti
daugiau nei viena ir daniausiai jos bna pasislp ir jas i pradi reikia surasti, ir
tik tada pritaikyti it fokus. Be to, jis ne visada suveikia (nors danai vos pabandius
i karto matosi ar suveiks, ar ne).
Pavyzdys (LitMO 2010). Duota trapecija ABCD su AB | CD ir AB + CD = BC.
rodyti, kad kamp B ir C pusiaukampins kertasi ant AD.
A B
C D
E
Sprendimas. Kaip ir sako patarimas, paymkime ant BC tak E tok, kad BE = AB
ir CE = CD. Tada trikampiai ABE ir CED yra lygiaoniai. Be to, AED = 180

AEBDEC = 180

180

ABE
2

180

DCE
2
=
ABE+DCE
2
= 90

. Kamp B ir
C pusiaukampins yra staiojo trikampio ABE kratini AE ir DE vidurio statmenys,
kurie akivaizdiai kertasi ant ambins vidurio tako, ko ir reikjo.
Panaiai galima elgtis ir su udaviniais su slyga A + B = C. Pirmiausia
reikt ireikti visus svarbiausius kampus brinyje per kelet kintamj, ir tada bandyti
geometrikai interpretuoti t slyg.
Nuo kurio tako pradti?
Retais atvejais pasitaiko, kad slyga liepia brtis gr, pvz. trikamp, kuris neturi
joki ypating bruo, bet i slygos paaikja, kad nusibr mes gauname netiksl
brin. Pabandykite nubrti brin iam udaviniui:
121
4.3. Panaieji trikampiai ir brinio papildymai Geometrija
Pavyzdys. Duotas trikampis ABC, ant kampo B pusiaukampins paimtas takas M
taip, kad AC = AM, BCM = 30

. Rasti AMB.
Nesunku matyti, kad tikrai ne bet kuriam trikampiui ABC tai pavykt padaryti:
MCB nra lygus 30

visiems trikampiams ABC. Kadangi neinome koki slyg rei-


kia, kad MCB = 30

, tai negalsime nusibrti absoliuiai tikslaus brinio, net ir su


matlankiu bei liniuote. Galite pagalvoti, kad tai menkas nuostolis - apytikslis brinys
yra taip pat puikus. Taiau yra brimo bdas, kuris ne tik padeda nusibrti tokius
brinius tiksliai, bet ir kartais suteikia nauj idj sprendimui. Tai yra BRIMAS
I KITO GALO.
Kaip pavadinimas sako, reikia brti i kito galo. Tam pirmiausiai briame ne
takus A, B, C, o kamp B ir jo pusiaukampin. Tada paimame bet kok tak M ant
pusiaukampins. Tada imame bet tak C ant kampo kratins taip, kad MCB = 30

.
Tada galiausiai CM vidurio statmens ir kitos kampo kratins sankirta paymima A.
Galite sitikinti kad dabar brinys tenkina visas slygas, nors mes pakeitme tik tak
brimo tvark.
Udaviniai
1. Lygiagretainyje ABCD AB +CD = AC. Ant kratins BC yra takas K toks, S
kad ADB = BDK. Raskite
BK
KC
.
2. Trapecijos ABCD (AD, BC - pagrindai) striain AC = AD +CD, o kampas S
tarp striaini yra lygus 60

. rodyti, kad trapecija yra lygiaon.


3. Duotas lygiaonis trikampis ABC, AB = BC. Ant kratini AB ir BC ati- S
tinkamai paimti takai K ir L taip, kad AK + LC = KL. Linija, lygiagreti
BC, nubrta per tak M, kuris yra atkarpos KL vidurio takas. i linija kerta
kratin AC take N. Rasti kamp KNL.
4. Duota trapecija ABCD, AD | BC. K yra bet koks takas ant AB. Nubrta S
linija per A, lygiagreti KC, ir linija per B, lygiagreti DK. rodyti, kad ios linijos
kertasi ant CD.
5. Duotas lygiagretainis ABCD, M DC vidurio takas, H - tako B projekcija S
AM. rodyti, kad BCH yra lygiaonis.
6. Duotas trikampis ABC, M - AC vidurio taskas. Takas D ant kratins BC toks, S
kad BMA = DMC. Jei CD +DM = BM, rodyti, kad ACB +ABM =
BAC.
7. Duotas trikampis ABC, ant kratins AC paimti takai K ir L taip, kad L yra S
AK vidurio takas, o BK - kampo LBC pusiaukampin. Jei BC = 2BL, rodyti,
kad KC = AB.
8. Duotas trikampis ABC. Linija, lygiagreti AC, kerta AB ir BC atitinkamai S
takuose K ir M. AM ir KC kertasi take O. Jei KM = MC ir AO = AK, tai
rodykite, kad AM = BK.
122
4.3. Panaieji trikampiai ir brinio papildymai Geometrija
9. Duotas ikilasis keturkampis ABCD toks, kad AC = BD, be to, BAC = S
ADB, CAD +ADC = ABD. Rasti kamp BAD.
10. Duotas trikampis ABC, AF pusiaukratin, D yra AF vidurio takas, E - CD S
ir AB sankirtos takas. Jei BD = BF = CF, rodyti, kad AE = DE.
11. ABCD yra ikilasis keturkampis su CBD = CAB, ACD = BDA. ro- S
dyti, kad ABC = ADC.
12. M ir N yra atitinkamai kvadrato ABCD kratini BC ir AD vidurio takai. S
K yra bet koks takas ant spindulio CA u tako A. KM ir AB kertasi take L.
rodyti, kad KNA = LNA.
13. Duotas trikampis ABC. A
1
, B
1
, C
1
yra atitinkamai BC, CA, AC vidurio takai. S
Tada ant C
1
B
1
pratsimo B
1
pus paimtas takas K toks, kad B
1
K =
BC
4
.
Duota, kad AA
1
= BC. rodyti, kad AB = BK.
14. Duotas trikampis ABC. D-kratins AC vidurio takas. Ant BC paimtas takas S
E toks, kad BEA = CED. Rasti
AE
DE
.
15. Duotas kvadratas ABCD, ant BC ir CD atitinkamai paimti takai E ir F S
taip, kad EAF = 45

. BD kerta AE take G, o FA take H. rodyti, kad


GH
2
= HD
2
+BG
2
.
16. Smailiajame trikampyje ABC ivesta auktin CH. Pasirod, kad AH = BC. S
rodyti, kad kampo B pusiaukampin, auktin AF i kampo A ir ties, einanti
per H ir lygiagreti BC, kertasi viename take.
17. Trikampyje ABC nubrtos pusiaukampins AA
1
, BB
1
, CC
1
. Jeigu C
1
A
1
yra S
BC
1
C pusiaukampin, tai rodykite, kad B
1
C
1
yra kampo AC
1
C pusiaukam-
pin.
18. Duotas ikilasis keturkampis ABCD toks, kad B = C ir CD = 2AB. Ant S
tiess BC parinktas takas X toks, kad BAX = CDA. rodyti, kad AD =
AX.
19. Duotas lygiakratis trikampis ABC. Ant AB, AC, BC atitinkamai parinkti S
X, Y, Z taip, kad BZ = 2AY , XY Z = 90

. rodykite, kad AX +CZ = XZ.


20. Ikilajame penkiakampyje ABCDE AE = AD, AB = AC, CAD = AEB + S
ABE. rodyti, kad CD dvigubai ilgesn u trikampio ABE pusiaukratin AM.
21. Duota trapecija ABCD su pagrindais AD, BC. P, Q - AD ir BC vidurio takai. S
Pasirod, kad AB = BC, ir be to, P guli ant kampo B pusiaukampines. rodyti,
kad BD = 2PQ.
22. Duotas keturkampis ABCD toks, kad CBD = CAB ir ACD = ADB. S
rodyti, kad i atkarp BC, AC, AD galima sudti statuj trikamp.
23. Duotas trikampis ABC, AL-pusiaukampin. Pasirode, kad AL = LB. Ant S
spindulio AL pasirinktas takas K toks, kad CL = AK. rodyti, kad AK = CK.
123
4.3. Panaieji trikampiai ir brinio papildymai Geometrija
24. Trapecijoje ABCD su pagrindais AD ir BC paimtas takas E ant kratins AB S
taip, kad
AE
BE
=
AD
BC
. Tako projekcija D ant tiess CE yra takas H. rodyti, kad
AH = AD.
25. Duotas statusis trikampis su staiu kampu A (AC > AB), auktine AD. Ant S
kratins BC paimtas takas E toks, kad ED = DA, o ant kratins AC paimtas
takas F toks, kad FE ED. Rasti kamp ABF.
26. Ant trikampio ABC kratini AB ir BC atitinkamai paimti takai X ir Y tokie, S
kad AX = BY ir XY B = BAC. BB
1
-trikampio ABC pusiaukampin i tako
B. rodyti, kad XB
1
| BC.
124
4.4. Apskritimai Geometrija
4.4 Apskritimai
iame skyriuje pradsime sprsti udavinius su apskritimais; gerai imanyti tokius u-
davinius yra labai svarbu, nes daugyb geometrijos udavini olimpiadose yra vienaip
ar kitaip su jais susij. Daugiausia dmesio skirsime brtiniams keturkampiams -
apibrtines gras nagrinsime kituose skyriuose.
Tai, kas svarbiausia
ia pateiksiu svarbiausius ir naudingiausius faktus, susijusius su apskritimais. Kai kuri
j dar iame skyrelyje nereiks, bet galbt prireiks vliau.
Teiginys. Kampas, besiremiantis apskritimo lank, yra dvigubai maesnis nei icent-
rinis to lanko kampas.

2
Teiginys. Jeigu ikilasis keturkampis ABCD yra brtinis ir F yra striaini sankir-
tos takas, o E yra AB ir CD sankirtos takas, tai tada trikampiai ABF ir CDF yra
panas. Be to, trikampiai AFD ir CFB taip pat yra panas. Ir galiausiai trikampiai
ADE ir CBE taip pat yra panas. Tuomet, i panaij trikampi kratini santykio
savybi mes gauname
BF FD = AF CF bei EA EB = ED EC.
A
B
C
D
O
E
F
Teiginys (Kampo tarp stygos ir liestins savyb). Kampas tarp apskritimo stygos ir
liestins, ivestos apskritimui viename i stygos gal, yra lygus brtiniam kampui,
besiremeniam t styg i kitos jos puss.
125
4.4. Apskritimai Geometrija
Taip pat teisinga yra ir atvirkia savyb: jeigu ACB = ABD ir D yra ant
atkarpos AC, tai AB lieia apie CBD apibrt apkritim take B.
C
A
B
D
Teiginys. Jeigu i tako A ivesime apskritimui dvi liestines, tai tada tos liestins bus
vienodo ilgio. Be to, jei ties per A kerta apskritim takuose C ir D, tai trikampiai
ABD ir ABC bus panas (kaip ir trikampiai ADE ir ACE). I j panaumo gauname
AC AD = AE
2
= AB
2
.
A
B
C
D
O
E
Teiginys. Lygs kampai apskritime remiasi lygius lankus. Dl to, pavyzdiui, trikam-
pio pusiaukampin dalija apie t trikamp apibrto apskritimo lank, kur atkerta nuo
apskritimo prieinga kratin, dvi lygias dalis.
126
4.4. Apskritimai Geometrija
B
C
D
O
E

Pavyzdiui, paveiksllyje viruje lankai CE ir ED yra vienodi, taigi CE = ED ir
todl CED yra lygiaonis.
Teiginys. Jeigu du trikampiai turi tokio pat dydio kamp ir tokio pat ilgio kratin
prie t kamp, tai apie tuos trikampius apibrt apskritim spinduliai yra vienodi.
Taip pat jei du trikampiai turi vienodo ilgio kratin ir viename trikampyje kampas
prie t kratin yra lygus a, o kitame 180

a, tai abie tuos trikampius apibrt


apskritim spinduliai taip pat vienodi.
Teiginys. Kampas, besiremiantis apskritimo skersmen, yra status.
Kaip rodyti, kad keturkampis yra brtinis
Labai danai tenka rodyti, kad keturkampis yra brtinis (arba keturi takai guli
ant vieno apskritimo). Tarkime, kad tos keturkampio virns yra A, B, C, D. Tada
pagrindiniai bdai tai padaryti yra ie:
rodyti, kad keturkampio prieing kamp suma yra lygi 180

. poym galima
suformuluoti ir taip: jeigu ikilojo keturkampio kampas yra lygus prieingo kampo
priekampiui, tai keturkampis yra brtinis.
127
4.4. Apskritimai Geometrija
rodyti, kad ABD = ACD (jei B ir C yra toje paioje AD pusje).
Pavyzdys. Trikampyje ABC nubrtas statmuo AD, o M, K, L yra BC, CA, AB
vidurio takai. rodyti, kad MKLD brtinis.
Sprendimas. KDL = KAL, nes D ir A simetriki KL atvilgiu. KML =
KAL, nes KALM lygiagretainis. Taigi KDL = KML, ir todl MKDL
brtinis.
Jei AC ir BD (nebtinai striains) kertasi take E, tai A, B, C, D yra ant vieno
apskritimo tada ir tik tada, jei AE EC = BE DE.
Pavyzdys. Trikampyje ABC nubrtas statmuo AD, o i D nuleisti statmenys
DE ir DF AB ir AC atitinkamai. rodyti, kad BEFC brtinis.
Sprendimas. Kadangi ADE = ABD, tai AD
2
= AE AB. Panaiai AD
2
=
AC AF. Todl AE AB = AC AF, taigi BEFC brtinis.
A
B C
D
E
F

(Retai naudojamas) Kakuri trij atkarp i aibs AB, BC, CD, DA, AC, BD
vidurio statmenys kertasi viename take (tos trys atkarpos turi nesudaryti tri-
kampio).
rodymas. Tegu tas sankirtos takas yra O. Jis yra vienodai nutols nuo kiekvie-
nos atkarpos, ant kurios vidurio statmens jis yra, gal. Todl O nutols vienodai
nuo trij por tak, ir mes darome ivad, kad visi atstumai OA, OB, OC, OD
vienodi. Tada apskritimas su centru O ir spinduliu OA eina per visus keturis
takus.
(Retai naudojamas) ABCD yra brtinis jeigu yra takas O, toks kad OA =
OB = OC = OD.
Yra keletas kit bd, bet jie naudojami reiau ir sunkesniuose udaviniuse; arti-
miausiuose skyriuose pilnai pakaks ir i.
128
4.4. Apskritimai Geometrija
Pavyzdiai
Pavyzdys. Ploktumoje nubrti du apskritimai taip, kad vienas nra kito viduje. Jiems
nubrtos dvi bendros iorins liestins: pirmoji lieia pirm apskritim take A, o ant-
rj take B. Antroji lieia pirm apskritim take C, o antr take D. AD kerta pirm
apskritim take E, o antr take F. rodyti, kad AE = FD.
Sprendimas. AF AD = AB
2
= CD
2
= DE DA, taigi AF = DE. I ia AE = FD.
A
C
B
D
F
E

Pavyzdys (2006 Lietuvos atranka Baltijos kelio olimpiad). Duotas trikampis ABC,
F, D, E yra atitinkamai kratini AB, BC, CA vidurio takai. rodyti, kad DAC =
ABE tada ir tik tada, jei AFC = ADB.
Sprendimas. Tegu M yra pusiaukratini susikirtimo takas. Tada DAC = ABE
MDF = FBM FBDM brtinis BDA = AFC.
Pavyzdys (Gerai inoma lema). Duotas ikilasis keturkampis ABCD toks, kad DC =
CB ir DAC = CAB. rodyti, kad tas keturkampis yra arba deltoidas, arba brtinis.
Sprendimas. Tegu DC = CB = a. Iveskime statmenis CF ir CE i C atitinkamai
AB ir AD. Pasiymime ant AE ir AF po du takus D
1
, D
2
, B
1
, B
2
, nutolusius nuo C
per a. Kadangi C yra ant kampo A pusiaukampins, tai CF = CE. Tada trikampiai
CFB ir CDE yra vienodi pagal 2 kratines ir kamp. Tokiu atveju mes turime keturis
skirtingus atvejus:
Brinyje B = B
1
ir D = D
1
. Tokiu atveju AD = AE DE = AF BF = AB.
Taigi keturkampis yra deltoidas.
Brinyje B = B
1
ir D = D
2
. Tada ABC = 180

CBF = 180

ADC,
taigi trikampis yra brtinis.
Brinyje B = B
2
ir D = D
1
. ia taip pat brtinis.
Brinyje B = B
2
ir D = D
2
. Dabar keturkampis ne ikilasis (isigims deltoi-
das).
129
4.4. Apskritimai Geometrija
A
B
1
C
D
1
E
F
B
2
D
2

Pavyzdys (Lietuvos TST 2010). Rombo ABCD striainje AC ir kratinje BC ati-


tinkamai parinkti takai M ir N tokie, kad DM = MN (N nesutampa su B). AC ir
DN kertasi take P, o tiess AB ir DM take R. rodyti, kad PR = DP.
Sprendimas. Pastebkime, kad keturkampis CDMN tenkina prie tai buvusios lemos
slygas. Kadangi CD ,= CN, tai keturkampis nra deltoidas ar isigims deltoidas,
taigi yra brinis. Tada RAP = BAC = DCA = ACB = MCN = MDN =
RDP = DAP. Taigi ADPR yra brtinis su DP = PR (i vieno aukiau buvusi
teigini).
Pavyzdys (Lietuvos TST 2006?). Duotas trikampis ABC, kampas A status. M yra
BC vidurio takas. Paimkime D ant AC taip, kad AD = AM. Tegu apie AMC ir
BDC apibrti apskritimai kertasi take P. rodyti, kad CP yra kampo ACB pusiau-
kampin.
Sprendimas. Paimsime tak P

kuris tenkina tas savybes, kurias reikia rodyti takui


P, tada rodysime, kad jis tenkina tas paias savybes, kaip ir takas P, ir galiausiai
parodysime, kad jie sutampa. Taigi tegu P

yra AMC apibrtinio apskritimo ir kampo


C pusiaukampins sankirta. Tada AP

= P

M. Trikampiai AP

D ir MP

B vienodi
pagal dvi kratines ir kamp. Taigi BP

= P

D. I lemos BCDP

yra arba brtinis,


arba BC = CD. Antras atvejis yra nemanomas, taigi P

guli ant abiej apskritim, ir


P

= P.
A
B
C
P

M
D

Udaviniai
1. ABC yra trikampis. Ant AB, BC, CA atitinkamai paimti takai K, L, M taip, S
kad BLK = CLM = BAC. BM ir CK kertasi take P. rodyti, kad
keturkampis AKPM yra brtinis.
130
4.4. Apskritimai Geometrija
2. Duotas trikampis ABC, M-BC vidurio takas, AA

, BB

, CC

yra auktins, S
AB ir A

kertasi take X, o MC

ir AC take Y . rodyti, kad XY | BC.


3. Duotas statusis trikampis ABC su staiu kampu A. M yra BC vidurio takas, S
o AH yra auktin. Linija per M, statmena AC, kerta apie AMC apibrt
apskritim take P. rodyti, kad BP dalija AH pusiau.
4. Ant staiojo trikampio ABC ambins AB iorje nubrtas kvadratas ABDE. S
Stataus kampo C pusiaukampin kerta DE take F. Rasti
EF
FD
, jeigu inoma, kad
AC = 1 ir BC = 3.
5. kamp brti du apskritimai su centrais A ir B taip, kad jie lieia kampo S
kratines ir vienas kit. rodyti, kad apskritimas, kurio skersmuo yra AB, taip
pat lieia kampo kratines.
6. Trikampyje ABC AB = BC. BH-auktin, M yra AB vidurio takas, K yra S
BH ir apie MBC apibrto apskritimo sankirtos takas. rodyti, kad BK =
3R
2
,
kur R yra apie ABC apibrto apskritimo spindulys.
7. Per apskritimo e centr nubrtas apskritimas f. A ir B - i apskritim S
sankirtos takai. Liestin apskritimui f take B kerta apskritim e take C.
rodyti, kad AB = BC.
8. Du apskritimai kertasi takuose A ir B. Take A abiems apskritimams ivestos S
liestins, kertanios apskritimus takuose M ir N. Tiess BM ir BN atitinkamai
dar syki kerta apskritimus takuose P ir Q. Irodyti, kad MP = NQ.
9. Kokiu kampu is staiojo trikampio stataus kampo matoma t trikamp brto S
apskritimo projekcija ambin?
10. AK - smailiojo trikampio ABC pusiaukampin, P ir Q - takai ant pusiaukam- S
pini (ar j tsini) BB

ir CC

tokie, kad PA = PK, QA = QK. rodykite, kad


PAQ = 90

BAC
2
.
11. Du apskritimai kertasi takuose A ir B. Nubrta jiems bendra iorin liestin S
lieia pirm apskritim take C, o antr take D. Tarkime, kad takas B yra
ariau CD negu takas A. CB kerta antrj apskritim antr kart take E.
rodyti, kad AD yra kampo CAE pusiaukampin.
12. Duotas brtinis keturkampis ABCD kurio striains statmenos, o apibr- S
tinio apskritimo centras yra O. rodyti, kad statmens i O AD ilgis dvigubai
trumpesnis nei kratin BC.
13. Ant apskritimo K su centru take O stygos AB paimtas takas C. Apie AOC S
apibrtas apskritimas kerta K take D. rodyti, kad CD = CB.
14. Lygiagretainio ABCD striains kertasi take O. rodykite, kad jei apie ABO S
apibrtas apskritimas lieia BC, tai apie BCO apibrtas apskritimas lieia CD.
131
4.4. Apskritimai Geometrija
15. Duotas trikampis ABC, apie j nubrtas apskritimas. Dvi tiess eina per tak S
A ir kerta atkarp BC takuose K ir M, o lank BC (t, kuris neturi tako A)
tiess AK ir AM kerta atitinkamai takuose L ir N. Jei KLMN yra brtinis,
tai rodykite, kad ABC lygiaonis.
16. Duotas apskritimas su centru O, j atkarpa AB lieia take A. AB yra pasukta S
aplink O ir taip gauta A

. rodyti, kad AA

eina per atkarpos BB

vidurio
tak.
A
A

B
B

O
17. Duotas brtinis keturkampis ABCD. K, L, M, N yra atitinkamai kratini S
AB, BC, CD, DA vidurio takai. P yra striaini susikirtimo takas. rodyti,
kad apie trikampius PKL, PLM, PMN, PNK apibrt apskritim spinduliai
yra vienodi.
18. Apskritimas S
1
su centru O
1
eina per kito apskritimo S
2
centr O
2
. Ant S
1
taip S
pat paimtas bet koks takas C, ir i to tako C ivestos liestins apskritimui S
2
kerta apskritim S
1
takuose A ir B. rodyti, kad AB O
1
O
2
.
19. Duotas rombas ABCD su A = 120

. M ir N yra takai atitinkamai ant kratini S


BC ir CD tokie, kad NAM = 60

. rodyti, kad apie trikamp NAM apibrto


apskritimo centras guli ant rombo striains.
20. Duotas trikampis ABC. Ant kratini AB ir BC atitinkamai paimti takai X S
ir Y . AY ir CX kertasi take Z. Pasirod, kad AY = Y C ir AB = ZC. rodyti,
kad B, Z, X, Y guli ant vieno apskritimo.
21. Duotas rombas ABCD. Ant linijos CD paimtas takas K kuris nesutampa C S
ar D taip, kad AD = BK. P yra tiess BD ir atkarpos BC vidurio statmens
sankirtos takas. rodyti, kad takai A, P, K guli ant vienos tieses.
22. Trikampyje ABC nubrtos auktins BE ir AD kertasi take H. X ir Y - S
atitinkamai CH ir AB vidurio takai. rodyti, kad XY DE.
23. Duotas trikampis ABC ir takas P jo viduje toks, kad ABP = ACP ir S
CBP = CAP. rodyti, kad P yra trikampio ABC auktini susikirtimo
takas.
132
4.4. Apskritimai Geometrija
24. Duotas brtinis keturkampis ABCD, ant spindulio AD u tako D paimtas S
takas E taip, kad AC = CE, BDC = DEC. rodyti, kad AB = DE.
25. Duotas lygiaonis trikampis ABC su AB = BC. Ant AB, BC, CA atitinkamai S
paimti takai C
1
, A
1
, B
1
tokie, kad BC
1
A
1
= CA
1
B
1
= A, P - atkarp BB
1
ir CC
1
sankirtos takas. rodyti, kad keturkampis AB
1
PC
1
yra brtinis.
26. Duotas statusis trikampis su staiu kampu B. Per tak B ivesta pusiaukratin S
BM. trikamp ABM brtas apskritimas lieia AM ir AB takuose K ir L.
Jei LK | BM, raskite kamp ACB.
27. Duotas trikampis ABC su auktinmis AA
1
, BB
1
. Kampo C pusiaukampin S
kerta AA
1
ir BB
1
atitinkamai takuose F ir L. rodyti, kad atkarpos FL vidurio
takas vienodai nutols nuo tak B
1
ir A
1
.
28. Duotas deltoidas ABCD, AB = BC, AD = DC. Ant striains AC paim- S
tas toks takas K, kad BK = KA. Jei keturkampis CDKB yra brtinis, tai
rodykite, kad CD = BD.
29. Duotas kvadratas ABCD. Ant AB paimtas takas K, ant CD - L, o ant KL S
- M. rodykite, kad apie AKM ir MLC apibrt apskritim sankirtos takas
(kitas negu M) yra ant striains AC.
30. Duotas trikampis ABC, jame ivestos auktin AH ir pusiaukampin BE. i- S
noma, kad BEA = 45

. rodyti, kad EHC = 45

.
31. Duotas trikampis ABC su pusiaukampinmis AL ir BM. Jei trikampi ACL S
ir BCM apibrtiniai apskritimai kertasi ant atkarpos AB, tai rodykite, kad
ACB = 60

.
32. Trikampio ABC pusiaukampins BD ir CE kertasi take O. rodykite, kad S
jeigu OD = OE, tai arba trikampis ABC lygiaonis, arba kampas A = 60

.
33. Duotas trikampis ABC su ABC = 60

. I - brto apskritimo centas, CL S


- pusiaukampin. Apskritimas, apibrtas apie trikamp ALI, kerta AC antr
kart take D. rodyti, kad BCDL - brtinis.
34. Duotas ikilasis eiakampis ABCDEF. inoma, kad AD = BE = CF. AD S
ir CF sankirtos takas yra P, BE ir CF sankirtos takas yra R, o AD ir BE -
takas Q. Jeigu AP = PF, BR = CR, DQ = EQ tai rodyti, kad eiakampis
yra brtinis.
35. AC ir BD yra du statmeni kakokio apskritimo skersmenys. Takas K ant S
apskritimo nesutampa su takais A, B, C, D. AK ir BD kertasi take M, o DK
ir CB - take N. rodyti, kad AC | MN.
36. Duotas smailusis trikampis ABC. Jame nubrtos auktins AA
1
, BB
1
, CC
1
. S
rodyti, kad C
1
projekcijos tieses AC, BC, BB
1
, CC
1
yra vienoje tieseje.
133
4.4. Apskritimai Geometrija
37. Smailajame trikampyje ABC nubrti statmenys BD ir CE. Apskritimas su S
skersmeniu AC kerta spindul DB take F, o apskritimas su skersmeniu AB kerta
spindul EC take G ir spindul CE take H ,= G. rodyti, kad FHG+FGA =
90

.
134
4.5. Plotai Geometrija
4.5 Plotai
Iki iol beveik visi udaviniai buvo apie kampus ir kratines. Taiau geometrija nra vien
tik kampai ir kratins - nedanai, taiau pasitaiko udavini apie plotus, perimetrus,
geometrines nelygybes, tapatybes ir panaiai. io skyrelio tema yra plot udaviniai.
Trikampio ABC plot, jei nepasakyta kitaip, ymsime S
ABC
.
Tai, kas svarbiausia
Teiginys. Trikampio plotas yra lygus pusei trikampio kratins ilgio, padauginto i
auktins, nuleistos t kratin, ilgio. Bene svarbiausia ivada i to yra ta, kad tri-
kampiai, turintys t pai ar tokio pat ilgio kratin, ir auktin, nuleist t kratin,
turi vienodus plotus. Pavyzdui paveiksllyje apaioje visi trikampiai su kratine AB
turi vienodus plotus tada ir tik tada, jei ties b yra lygiagreti tiesei AB:
A B
b
Tai yra labai naudingas faktas interpretuojant slyg geometrikai ar bandant rodyti
koki nors tapatyb su plotais.
Pavyzdys. Duotas trikampis ABC, ant BC paimtas bet koks takas E, o ant CA
paimtas bet koks takas F. M ir N yra atitinkamai AE ir BF vidurio takai. rodyti,
kad S
CFM
= S
CEN
.
Sprendimas. Tegu L yra FE vidurio takas. Tada LN | CB, taigi S
CEN
= S
CLE
=
S
CFE
2
. Panaiai ir S
CFM
=
S
CFE
2
.
A
B
C
E
F
L
M
N

Teiginys. Trikampio plotas yra lygus dviej jo kratini sandaugai padaugintai i kam-
po tarp j sinusui ir padalinus i dviej; todl jeigu turime du trikampius, vien su
kratinmis a, b ir kampu tarp j, ir kit su kratinmis a, b ir kampu 180

tarp
j, tai t trikampi plotai lygs.
135
4.5. Plotai Geometrija
Teiginys. Ikilojo keturkampio plotas yra lygus striaini sandaugai, padaugintai i
kampo tarp striaini sinuso ir padalintai i dviej. Dl to, pavyzdiui, jeigu ketur-
kampio striains statmenos, tai jo plotas lygus striaini sandaugos pusei.
Pavyzdys. Keturkampis striainmis padalintas keturis trikampius. rodyti, kad
prieing trikampi plot sandaugos lygios.
Sprendimas. Tegu striains dalija viena kit keturias atkarpas, kuri ilgiai yra
a, b, c, d, o kampas tarp striaini yra . Tada iekomos plot sandaugos bus lygios
abcd(sin )
2
, abcd(sin(180

))
2
, o bet tai yra tas pats, nes sin = sin(180

).

Dar vienas naudingas triukas sprendiant vairius udavinius (ne tik geometrijos)
kuriuose reikia rodyti koki nors lygyb yra pridti ar atimti t pat dyd prie abiej
lygybs pusi. Geometrijoje kartais to pakanka isprsti udaviniui.
Pavyzdys. Duotas lygiagretainis ABCD, ant BC paimtas bet koks takas E, AE ir
BD kertasi take F. rodyti, kad S
BFE
+S
ECD
= S
AFD
.
Sprendimas. Pridedame prie abiej pusi po S
FED
ir viskas pasidaro akivaizdu.
A
B
D
C
E
F

Udaviniai
1. rodykite, kad i vis keturkampi, brt ksuoto spindulio apskritim, di- S
diausi plot turi kvadratas.
2. Duotas trikampis ABC. Per jo virnes A ir B ivestos dvi tiess, kurios padalina S
it trikamp 4 gras: 3 trikampius ir vien keturkamp. inoma, kad trij i
i gur plotai vienodi. rodykite, kad tarp t trij yra keturkampis.
3. Duotas brtas keturkampis ABCD, kurio striains yra statmenos. rodyti, S
kad laut AOC dalija keturkamp dvi lygiaplotes dalis.
4. Ikilajame eiakampyje ABCDEF AB | CF, CD | BE, EF | AD. rodyti, S
kad trikampi ACE ir BDF plotai lygs.
5. Duotas kvadratas ABCD su kurio kratins ilgis 1. Ant AB, BC, CD, DA S
atitinkamai paimti takai K, L, M, N taip, kad KM | BC ir NL | AB. Jei
BKL perimetras yra lygus 1, rasti trikampio DMN plot.
136
4.5. Plotai Geometrija
6. Ikilojo keturkampio striains dalija j keturis maus trikampius. Pasirod, S
kad dviej prieing trikampi plot suma yra lygi kit dviej trikampi plot
sumai. rodyti, kad viena i striaini dalija kit pusiau.
7. Ikilajame eiakampyje AC

BA

CB

AB

= AC

, BC

= BA

, CA

= CB

ir S
A+B +C = A

+B

+C

. rodykite, kad ABC plotas yra lygus pusei


eiakampio ploto.
8. Duota trapecija ABCD su pagrindais AB ir CD. M-AD vidurio takas. rodyti, S
kad BCM plotas yra lygus pusei trapecijos ploto.
9. Duotas ikilasis keturkampis ABCD. Paimti takai E, F ant BC tokie, kad S
BE = EF = FC. Paimti takai H, G ant AD tokie, kad AH = HG = GD.
rodyti, kad S
EFGH
=
S
ABCD
3
.
10. brtiniame keturkampyje ABCD BC = CD. rodykite, kad jo plotas lygus S
AC
2
sin A
2
.
11. Duotas brtinis eiakampis ABCDEF. Pasirod, kad AB = BC, CD = DE, S
EF = FA. rodyti, kad trikampio BDF plotas lygus pusei eiakampio ploto.
12. Duotas smailusis trikampis ABC, O-apibrto apskritimo centras, BO kerta S
apibrtin apskritim antr kart take D, o auktins i virns A tsinys ker-
ta apskritim take E. rodyti, kad trikampio ABC plotas lygus keturkampio
BECD plotui.
13. Ant lygiagretainio kratini paimta po vien tak. Keturkampio, kurio virns S
yra tuose takuose, plotas yra dvigubai maesnis nei lygiagretainio. rodyti, kad
bent viena keturkampio striain yra lygiagreti lygiagretainio kratinei.
14. Tegu ABCDE yra ikilasis penkiakampis toks, kad AB = AE = CD = 1, S
ABC = DEA = 90

ir BC +DE = 1. Rasti penkiakampio plot.


137
4.6. Apibrtins gros Geometrija
4.6 Apibrtins gros
Apskritim skyriuje daugiausia dmesio buvo skiriama brtinms groms, t.y. toms,
kurios buvo apskritim viduje. io skyrelio tema yra apibrtins gros, todl ia
svarbiausia bus tai, kas yra apskritimo iorje.
Keletas svarbiausi savybi
Pati svarbiausia io skyrelio savyb yra i:
Teiginys. Dvi liestins apskritimui i tako yra vienodo ilgio.
ir jai panai
Teiginys. Bendros iorins ar vidins liestins dviems apskritimams yra vienodo ilgio.
Kita labai svarbi savyb yra i:
Teiginys. Jei apskritimui su centru O take A ivesta liestin, tai ta liestin statmena
AO.
Vien su iais teiginiais galima issprsti nemaai udavini.
Pavyzdys. rodykite, kad jei ikilasis keturkampis yra apibrtinis, tai prieing kra-
tini sumos lygios.
Sprendimas. Tegu keturkampis ABCD yra apibrtinis, o brtas apskritimas lieia
AB, BC, CD, DA atitinkamai takuose A

, B

, C

, D

. Tada AB +CD = AA

+A

B +
CC

+C

D = AD

+BB

+CB

+DD

= AD +BC.
Teisingas ir prieingas faktas:
Teiginys. Jeigu ikilojo keturkampio prieing kratini sumos lygios, tai tas ketur-
kampis yra apibrtinis.
Taip pat yra teisingas kiek kitoks, bet taip pat svarbus faktas:
Teiginys. Ikilasis n-kampis yra apibrtinis tada ir tik tada jei jo kakuri n1 kamp
pusiaukampins kertasi viename take.
Todl, pavyzdiui, visi trikampiai yra apibrtiniai, nes j dviej kamp pusiau-
kampins kertasi viename take. Nepamirkite ir io fakto:
Teiginys. Jeigu liestins i dviej skirting tak tam paiam apskritimui yra vienodo
ilgio, tai atstumai nuo t tak iki apskritimo centro yra vienodi. Be to, keturi takai,
kuriuose keturios liestins i t dviej tak lieia apskritim yra lygiaons trapecijos
virns.
138
4.6. Apibrtins gros Geometrija
Iorins pusiaukampins
Iki iol, mindami gros kampo pusiaukampin, turdavome omeny ties, kuri dalina
gros vidin kamp pusiau ir eina i gros iors gros vid. Bet yra ir iorins pu-
siaukampins, kurios dalija gros kampo priekamp pusiau. Jos yra tiess, kurios visos
yra gros iorje. Pavyzdiui, ties d paveiksllyje emiau yra trikampio ABC kampo
C iorin pusiaukampin, o ties a yra kampo C pusiaukampin (Jeigu nepasakyta,
kad pusiaukampin yra iorin, tai ji yra paprasta):
A B
C
F
a
d
Teiginys. Kampo pusiaukampin ir iorin pusiaukampin yra statmenos viena kitai.
Iorins pusiaukampins turi savyb, labai panai paprast pusiaukampini:
Teiginys. Tegu trikampio ABC (BA > BC) kampo B iorin pusiaukampin kerta
spindul AC take D. Tada
CD
AD
=
CB
AB
Iorins pusiaukampins susijusios su pribrtinias apskritimais:
Pribrtiniai apskritimai
Mes inome, kad kiekvien apskritim galima brti apskritim, kuris yra to trikampio
viduje ir lieia visas tris trikampio kratines. Taiau yra trys apskritimai, kuri kiek-
vienas lieia vien trikampio kratin ir kit dviej kratini tsinius, kaip paveiksllyje
apaioje:
139
4.6. Apibrtins gros Geometrija
Paprastai jei nepasakyta kitaip, pribrtinio apskritimo, kuris lieia kratin BC (
ne jos tsin), centras ymimas I
A
. Panaiai kiti centrai ymimi I
B
ir I
C
. J spinduliai
atitinkamai ymimi r
A
, r
B
, r
C
.
Teiginys. Pribrtinio apskritimo prieais kamp A centras guli ant kampo A pusi-
aukampins ir ant kamp B ir C iorini pusiaukampini.
rodymas. Kadangi tas apskritimas lieia AB ir AC, tai jo centras yra ant kampo
A pusiaukampins. Taip pat kadangi jis lieia tieses AB ir BC, tai jo centras yra
ant kampo B iorins pusiaukampins. Panaiai jis yra ir ant kampo C iorins
pusiaukampins.
I ia seka tokia ivada:
Teiginys. Trikampio kampo pusiaukampin ir kit dviej kamp iorins pusiaukam-
pins kertasi viename take ( tai galima rodyti ir be pribrtini apskritim: Kadangi
dviej i i trij tiesi sankirtos takas vienodai nutols nuo vis trikampio kratini,
tai ir treia ties eina per tak).
Udaviniai
1. ia svarbus udavinys - siminkite iuos rezultatus. Tegu trikamp ABC br- S
tas apskritimas su centru I lieia kratines AB, BC, CA atitinkamai takuose
E, F, G, o pribrtinis apskritimas prie virn A lieia tas paias kratines ta-
kuose L, M, N (kaip paveiksllyje). Jei AB = c, AC = b ir BC = a , tai
tada
rodykite, kad AL = AN = s kur s =
a+b+c
2
pusperimetris.
rodykite, kad GC = BM = sc. Panaiai rodykite, kad NC = BE = sb
ir AE = AG = s a.
rodykite, kad AI AI
A
= AB AC.
rodykite, kad S = r
A
(s a), kur S yra ABC plotas
rodykite Herono formul: S =
_
s(s a)(s b)(s c)
A
B
C
E
F
G
L
M
N
I Ia
140
4.6. Apibrtins gros Geometrija
2. Trikampio ABC pribrtini apskritim spinduliai yra I
A
, I
B
, I
C
, o brtinio S
apskritimo centras yra I. rodykite, kad trikampio I
A
I
B
I
C
auktini susikirtimo
takas yra I.
3. Ikilojo keturkampio prieing kratini sumos lygios. rodyti, kad trikampi, S
gaut nubrus vien striain, brtiniai apskritimai lieiasi.
4. ABCD yra apibrtinis keturkampis, kurio prieing kratini sandaugos lygios. S
Kampas tarp vienos i kratini ir striains yra 25

. Rasti kamp tarp tos


kratins ir kitos striains.
5. Duotas kvadratas ABCD. Ant kratins BC yra takas M, o ant kratins DC - S
takas K taip, kad trikampio CMK perimetras yra dvigubai ilgesnis u kvadrato
kratin. Rasti kamp MAK.
6. Duotas apibrtinis keturkampis ABCD. Kratins AB, BC, CD, DA lieia t S
apskritim atitinkamai takuose K, L, M, N. KM ir LN kertasi take S. Jeigu
SKBL yra brtinis, tai rodykite, kad SNDM taip pat brtinis.
7. Duotas trikampis ABC. Ivestos tiess, simetrikos tiesei AC tiesi BC ir S
BA atvilgiu, ir jos kertasi take K. rodyti, kad apie trikamp ABC apibrto
apskritimo centras guli ant tieses BK.
8. Trikampyje ABC nubrtos pusiaukampins AD, BE ir CF. Jei BAC = 120

, S
tai rodykite, kad ED FD.
9. Per smailiojo trikampio ABC (AC > AB) virn A nubr pusiaukampin S
AM ir iorin pusiaukampin AN bei liestin AK apskritimui, apibrtam apie
ABC (takai M, K, N yra ant spindulio CB). rodykite, kad MK = KN.
10. trapecij galima brti apskritim. rodyti, kad apskritimai, kuri skersmenys S
yra trapecijos onins kratins, lieia vienas kit.
11. Takas O yra pribrtinio apskritimo, lieianio trikampio ABC kratin AC ir S
kratini BA ir BC tsinius, centras. D - apskritimo, einanio per takus B, A, O,
centras. rodykite, kad takai A, B, C ir D yra ant vieno apskritimo.
12. Penkiakampis ABCDE apibrtas aplink apskritim S. AB = BC = CD, BC S
lieia S take K. rodyti, kad EK BC.
13. Trikampyje ABC nubr pusiaukampines AD ir BE. Jei DE yra kampo ADC S
pusiaukampin, tai raskite kamp BAC.
14. Duotas trikampis ABC, ant spindulio CB u tako B paimtas takas D toks, S
kad BD = AB. Kamp A ir B iorins pusiaukampins kertasi take M.
rodyti, kad takai M, A, C, D guli ant vieno apskritimo.
15. Apibrtiniame penkiakampyje ABCDE striains AD ir CE kertasi take O, S
kuris yra brto apskritimo centras. rodyti, kad BO DE.
141
4.6. Apibrtins gros Geometrija
16. Duotas smailusis trikampis KEL, j brtas apskritimas su spinduliu R. iam S
apskritimui ivestos 3 liestins taip, kad KEL yra padalintas 3 staius trikampius
ir viena eiakamp, kurio perimetras yra Q. Rasti tris staiuosius trikampius
brt apskritim spinduli sum.
17. Duotas apibrtinis keturkampis ABCD. j brtas apskritimas lieia kratines S
AB, BC, CD, DA atitinkamai takuose E, F, G, H. rodykite, kad linija, jungian-
ti trikampi HAE ir FCG brtini apskritim centrus yra statmena linijai,
jungianiai trikampius EBF ir GDH brt apskritim centrus.
18. kamp brtas apskritimas su centru O. Per tak A, simetrik takui O S
vienos i kampo kratini atvilgiu, nubr apskritimui dvi liestins, kurios kerta
labiau nuo tako A nutolusi kampo kratin takuose B ir C. rodyti, kad apie
ABC apibrto apskritimo centras yra ant duotojo kampo pusiaukampins.
19. Ant trikampio ABC kratins BC paimtas takas D. trikampius ABD ir ACD S
brti apskritimai, ir nubrta bendra jiems liestin (kita nei BC), kertanti AD
take K. rodykite, kad atkarpos AK ilgis nepriklauso nuo tako D pasirinkimo.
142
4.7. Vienareikmiki udaviniai Geometrija
4.7 Vienareikmiki udaviniai
Olimpiadose retkariais pasitaiko vienareikmik udavini. Tai tokie udaviniai,
kur inomi visi kampai tarp vis tiesi, arba kitais odiais tariant, visi kampai vie-
nareikmiki. Dl ios prieasties juos patogu sprsti trigonometriniais metodais. Kita
vertus, juos danai trumpiau ir graiau galima isprsti geometriniais metodais. Taiau
bandydami surasti tok sprendim galite prarasti daug laiko, kai tuo tarpu trigonometri-
nis sprendimas greiiausiai duos vaisi. Tuo ie udaviniai primena galvoskius arba kai
kuriuos kombinatorikos udavinius - isprsti galima tik gudriai pastebjus sprendim,
arba darant ilgai ir nuobodiai. Olimpiadose sutikus tok udavin reikt tiktis, kad
yra gana paprastas geometrinis sprendimas, nes udavinys, kuris yra isprendiamas tik
trigonometriniais metodais, yra prarands savo olimpiadikum. ( Tai vienas didij
skirtum tarp realaus pasaulio udaviniai nuo olimpiadini - olimpiadiniai udaviniai
visada turi pakankamai trump sprendim ). Dl i prieasi teorijos iame skyriuje
yra nedaug.
Pavyzdys. Duotas trikampis ABC su AB = AC ir BAC = 100

. BD yra pusiau-
kampin. rodyti, kad BD +DA = BC.
Sprendimas. Tegu DC vidurio statmuo kerta BC take F. Tada DCF yra lygiaonis, ir
todl DFC = 100

. Tada ABFD yra brtinis, o BFD lygiaonis. Kadangi BD yra


kampo B pusiaukampin, tai AD = DF = FC. Taigi BD +DA = BF +FC = BC.

A
B
C D
F
80

80

40

100

20

Pavyzdys. Duotas keturkampis ABCD toks, kad AD = CD, A = 75

, D =
60

, C = 135

. E yra CD vidurio takas. Rasti


BE
ED
Sprendimas. Tegu F yra CA vidurio takas. Mes nesunkiai suskaiiuojame, kad B =
90

, BFE = BFC+CFE = 30

+60

= 90

, taigi FE = FC = FB, ir i Pitagoro


teoremos randame
BE
ED
=
BE
EF
=

2.
143
4.7. Vienareikmiki udaviniai Geometrija
A
B
C
E
F
D
60

75

135

Pavyzdys (Turkijos TST 1995). Ikilajame keturkampyje ABCD duoti kampai CAB =
40

, CAD = 30

, DBA = 75

, DBC = 25

. Raskite BDC.
Sprendimas. Nesunkiai suskaiiuojame, kad ABC lygiaonis su AB = BC. Imame
tak E ant AD tok, kad AEB = 70

. Tada vl nesunkiai suskaiiuojame, kad AEB


lygiaonis, EBC lygiakratis (pagal kamp ir dvi lygias kratines), o EBD lygiaonis
su ED = EB = EC. Taigi E yra apie CDB apibrto apskritimo centras, ir i ia
CDB =
CEB
2
= 30

.
A
B
D
C
E
30

40

40

35

25

25

35

40

Toliau pateikta savyb yra naudinga sprendiant vairius geometrijos udavinius,


bet ypa naudinga sprendiant vienareikmikus:
Teiginys (inotina lema). Tarkime, kad turime ikilaj keturkamp ABCD tok, kad
BC = AB ir ADC +
ABC
2
= 180

. Tada BC = BD = BA.
rodymas. Paimkime apskritim su centru B ir spinduliu AB. Tada is apskritimas
eina per takus A ir C. Tegu AB antr kart kerta t apskritim take E. Tada
AEC + ADC =
ABC
2
+ ADC = 180

. Taigi AECD brtinis ir todl BA =


BC = BD.
144
4.7. Vienareikmiki udaviniai Geometrija
A
B
C
D
E
Pavyzdys. Keturkampyje ABCD AB = BC = 1. Kampas B = 100

, D = 130.
Rasti BD
Sprendimas. Keturkampis ABCD tenkina visas slygas, mintas viruje:
ABC
2
+
ADC = 50

+ 130

= 180

ir AB = BC. Taigi BD = BC = BA = 1.
Udaviniai
1. Duotas kvadratas ABCD. Jo viduje paimtas takas M toks, kad MAC = S
MCD = u. Rasti MBA.
2. Duotas statusis lygiaonis trikampis ABC su AB = AC ir BAC = 90

. S
Nubrta pusiaukratin BM, o jai per tak A ivestas statmuo. rodykite, kad
is statmuo dalina BC santykiu 2:1.
3. (Langljaus udavinys) Duotas trikampis ABC su B = 20

, A = C = 80

. S
Ant AB ir BC paimti takai E ir D atitinkamai taip, kad CAD = 60

ir
ACE = 50

. Rasti kamp ADE.


4. Duotas trikampis ABC su kampais B = 90

, A = 50

, C = 40

. Paimti S
takai K ir L ant BC taip, kad KAC = LAB = 10

. Rasti
KC
BL
.
5. Duotas deltoidas ABCD, AB = BC, AD = DC, ADC = 3ACB, AE- S
trikampio ABC pusiaukampin, DE ir AC kertasi take F. rodyti, kad CEF
lygiaonis.
6. Duotas keturkampis ABCD toks, kad AB = BD, BCA = 65

, ACD = 50

. S
Rasti ABD.
7. Lygiakraiam trikampiui ABC ant kratins BC trikampio iorje nubrtas S
pusapskritimis. Per tak A ivestos tiess dalina t pusapskritimio lank tris
lygias dalis. rodykite, kad tos tiess taip pat dalina BC tris lygias dalis.
145
4.7. Vienareikmiki udaviniai Geometrija
8. Trikampyje ABC A = 20

, AB = AC. Ant kratins AB paymta atkarpa S


AD, lygi BC. Rasti BCD.
9. Kvadrato ABCD viduje paimtas takas M taip, kad MCD = MDC = 15

. S
Rasti kamp AMB
10. Duotas keturkampis ABCD toks, kad DAC = DBA = 45

, AB = BC = S
CA. Rasti ADC.
11. Trikampyje ABC, A = 30

, C = 45

. Ant AC paimtas takas D toks, kad S


CD = BA. Rasti ABD.
12. Duotas keturkampis ABCD toks, kad BCA = 21

, CDA = 78

, CAD = S
39

, BC = CD. Rasti BAC.


13. Ikilajame keturkampyje ABCD, kuris nra trapecija, kampai tarp striains S
AC ir kratini yra 55

, 55

, 16

, 19

kakokia tvarka. Rasti visus manomus smai-


lius kampus tarp AC ir BD.
14. P - vidinis trikampio ABC takas (AB = BC). ABC = 80

, PAC = 40

, S
ACP = 30

. Rasti BPC.
15. ( IMO 1975 motyvais) Duotas bet koks trikampis ABC. Jo iorje sukonst- S
ruoti trikampiai ABR, BCP, ACQ, taip, kad BCP = ACQ = 30

, CBP =
CAQ = 60

x, RBA = RAB = x. rodyti, kad PR = QR.


16. Duotas keturkampis ABCD toks, kad AB = AD, CBD = 30

, BAC = 48

, S
DAC = 16

. Rasti ACD.
17. Duotas keturkampis ABCD toks, kad jo striains statmenos, ir BAC = S
20

, DAC = 10

, BCA = 50

. Rasti BDC.
18. Duotas statusis trikampis ABC su A = 50

, C = 40

. Paimti takai D ir E S
ant atitinkamai BC ir BA tokie, kad BAD = 20

, BCE = 10

. Rasti EDA.
146
4.8. Geometrins nelygybs Geometrija
4.8 Geometrins nelygybs
Geometrijos ir nelygybi temos susikerta geometrini nelygybi udaviniuose. Juos ga-
lima iskaidyti dvi pagrindines kategorijas: algebrins nelygybs, kuri kintamieji yra
trikampio komponentai (ios nelygybs daniausiai bna ciklins kamp atvilgiu), ir
nelygybs, lyginanios ilgius bei plotus. iame skyiuje nagrinsime antrojo tipo nely-
gybes. Tokie udaviniai olimpiadose pasitaiko ne itin danai, bet jie bna vairiausio
sunkumo. Be to, tai vieni t udavini, kuriuos danai galima paversti algebros
udavin ir bandyti sprsti algebriniais metodais. Taiau iame skyriuje nagrinsime
geometrinius j sprendimo bdus, kurie nors reikalauja iokio tokio imoningumo, yra
trumpesni ( Nors tikrai ne visas nelygybes manoma taip isprsti - kai kurios daromos
tik algebrinias metodais).
Teiginys. Keletas gerai inom nelygybi:
Trikampio nelygyb: Jeigu trikampio kratini ilgiai yra a, b, c, tai tada a +b > c,
b +c > a, a +c > b.
Jeigu ABC yra trikampis, R - apie t trikamp apibrto apskritimo spindulys, r
- brto apskritimo spindulys, tai tada R 2r. Lygybs atvejis tada ir tik tada,
kai trikampis yra lygiakratis. rodymas duotas emiau.
Jeigu ant trikampio ABC kratins AB paimtas takas D ( nesutampantis su A
ar B), tai tada arba AC > CD, arba BC > CD, arba AC > CD ir BC > CD.
Bet kokiu atveju, AC +BC > CD.
Apskritimo styga visada trumpesn u skersmen.
Kampo kosinusas ir sinusas visada yra intervale [1, 1].
Pavyzdys. Duotas trikampis ABC. rodykite, kad R 2r.
Sprendimas. Tegu K, L, M yra trikampio kratinio vidurio takai. Tada KLM yra
dvigubai maesnis nei ABC, taigi R
KLM
=
R
2
. Tegu yra apie KLM apibrtas
apskritimas. Nubrkime tris liestines apskritimui , lygiagreias AB, BC, CD taip,
kad j sankirtos yra trikampio, kurio viduje yra trikampis ABC, virns. Tegu is
trikampis yra QPR, ir jis akivaizdiai panaus ABC ir u j nemaesnis. Taigi i
atitinkam element panaumo
R
2
= R
KLM
r.
Pavyzdys. Duotas taisyklingasis eiakampis. rodykite, kad suma atstum nuo jo
virni iki centro yra maesn nei tokia suma iki bet kurio kito tako.
Sprendimas. Tegu ABCDEF yra tas eiakampis, O-bet koks takas. Tada (OA +
OD) + (OB +OE) + (OC +OF) AD +BE +CF.
Dalis nelygybi gali bti isprendiamos vien prisibriant ir pritaikant trikampio
nelygyb.
147
4.8. Geometrins nelygybs Geometrija
Pavyzdys (LitMO 2011 rajono etapas). Duotas lygiaonis trikampis ABC, AB = AC.
Ant spindulio BC u tako C paimtas takas E, o ant atkarpos BC paimtas takas F
taip, kad BF = CE. rodyti, kad AB +AC < AE +AF.
Sprendimas. Tereikia nubrti trikamp kuriam galtume taikyti trikampio nelygyb.
Tam imame tak G ant spindulio AB u tako B taip, kad BG = AB. Tada trikampiai
ACE ir BFG vienodi pagal dvi kratines ir kamp, taigi AB + AC = AB + BG =
AG < AF +FG = AF +AE, ko ir reikjo.
A
B
C
E
G
F

Pavyzdys. Duotas trikampis ABC. Kampo B pusiaukampin pratsta iki susikirtimo


su apibrtiniu apskritimu take D. rodyti, kad 2BD > AB +BC
Sprendimas.
A
B
D
C
E
io udavinio sunkumas tame, kad kitaip nei trikampio nelygybje, sum turime kitoje
lygybs pusje, todl gali pasirodyti, kad su trikampio nelygybe nieko nepavyks. Taiau
mes galime parayti 2BD = BD + BD, ir pabandyti surasti trikamp su dvejomis
kratinmis, lygiomis BD, ir treia kratine, lygia AB+BC. Tam mes paymime tak
E ant BA tsinio taip, kad AE = CB. Tada ADE ir CBD vienodi pagal 2 kratines
ir kamp. BDE ir yra iekomas trikampis.

148
4.8. Geometrins nelygybs Geometrija
Pavyzdys. a, b, c yra kakokio trikampio kratins. rodyti, kad
1
a+b
,
1
a+c
,
1
c+b
taip pat
yra kakokio trikampio kratins.
Sprendimas. Utenka parodyti, kad galioja
1
a+b
+
1
b+c
>
1
a+b
ir ciklins perstatos.
Iprastin vardiklius gauname (b a)(b c) < (a + c)(a + c), kas yra akivaizdu, nes
b a < a +c ir b c < a +c.
Udaviniai
1. Ant tiess a rasti tak G tok, kad AG+BG but maiausias, kur A ir B yra S
takai toje paioje tiess pusje.
2. Duotas trikampis ABC, takas O jo viduje. rodykite, kad AB + BC + CA > S
AO +BO +CO >
AB+BC+CA
2
.
3. Duotas trikampis ABC, AM - pusiaukratin. rodyti, kad AB +AC 2AM. S
4. rodyti, kad trikampio pusiaukratini ilgi suma yra maesn u trikampio S
perimetr, bet didesn u
3
4
perimetro.
5. Duotas trikampis ABC su ACB = 70

, o jo viduje yra takas M toks, kad S


BAM = ABC, AMB = 100

. rodyti, kad BM < AC.


6. Duotas trikampis, o j brtas kvadratas taip, kad jo dvi virns yra ant vienos S
kratins, o ant kit kratini po vien virn. Tegu a yra kvadrato kratins
ilgis, o r brtinio apskritimo spindulys. rodyti, kad

2r < a < 2r.
7. Trikampis ABC lygiakratis su AB = 1. Takas O yra trikampio viduje. rody- S
kite, kad 2 OA+OB +OC.
8. Duotas keturkampis, o jo viduje takas. rodyti, kad atstum nuo to tako iki S
keturkampio virni suma nevirija D
1
+D
2
+P, kur P - keturkampio perimetras,
D1, D2 - striaini ilgiai.
9. Duotas keturkampis su kratinmis a, b, c, d (tokia tvarka). rodyti, kad S < S
(a+b)(c+d)
4
, kur S yra keturkampio plotas.
10. Ant kvadrato, kurio kratins ilgis 1, kiekvienos kratins yra pastatytas statusis S
trikampis, kuri ambin yra to kvadrato kratin. T 4 staij trikampi sta-
tieji kampai yra A, B, C, D, o staiuosius trikampius brt apskritim centrai
yra O
1
, O
2
, O
3
, O
4
. rodyti, kad ABCD plotas nevirija 2, o O
1
O
2
O
3
O
4
plotas
nevirija 1. .
11. Duotas brtinis keturkampis ABCD toks, kad BC = CD. E - kratins AC S
vidurio takas. rodyti, kad BE +DE AC.
12. taisyklingj septynkamp brtas apskritimas ir aplink taip pat apibrtas S
apskritimas. Tada pats septynkampis yra iede, suformuotame i dviej apskriti-
m. Tas pats padaryta su taisyklinguoju 17-kampiu. Taip jau nutiko, kad abiej
ied plotai lygs. rodyti, kad septynkampio ir septyniolikakampio kratins yra
vienodo ilgio.
149
4.8. Geometrins nelygybs Geometrija
13. Staiakamp ABCD kurio plotas 1, sulenk taip, kad takas C sutapo su taku S
A. rodyti, kad gauto penkiakampio plotas yra maesnis negu
3
4
.
14. Trikampyje ABC paimta pusiaukratin AM. Ar galejo taip nutikti, kad tri- S
kampi AMC ir AMB brtini apskritim spinduliai skiriasi lygiai du kartus?
15. Duotas trikampis ABC su auktinmis AA
1
, BB
1
, CC
1
ir pusiaukratinmis S
AA
2
, BB
2
,CC
2
. rodyti, kad i atkarp A
1
B
2
, B
1
C
2
, C
1
A
2
galima sudti trikam-
p.
16. Trikampis A
1
A
2
A
3
brtas apskritim su spinduliu 2. rodykite, kad ant S
lank A
1
A
2
, A
2
A
3
, A
3
A
1
galima atitinkamai paimti takus B
1
, B
2
, B
3
taip, kad
eiakampio A
1
B
1
A
2
B
2
A
3
B
3
ploto skaitin vert bt lygi trikampio A
1
A
2
A
3
perimetro skaitinei vertei.
17. Rasti trikamp su kratini ilgiais a, b, c ir apibrto apskritimo spinduliu R, S
kuris tenkint R(b +c) = a

bc.
18. Trikampio ABC viduje yra du apskritimai, kuri vienas lieia AB ir BC, o kitas S
AC ir BC, o abu irgi lieiasi iorikai. rodyti, kad j spinduli suma didesn nei
brto apskritimo spindulys.
19. Duotas trikampis ABC, AB > BC. Nubtos pusiaukampins AK ir CM. S
rodyti, kad AM > MK > KC.
20. Duotas trikampis ABC. Ties kerta jo kratines AB ir BC atitinkamai takuose S
M ir K, ir dalina ABC plot pusiau. rodyti, kad
MB+BK
AM+CA+CK
>
1
3
.
150
5 SKYRIUS
SPRENDIMAI
Skaii teorija
Dalumas
1. Jei n[3a, tai n[12a ir n[12a + 5b 12a = 5b. Aiku, kad i n[5b seka ir n[10b.
2. Pastebkime, kad b galima ireikti kaip 3(2a +5b) 2(3a +7b), o a kaip 5(3a +
7b) 7(2a + 5b), vadinasi, abu jie i n dalinsis.
3. Ne, jos visos trys neteisingos.
a) Jei x[a +b, tai nebtinai x[a ir x[b. Pavyzdiui, 5[2 + 3, bet 5 2 ir 5 3.
b) Jei x[a b, tai nebtinai x[a arba x[b. Pavyzdiui, 6[2 3, bet 6 2 ir 6 3. Kaip
bebt, i savyb teisinga, kai x pirminis (jei dviej skaii sandauga dalijasi i
pirminio skaiiaus, tai i to pirminio dalijasi nors vienas i skaii).
c) Jei x[a ir y[a, tai nebtinai xy[a. Pavyzdiui, 4[12 ir 6[12, bet 24 12.
4. Taip gauto skaiiaus skaitmen suma yra lygi 45, tad pagal dalumo poym jis
i 9 dalinsis.
5. Pritaikome dalybos i 11 poym: a b + b a = 0 dalijasi i 11, vadinasi ir
skaiius abba dalinsis i 11.
6. a) Jei vietoje vaigduts raysime x, tai gauto skaiiaus skaitmen suma bus
lygi 15 +x. Ji dalinsis i 9 kai x = 3
b) Pagal dalumo poym i 8 turi dalintis 45. Kadangi 400 dalijasi i 8 ir 56
dalijasi i 8 tai vietoje vaigduts galime rayti 6.
c) Alternuojanti suma vietoj vaigduts raius x yra lygi 3 x. Ji dalinsis i 11,
kai x = 3.
151
Sprendimai
7. Pakanka pastebti, kad 10a +b yra lygus 10(a + 4b) 13 3b.
8. Atmet lyginius ir dalius i 5 skaiius gauname, kad lieka patikrinti 181, 183, 187,
189, 191, 193, 197 ir 199. Pagal dalumo poymius 183 ir 189 dalijasi i 3, o 187 i
11. I 7 itame intervale dalijasi skaiiai 182, 189 ir 196, o i 13 tik 182. Vadinasi,
skaiiai 181, 191, 193, 197 ir 199 nesidalija i pirmini, maesni u

199 14,
todl yra pirminiai.
9. Iskaidykime: n
2
+ 5n + 6 = (n + 2)(n + 3). Kadangi su visomis natraliosiomis
n reikmmis abu dauginamieji yra didesni u 1, tai j sandauga niekada nebus
pirminis skaiius.
10. Iskaidykime dauginamaisiais:
a
3
+ 2a +b
3
+ 2b = 2(a +b) + (a +b)(a
2
ab +b
2
) = (a +b)(a
2
ab +b
2
+ 2).
Kadangi a +b dalijasi i n, tai ir duotas reikinys i n dalinsis.
11. Pagal Euklido algoritmo ivad tokiu bdu galima ireikti dbd(8, 5) = 1. Bet
tuomet galima ireikti ir bet kur skaii a pakanka vietoje x ir y, naudojam
vieneto iraikoje, imti ax ir ay.
12. Negali. Jei jo lyginse pozicijose esani skaitmen sum paymsime x, o ne-
lyginse y, tai gausime, kad x y turi dalintis i 11. Kadangi x + y = 5, tai
11 < x y < 11, lieka tiktai variantas x y = 0. Bet tokiu atveju x ir y turt
bti arba abu lyginiai, arba abu nelyginiai, o tai prietaraut tam, kad j suma
nelygin.
13. Skaiius p

1
1
p

n
n
turi (
1
+ 1) (
n
+ 1) dalikli. Kad dalikli skaiius bt
nelyginis, visi
1
, . . . ,
n
turi bti lyginiai. Taiau tuomet skaiius bus sveikojo
skaiiaus kvadratas (p

1
/2
1
p

n
/2
n
)
2
.
14. Jei trupmena
a
b
yra suprastinama, tai dbd(a, b) = d > 1. Tada, kadangi d[a ir
d[b, tai d[ab ir d[a+b, vadinasi, ir trupmena
ab
a+b
bus suprastinama. Atvirkias
teiginys nra teisingas. I dbd(a b, a +b) = d > 1 galime gauti, kad d[2a ir d[2b,
o i ia ir idj kontrapavyzdiui:
53
5+3
suprastinama, o
5
3
ne.
15. Paymkime dbd(a, b) = d ir a = da
1
, b = db
1
. Kadangi d[b, o dbd(a
1
, b) = 1,
tai mbk(a, b) = mbk(da
1
, b) = a
1
b. Lieka patikrinti:
mbk(a, b) dbd(a, b) = d a
1
b = a b
16. Jei 11[3x + 7y ir 11[2x + 5y, tai 11[3(2x + 5y) 2(3x + 7y), t.y. 11[y. Taiau jei
11[2x + 5y ir 11[y, tai 11[2x = 11[x. Gavome, kad 11[x ir 11[y, todl tikrai
11
2
[x
2
+ 3y
2
.
17. Paymkime skaiiaus skaitmen esani lyginse vietose sum a ir nelyginse
b. Pagal dalumo i 9 poym a + b turi dalintis i 9. Pastebkime, kad a + b
negali bti lygus 9, nes tada vienas i j turt bti lyginis, kitas nelyginis, ir j
skirtumas a b nebt lygus 0 ir nesidalint i 11 (11 < a b < 11). Vadinasi,
a +b turi bti lygus bent 18.
152
Sprendimai
18. Aiku, kad n = 12! + 2 tenkins slyg.
19. vertinkime grubiai d dyd. Kadangi a yra imtaenklis, tai b nevirys 100 9.
Tuomet jo skaitmen suma, c, nevirys 3 9, o io skaitmen suma, d, nevirys
2 +9 = 11. Pagal dalumo i 9 poym 9[a = 9[b = 9[c = 9[d. Vienintelis
teigiamas skaiius besidalijantis i 9 ir nedidesnis u 11 yra 9.
20. Raskime paskutin skaiiaus 27
28
skaitmen. 27
1
paskutinis skaitmuo 7, 27
2
- 9,
27
3
- 3, 27
4
- 1, 27
5
- 7, . Matome, kad paskutinis skaitmuo keliant laipsniais
kartojasi kas keturis, vadinasi, 28 laipsnio bus toks pat kaip ir 4, t.y. 1. Tuomet
n paskutinis skaitmuo bus lygus 5, vadinasi, jis dalinsis i 5.
21. Kadangi p pirminis, tai jokie maesni u j skaiiai i p nesidalins. Tuomet i p
nesidalins ir k! ir (p k)!. Kadangi i p dalinasi p!, t.y. trupmenos skaitiklis, bet
nesidalija trupmenos vardiklis, tai suprastinus p neisiprastins, ir gautas skaiius
i p dalinsis.
22. a) Pertvark n
2
+ 1 = (n 1)(n + 1) + 2 gauname, kad dbd(n
2
+ 1, n + 1) =
dbd(2, n + 1). Pastarasis bus didesnis u 1 tada, kai n bus nelyginis, o j iki 100
bus 50.
b) Pertvark n
2
+ 1 = (n 2)(n + 2) + 5 gauname, kad dbd(n
2
+ 1, n + 2) =
dbd(5, n + 2). Pastarasis bus didesnis u 1 tada, kai n + 2 dalinsis i 5. Toki
skaii bus 20 - 3, 8, . . . 98.
23. Perraykime:
n
3
+ 3
n
2
+ 7
=
(n
2
+ 7)n 7n + 3
n
2
+ 7
= n
7n 3
n
2
+ 7
.
Matome, kad duotas skaiius bus sveikasis tik tada, kai sveikasis bus
7n3
n
2
+7
. Pa-
stebkime, kad kai [n[ > 6, tai vardiklis tampa moduliu didesnis u skaitikl, tad
trupmena tikrai nebus sveikasis skaiius. Lieka patikrinti likusias reikmes, i
kuri tinka tik n = 2 ir n = 5.
24. Pirma, aikumo dlei, parodysime, kad teiginys teisingas su n = 3 (su n = 2, ir
n = 1 jis teisingas pagal dalumo i 9 ir 3 poymius). Uraykime
11 . . . 1
. .
27
= 1 00 . . . 0
. .
8
1 00 . . . 0
. .
8
1 11 . . . 1
. .
9
.
Deinje pusje pirmojo dauginamojo skaitmen suma lygi 3, todl jis dalijasi i
3, o antrasis dauginamasis dalijasi i 9, vadinasi, sandauga dalijasi i 27. Bendru
atveju naudosime indukcij. Ura
11 . . . 1
. .
3
n
= 1 00 . . . 0
. .
3
n1
1
1 00 . . . 0
. .
3
n1
1
1 11 . . . 1
. .
3
n1
ir tar, kad 11 . . . 1
. .
3
n1
dalijasi i 3
n1
gauname, kad 11 . . . 1
. .
3
n
dalijasi i 3
n
.
153
Sprendimai
25. I slygos aiku, kad skaiius turi dalintis bent i 2, 3 ir 5. Kadangi iekome ma-
iausio tokio skaiiaus, tai galime tarti, kad daugiau pirmini dalikli skaiius ne-
turs, nes i to kad 2
a
3
b
5
c
q tenkina slyg gautume ir kad 2
a
3
b
5
c
tenkina slyg, o
jis maesnis. Pagal slyg 2
a1
3
b
5
c
turi bti kvadratas, 2
a
3
b1
5
c
- kubas, 2
a
3
b
5
c1
penktasis laipsnis. Vadinasi 2[a1, 2[b, 2[c, 3[a, 3[b1, 3[c, 5[a, 5[b, 5[c1. Kiek-
vieno i a, b, c iekome atskirai. Maiausias nelyginis i 3 ir 5 besidalijantis skaiius
yra 15, vadinasi a = 15. Analogikai b = 10, c = 6. Gavome, kad maiausias skai-
ius tenkinantis slyg yra 2
15
3
10
5
6
.
26. Skaiius 75 isiskaido kaip 3 5 5, vadinasi jis turs ne daugiau kaip 3 skirtingus
pirminius daliklius, i kuri du yra 5 ir 3. Maiausias skaiius, kur gauname
dviej pirmini dalikli atveju yra 3
14
5
4
, maiausias skaiius, kur gauname trij
pirmini dalikli atveju, yra 2
4
3
4
5
2
. is ir bus maiausias.
27. Pastebkime, kad 5n+3 uraomas kaip 4(2n+1)(3n+1). Paymj 2n+1 = a
2

ir 3n + 1 = b
2
gausime, kad 5n + 3 isiskaido kaip (2a b)(2a + b). Jis nebus
pirminis, jei 2a b > 1. Patikrinkime atvej, kai 2a = b + 1. sistat gausime
lygi sistem
_
2n + 1 = a
2
,
3n + 1 = (2a 1)
2
.
Isprend randame vienintl sveikj sprendin a = 1, n = 0.
28. Tarkime prieingai, kad toki pirmini skaii yra baigtinis skaiius. Paymkime
juos p
1
, p
2
, . . . , p
k
ir nagrinkime skaii 4p
1
p
2
p
k
1. Jis nesidalins i n vieno
pirminio p
1
, . . . , p
k
, vadinasi, visi jo pirminiai dalikliai bus pavidalo 4k+1. Taiau
toki dalikli ir j laipsni sandauga bus pavidalo 4k + 1, vadinasi, negali bti
lygi 4p
1
p
2
p
k
1. Prietara.
29. Jei sudaugin gavome 1, tai priepaskutinis skaiius turjo bti pavidalo 111.
rodysime, kad tokio tipo skaiiaus negalime gauti daugindami skaitmenis. Tam
uteks parodyti, kad jis turi pirmini dalikli didesni u 7. Ities, jei 111
dalijasi i 3, tai dalijasi ir i 111, ir i 37. Jei 111 dalijasi i 7, tai dalijasi i
111111, ir taip pat dalijasi i 37. Jei nesidalija nei i 3, nei i 7, tai tikrai dalijasi
i pirminio didesnio u juos. Vadinasi, slyg tenkina tik skaiiai pavidalo 111.
30. Pastebkime, kad pirminiai p ir q yra panaaus dydio, t.y. p q +6 ir q p+7.
Taip pat, p ir q negali bti labai dideli, nes bet kokio skaiiaus didiausias daliklis
neskaitant paties skaiiaus yra bent dvigubai u j maesnis. Pasinaudokime tuo:
kadangi p[q + 6 ir p q 7, tai arba q 7 bus didesnis nei pus q + 6 ir p turs
bti lygus q +6, arba q 7 bus nedidesnis nei q +6. Pirmuoju atveju i p = q +6
gauname q[q+13, i kur q = 13, p = 19. Antruoju atveju q7 turi bti nedidesnis
u pus q + 6, arba sutvarkius, q 20. Vadinasi, lieka patikrinti tik q reikmes
2, 3, 5, 7, 11, 13, 17, 19. Tai padaryti nesunku: nei viena i j, iskyrus jau rast
13, netinka.
31. Pastebkime, kad didiausias n daliklis nevirija n, antras pagal dyd nevirija
154
Sprendimai
n
2
, treias pagal dyd nevirija
n
3
ir taip toliau. Tuomet gausime, kad
d
k
d
k1
+ +d
2
d
1
<
n
1
n
2
+
n
3
n
4
+ +
n
k
n
k + 1
= n
2
(
1
1
1
2
+
1
2
1
3
+ +
1
k
1
k + 1
).
vertinkime sum:
1
1
1
2
+
1
2
1
3
+ +
1
k
1
k + 1
=
1
1

1
2
+
1
2

1
3
+ +
1
k

1
k + 1
= 1
1
k + 1
< 1.
rodysime, kad d
1
d
2
+ d
2
d
3
+ + d
k1
d
k
dalo n
2
tada ir tik tada, kai n yra
pirminis. Tarkime prieingai, tegu n sudtinis, ir tegu p yra maiausias pirminis
n daliklis. Tuomet
n
2
> d
1
d
2
+d
2
d
3
+ +d
k1
d
k
> n
n
p
,
prietara, nes
n
2
p
yra antras pagal dyd n
2
daliklis.
Lyginiai
1. a) 1+11+111+1111+11111 = 1+(9+2)+(108+3)+(1107+4)+(11106+5)
6 (mod 9).
b) 555 777 + 666 888 6 3 + 0 6 0 (mod 9).
c) 3
99
1 (mod 2), 0 (mod 3), 1 (mod 4), 2 (mod 5), 3 (mod 6),
1 (mod 7).
d) 7
4
1 (mod 10) = 7
777
7 (mod 10).
2. rodysime naudodamiesi apibrimu. Iskaidykime skirtum:
ab +cd ad bc = a(b d) +c(d b) = (a c)(b d).
Kadangi a c[(a c)(b d), tai i ties ab +cd ad +bc (mod a c).
3. Jei skaiius lyginis, tai jo kvadrato dalybos i 4 liekana bus 0, jei nelyginis (
1 (mod 4)), tai 1.
4. Iskaidykime n
5
n = (n 1)n(n + 1)(n
2
+ 1). Akivaizdu, kad duotas reikinys
dalijasi i 2 ir 3. rodysime, kad dalijasi ir i 5. Jei n lygsta 1, 0 arba -1, tai
tuomet i 5 dalijasi atitinkamai n 1, n, n +1, o jei n lygsta 2 moduliu 5 tai i
5 dalijasi n
2
+ 1 (n
2
+ 1 4 + 1 0 (mod 5)).
5. Dalindami kvadrat i trij gausime tiktai liekanas 0 arba 1. Jas sumuojant nul
galima gauti vieninteliu bdu, kai abu dmenys lygs 0.
6. Dalindami kvadrat i septyni, gausime liekanas 0, 1, 2 arba 4. Kaip ir praeitame
udavinyje, jas sumuojant, nul galima gauti tik, kai abu dmenys lygs 0.
7. Nelyginis skaiius moduliu 8 gali duoti liekanas 1 ir 3. Tuomet jo kvadratas
duos liekanas (1)
2
1 (mod 8) ir (3)
2
9 1 (mod 8).
155
Sprendimai
8. Kadangi 6[x
3
x, tai x
3
x (mod 6). Tuomet ir a
3
+b
3
+c
3
a+b+c (mod 6).
9. Kadangi a nesidalija i 2 tai a 1 (mod 8) arba a 3 (mod 8). Abiem
atvejais pakl abi lygybs puses kvadratu gauname a
2
1 (mod 8). Analogikai,
kadangi a nesidalija i 3, tai a 1 (mod 3), vadinasi a
2
1 (mod 3). Gavome,
kad a
2
1 dalijasi i 3 ir 8, vadinasi, a
2
1 (mod 24).
10. Kvadratai duoda liekanas 1, 0 moduliu 4, o dviej nelygini skaii kvadrat
suma duoda liekan 2.
11. Iskaidykime 5 3 2
3
[n(n
2
+ 1)(n + 1)(n 1). Nesunku sitikinti, kad su
visomis n reikmmis n(n
2
+ 1)(n + 1)(n 1) dalijasi i 5 ir 3. Patikrinkime,
kada dalijasi i 8. Kai n nelyginis, tai trys dauginamieji (n
2
+ 1)(n + 1)(n 1)
lyginiai, todl i 8 dalinsis. Kai n lyginis, tai vienintlis lyginis dauginamasis yra
n, vadinasi sandauga dalinsis i 8 kai n dalinsis i 8. Gavome, kad 120[(n
5
n),
kai su visomis nelyginmis ir i atuoni besidalijaniomis reikmmis.
12. Jei abu pirminiai p ir q nesidalija i 3, tai j kvadratai lygsta 1 moduliu 3. Taiau
tuomet p
2
2q
2
1 2 1 , 1 (mod 3). Vadinasi bent vienas i j dalijasi
i trij, t.y. yra lygus trims. Patikriname: su q = 3 sprendini nra, o su p = 3
gauname q = 2.
13. I pradi raskime, su kuriomis n reikmmis duotas daugianaris dalijasi i 11.
Tam utenka perrinkti 11 liekan gausime, kad tinka tik n 4 (mod 11). stat
n = 11k + 4 gausime 11
2
k
2
+ 11
2
k + 33, kas su jokia k reikme nesidalija i 121.
14. Uraykime reikin kaip (101)(10
n1
+ +10+1)+45n. Padalij i 9 matome,
kad dalmuo dar dalijasi i 3: 10
n1
+ +10 +1 +5n 1 +1 + 1 +5n 6n
0 (mod 3).
15. Pastebkime, kad su bet kuriuo k yra teisinga 10
k
1
k
1 (mod 9). Tuomet
a
1
a
2
. . . a
n
= a
1
10
n1
+a
2
10
n2
+ +a
n
1 10+a
n
a
1
+a
2
+ a
n
(mod 9).
16. Perraykime slyg kaip n[a b. Aiku, kad jei d[n ir d[a, tai d turi dalinti ir b.
Lygiai taip pat, jei d[n ir d[b, tai d turi dalinti ir a. Vadinasi i ties dbd(a, n) =
dbd(b, n).
17. Pastebj, kad 899 = 9001 = (301)(30+1) galime iekoti, su kuriomis n reik-
mmis duotas reikinys dalijasi i 29 ir 31 atskirai. Kadangi 36
n
7
n
(mod 29)
ir 24
n
(5)
n
(mod 29), tai
36
n
+ 24
n
7
n
5
n
(5)
n
5
n
(mod 29),
ir lygsta nuliui, kai n lyginis. Analogikai
36
n
+ 24
n
7
n
5
n
(7)
n
7
n
(mod 31),
ir lygsta nuliui taip pat, kai n lyginis. Vadinasi duotas reikinys dalinsis i 899 su
visomis lyginmis n reikmmis.
156
Sprendimai
18. inome, kad p[
_
p
k
_
=
p!
k!(pk)!
, su visomis reikmmis 0 < k < p, todl
(a +b)
p
= a
p
+
_
p
1
_
a
p1
b + +
_
p
p 1
_
ab
p1
+b
p
a
p
+ 0 + + 0 +b
p
a
p
+b
p
(mod p).
19. I x +y x (mod y) seka (x +y)
n
x
n
(mod y). Jei daugianar q usiraysime
kaip q(x) = a
n
x
n
+. . . a
1
x +a
0
, tai gausime, kad
a
n
(x +y)
n
+. . . a
1
(x +y) +a
0
a
n
x
n
+. . . a
1
x +a
0
(mod y).
20. Raskime skaiiaus 1010 101 dalybos i 9999 liekan. Tai padaryti labai pa-
prasta pastebjus, kad skaiius uraomas kaip 10
0
+10
2
+10
4
+10
6
+ , ir kad
10
4
1 (mod 9999). Tuomet dalybos liekana bus 1+100+1+100+ . Kadangi
9999 = 101 99, tai norint, kad liekana bt 9999 kartotinis reiks 99k dmen
1 + 100. Vadinasi, skaiius turs 4 99k 1 skaitmen.
21. Parodysime, kad tinka tik p = 2, 3, 5. Nesunku sitikinti, kad ios reikms tinka,
o p = 7, 11 netinka, tad tarkime, kad p > 11 ir tuomet 11+p
2
> 144. Pastebkime,
kad p
2
1 (mod 3) ir p
2
1 (mod 4), todl p
2
+ 11 0 (mod 12). I ia seka,
kad p
2
+ 11 turi daliklius 1, 2, 3, 4, 6, 12 ir
p
2
+11
1
, . . . ,
p
2
+11
12
, taigi daugiau nei 11.
Oilerio teorema
1. Taikykime maj Ferma teorem. Pagal j 3
12
1 (mod 13), 7
16
1 (mod 17)
ir 9
18
1 (mod 19). Skaiiuojame:
3
33
3
212
3
9
3
9
27 27 27 1 (mod 13),
7
77
7
416
7
13
7
13
49
6
7 (2)
6
7 6 (mod 17),
9
99
9
518
9
9
81
4
9 5
4
9 1 (mod 19).
2. Pagal Oilerio teorem 11
8
1 (mod 15) (11 ir 15 tarpusavyje pirminiai, (15) =
8). Raskime, koki liekan gausime dalindami laipsn 11
11
i 8. Kadangi (8) = 4,
tai
11
11
11
3
3
3
3 (mod 8).
Tuomet
11
11
11
11
3
(4)
3
11 (mod 15).
3. Prisiminkime, kad jei a b (mod n), tai dbd(a, n) = dbd(b, n). Tuomet aiku,
kad jei dbd(a, n) > 1, tai dbd(a
n
) > 1 ir a
n
, 1 (mod n), nes dbd(1, n) = 1.
4. Pagal maj Ferma teorem n
pq
n
p
(mod q) ir n
q
n (mod q), todl n
pq

n
p
n
q
+n 0 (mod q). Analogikai ir n
pq
n
p
n
q
+n 0 (mod p).
157
Sprendimai
5. rodysime, kad a
47
+b
57
+c
47
dalijasi i 2 ir i 47, todl nra pirminis. Kadangi
keliant laipsniu skaiiaus lyginumas nesikeiia, tai aiku, kad jei skaii suma buvo
lygin, tai ir 47-tj laipsni suma taip pat bus lygin. O pagal maj Ferma
teorem turime x
4
7 x (mod 47), todl a
47
+b
57
+c
47
a +b +c 0 (mod 47).
6. Teiginys teisingas atveju p = 3 (pakanka imti skaiius, kuri skaitmen skaiius
dalijasi i 3), tad tarsime, kad p 7. Perraykime 11 . . . 11 kaip
10
n
1
9
, kur n
skaitmen skaiius. Kadangi vardiklis nesidalija i nagrinjamo pirminio p, tai
pakanka parodyti, kad su be galo daug n reikmi 10
n
1 dalijasi i p. Pagal
slyg dbd(10, p) = 1, todl galime taikyti maj Ferma teorem. Gausime
10
p1
1 (mod p), ir tuo paiu, 10
(p1)k
1 (mod p), vadinasi 11 . . . 11 dalinsis
i p kai tik skaitmen skaiius dalinsis i p 1.
7. Pastebkime, kad pagal Oilerio teorem tiks bet koks n, kuris yra tarpusavyje
pirminis su 2003 ir tenkina (n)[n. Abu kriterijus tenkina dvejeto laipsniai 2
k
,
k N.
8. Pairkime, kokius skaitmenis galime naudoti, nordami negauti sudtinio skai-
iaus. 9 negalima naudoti pagal slyg, taip pat netiks 2, 4, 5, 6, 8 ir 0, tad lieka
tik 1, 3 ir 7. Vienetas ir septynetas duoda liekan 1 moduliu 3, todl juos dau-
giausia galsime urayti du kartus, kitaip skaitmen suma dalinsis i 3 ir skaiius
bus sudtinis. Vadinasi, nuo kakurios vietos visi skaitmenys turs bti trejetai.
Skaii iki tos vietos paymj A tursime, kad A yra pirminis, bet inome, kad
kiekvienam pirminiam egzistuoja pavidalo 11 . . . 11 (o todl ir pavidalo 33 . . . 33)
skaiius, besidalinantis i to pirminio. Vadinasi, po kakurio trejeto priraymo
gausime skaii besidalijant i A, t.y. sudtin.
9. Tegu p pirminis a
1
1 + a
2
2 + + a
m
m daliklis. Pagal maj Ferma teorem
a
k(p1)
1 (mod p), todl a
1
1
n
+ a
2
2
n
+ + a
m
m
n
dalinsis i p su visomis
n = k(p 1) + 1, k = 1, 2, . . . reikmmis.
10. sistat n = p gausime f(p)
p
p (mod f(p)), arba p 0 (mod f(p)) =
f(p) = p arba f(p) = 1. Pastebkime, kad jei kakokiai reikmei f(n) ,= n, tai
f(p) = p gali galioti tik baigtiniam skaiiui pirmini, nes kiekvienam i j yra
teisinga f(n)
p
n (mod p) = f(n) n (mod p) = p[f(n) n. Vadinasi,
tiks arba funkcija f(n) = n, arba funkcijos, kurios baigtiniam skaiiui pirmini
p
1
, . . . p
k
tenkina f(p
k
) = p
k
, visiems kitiems pirminiams f(p) = 1, o sudtiniams
a turi galioti f(a) a (mod p
i
), i = 1 . . . k. f(p)[p, f(p) = p tik baigtin skaii
kart, p[f(n) n.
11. Parodysime, kad kiekvienam pirminiam p atsiras toks n, kad p[a
n
. Atvejai p = 2
(tinka visos n reikms) ir p = 3 (tinka lygins n reikms) paprasti, tad tarkime,
kad p 5. Spjame, kad p[a
p2
. Kad galtume taikyti maj Ferma teorem
padauginkime a
p2
i 6 (kadangi p 5, tai p[a
p2
p[6a
p2
):
6(2
p2
+3
p2
+6
p2
1) = 3 2
p1
+2 3
p1
+6
p1
6 2+3+16 0 (mod p).
158
Sprendimai
Kin liekan teorema
1. Pirmojoje sistemoje i pirmos lygties gauname, kad iekomas r dalinasi i 5. I
antros lygties inome, kad jis taip pat turi bti lygus 7k +4. Pervelg pirmsias
reikmes randame, kad tinka 25, o jis taip pat tenkina ir treij lygt.
Antrj sistem sutvarkome kaip ir pavyzdyje. Pirm lygt dauginame i 2, antr
i 5, trei i 4. Gausime sistem
_

_
r 2 (mod 5),
r 5 (mod 7),
r 4 (mod 11).
Pirmas dvi lygtis tenkina r = 12, taiau i reikm netenkina treios. Kit pirmj
dviej lygi sprendin rasime prie 12 pridj 5 7, bet ir is netiks. Vl ir vl
priddami po 35 galiausiai rasime, kad tinka r = 257.
Nemaai pasidarbavus, i karto kyla mini, kaip buvo galima proces pagreitinti.
Pirma, galima buvo nepertvarkyti lygties, o paymti 3r = x ir iekoti tokio
sprendinio, kuris dalijasi i 3. tai gana paprasta, nes pradins lygties sprendiniai
yra 1 + 5 7 11 k. Antra, kadangi 12 1 (mod 11), o priddavome po 35
2 (mod 11), tai galjome i karto suskaiiuoti, kad 4 (mod 11) gausime pridj 35
septynis kartus.
2. Kadangi 450 isiskaido kaip 9 50, tai uteks rasti liekanas atskirai moduliu 9 ir
50 ir pasinaudoti kin liekan teorema. Liekana moduliu 50 yra 9, o moduliu 9
1+2+ +2009 =
20092010
2

23
2
3. Nesunku atspti, kad abu lyginius tenkina
309.
3. Pagal maj Ferma teorem 5x
13
+ 13x
5
+ 9ax 5x + 9ax (mod 13) ir 5x
13
+
13x
5
+ 9ax 13x + 9ax (mod 5). Vadinasi, kad daugianaris dalintsi i 65 su
visomis x reikmmis, 5 + 9a turi dalintis i 13, ir 13 + 9a i penki. Gauname
lygini sistem, kuri galima sprsti prastai, paymjus 9a = t, bet veriau iek
tiek pagudrauti. Padaugin pirmos lygties abi puses i dviej, gausime 18a
10 (mod 13), arba, a 2 (mod 13). Padaugin antros lygties abi puses i 4,
gausime 36a 3 4 (mod 5), arba, a 2 (mod 5). Matome, kad a = 2 yra
sprendinys, taiau mums reikia natraliojo. Maiausias toks pagal kin liekan
teorem bus 2 + 65 = 63.
4. Uraykime a = 2

1
3

2
p

k
k
, kur p
k
didiausias pirminis nevirijantis 1997.
Skaiius a bus natraliojo skaiiaus laipsnis, jei visi
1
,
2
, . . . ,
k
dalinsis i ka-
kokio pirminio q
1
. Skaiius 2a bus natraliojo skaiiaus laipsnis, jei visi skaiiai

1
+ 1,
2
, . . . ,
k
dalinsis i kakokio pirminio q
2
. Taip tsdami, kiekvienam
i
gausime 1997 lygini sistem, kuri pagal kin liekan teorem turs sprendin.
Rad visus
i
rasime ir a, tad natralusis skaiius tenkinantis slyg egzistuoja.
5. Pastebkime, kad kiekvienam n uteks rasti skaii r, su kuriuo r(r + 1) + 1 =
r
2
+r + 1 turt bent n skirting pirmini dalikli.
159
Sprendimai
Jei p
1
[r
2
1
+r
1
+1, p
2
[r
2
2
+r
2
+1, p
n
[r
2
n
+r
n
+1, tai pagal Kin liekan teorem
rad tok r, kad
_

_
r r
1
(mod p
1
)
r r
2
(mod p
2
)
. . .
r r
n
(mod p
n
)
Tursime p
1
p
2
p
n
[r
2
+r+1. Lieka rodyti, kad daugianaris x
2
+x+1 turi be galo
daug pirmini dalikli (daugianario p(x) daliklis yra skaiius p, kuriam egzistuoja
toks a, kad p[p(a) ). Tarkime prieingai, tegu daugianaris x
2
+x +1 turi baigtin
skaii pirmini dalikli. Analogikai naudodamiesi Kin liekan teorema rasime
tok x
0
, kad x
2
0
+x
0
+1 dalintsi i j vis. Taiau tuomet (x
0
+1)
2
+(x
0
+1) +1
nesidalins n i vieno, o taip bti negali.
6. Takas bus nematomas, jei jo koordinats nra tarpusavyje pirminiai skaiiai,
t.y. turi bendr dalikl. Tuo ir pasinaudosime. Tegu p
1
, p
(n+1)
2 skirtingi pirminiai
skaiiai. Pagal kin liekan teorem, lygini sistema turs sprendin:
_

_
x 0 (mod p
1
)
y 0 (mod p
1
)
x 0 (mod p
2
)
y + 1 0 (mod p
2
)
. . .
x 0 (mod p
n+1
)
y +n 0 (mod p
n+1
)
. . .
x +n 0 (mod p
(n+1
2
)
)
x +n 0 (mod p
(n+1)
2)
Aiku, kad kvadrato, kurio apatinis kairysis kampas yra sistemos sprendinys (x, y),
o kratins ilgis n, kiekvieno vidaus tako koordinai pora turi bendr dalikl,
t.y. takas yra nematomas.
Liekan grup
1. Generatoriai bus keturi 2, 6, 7 ir 8.
2. Tarkime prieingai, kad liekanos a eil d yra maesn u jos atvirktins a
1
eil
d

. Taiau tuomet (a
1
)
d
(a
d
)
1
1
1
1 prietara.
3. Grups nesudarys, nes liekanos, kurios nra tarpusavyje pirmins su n, neturi
atvirktini liekan.
4. Sudtiniams skaiiams negalioja teiginys, kad jei x yra daugianario (a x)q(x)
aknis, tai x btinai yra arba ax aknis arba q(x) aknis. Btent tai ir matome
duotuoju atveju daugianario x
2
+ x = x(x + 1) aknimis yra 2 ir 3, nors ios
liekanos nra nei vieno i daugianari x ir x + 1 aknys.
160
Sprendimai
5. Jei a eil bt maesn nei p1, tai ji, bdama p1 daliklis, bt ir
p1
q
daliklis
su kakokiu q, o tada ir a
p1
q
lygt 1. Kadangi taip nra, tai a turi btinai bti
generatorius. kit pus teiginys akivaizdus jei a generatorius, tai, inoma,
keldami jo laipsniu, maesniu nei p 1, negausime 1.
6. Tegu g generatorius. I prie tai buvusio udavinio gauname, kad tie generato-
riaus laipsniai, kurie yra tarpusavyje pirminiai su p 1 bus generatoriai, o tie,
kurie nra, nebus. I viso tarpusavyje pirmini laipsni bus (p 1) (tarp kuri
ir g
1
), vadinasi, tiek bus ir generatori.
7. Jei 2 nebt generatorius, tai jis turt tenkinti 2
14
1 arba 2
4
1 (mod 29),
bet taip nra 2
14
1 (mod 29) ir 2
4
16 (mod 29).
a) Iekosime sprendini pavidalo 2
k
. Kadangi 2 yra generatorius, tai 2
7k
lygs
vienetui tik tada, kai 7k dalinsis i 28. Taip bus atvejais x = 2
4
, x = 2
8
, x = 2
12
,
x = 2
16
, x = 2
20
, x = 2
24
ir x = 2
28
.
b) Visi duotos lygties sprendiniai bus ir lygties (x 1)(x
6
+ x
5
+ + x + 1)
0 (mod 29), t.y. x
7
1 (mod 29) sprendiniais. ios lygties sprendinius gavome
a) dalyje, lieka tik i j imesti 2
28
1.
8. Generatoriaus atvirktin liekana taip pat bus generatorius, tad j sandauga bus
lygi 1, nebent atsiras generatori, kurie yra sau atvirktiniai. Tokios liekanos yra
tik 1 ir 1. Pirmoji i j niekada nebus generatorius, o 1 yra generatorius tik
liekan grups moduliu 3. Pastebkime, kad (p 1) gyja nelygin reikm taip
pat tik kai p 1 = 2.
9. Lygties sprendiniai bus tie, kuri eil moduliu 19 dalins 17. Kadangi element
eil turi dalinti dar ir grups eil (t.y. 18), tai tiks tik x = 1.
10. Atveju kai p 1 dalo k pagal maj Ferma teorem, gausime 1
k
+ 2
k
+ ... +
(p 1)
k
1 + 1 + + 1 p 1 1 (mod p). Atveju, kai p 1 nedalo k
pasinaudosime tuo, kad liekan grup moduliu p yra ciklin. Generatori pay-
mj g, nagrinjam sum galime perrayti kaip 1 +g
k
+g
2k
+g
3k
+ +g
(p2)k
.
Susumav gausime
(g
k
)
p1
1
g
k
1
. Pagal maj Ferma teorem skaitiklis lygus nuliui,
o vardiklis, kadangi p 1 nedalo k, nelygus.
11. Grups moduliu p eil yra 2
n
, vadinasi, bet kurio elemento eil bus dvejeto laips-
nis. Jei kuris nors nelyginis generatoriaus g laipsnis g
N
nebt generatorius, tai
jo eil bt lygi 2
n
. Taiau tuomet gautume, kad g
2
n
(N)
1, kas negali bti
teisinga, nes 2
n
2
n
N. Jei tarsime, kad 3 nra generatorius, tai pagal a) dal jis
turs bti lyginis generatoriaus laipsnis, kaip ir 1, kuris irgi nra generatorius.
Vadinasi, sandauga 3 bus lyginis generatoriaus laipsnis, t.y. kvadratas. Norda-
mi rodyti c) dalies tvirtinim pakelkime duot lygyb kubu ir pasinaudokime b)
dalimi. Gausime
8u
3
(a 1)(a
2
2a + 1) (a 1)(2a 2) 2(a
2
1) 8 (mod p).
Suprastin i 8 (p nelyginis) gausime u
3
1 (mod p). Bet treios eils element
grup turti negali, nes 3 2
n
, prietara.
161
Sprendimai
12. Pirmiausia pakelkime a + 1 etuoju laipsniu ir sitikinkime, kad gausime 1:
(a + 1)
6
(a
3
+ 3(a
2
+a + 1) 2)
2
(1)
2
1 (mod p).
Lieka sitikinti, kad a + 1 eil negali bti 2 arba 3. Ities, antros eils elementas
yra tik 1, tad iuo atveju a bt lygus 2, o (2)
3
8 , 1 (mod p). Treios
eils negali bti, nes, kaip jau matme, (a + 1)
3
1 (mod p).
13. i grup turi bent tris antros eils liekanas. Viena i j 1, o kitos dvi tenkina
lygini sistemas:
_
r
1
1 (mod p),
r
1
1 (mod q);
_
r
2
1 (mod p),
r
2
1 (mod q).
Parodysime, kad ciklin grup negali turti antros eils liekan be 1. I ties,
tegu grups eil 2k ir g
a
1, kur a ,= k ir a < 2k. Tuomet g
2a
1 ir g
2k
1,
vadinasi ir g
2a2k
1 prietara, nes 0 < [2a 2k[ < 2k.
14. Pastebj, kad lygins n reikms tikrai netinka, udavin galime performuluoti
taip: rodykite, kad dvejeto eil moduliu n nedalo n. I pirmo vilgsnio tai atrodo
kiek keista, nes dvejeto eil dalo (n), o (n) ir n turi gana didel bedr dalikl.
Nepaisant to, parodysime, kad dvejeto eil dalijasi bent i vieno skaiiaus, i kurio
nesidalija n. Paymkime p
0
maiausi pirmin n dalikl. Jei 2
a
1 (mod n), tai
2
a
1 (mod p
0
). Dvejeto eil moduliu p
0
yra p
0
1 daliklis, i kurio, aiku, turi
dalintis a, bet i kurio nesidalins n, nes jis maesnis u maiausij p
0
.
15. Pirma, teigin rodykime pirmini skaii laipsniams. Jei p 3 pirminis, tai jo
liekan, tarpusavyje pirmini su p

grup yra ciklin, todl visas sumoje esanias


liekanas galime urayti kaip 1, g, g
2
, . . . , g
(p

)1
. Tuomet j kub suma bus lygi
1 +g
3
+g
3
2 + g
3((p

)1)
=
(g
3
)
(p

)
1
g
3
1
.
Pagal Oilerio teorem, skaitiklis lygus nuliui, o vardiklis nuliui nelygus, vadinasi,
suma tikrai dalinsis i p

. Su dvejeto laipsniais samprotausime kiek kitaip: visas


liekanas, tarpusavyje pirmines su 2

(t.y. nelygines), pakl kubu gausime t


pat liekan rinkin. Ities, nelygins liekanos kubas bus nelygin liekana, o jei
a
3
b
3
(mod 2

) tai (a b)(a
2
+ ab + b
2
) 0 (mod 2

) = a b (mod 2

),
nes a
2
+ ab + b
2
-nelyginis. Lieka pastebti, kad vis nelygini liekan moduliu
2

suma bus nulis tam pakanka sumuoti poromis maiausi su didiausia, antr
su priepaskutine ir t.t.
Bendru atveju iskaidykime n dauginamaisiais: n = 2

0
p

1
1
p

k
k
. rodysime,
kad nagrinjama suma dalijasi i kiekvieno pirminio laipsnio p

i
i
. Tam nagrinki-
me j moduliu p

i
i
. I viso sumoje yra (n) dmen, tad moduliu p

i
i
dauguma j
sutaps. Pasinaudoj kin liekan teorema sitikinsime, kad sutaps taisyklingai,
t.y. kiekvien liekan gausime lygiai
(n)
(p

i
i
)
kart. Ities, liekan i gausime i t
162
Sprendimai
ir tik i t skaii x, kurie tenkins lygini sistem:
_

_
x r
1
(mod 2

)
x r
2
(mod p

1
1
)
.
.
.
x i (mod p

i
i
)
.
.
.
x r
k
(mod m
k
),
kur r
j
bet kokios liekanos tarpusavyje pirmins su p
j
. Kadangi kiekvienam i toki
sistem bus po tiek pat, tai ir liekan moduliu n teks po tiek pat. Taiau tuomet
suma moduliu p

i
i
bus lygi
(n)
(p

i
i
)
0 pagal tai, k rodme anksiau.
16. Aiku, kad daugianariai q(x) = 1 ir q(x) = 1 tenkina slyg. Parodysime,
kad joki kit slyg tenkinantis daugianaris gyti negali. Tarkime prieingai,
tegu q(a) ,= 1. Tada q(a) dalijasi i kakokio nelyginio pirminio (i 2 dalintis
negali, nes 2
n
1 nelyginis), kur paymkime p. Pastebkime, kad tuomet visoms
sveikoms k reikmms q(a + pk) dalinsis i p, vadinasi ir 2
a+pk
1 dalinsis i p.
Taiau to bti negali, nes jei 2
a
1 (mod p), tai 2
a+p
2
a
2
p1
2 2 (mod p).
17. Jei p = 2, tai q[4 + 2
q
= 4 + 2 0 (mod q) = q = 2, 3. Abu atvejai
tinka. Tegu p, q > 2. I 2
p
+ 2
q
0 (mod p) pagal maj Ferma teorem seka
2 + 2
q
0 (mod p) = 2
q1
1 (mod p). Paymkime ord
p
(2) liekanos 2
eil moduliu p. Tuomet 2
ord
p
(2)/2
1 (mod p), todl i 2
q1
1 (mod p) seka
q 1 = ord
p
(2)/2m, kur m nelyginis. Kadangi elemento eil dalo grups eil, tai
ord
p
(2)[p 1 = 2(q 1)[(p 1)m. Analogikai gauname ir 2(p 1)[(q 1)m.
Paymj r ir s didiausius dvejeto laipsnius i kuri dalijasi q1 ir p1 gauname
r > s ir s > r prietara.
18. rodysime, kad x
2
n
+y
2
n
su kakokiu n dalijasi i 257 = 2
8
+1. Nagrinkime z =
x y
1
kaip grups moduliu 257 liekan. Kadangi ios grups eil yra 2
8
, tai z eil
bus 2
s
, kur 2 s 8 (s ,= 0, nes x , y (mod 257) ir s ,= 1, nes x , y (mod 257)
dl apribojimo 2 x, y 100 ). Tuomet z
2
s1
1 = x
2
s1
+ y
2
s1
0.
Lieka patikrinti, ar x
2
s1
+ y
2
s1
nra tiesiog lygus 257. Vienintelis atvejis, kai
taip gali nutikti, yra 1
2
+ 16
2
, bet jis netenkina slygos x, y 2.
19. Iekokime skaii m ir n uraom kaip m = ad ir n = bd, kur d tarpusavyje
pirminis su a ir su b. Tuomet slygos a n, b m bus tenkinamos, o m[n
2
+
n, n[m
2
+m persirays kaip a[bd + 1 ir b[ad + 1, arba
_
bd 1 (mod a),
ad 1 (mod b).
Kadangi a ir b tarpusavyje pirminiai, tai lygini sistem galime perrayti kaip
_
d b
1
(mod a),
d a
1
(mod b).
Pastaroji turi sprendin pagal Kin liekan teorem, vadinasi iekomi m ir n tikrai
egzistuoja.
163
Sprendimai
20. Tegu p maiausias n daliklis. rodysime, kad jis lygus septyniems. Pastebkime,
kad p negali bti lygus 2 ir 3, nes p[3
n
+4
n
. Pagal maj Ferma teorem p[4
p1

3
p1
ir i slygos p[4
2n
3
2n
, todl
p[ dbd(4
2n
3
2n
, 4
p1
3
p1
) = 4
dbd(2n,p1)
3
dbd(2n,p1)
.
Kadangi dbd(2n, p 1) = 2, tai p[4
2
3
2
= p = 7.
21. Kadangi liekan pavidalo a
n
(mod p), kur dbd(a, p) yra
p1
dbd(p1,n)
, tai lygtis turs
sprendini, jei dbd(p 1, 3) arba dbd(p 1, 37) bus lygus 1. Kad taip nebt,
p 1 turi dalintis i 3 ir i 37, bet tuomet p bus didesnis u 100.
Kvadratins liekanos
1. Skaiiuokime:
_
79
101
_
=
_
101
79
_
=
_
22
79
_
=
_
2
79
__
11
79
_
=
_
79
11
_
=
_
2
11
_
= 1.
2. Jei p[a
2
+ 12, tai a
2
12 (mod p). Iekome moduliu kuri pirmini p, liekana
12 bus kvadratin:
_
12
p
_
=
_
1
p
__
2
p
_
2
_
3
p
_
= (1)
p1
2
(1)
p1
2
_
p
3
_
=
_
p
3
_
.
3. Kvadratinmis bus lyginiai generatoriaus laipsniai, o nekvadratinmis - nelyginiai,
tad tikrai pus bus toki ir pus kitoki.
4. Palik nuoalyje atvejus p = 2 ir p = 3 iekome kit:
_
6
p
_
=
_
2
p
__
3
p
_
= (1)
p
2
1
8
(1)
p1
2
_
p
3
_
.
Sandauga bus lygi 1 kai p 1, 3 (mod 8) ir p 1 (mod 3), arba kai p
5, 7 (mod 8) ir p 2 (mod 3). Sujung gauname, kad tiks p 1 (mod 24)
ir p 5 (mod 24).
5. Skaiius N dalinsis i 2 ir 3 bet nesidalins i 4, todl N 1 2 (mod 3) ir
N + 1 3 (mod 4). Taiau nei 2 moduliu 3, nei 3 moduliu 4 nra kvadratins
liekanos.
6. Pakanka perrayti lygt kaip (x +
b
2a
)
2

b
2
4ac
4a
2
(mod p).
7. Daugianario reikms visuomet nelygins, tad pakaks nagrinti moduliu kurio
nelyginio pirminio diskriminantas 67 yra kvadratin liekana. Pirmasis toks pir-
minis bus 17, ir jis tikrai daugianar dalins, pavyzdiui, kai statysime reikm
n = 2.
8. rodysime, kad jei p[a
2
+b
2
, tai p[a ir p[b. Tarkime prieingai, tegu, pavyzdiui,
p b. Tuomet i a
2
+b
2
0 (mod p) gausime (ab
1
)
2
1 (mod p) prietara,
nes 1 nra kvadratin liekana moduliu pirmini duodani liekan 3 moduliu 4.
164
Sprendimai
9. Imkime bet kur pirmin n dalikl q. Jei q nelyginis, tai pagal kvadratinio ap-
veriamumo teorem
_
q
p
_
= (1)
2n
q1
2
_
p
q
_
= 1
_
1
q
_
= 1. Jei q lyginis, t.y. 2,
tai tuomet p 1 (mod 8) ir
_
2
p
_
= 1.Kadangi bet koks n daliklis bus sandauga
pirmini dalikli, t.y. kvadratini liekan, tai ir jis pats bus kvadratin liekana.
10. Nagrinkime du atvejus. Kai p = 4k + 3, gausime i viso 2k + 1 dauginamj.
Kadangi 1 nra kvadratin liekana moduliu p, tai tarp dauginamj bus tik viena
liekana, kuri yra pati sau atvirktin (liekana 1). Visos likusios bus atvirktins
poromis (kvadrato atvirktin yra kvadratas), tad sudaugin i ties gausime 1.
Kai p = 4k + 1, gausime i viso 2k dauginamj. Kadangi 1 iuo atveju jau
yra kvadratin liekana, tai bus dvi liekanos, kurios yra sau atvirktins (1 ir 1).
Likusios vl bus atvirktins poromis, tad vis sandauga bus lygi 1.
11. Pastebkime, kad duota sandauga yra vis kvadratini liekan moduliu p san-
dauga. I ties i viso yra (p 1)/2 liekan, visos jos kvadratins, ir jokios dvi
nesutampa, nes jei a
2
b
2
(mod p), tai arba a b (mod p) arba a b (mod p).
Pastaroji lygyb negali bti teisinga, nes ir a ir b nelyginiai skaiiai tarp 1 ir p2.
Lieka pasinaudoti praeitu udaviniu.
12. Liekana 4 bus bikvadratin moduliu p, kai lygtis x
4
+ 4 0 (mod p) turs
sprendin. Pasinaudoj duota lygybe gauname, kad taip bus tada ir tik tada, kai
sprendin turs viena i lygi (x 1)
2
+ 1 = 0, kas yra ekvivalentu 1 buvimui
kvadratine liekana moduliu p.
13. Jei pirminis p dalo duot reikin, tai tuomet x
4
x
2
+1 0 (mod p). Perraykime
i lygyb dviem bdais: (x
2
1)
2
x
2
(mod p) ir (x
2
+1)
2
3x
2
(mod p). I
pirmosios gausime, kad 1 yra kvadratin liekana moduliu p, t.y. p 1 (mod 4),
o i antrosios, kad 3 yra kvadratin liekana moduliu p, t.y. p 1 (mod 3).
14. Iskaid daugianar dauginamaisiais gauname (x
2
2)(x
2
3)(x
2
6). Pirminis
p nedalins jo, kai ir 2, ir 3, ir 6 bus nekvadratins liekanos. Taiau to bti negali,
nes dviej nekvadratini liekan sandauga yra kvadratin liekana.
15. Kadangi q pirminis, tai q[2
q1
1, t.y. 2
2p
1 (mod q). Vadinasi, 2
p
bus lygus
arba 1 arba 1 moduliu q. Parodysime, kad atrasis variantas negalimas. Kandagi
p 3 (mod 4), tai q 7 (mod 8), bet tuomet 2 yra kvadratin liekana moduliu
q, o 1 nra, kas prietaraut 2
p
1 (mod q).
16. a) Tegu q pirminis a daliklis (pagal slyg nelyginis). Kadangi p b
2
(mod q) ir
p 1 (mod 4), tai
_
q
p
_
=
_
p
q
_
=
_
b
2
q
_
= 1. Kadangi visi pirminiai a dalikliai yra
kvadratins liekanos, tai ir a bus kvadratin liekana.
b) Tegu q pirminis a+b daliklis. Usira lygyb p = a
2
+b
2
= (a+b)(ab) +2b
2
matome, kad p 2b
2
(mod q), arba
_
q
p
_
=
_
p
q
_
=
_
2
q
_
. Jei a + b turi lygin
skaii pirmini dalikli (skaiiuojant kartotinumus), kurie lygsta 3 moduliu 8,
tai tuomet
_
a+b
p
_
= 1 ir a+b 1 (mod 8), o jei nelygin, tai tuomet
_
a+b
p
_
= 1
ir a +b 3 (mod 8).
c) Duota lygyb seka i (a +b)
2
2ab = p.
d) Pakanka prie tai gaut lygyb pakelti laipsniu (p 1)/4.
165
Sprendimai
Ura lygyb a
2
b
2
a
2
f
2
(mod p) ir suprastin gausime f
2
1 (mod p).
Sujung pastebjim su antra ir ketvirta lygybmis gausime
f
(a+b)
2
1
4
(1)
(a+b)
2
1
8
(a +b)
p1
2
(2ab)
p1
4
(2a
2
f)
p1
4
2
p1
4
f
p1
4
(mod p),
k suprastin gausime 2
p1
4
f
ab/2
(mod p). Galiausiai lieka pastebti, kad
f
ab/2
1 (mod p) tik tada, kai b dalijasi i 8, kas ir reikia, kad p uraomas kaip
A
2
+ 64B
2
.
17. Kai p = 2 tai A nra kvadratas, tad tarkime, kad p 3 . Pagal Ferma teorem
7p + 3
p
4 1 (mod p). Jei A kvadratas, tai 1 kvadratin liekna moduliu p,
todl p = 4k + 1. Taiau tuomet A 7 + (1) 4 2 (mod 4), ko bti negali,
nes 2 nra kvadratin liekana moduliu 4.
18. Parodysime, kad lygtis visuomet turi sprendini. Tarkime prieingai, tegu su
kakokiu p lygtis sprendini neturi, t.y. su visomis x ir y reikmmis x
2
+ y
2
,
2003 (mod p), arba x
2
, 2003y
2
(mod p). Kadangi moduliu p lygiai
p+1
2
pusiau
liekan yra kvadratins (su nuliu), tai ir kair ir dein lygties puss gys po
p+1
2
skirting reikmi. Kadangi
p+1
2
+
p+1
2
> p, tai bent dvi jos sutaps - prietara.
19. Skaiiaus m skaitmen suma negali bti lygi 1, parodysime, kad negali bti lygi
ir dviem. Tarkime prieingai, tuomet egzistuos tokios a ir b reikms, su kuriomis
10
a
+10
b
dalinsis i 2003, t.y. 10
a
10
b
(mod 2003). Kadangi 10 yra kvadratin
liekana moduliu 2003 (
_
10
2003
_
=
_
2
2003
_ _
5
2003
_
=
_
3
5
_
= 1), tai gauname, kad ir
1 yra kvadratin liekana moduliu 2003; prietara.
Parodyti, kad S(m) = 3, nra labai paprasta, nes tenka dauginti gana nema-
us skaiius, norint sitikinti, kad 10 laipsniai gyja pakankamai daug skirtin-
g liekan. Konkreiau, norint parodyti, kad 10 eil moduliu 2003 yra 1001
reikia parodyti, kad 10
77
, 10
91
ir 10
143
nelygsta vienetui. Greiiausia yra ras-
ti laipsnius 10
7
, 10
14
, 10
28
, 10
56
, 10
112
, tuomet gausime, kad 10
77
10
7
10
14
10
56
,
10
91
10
77
10
14
, 10
143
= 10
112
10
28
10
3
. Parodius tai, lieka pastebti, kad tuomet
10 laipsniais galsime urayti visas kvadratines liekanas, tarp j ir 1600, 400 ir 3.
Diofantins lygtys
Dvi lygties puss
1. Nagrinkime lygt moduliu 3. Gausime x
2
2 (mod 3), o taip bti negali.
Vadinasi lygtis sveikj sprendini neturi.
2. Iekokime tik teigiam sprendini, nes rad juos, rasime ir neigiamus. Iskaidy-
kime dauginamaisiais: (x y)(x +y) = 100. Kadangi x y ir x +y yra vienodo
lyginumo, ir j sandauga lygi 100, tai jie tegali bti lygs 2 ir 50 arba 10 ir 10.
Gauname sprendinius (26, 24) ir (10, 0). Lieka tik pridurti, kad ie sprendiniai
tiks ir paimti su visomis manomomis enkl kombinacijomis.
166
Sprendimai
3. Nagrindami lygt moduliu 4 gauname, kad dviej kvadrat suma turi bti lygi
trims. Kadangi kvadratai moduliu 4 gyja tik liekanas 0 ir 1, tai taip niekada
nebus. Lygtis sprendini neturi.
4. Pastebkime, kad x turi bti lyginis. Taiau tuomet kairioji lygties pus dalinsis
i 4, o deinioji - ne. Sprendini nra.
5. Pastebkime, kad jei x > 2 arba x < 0, tai x
2
> 2x + 2. Taip pat, jei y > 3 arba
y < 0, tai y
2
> 3y + 2. Vadinasi, arba x turi bti lygus 0, 1, 2, arba y turi bti
lygus 0, 1, 2, 3. Patikrin randame sprendinius (0, 1), (0, 4), (2, 1) ir (2, 4).
6. Iskaidykime dauginamaisiais: (x y)(x z
2
) = 1987. I ia nesunku rasti didel
sprendin, pvz (100
2
+ 1, 100
2
1986, 100).
7. 3
y
moduliu 8 lygsta tik 3 arba 1, tad lygtis neturs sprendini su x 3. Patikrin
maesnes reikmes randame sprendinius (2, 1) ir (1, 0).
8. Pastebkime, kad jei x > 1, tai y turi bti lyginis, o tuomet, paymj y =
2z, galime iskaidyti lygties deinij pus : 2
x
= (3
z
1)(3
z
+ 1). Vienas i
dauginamj nesidalins i 4 todl turs bti lygus dviems. Gauname sprendinius
(3, 2) ir (i atvejo x = 1) (1, 1).
9. Kairioji pus bus didesn u deinij, jei tik y bus didesnis u 9, todl utenka
patikrinti devynias reikmes. Tai padaryti paprasta persiraius lygt kaip kvadra-
tin (x
2
+x(2y 18) +y
2
81) ir suskaiiavus diskriminant - 4 9 (182y). Tiks
reikms y = 1, y = 7 ir y = 9 (pastaroji netinka, nes x turt bti 0). Gausime
sprendinius (20, 1) ir (8, 7).
10. Kairioji lygybs pus yra kvadratas, o deinioji, jei z > 1, duoda liekan 3
moduliu 4. Vadinasi z gali bti lygus tik vienam, i kur randame sprendinius
(1, y, 1), y N.
11. Iskaidykime dauginamaisiais: (y
2
3)(2x
2
+1) = 9. Dauginamasis 2x
2
+1 dalo
9 tik kai x = 0, x = 1 arba x = 2. Tinka tik pastarasis, randame sprendin
(2, 2).
12. Iskaid dauginamaisiais 1989 = 13 17 9 matome, kad x turi dalintis i 17, o y i
13. Pakeit x = 17a, y = 13b gauname lygt 17a
2
+13b
2
= 17 13 9
2
, i kurios vl
gauname, kad a = 13k, b = 17l. stat ir suprastin gauname 13k
2
+ 17l
2
= 81.
Pastaroji labai paprasta, randame, kad k = 1, l = 2, vadinasi pradins lygties
sprendinys bus (17 13, 2 17 13).
13. Naudosime terpimo tarp kvadrat (iuo atveju ketvirtj laipsni) triuk. Kai
x teigiamas, tai x
4
< 1 + x + x
2
+ x
3
+ x
4
< (x + 1)
4
,o kai x neigiamas, tai
(x + 1)
4
< 1 +x +x
2
+x
3
+x
4
x
4
(lygyb gyjama tik kai x = 1). Vadinasi
lieka patikrinti dvi reikmes x = 0, x = 1, i kuri gauname sprendinius (1, 1)
ir (0, 1).
14. Dein lygties pus beveik visuomet didesn u kairij. T paprasta inaudoti
persiraius lygt kaip kvadratin (5a
2
+ a(5b 7) + 5b
2
14b) ir suskaiiauvus
167
Sprendimai
diskriminant: 15(5b
2
14b)+49. Jis nebus neigiamas tik kai b tenkins 0 b 3,
patikrin ias reikmes gauname du sprendinius - (0, 0) ir (1, 3).
15. Kadangi kairioji pus sveikas skaiius, tai x turi bti nemaesnis u y. Jei jie
lygus, tai tinka tik (1, 1), tad tarkime, kad x > y. Tuomet gausime, kad xy turi
bti didesnis u y, ir kad y[x. Paymj x = ky gauname (ky)
y
= y
(k1)y
, arba
k = y
k2
. i lygtis turi tik du sprendinius k = 3, y = 3 ir k = 4, y = 2, nes jei
k > 4 tai k < 2
k2
y
k2
. Pakeit atgal, gauname pradins lygties sprendinius
(6, 3) ir (8, 2).
16. Udavinys ekvivalentus tokiam - iskaidykite 6! paeiliui einani skaii sandau-
g. Daugiausia j galima iskaidyti 6 dauginamuosius, tuomet gausime sprendin
(1, 6). penkis ir keturis dauginamuosius iskaidyti nepavyks, nes jei visi bus
maesni u 6, tai sandauga bus per maa, o jei didesni, tai turs arba dalintis
i 7 (arba 11, arba 13) arba sandauga jau bus per didel. tris dauginamuosius
iskaidyti galima - 6! = 8 9 10, du ne (26 27 < 720 < 27 28), vien, aiku,
galima. Randame dar du sprendinius: (7, 10) ir (6! 1, 6!)
17. Sukelkime visk vien lygties pus : x
2
3x +y
2
3y +z
2
3z +t
2
3t = 0.
Maiausios reikms kurias gali gyti reikinys x
2
3x yra 4, 0, 2, o visos likusios
ne maesns u deimt. Susumav gausime nul tik arba atveju 0 +0 +0 +0 arba
0 2 2 + 4, tad sprendiniai bus (0, 0, 0, 0) ir visos manomos kombinacijos i 0,
1 arba 2, 1 arba 2, 1 arba 4 (pvz. (0, 1, 1, 1), (4, 2, 0, 1), ).
18. Parodysime, kad kairioji lygties pus yra beveik visuomet didesn u deinij.
Kadangi x > y, tai xy + 61 < x
2
+ 61. I kitos puss, x
3
y
3
x
3
(x 1)
3
=
3x
2
3x +1, kas yra daugiau u x
2
+61, kai x > 6. Vadinasi, lieka patikrinti tik
kelet reikmi, k padar randame vienintl sprendin (6, 5).
19. Jei b = 0, tai lygtis uraoma kaip 2
a
= (c3)(c+3). Vienintliai dvejeto laipsniai
besiskiriantys per 6 yra 2 ir 8, randame sprendin (4, 0, 5). Tegu b > 0, tuomet c
dalijasi i trij ir b 2. Paymj b2 = d, c = 3n gauname 2
a
3
d
= (n1)(n+1).
Kadangi dbd(n1, n+1) 2, tai vienas i dauginamj nesidalija i 3. Tada jis
arba yra lygus 1, arba dalijasi i 2. Jei lygus vienetui, tai tuomet n = 2, randame
sprendin (0, 3, 6). Jei dalijasi i dviej, tai tuomet n nelyginis ir a 2. Paymj
n = 2k 1 ir a 2 = e gauname 2
e
3
d
= k(k +1). Kadangi k ir k +1 tarpusavyje
pirminiai, tai arba k = 2
e
, k + 1 = 3
d
arba k = 3
d
, k + 1 = 2
e
. Pirmu atveju
gauname lygt 3
d
= 2
e
+ 1, antru 2
e
= 3
d
+ 1.
20. 3
x
(1)
x
(mod 4), todl, jei y > 1 (y = 1 tinka, tuomet x = 1), tai x turi bti
lyginis. Paymj x = 2a gauname lygt 2
y
= (3
a
1)(3
a
+1). Abu dauginamieji
esantys deinje pusje turi bti dvejeto laipsniai, bet besiskiriantys per du yra
tik 2 ir 4. Vadinasi a = 1, x = 2, y = 3.
21. Iskaidykime x
3
= 4(y 1)(3y
2
+ 3y 1). Kadangi su visomis y reikmmis
3y
2
+3y 1 2 (mod 3), tai jis turs pirmin dalikl duodant liekan 2 moduliu
3. Taiau kairioji lygties pus tokio daliklio turti negali, nes 3 negali bti
kvadratin liekana moduliu pirminio p 2 (mod 3). Vadinasi, 3y
2
+ 3y 1 turi
168
Sprendimai
bti lygus 1, todl y = 0 arba y = 1. Tinka tik pirmasis, randame sprendin
(1, 0).
Algebra
Nelygybs
Pirmieji ingsniai
1. I a
2
+b
2
2ab: x
3
+y
3
= (x +y)(x
2
xy +y
2
) (xy)(2xy xy) = xy(x +y).
2. Nelygyb ekvivalenti
1
2
(a b)
2
+
1
2
(b c)
2
+
1
2
(c a)
2
0, kas yra akivaizdu.
3. Nelygyb ekvivalenti
a
2
4
+
_
a
2
4
b
_
2
+
_
a
2
4
c
_
2
+
_
a
2
4
d
_
2
0, kas yra aki-
vaizdu. Lygyb galios, kai a = b = c = d = 0.
4. Nelygyb ekvivalenti a
3
+b
3
+c
3
3abc = (a+b+c)(a
2
+b
2
+c
2
abbcac) 0.
I udavinio nr. 2 rezultato seka, kad ji yra teisinga.
5. Padauginame nelygyb i ab(a + b). Gausime a
2
xy + a
2
y
2
+ b
2
yx + b
2
x
2

a
2
xy + b
2
xy + 2abxy (ay bx)
2
0, kas yra akivaizdu. Lygyb galios, kai
a
x
=
b
y
. Pagal matematins indukcijos princip, nelygyb galime praplsti:
a
2
1
b
1
+
a
2
2
b
2
+
a
2
3
b
3
+... +
a
2
n
b
n

(a
1
+a
2
)
2
b
1
+b
2
+
a
2
3
b
3
+... +
a
2
n
b
n

(a
1
+a
2
+a
3
)
2
b
1
+b
2
+b
3
+... +
a
2
n
b
n
...

(a
1
+a
2
+a
3
+... +a
n
)
2
b
1
+b
2
+b
3
+... +b
n
.
Lygyb galios, kai
a
1
b
1
=
a
2
b
2
= ... =
a
n
b
n
.
6. Nelygyb keliame kvadratu ir dauginame i x
2
y
2
(x
2
+y
2
). Gausime y
4
+x
2
y
2
+
2xy(x
2
+y
2
) +x
4
+x
2
y
2
8x
2
y
2
. Pastebkime, kad sudj akivaizdias nelygybes
x
4
+y
4
2x
2
y
2
ir 2xy(x
2
+y
2
) 4x
2
y
2
, gausime tai, k reikjo rodyti.
7. Nelygyb ekvivalenti 10a
2
+10b
2
+c
2
4ab+4ac+4bc. Belieka tik pasukti galv,
kaip sukonstruoti nelygyb i akivaizdi kit:
8a
2
+
1
2
c
2
4ac;
8b
2
+
1
2
c
2
4bc;
2a
2
+ 2b
2
4ab.
8. Naudosime udavinio nr. 2 rezultat: S a
2
+ b
2
+ c
2
= 1. Minimumas S = 1
pasiekiamas, kai a = b = c =
1

3
.
169
Sprendimai
9. = (2a 1)
2
+ (a + c)
2
+ (2c + 1)
2
+ 6b
2
2 2. Minimumas yra 2,
pasiekiamas, kai a =
1
2
, b = 0, c =
1
2
.
10. Naudojame a + b 2

ab:
1
1x
2
+
1
1y
2

2

(1x
2
)(1y
2
)
. Pastebkime, kad
(1 x
2
)(1 y
2
) = 1 x
2
y
2
+x
2
y
2
1 2xy +x
2
y
2
= (1 xy)
2
. Tai ir ubaigia
rodym.
11.
1
a
+
1
b
+
1
c
a +b +c
3(ab+bc+ac)
a+b+c
3abc. Tuomet, belieka rodyti a +b +c
3(ab+bc+ac)
a+b+c
a
2
+b
2
+c
2
ab +bc +ac, o remiantis ud. nr. 2, tai yra rodyta.
12. Tegu E = (x + y a)
2
+ (x + z b)
2
+ (y + z c)
2
+ (x d)
2
+ (y e)
2
+
(z f)
2
+ 2(x + y + z k)
2
+ C C. Kvadratus parinkome tokius, kad visk
sudauginus koecientai prie kvadrat ir nari xy, xz, yz atitikt originali E
iraik, nepriklausomai nuo a, b, c, d, e, f, k. Tuomet
_

_
2xa 2xb 2xd 4xk = 52x,
2ya 2yc 2ye 4yk = 60y,
2zb 2zc 2zf 4zk = 64z

_
a +b +d + 2k = 26,
a +c +e + 2k = 30, (1)
b +c +f + 2k = 32,
Kad E minimumas bt C, visi kvadratai turi bti lygs 0:
_

_
x +y a = 0,
x +z b = 0,
y +z c = 0,
x d = 0,
y e = 0,
z f = 0,
x +y +z k = 0,

_
d +e = a,
d +f = b,
e +f = c,
d +e +f = k,
(2)
I (1) ir (2) sudar bendr sistem ir j isprend gausime a = 4, b = 5, c = 7,
d = 1, e = 3, f = 4, k = 8, o a
2
+ b
2
+ c
+
d
2
+ e
2
+ f
2
+ k
2
= 244. Taigi,
E = (x +y 4)
2
+ (x +z 5)
2
+ (y +z 7)
2
+ (x 1)
2
+ (y 3)
2
+ (z 4)
2
+
2(x + y + z 8)
2
244 + 244. Vadinasi, E minimumas yra 244, o
jis pasiekiamas, kai x = 1, y = 3, z = 4.
13.

cyc
a
3
a
2
+ab +b
2

a
3
0. Naudojame udavinio nr.1 rezultat:

cyc
a
3
a
2
+ab +b
2

a
3
=

cyc
3a
3
a
3
a
2
b ab
2
3(a
2
+ab +b
2
)


cyc
2a
3
a
3
b
3
3(a
2
+ab +b
2
)
=

cyc
a b
3
= 0.
170
Sprendimai
14. Naudojame udavinio nr. 1 rezultat:
KAIR PUS
1
ab(a +b) +abc
+
1
bc(b +c) +abc
+
1
ac(a +c) +abc
=
c
abc(a +b +c)
+
a
abc(a +b +c)
+
b
abc(a +b +c)
=
a +b +c
abc(a +b +c)
=
1
abc
.
15. Lema. Jei x, y - teigiami realieji, tai x
5
+y
5
x
2
y
2
(x +y).
Lemos rodymas.
x
5
+y
5
= (x +y)(x
4
x
3
y +x
2
y
2
xy
3
+y
4
)
= (x +y)((x y)
2
(x
2
+xy +y
2
) +x
2
y
2
)
x
2
y
2
(x +y).
Naudodami slyg abc = 1, nelygyb pertvarkome:
KAIR PUS =

cyc
a
2
b
2
c
a
5
+b
5
+a
2
b
2
c


cyc
a
2
b
2
c
a
2
b
2
(a +b) +a
2
b
2
c
=

cyc
c
a +b +c
= 1.
16. Lema 1. b
3
c +bc
3
b
4
+c
4
.
Lemos 1 rodymas. b
3
(b c) + c
3
(c b) 0 (b c)
2
(b
2
+ bc + c
2
) 0. Jei
bc 0, nelygyb akivaizdi, o jei bc < 0, tenka rodinti b
2
+bc +c
2
0: nelygyb
ekvivalenti (b +c)
2
bc, kas yra akivaizdu.
Lema 2. a
2
bc
1
2
a
2
b
2
+
1
2
a
2
c
2
.
Lemos 2 rodymas. (ab ac)
2
0, kas yra akivaizdu.
Naudodami slyg abc 1, nelygyb pertvarkome:
KAIR PUS

cyc
a
5
a
2
abc
a
5
+abc(b
2
+c
2
)
=

cyc
a
4
a
2
bc
a
4
+b
3
c +bc
3

cyc
a
4

1
2
a
2
b
2

1
2
a
2
c
2
a
4
+b
3
c +bc
3
(Lema 2)

a
4
a
2
b
2
a
2
c
2
+b
4
b
2
c
2
+c
4
a
4
+b
4
+c
4
(Lema 1)
=
1
2

(a
2
b
2
)
2
+ (b
2
c
2
)
2
+ (c
2
a
2
)
2
a
4
+b
4
+c
4
0.
171
Sprendimai
17. Pastebime, kad galioja tapatyb:
(a
2
+b
2
+c
2
)
2
3(a
3
b +b
3
c +c
3
a) =
1
2

cyc
(a
2
2ab +bc c
2
+ca)
2
0.
Vidurki nelygybs
1. Naudosime AM-GM nelygyb:
S = ab +
1
16ab
+
15
16ab
2
_
ab
1
16ab
+
15
16
_
a+b
2
_
2

1
2
+
15
16
1
4
= 4
1
4
.
Minimumas yra 4
1
4
, pasiekiamas, kai a = b =
1
2
.
2. Naudosime AM-GM nelygyb:
S = a +
1
4a
+b +
1
4b
+c +
1
4c
+
3
4
_
1
a
+
1
b
+
1
c
_
2
_
a
1
4a
+ 2
_
b
1
4b
+ 2
_
c
1
4c
+
3
4
3
3
_
1
abc
3 +
9
4

1
a+b+c
3
3 +
9
4

1
1
2
= 7
1
2
.
S minimumas yra 7
1
2
, ir jis pasiekiamas, kai a = b = c =
1
2
.
3. Naudosime AM-GM nelygyb:
S =

cyc
3
_
9
4

3
_
(a +b)
2
3

2
3


cyc
3
_
9
4

a +b +
2
3
+
2
3
3
=
3
_
9
4

2(a +b +c) + 4
3
=
3
_
9
4

6
3
=
3

18.
Maksimumas yra
3

18, o jis pasiekiamas, kai a = b = c =


1
3
.
4. Galsime naudoti AM-GM nelygyb, nes a 2 0; b 6 0; c 12 0:
+
_

_
bc

a 2 =
bc

2
_
(a 2) 2
bc

2

(a2)+2
2
=
abc
2

2
,
ca
3

b 6 =
ca
3

9
3
_
(b 6) 3 3
ca
3

9

(b6)+3+3
3
=
abc
2
3

9
,
ab
4

c 12 =
ab
4

64
4
_
(c 12) 4 4 4
ab
4

64

(c12)+4+4+4
4
=
abc
8

2
,

1
abc

_
abc
2

2
+
abc
2
3

9
+
abc
8

2
_
=
5
8

2
+
1
2
3

9
.
gauna maksimali reikm, kai a = 4, b = 9, c = 16. Ji lygi
5
8

2
+
1
2
3

9
.
172
Sprendimai
5. Taikydami AM-GM nelygyb prarandame jos lygybs atvej, taiau jis mums ir
nereikalingas.
Turime
k

k + 1
k
= k

_
k + 1
k
1 1 . . . 1
. .
k1
<
1
k
_
k + 1
k
+ (k 1)
_
= 1 +
1
k
2
.
Tuomet I < n 1 +
1
2
2
+
1
3
2
+ . . . +
1
n
2
< n 1 +
1
12
+
1
23
+ . . . +
1
(n1)n
=
n 1 +
_
1
1

1
2
_
+
_
1
2

1
3
_
+. . . +
_
1
n1

1
n
_
= n 1 +
_
1
1
n
_
< n.
6. Pagal AM-GM:
+
_

_
7
a
3
b
2
+ 2
b
2
c
+
c
2
a
10
10
_
a
21
b
4
c
2
ab
14
c
2
= 10
a
2
b
,
7
b
3
c
2
+ 2
c
2
a
+
a
2
b
10
10
_
b
21
c
4
a
2
bc
14
a
2
= 10
b
2
c
,
7
c
3
a
2
+ 2
a
2
b
+
b
2
c
10
10
_
c
21
a
4
b
2
ca
14
b
2
= 10
c
2
a
,
Sudedame ir gauname tai, k reikjo rodyti.
Pastaba. udavin galima daug paprasiau rodyti, naudojant nesunkiai rodom
lem: Su realiaisiais teigiamais a, b, c galioja
a
2
b
+
b
2
c
+
c
2
a
a +b +c.
7. Pagal AM-GM nelygyb, galioja ios nelygybs:
+
_

_
b+c

a
+ 2

a =
b

a
+

a +
c

a
+

a 2

b + 2

c,
c+a

b
+ 2

b =
c

b
+

b +
a

b
+

b 2

c + 2

a,
a+b

c
+ 2

c =
a

c
+

c +
b

c
+

c 2

a + 2

b,

a +

b +

c 3
3
_

abc = 3.
Visk sudj gausime norim rezultat.
8. Pagal AM-GM:
+
_

_
a
3
b
3
+
a
3
b
3
+ 1 3
3
_
a
6
b
6
= 3
a
2
b
2
,
b
3
c
3
+
b
3
c
3
+ 1 3
3
_
b
6
c
6
= 3
b
2
c
2
,
c
3
a
3
+
c
3
a
3
+ 1 3
3
_
c
6
a
6
= 3
c
2
a
2
,
2
_
a
3
b
3
+
b
3
c
3
+
c
3
a
3
_
+ 3 2
_
a
2
b
2
+
b
2
c
2
+
c
2
a
2
_
+
_
a
2
b
2
+
b
2
c
2
+
c
2
a
2
_
2
_
a
2
b
2
+
b
2
c
2
+
c
2
a
2
_
+ 3
3

a
2
b
2

b
2
c
2

c
2
a
2
= 2
_
a
2
b
2
+
b
2
c
2
+
c
2
a
2
_
+ 3.
9. Pagal AM-GM:
173
Sprendimai
+
_

_
3
a
2
b
5
+ 2
1
a
3
5
5
_
a
6
b
15
a
6
= 5
1
b
3
,
3
b
2
c
5
+ 2
1
b
3
5
5
_
b
6
c
15
b
6
= 5
1
c
3
,
3
c
2
a
5
+ 2
1
c
3
5
5
_
c
6
a
15
c
6
= 5
1
a
3
.
Sudj gausime tai, k reikjo rodyti.
10. Naudosime AM-GM nelygyb:
KAIR PUS = (a +b +a +c)(a +b +b +c)(a +c +b +c)
2
_
(a +b)(a +c) 2
_
(a +b)(b +c) 2
_
(a +c)(b +c)
= 8(a +b)(a +c)(b +c)
= 8(1 a)(1 b)(1 c).
11. Duota nelygyb ekvivalenti
a
b
+
b
c
+
c
a
+
b
a
+
c
b
+
a
c

2(a+b+c)
3

abc
. Pagal AM-GM
nelygyb:
+
_

_
a
b
+
a
b
+
b
c
3
3
_
a
b

a
b

b
c
=
3a
3

abc
,
b
c
+
b
c
+
c
a
3
3
_
b
c

b
c

c
a
=
3b
3

abc
,
c
a
+
c
a
+
a
b
3
3
_
c
a

c
a

a
b
=
3c
3

abc
,

a
b
+
b
c
+
c
a

a +b +c
3

abc
. (1)
Taip pat:
+
_

_
b
a
+
b
a
+
a
c
3
3
_
b
a

b
a

a
c
=
3b
3

abc
,
c
b
+
c
b
+
b
a
3
3
_
c
b

c
b

b
a
=
3c
3

abc
,
a
c
+
a
c
+
c
b
3
3
_
a
c

a
c

c
b
=
3a
3

abc
,

b
a
+
c
b
+
a
c

a +b +c
3

abc
. (2)
Sudj (1) ir (2) gausime tai, k reikjo rodyti.
12. Pagal AM-GM:
_

_
1 +
2a
3b
=
1
3
+
1
3
+
1
3
+
a
3b
+
a
3b
5
5
_
_
1
3
_
3

_
a
3b
_
2
=
5
3
_
a
b
_2
5
,
1 +
2b
3c
=
1
3
+
1
3
+
1
3
+
b
3c
+
b
3c
5
5
_
_
1
3
_
3

_
b
3c
_
2
=
5
3
_
b
c
_2
5
,
1 +
2c
3d
=
1
3
+
1
3
+
1
3
+
c
3d
+
c
3d
5
5
_
_
1
3
_
3

_
c
3d
_
2
=
5
3
_
c
d
_2
5
,
1 +
2d
3a
=
1
3
+
1
3
+
1
3
+
d
3a
+
d
3a
5
5
_
_
1
3
_
3

_
d
3a
_
2
=
5
3
_
d
a
_2
5
,
S =
_
1 +
2a
3b
_ _
1 +
2b
3c
_ _
1 +
2c
3d
_ _
1 +
2d
3a
_

625
81

_
a
b

b
c

c
d

d
a
_2
5
=
625
81
. S Mini-
mumas yra
625
81
. Jis pasiekiamas, kai a = b = c = d > 0.
174
Sprendimai
13. Naudodami slyg, veriame nelygyb homogenine:
a
3
b(2c+a)
+
b
3
c(2a+b)
+
c
3
a(2b+c)

a+b+c
3
. Pagal AM-GM:
+
_

_
9a
3
b(2c+a)
+ 3b + (2c +a) 3
3
_
9a
3
b(2c+a)
3b(2c +a) = 9a,
9b
3
c(2a+b)
+ 3c + (2a +b) 3
3
_
9b
3
b(2a+b)
3c(2a +b) = 9b,
9c
3
a(2b+c)
+ 3a + (2b +c) 3
3
_
9c
3
b(2b+c)
3a(2b +c) = 9c,
Sudj ir sutvark nelygyb ir gausime tai, k reikjo rodyti.
14. Nelygyb galime paversti homogenine, naudodami duot slyg:
c(a +b) +ab
a(a +b)
+
a(b +c) +bc
b(b +c)
+
b(c +a) +ca
c(c +a)

9
2

a
b
+
b
c
+
c
a
+
b
a +b
+
c
b +c
+
a
c +a

9
2

a +b
b
+
b +c
c
+
c +a
a
+
b
a +b
+
c
b +c
+
a
c +a

15
2
(1)
Naudosime AM-GM nelygyb:
KAIR PUS(1) =
a +b
4b
+
b +c
4c
+
c +a
4a
+
b
a +b
+
c
b +c
+
a
c +a
+
3
4
_
a +b
b
+
b +c
c
+
c +a
a
_
6
6

a +b
4b

b +c
4c

c +a
4a

b
a +b

c
b +c

a
c +a
+
3
4
_
_
3
3

a
b

b
c

c
a
+ 3
_
_
=
15
2
.
15. Pagal AM-GM nelygyb:
3 = ab +bc +ac 3
3

a
2
b
2
c
2
abc 1.
Pagal duot slyg ir turim rezultat:
KAIR PUS =

cyc
1
1 +a(3 bc)
=

cyc
1
1 + 3a abc


cyc
1
3a
=
ab +ac +bc
3abc
=
1
abc
.
16. Nelygyb keliame kvadratu ir sutvarkome:
a
2
b
2
c
2
+
b
2
c
2
a
2
+
c
2
a
2
b
2
a
2
+ b
2
+ c
2
.
Pagal AM-GM:
+
_

_
a
2
b
2
c
2
+
b
2
c
2
a
2
2
_
a
2
b
2
c
2

b
2
c
2
a
2
= 2b
2
,
b
2
c
2
a
2
+
c
2
a
2
b
2
2
_
b
2
c
2
a
2

c
2
a
2
b
2
= 2c
2
,
c
2
a
2
b
2
+
a
2
b
2
c
2
2
_
c
2
a
2
b
2

a
2
b
2
c
2
= 2a
2
,
175
Sprendimai

a
2
b
2
c
2
+
b
2
c
2
a
2
+
c
2
a
2
b
2
a
2
+b
2
+c
2
.
17. Pagal AM-GM nelygyb:
(x +y)(x +z) = xy + (x
2
+zy) +xz xy + 2x

yz +xz = (

xy +

xz)
2
.
Taigi,

cyc
x
x +
_
(x +y)(x +z)

cyc
x
x +

xy +

xz
=

cyc

x +

y +

z
= 1.
18. Padaugin i 2 ir prie abiej nelygybs pusi pridj x
2
+y
2
+z
2
, gausime
x
2
+ 2

x +y
2
+ 2

y +z
2
+ 2

z 3.
I AM-GM nelygybs:

cyc
x
2
+

x +

cyc
3
3

x
3
= 9.
T ir reikjo rodyti.
19. Pagal AM-GM:
+
_

_
a
3
(1+b)(1+c)
+
1+b
8
+
1+c
8
3
3
_
a
3
(1+b)(1+c)

1+b
8

1+c
8
=
3a
4
,
b
3
(1+c)(1+a)
+
1+c
8
+
1+a
8
3
3
_
b
3
(1+c)(1+a)

1+c
8

1+a
8
=
3b
4
,
c
3
(1+a)(1+b)
+
1+a
8
+
1+b
8
3
3
_
c
3
(1+a)(1+b)

1+a
8

1+b
8
=
3c
4
,

a
3
(1+b)(1+c)
+
b
3
(1+c)(1+a)
+
c
3
(1+a)(1+b)

a+b+c
2

3
4

3
3

abc
2

3
4
=
3
4
.
20. Naudodami AM-GM nelygyb gauname:
_
a
6
b
3
+
b
6
c
3
+ 4
_
+
_
b
6
c
3
+
c
6
a
3
+ 4
_
+
_
c
6
a
3
+
a
6
b
3
+ 4
_
6
_
6
_
a
6
b
3
c
3
+
6
_
b
6
c
3
a
3
+
6
_
c
6
a
3
b
3
_
=
18
2
_
a
6
b
3
+
b
6
c
3
+
c
6
a
3
_
+ 12 18
a
6
b
3
+
b
6
c
3
+
c
6
a
3
3.
21. Pastebkime, kad
(ab+cd)
2
0 (a+b+c+d)
2
4(ab+bc+cd+da) = 4 a+b+c+d 2.
Pagal AM-GM:
+
_

_
36a
3
b+c+d
+ 2(b +c +d) + 6a + 3 4
4
_
36a
3
b+c+d
2(b +c +d) 6a 3 = 24a,
36b
3
c+d+a
+ 2(c +d +a) + 6b + 3 4
4
_
36b
3
c+d+a
2(c +d +a) 6b 3 = 24b,
36c
3
d+a+b
+ 2(d +a +b) + 6c + 3 4
4
_
36c
3
d+a+b
2(d +a +b) 6c 3 = 24c,
36d
3
a+b+c
+ 2(a +b +c) + 6d + 3 4
4
_
36d
3
a+b+c
2(a +b +c) 6d 3 = 24d,
KAIR PUS
a +b +c +d
3

1
3

2
3

1
3
=
1
3
.
176
Sprendimai
22. Pagal AM-GM:
+
_

_
bc
a
2
=
3
_
b
7
a
2
c
2

c
7
a
2
b
2

1
a
2
b
2
c
2

1
3
_
b
7
a
2
c
2
+
c
7
a
2
b
2
+
1
a
2
b
2
c
2
_
,
ca
b
2
=
3
_
c
7
b
2
a
2

a
7
b
2
c
2

1
a
2
b
2
c
2

1
3
_
c
7
b
2
a
2
+
a
7
b
2
c
2
+
1
a
2
b
2
c
2
_
,
ab
c
2
=
3
_
a
7
c
2
b
2

b
7
c
2
a
2

1
a
2
b
2
c
2

1
3
_
a
7
c
2
b
2
+
b
7
c
2
a
2
+
1
a
2
b
2
c
2
_
,
abc =
3
_
b
7
a
2
c
2

c
7
a
2
b
2

a
7
b
2
c
2

1
3
_
b
7
a
2
c
2
+
c
7
a
2
b
2
+
a
7
b
2
c
2
_
,
Sudj gausime tai, k reikjo rodyti.
Cauchy-Schwarz nelygyb
1. Paymime a
1
= , a
2
+a
3
= , a
4
+a
5
+a
6
= ir a
7
+a
8
+a
9
+a
10
= . Tuomet
+ + + = 1 ir . Pagal Cauchy-Schwarz nelygyb:
Z = a
2
1
+ a
2
2
+ ... + a
2
10

2
+

2
2
+

2
3
+

2
4
12Z 12
2
+ 6
2
+ 4
2
+ 3
2
.
Pastebime, kad
1
4
, +
1
2
, ++
3
4
, be to ++ + = 1. Teisingai
padaugin ir sudj gausime 12+6 +4 +3
25
4
. Na o pagal Cauchy-Schwarz
nelygyb:
12Z
(12 + 6 + 4 + 3)
2
25

25
2
4
2
25
=
25
16
.
Taigi Z minimumas yra
25
192
, o jis pasiekiamas, kai a
1
=
1
4
, a
2
= a
3
=
1
8
, a
4
=
a
5
= a
6
=
1
12
ir a
7
= a
8
= a
9
= a
10
=
1
16
.
2. Paymkime =

a +

b +

c +

d. Pagal Cauchy-Schwarz nelygybs Engel


form:

2
10
=
(

a +

b +

c +

d)
2
10
a +
b
2
+
c
3
+
d
4
12

2
10
12a + 6b + 4c + 3d
= 3(a +b +c +d) + (a +b +c) + 2(a +b) + 6a
3 30 + 14 + 2 5 + 6 1 = 120
10.
3. Nelygyb transformuojame naudodami duot slyg ir tada sprendiame naudo-
dami Cauchy-Schwarz nelygyb:
KAIR PUS =
b
2
c
2
a(b +c)
+
a
2
c
2
b(a +c)
+
a
2
b
2
c(a +b)

(ab +bc +ac)


2
2(ab +bc +ac)
=
ab +bc +ac
2

3
3

a
2
b
2
c
2
3
(AM-GM)
=
3
2
.
177
Sprendimai
4. Pagal Cauchy-Schwarz nelygyb:
KAIR PUS =

cyc
_
x
n
(3x
1
+x
2
)

_
_

cyc
x
n
__

cyc
3x
1
+x
2
_
=

_
4
_

cyc
x
n
_
2
= 2(x
1
+x
2
+... +x
n
).
5. Pertvark taikome Cauchy-Schwarz nelygyb:
(a +b +c)
_
a
(b +c)
2
+
b
(a +c)
2
+
c
(a +b)
2
_

_
a
b +c
+
b
a +c
+
c
a +b
_
2

9
4
.
Paskutin nelygyb remiasi Nesbitto nelygybe, o tai ir ubaigia rodym.
6. Pagal Cauchy-Schwarz nelygyb:
x
ay +bz
+
y
az +bx
+
z
ax +by

(x +y +z)
2
x(ay +bz) +y(az +bx) +z(ax +by)
=
(x +y +z)
2
(xy +yz +xz)(a +b)

3
a +b
.
Paskutin nelygyb teisinga pagal
(x +y +z)
2
3(xy +yz +xz).
7. Nelygyb pertvarkome, tada taikome Cauchy-Schwarz nelygyb, tada vl per-
tvarkome:
KAIR PUS =

cyc
a
2
a +ab
2
c

(a +b +c)
2
a +b +c +abc(a +b +c)
=
a +b +c
abc + 1
.
Belieka rodyti
2(a +b +c) 3abc + 3,
kas pagal duot slyg yra ekvivalentu
(a +b +c)
3
27abc,
kas seka i AM-GM nelygybs.
8. rodymas remiasi matematine indukcija. Akivaizdu, kad jei nelygyb teisinga su
n = k, tai teisinga ir su n = k + 1. Taigi, belieka rodyti kai n = 2:
_
a
2
1
+b
2
1
+
_
a
2
2
+b
2
2

_
(a
1
+a
2
)
2
+ (b
1
+b
2
)
2
.
178
Sprendimai
Atskliaudus ir sutvarkius:
(a
2
1
+a
2
2
)(b
2
1
+b
2
2
) (a
1
b
1
+a
2
b
2
)
2
,
kas yra tiesiog Cauchy-Schwarz nelygyb. i nelygyb taip pat galima rodyti
naudojantis Pitagoro teorem, ia rodymo nepateiksime, bet galite pabandyti j
patys atrasti.
9. Lemma. 3(a
3
+b
3
+c
3
) (a +b +c)(a
2
+b
2
+c
2
).
Lemos rodymas. Naudojame AM-GM nelygyb: 3(a
3
+ b
3
+ c
3
) =

cyc
a
3
+

sym
a
3
+a
3
+b
3
3


cyc
a
3
+

sym
3a
2
b
3
= (a +b +c)(a
2
+b
2
+c
2
).
Pagal Cauchy-Schwarz nelygyb ir lem:
(DEIN PUS)
2
(a
2
+b
2
+c
2
)((b +c) + (a +c) + (a +b))
= 2(a
2
+b
2
+c
2
)(a +b +c)
6(a
3
+b
3
+c
3
) = 6(KAIR PUS).
Kita vertus, pagal AM-GM:
DEIN PUS 3
3
_
abc
_
(b +c)(a +c)(a +b)
3
3
_
abc

8abc
= 3
3
_
2

8 2 = 6.
Gauname, kad:
6(DEIN PUS) (DEIN PUS)
2
6(KAIR PUS)
KAIR PUS DEIN PUS, k ir reikjo rodyti.
10. Pagal Cauchy-Schwarz nelygyb:
(x +y)(z +x) (

xy +

xz)
2
.
Taip sumain vis trupmen vardiklius gausime:

cyc
x
x +
_
(x +y)(x +z)

cyc
x
x +

xy +

xz
=

cyc

x +

y +

z
= 1.
11. Pagal Cauchy-Schwarz nelygyb:
KAIR PUS =

cyc
a
2
ab+ac

(a+b+c+d+e+f)
2
ab+ac+bc+bd+cd+ce+de+df+ef+ea+fa+fb
.
Pavadinkime gaut vardikl V . Tada:
2V = (a +b +c +d +e +f)
2
(a +d)
2
(b +e)
2
(c +f)
2
.
Taiau vl i Cauchy-Schwarz nelygybs:
(1 + 1 + 1)
_
(a +d)
2
+ (b +e)
2
+ (c +f)
2
_
(a +b +c +d +e +f)
2
.
Taigi, V
1
3
(a +b +c +d +e +f)
2
, kas ubaigia rodym.
179
Sprendimai
12. Cauchy-Schwarz nelygyb naudosime dukart. Pirmiausia,
ax +by +cz
_
a
2
+b
2
+c
2

_
x
2
+y
2
+z
2
.
Taigi,
KAIR PUS

cyc
a
2

cyc
x
2
+

cyc
ab

cyc
xy

cyc
x
2
+ 2

cyc
xy

cyc
a
2
+ 2

cyc
ab
= (a +b +c)(x +y +z)
= a +b +c.
13. Pirmiausia pertvarkome:

cyc
a +b +c
b +c
+

cyc
a
b +c

cyc
2a
b
3 + 2

cyc
a
b +c

cyc
2a
b

cyc
a
b

a
b +c

3
2

ac
b(b +c)
+
ab
c(a +c)
+
bc
a(a +b)

3
2

a
2
c
2
abc(b +c)
+
a
2
b
2
abc(a +c)
+
b
2
c
2
abc(a +b)

3
2
.
Paskutinei nelygybei pritaik Cauchy-Schwarz nelygyb gausime:
a
2
c
2
abc(b +c)
+
a
2
b
2
abc(a +c)
+
b
2
c
2
abc(a +b)

(ab +bc +ac)
2
2abc(a +b +c)

3
2
.
Paskutinei nelygybei rodyti naudojome gerai inom fakt, kad realiesiems x, y, z
galioja (x +y +z)
2
3(xy +xz +yz).
14. Padaugin nelygyb i -2 ir prie abiej pusi pridj po 3, gausime ekvivaleni
nelygyb
a
2
+b
2
2 +a
2
+b
2
+
a
2
+c
2
2 +a
2
+c
2
+
c
2
+b
2
2 +c
2
+b
2

3
2
. (1)
Naudosimes Cauchy-Schwarz nelygybe:
KAIR PUS (1)
_

a
2
+b
2
+

a
2
+c
2
+

b
2
+c
2
_
2
6 + 2(a
2
+b
2
+c
2
)
=
2(a
2
+b
2
+c
2
) + 2

cyc
_
(a
2
+b
2
)(a
2
+c
2
)
6 + 2(a
2
+b
2
+c
2
)

2(a
2
+b
2
+c
2
) + 2

cyc
(a
2
+bc)
6 + 2(a
2
+b
2
+c
2
)
=
(a +b +c)
2
+ 3(a
2
+b
2
+c
2
)
6 + 2(a
2
+b
2
+c
2
)
=
3(3 +a
2
+b
2
+c
2
)
2(3 +a
2
+b
2
+c
2
)
=
3
2
.
180
Sprendimai
15. Visur taikysime Cauchy-Schwarz nelygyb. Pastebkime, kad
(a
2
+ 2)(b
2
+ 2) = (a
2
+ 1)(1 +b
2
) +a
2
+b
2
+ 3
(a +b)
2
+
(a +b)
2
2
+ 3
=
3
2
((a +b)
2
+ 2).
Tuomet
(a
2
+ 2)(b
2
+ 2)(c
2
+ 2)
3
2
((a +b)
2
+ 2)(2 +c
2
)

3
2
(

2(a +b) +

2c)
2
= 3(a +b +c)
2
.
Specialios technikos
1. Pirma mintis - atlikti homogenizuojant keitin a =
x
y
, taiau netrunkame sitikinti
kad tai nieko gero neduoda, todl tenka pasukti galv iekant kitokio kelio. Ir tai
- keitinys a =
1
x
, b =
1
y
, c =
1
z
isprs problem. inoma, nepamirkime, kad
vistiek xyz = 1. Nelygyb tampa
1 +
3
xy +yz +zx

6
x +y +z
.
Kadangi
xy +yz +xz
1
3
(x +y +z)
2
,
tai belieka rodyti:
1 +
9
(x +y +z)
2

6
x +y +z
,
kas seka i AM-GM.
2. Nesunku pamatyti, kad reikia pasikeisti a =
2x
y
, b =
2y
z
, c =
2z
a
. Gausime
nelygyb:
2x 2y
2x +y
+
2y 2z
2y +z
+
2z 2x
2z +x
0
y
2x +y
+
z
2y +z
+
x
2z +x
1.
Pagal Cauchy-Schwarz nelygyb:

cyc
x
2z +x

(x +y +z)
2
x(2z +x) +y(2x +y) +z(2y +z)
= 1.
3. Kadangi abc = 1, keiiame a =
x
y
, b =
y
z
, c =
z
x
. Tuomet gausime, kad reikia
rodyti

cyc
z
2
y
2
+xz

3
2
.
181
Sprendimai
Pritaikome Cauchy-Schwarz nelygyb:

cyc
z
4
z
2
y
2
+xz
3

(x
2
+y
2
+z
2
)
2
x
2
y
2
+x
2
z
2
+y
2
z
2
+xz
3
+yx
3
+zy
3
.
Belieka rodyti
2(x
2
+y
2
+z
2
)
2
3(x
2
y
2
+x
2
z
2
+y
2
z
2
+xz
3
+yx
3
+zy
3
),
kas ekvivalentu i dviej nelygybi (kurios galioja pagal AM-GM nelygyb) su-
mai:

cyc
x
4

cyc
x
3
y
ir

cyc
x
4
+x
2
y
2
2

cyc
x
3
y.
4. Duota nelygyb yra homogenin, todl j rodysime kai a
2
+ b
2
+ c
2
+ d
2
= 1.
Nelygyb tampa:
a
1 a
2
+
b
1 b
2
+
c
1 c
2
+
d
1 d
2

3

3
2
.
Pagal AM-GM nelygyb:
2a
2
(1 a
2
)(1 a
2
)
_
2a
2
+ 1 a
2
+ 1 a
2
3
_
3
=
_
2
3
_
3
a(1 a
2
)
2
3

a
1 a
2

3

3
2
a
2
.
Taigi:
a
1 a
2
+
b
1 b
2
+
c
1 c
2
+
d
1 d
2

3

3
2
(a
2
+b
2
+c
2
+d
2
) =
3

3
2
.
5. Kadangi turime homogenin nelygyb, nemaindami bendrumo tariame, kad
a +b +c = 3. Pertvark gausime:
(3 +a)
2
2a
2
+ (3 a)
2
+
(3 +b)
2
2b
2
+ (3 b)
2
+
(3 +c)
2
2c
2
+ (3 c)
2
8

a
2
+ 6a + 9
a
2
2a + 3
+
b
2
+ 6b + 9
b
2
2b + 3
+
c
2
+ 6c + 9
c
2
2c + 3
24
3 +
8a + 6
(a 1)
2
+ 2
+
8b + 6
(b 1)
2
+ 2
+
8c + 6
(c 1)
2
+ 2
24.
Kadangi (x 1)
2
+ 2 2 visiems x, tai belieka rodyti
8(a +b +c) + 18 42,
kas pagal slyg a +b +c = 3 yra tapatyb.
182
Sprendimai
6. Pasikeiskime x =
1
a
, y =
1
b
, z =
1
c
. Slyga taps xy + xz + yz = 1. Pagrindin
nelygyb:
x

1 +x
2
+
y
_
1 +y
2
+
z

1 +z
2

3
2
,
arba
x
_
x
2
+xz +xy +yz
+
y
_
y
2
+xz +xy +yz
+
z
_
z
2
+xz +xy +yz

3
2
,
arba
x
_
(x +y)(x +z)
+
y
_
(y +x)(y +z)
+
z
_
(z +x)(z +y)

3
2
.
Pagal AM-GM nelygyb:

cyc
x
_
(x +y)(x +z)
=

cyc
x
_
(x +y)(x +z)
(x +y)(x +z)


cyc
1
2

x(x +y) +x(x +z)
(x +y)(x +z)
=
1
2

cyc
x
x +z
+
x
x +y
=
3
2
.
7. Pasikeiskime a =
x
y
, b =
y
z
, c =
z
t
, d =
t
u
, e =
u
a
. Tada po nedideli pertvarkym
gausime:

cyc
a +abc
1 +ab +abcd
=

cyc
1
y
+
1
t
1
x
+
1
z
+
1
u
.
O tada dar pakeit
1
x
= a
1
,
1
y
= a
2
,
1
z
= a
3
,
1
t
= a
4
,
1
u
= a
5
ir paprastumo dlei
paymj S = a
1
+a
2
+a
3
+a
4
+a
5
, gausime, kad reikia rodyti

cyc
a
2
+a
4
a
1
+a
3
+a
5

10
3
. (1)
Dabar taikome Cauchy-Schwarz nelygyb, neymiai pertvarkome vardikl ir dar
kart taikome Cauchy-Schwarz nelygyb:
KAIR PUS(1)
4S
2

cyc
(a
2
+a
4
)(a
1
+a
3
+a
5
)
=
4S
2
2S
2

cyc
(a
1
+a
3
)
2

4S
2
2S
2

4S
2
5
=
10
3
.
183
Sprendimai
8. Neprarasdami bendrumo tariame, kad a + b + c + d = 1. Tuo naudodamiesi
rodysime, kad
(a +b)(b +c)(c +d)(d +a) abc +bcd +cda +dab.
Tai reikalauja tiesiog pertvarkyti nelygyb ir pritaikyti fakt x
2
0:
(a +b)(b +c)(c +d)(d +a) = a
2
c
2
+b
2
d
2
+ 2abcd +

cyc
abc(a +b +c)
= (ac bd)
2
+

cyc
abc(a +b +c +d)


cyc
abc.
Dabar rodysime
_

cyc
abc
_
3
16a
2
b
2
c
2
d
2
(a +b +c +d).
Pakeit abc = x, bcd = y, cda = z, dab = t, gauname
(x +y +z +t)
3
16(xyz +yzt +ztx +txy).
Taikykime AM-GM nelygyb:
KAIR PUS =
=

cyc
x
3
+
3
2

sym
x
2
y + 6

cyc
xyz
=
1
3

cyc
x
3
+y
3
+z
3
+
1
4

sym
x
2
y +x
2
z +y
2
x +y
2
z +z
2
x +z
2
y + 6

cyc
xyz


cyc
xyz +
3
2

sym
xyz + 6

cyc
xyz
= 16

cyc
xyz.
9. Slyga a, b, c [0, 1] sueruoja apie trigonometrin keitin. Ir ities, pasikeit
a = sin
2
x, b = sin
2
y, c = sin
2
z, kur x, y, z [0,

2
], gauname tai, k reikia:
sin xsin y sin z + cos xcos y cos z < sin xsin y + cos xcos y = cos(x y) < 1.
10. Pakeit a = y + z, b = x + z, c = x + y, padalin i xyz ir sutvark nelygyb
gausime, jog tereikia rodyti
x
2
y
+
y
2
z
+
z
2
x
+
x
2
z
+
y
2
x
+
z
2
y
2x + 2y + 2z.
Taiau tai yra dviej nelygybi, kurios tiesiogiai rodomos su Cauchy-Schwarz
nelygybe, suma:
x
2
y
+
y
2
z
+
z
2
x
x +y +z
ir
x
2
z
+
y
2
x
+
z
2
y
x +y +z.
184
Sprendimai
11. Nelygyb dauginame i 4, pertvarkome, tada taikome Cauchy-Schwarz nelygybs
Engel form, nes i trikampio nelygybs seka, kad visi vardikliai teigiami:
4 (KAIR PUS) = 3 +
a +b c
3a b +c
+
b +c a
3b c +a
+
c +a b
3c a +b
3 +
(a +b +c)
2

cyc
(a +b c)(3a b +c)
= 3 +
(a +b +c)
2

cyc
3a
2
ab +ac + 3ab b
2
+bc 3ac +bc c
2
= 4.
12. Atliekame Ravi keitin: a = x +y, b = y +z, c = z +x. Gausime:
3
_
_
(x +y)(x +z) +
_
(x +y)(y +z) +
_
(z +y)(x +z)
_
2
_
x +

y +

z
_
2
.
Bet pagal AM-GM nelygyb:
_
(x +y)(x +z) =
_
x
2
+xy +xz +yz
_
x
2
+ 2x

yz +yz = x +

yz.
Analogikai pasielg su likusiais nariais gausime nauj nelygyb, kuriai vl taikome
AM-GM nelygyb:
3(x +y +z) + 3(

yz +

xz +

xy) 2(x +y +z) + 4(

yz +

xz +

xy)
= 2
_
x +

y +

z
_
2
.
13. Pertvarkykime kairs puss dmenis, kad jie tapt apversti ir ikart taikykime
AM-GM nelygyb:
a
1 +b
2
=
a +ab
2
ab
2
1 +b
2
= a
ab
2
1 +b
2
a
ab
2
2b
= a
ab
2
.
Analogikai pertvarkius likusius dmenis, nelygyb pavirs

cyc
a
1 +b
2
a +b +c
1
2

cyc
ab
3
2
.
Paskutin nelygyb rodome pasinaudoj faktu
ab +bc +ca
(a +b +c)
2
3
= 3.
14. Pertvarkome, taikome AM-GM:

cyc
a +ab
2
c ab
2
c
1 +b
2
c


cyc
a
ab
2
c
2b

c
=

cyc
a
1
2
b

ac a


cyc
a
1
4
b(ac +a)
= a +b +c +d
1
4

cyc
abc
1
4

cyc
ab.
185
Sprendimai
Pagal AM-GM nelygyb:

cyc
abc
1
16
(a +b +c +d)
3
= 4,
o pagal Cauchy-Schwarz nelygyb:

cyc
ab = (a+b+c+d)
2
(a+c)
2
(b+d)
2
(a+b+c+d)
2

(a +b +c +d)
2
2
= 4.
Taigi,
a
1 +b
2
c
+
b
1 +c
2
a
+
c
1 +d
2
a
+
d
1 +a
2
b
a +b +c +d 2 = 2.
15. Pertvarkome, taikome AM-GM:

cyc
1
a
3
n
+ 2
=
n
2

1
2

cyc
a
3
n
a
3
n
+ 2

n
2

1
2

cyc
a
3
n
3a
n
=
n
3
.
16. Naudosime Cauchy Reverse Technique:

cyc
a + 1
b
2
+ 1
=

cyc
a + 1
ab
2
+b
2
b
2
+ 1

cyc
a + 1
ab +b
2
.
Pagal Cauchy-Schwarz nelygyb:

cyc
ab =
1
2
((a +b +c +d)
2
(a +c)
2
(b +d)
2
)

1
2
((a +b +c +d)
2

(a +b +c +d)
2
2
) = 4.
Taigi:

cyc
a + 1
b
2
+ 1
a +b +c +d + 4
4 +a +b +c +d
2
= 4.
17. Lema. x(2 x) 1, su realiais x.
Lemos rodymas. (x 1)
2
0
Pertvarkome pagrindin nelygyb ir taikome lem:

cyc
1
2 a
=
3
2
+

cyc
a
2
2a(a 2)

3
2
+

cyc
a
2
2
= 3.
18. Pertvarkome ir du kartus taikome AM-GM bei nelygyb ab +bc +ac
(a+b+c)
2
3
:

cyc
a
2
a + 2b
3
=

cyc
a
2b
3
a
a + 2b
3


cyc
a
2b
3
a
3
3

ab
6
=

cyc
a
2
3
3

b
3
a
2


cyc
a
2
9
(ab +ab +b)
a +b +c
2
27
_
2(a +b +c)
2
+ 3(a +b +c)
_
= 1.
186
Sprendimai
Funkcins lygtys
sistatykime x = 0
1. sistatykime y = 0, gausime f(x) = x
2
. Patikrin matome, kad i funkcija tinka.
2. sistatykime y = 0. Gausime, kad su visais x turi bti f(x) = 1, taiau i funkcija
lygties netenkina. Sprendini nra.
3. sistat x = 0 gauname f(y) = (y+1)f(0), t.y. vienintels funkcijos kurios galt
tikti yra f(x) = c(x + 1). Patikrin gauname, kad tinka tik c = 0, t.y. f(x) = 0.
4. sistatykime vietoje y bet kok nelyg nuliui skaii, pavyzdiui 1. Gausime
f(x) = f(1)x, vadinasi, iekomos funkcijos bus pavidalo f(x) = cx, kur c reali
konstanta. Patikrin gauname, kad visos tokios funkcijos tinka.
5. sistatykime y = 1, gausime f(x + f(1)) = 0. Kadangi f(1) yra konkretus
skaiius, tai x = f(1) gyja visas realias reikmes, i kur gauname, kad funkcija
turi tenkinti f(x) = 0. Patikrin matome, kad is sprendinys tinka.
6. sistatykime x = x. Gausime xf(x) + f(x) + 1 = 0. I pradins lygties
isireik f(x) ir sistat gausime f(x) =
1+x
1+x
2
, kas ir yra sprendinys.
7. sistat x =
x1
x
ir x =
1
1x
kartu su pradine turime tris lygtis, i kuri papluj
isireikiame f(x). Gauname f(x) =
x
3
3x
2
+2x1
2x2x
2
.
8. sistat x = 1 ir z = 1 gauname f(t) = tf(1), t.y. funkcija gali bti tiktai
pavidalo f(x) = ax, a R. Patikrin matome, kad visos tokios funkcijos tinka.
9. sistatykime y = 0 ir y = 1. I gaut lygybi gauname, kad f(x) - tiesin funkcija
(f(x) = (f(1) f(0))x + f(0)). Patikrin matome, kad funkcijos f(x) = ax + b
tinka su visais a, b R.
10. stat vietoje t bet koki reikm, su kuria f(t) ,= 0, gauname, kad f(x) - tiesin
funkcija. Patikrin gauname, kad tinka tik f(x) = 1 x ir f(x) = 1 +x.
11. Taip:
f(x) =
_
x, x < 0,
x
3
, x 0.
12. sistat x = y gauname f(f(0)) = x
2
f(x)
2
. sistat x = y = 0 gauname
f(f(0)) = f(0)
2
. I i dviej lygybi gauname f(0)
2
x
2
= f(x)
2
0, kas
negalioja su visais x R. Vadinasi funkcij tenkinani lygt nra.
13. Kadangi visiems realiesiems a, b egzistuoja tokie x,y, kad x +y = a ir x y = b,
tai lygt galime urayti b
2
f(a) = a
2
f(b). Jei egzistuoja toks b
0
, kad f(b
0
) ,= 0,
tai j stat vietoje b gauname f(a) = ca
2
, kur c - konstanta (jei neegzistuoja, tai
f(x) = 0). sistat gauname, kad tinka visos c reikms, vadinasi, sprendiniai yra
f(x) = cx
2
.
187
Sprendimai
14. stat x = 0 ir stat y = 0 gauname f(f(x)) = f(f(0)) + x ir f(x + f(0)) =
f(f(x)), i kur f(x +f(0)) = f(f(0)) +x = f(x) = x +c.
15. sistatykime x = y ir y = x (t.y sukeiskime kintamuosius vietomis). Gausime
f(x + y) = 3
x
f(y) + 2
y
f(x). Atm i ios lygybs pradin ir sistat y = 1
gausime f(x) = 3
x
2
x
. Patikriname - tinka.
16. Rasti bent vien funkcij nra visai paprasta, taiau kiek pamst matome, kad
f(x) = x +
1
x
. Taip pat tiks ir f(x) = x
2
+
1
x
2
ir f(x) = x
3
+
1
x
3
.
statykime x = 1 ir y = 1. Gausime f(1)
2
= 2f(1). Kadangi funkcija gyja tik
teigiamas reikmes, tai f(1) = 2.
statykime y = x. Gausime, kad f(x)
2
= f(x
2
) + 2. Kadangi su visais x R
f(x
2
) 0, tai su visais x R f(x)

2. Dabar, kadangi su visais x R
f(x
2
)

2, tai su visais x R f(x)
_
2 +

2. Taip tsdami, gauname, kad


su kiekvienu x ir kiekvienu n N
f(x)

2 +
_
2 + +
_
2 +

2
. .
n
.
Kadangi, kai n artja begalyb,

2 +
_
2 + +
_
2 +

2
. .
n
artja 2 (seka aki-
vaizdiai didjanti, maesn u 2 turi rib. J randame isprend

2 +x =
x x = 2), tai f(x) 2.
c.) dalyje utenka pasinaudoti sistaius f(x)
2
= f(x
2
) + 2, ir norint sitikinti,
kad f
2
(x) 2 0 - b.) dalyje gauta nelygybe.
17. Atkreipsime dmes, kad jei nebt lygties apribojimo vien teigiamiems skai-
iams, tai sistat y = 0 i karto gautume, kad f(x) = c. Taiau apribojimas yra,
todl suktis teks kiek kitaip.
Fiksuokime sum ir irkime, kaip kinta sandauga. T.y. sistatykime pvz., y =
2x. Gausime f(x(2x)) = f(2). Kadangi galime statyti tik teigiamas reikmes,
tai i lygyb yra teisinga tik, kai x (0, 2). iame intervale, kintant x reikmei,
reikinio x(2 x) reikm kinta nuo 0 iki 1, t.y. x(2 x) (0, 1]. Tad gauname,
kad f(x) yra pastovi intervale (0, 1]. Lieka pastebti, kad ji periodin: stat
y = 1 gausime f(x) = f(x + 1), todl pastovi ir visur.
18. Fiksuokime sum. Tegu x = 2y. Tuomet f(2) = f(
2
x(2x)
). Kai x kinta nuo
iki reikinys
2
x(2x)
kinta intervaluose (, 0) [
1
2
, ), kartu ir funkcija tuose
intervaluose pastovi. Likusi dal (0,
1
2
) galime prijungti naudodami f(x +
1
2
) =
f(
1
x
+ 2). Pastebsime, kad funkcijos reikm take 0 taip ir lieka neapibrta.
Atsakymas
f(x) =
_
a, x ,= 0
b, x = 0 a, b R
188
Sprendimai
19. Atsikratykime penketo: sistatykime f(x) = g(x)
5
2
. Gausime, kad g tenkina
lygt g(2x + 1) = 3g(x). Paband pirmas kelet reikmi gausime, kad
g(1) = 3g(0), g(3) = 3g(0), g(7) = 3
2
g(0), g(15) = 3
3
g(0).
siirj pamatysime, kad taip tsdami gausime
g(2
n
1) = 3
n
g(0).
Paymj 2
n
1 = x gauname n = log
2
(x+1) arba g(x) = 3
log
2
x+1
g(0). I f(0) =
0 seka, kad g(0) =
5
2
ir susitvark su neigiam skaii keliamais nepatogumais
gauname, kad
f(x) = 3
log
2
|x+1|

5
2

5
2
tenkina lygt.
20. sistatykime y = 0 ir y = 1:
f(x
3
) = (x
2
+x + 1)(f(x) f(1)) +f(1),
f(x
3
) = x
2
(f(x) f(0)) +f(0).
Lygybs teisingos su visomis x reikmmis, tad sulygin deinisias puses gausime
f(x) = xf(1) + (1 x)f(0),
t.y. funkcija tiesin. Patikrin matome, kad lygt tenkina visos funkcijos f(x) =
ax +b, kur a, b R.
21. is udavinys, nors ir paprastas, yra gerai inomi spstai. I pirmo vilgsnio
padaryta ivada, kad sprendinai yra tik f(x) = 1 ir f(x) = 1 nra teisinga.
Atidiau pavelgus tampa aiku, kad viskas, k galima pasakyti apie funkcij, yra
tai, kad bet kuriame take ji gyja reikm 1 arba 1. Uraius t matematikiau,
sprendiniai atrodo kaip
f(x) =
_
1, x A,
1, x , A,
kur A bet koks R poaibis.
Funkcij tipai
1. Jei funkcija grietai didjanti, tai visiems skirtingiems a > b tursime f(a) > f(b),
todl funkcija negis vienod reikmi.
Bijektyvi funkcija nebtinai turi bti monotonika. Pavyzdiui, f(x) =
1
x
, kai
x ,= 0, ir f(0) = 0.
2. Negali, nes, pavyzdiui, stat x = 0 matome, kad f(y) = f(y) su visais y.
3. stat x = x ir y = y gauname f(x + y) = f(x y) f(t) = f(t)
t R.
189
Sprendimai
4. Lygins monotonins yra tik f(x) = c, f : A R, lygin injektyvi tik f(0) = c,
f : 0 R (jei dar bent viena tak pora priklausyt apibrimo sriiai i karto
gautume neinjektyvi). Lygins surjektyvios pavyzdys gali bti f(x) = ln[x[, kai
x ,= 0, f(0) = 0.
5. Tegu a > b. stat x = b, y = a b gausime f(a)f
2
(a b) = f(b). Kadangi
f(a b)
2
1, tai f(a) f(b).
6. Jei inome, kad funkcija yra surjektyvi, tai egzistuoja toks a, kad f(a) = 1. stat
gauname f(a 1) = a 1 = a, vadinasi f(1) = 1.
7. rodysime, kad f(x) = x. Tarkime prieingai - tegu egzistuoja toks a, kad
f(a) > a. Tuomet, kad kadangi f yra didjanti, tai
f(a) > a f(f(a)) > f(a) f(f(a)) > f(a) > a f(f(a)) ,= a - prietara.
Tardami, kad f(a) < a, prietar gauname analogikai.
8. stat x = 0 ir x = 1 gauname, kad f(a+b) = f(b), todl pasinaudoj injektyvumu
gauname a = 0. stat x = b gauname f(b)f(1 b) = f(b), tad arba f(1 b) = 1,
arba f(b) = 0. Taiau f(b) negali bti lygus nuliui, nes gautume f(x)f(1x) = 0,
o i ia be galo daug reikmi, su kuriomis funkcija lygi nuliui, kas prietarauja
injektyvumui. Galiausiai pastebkime, kad funkcija nulio i vis negyja, nes jei,
tarkime, f(c) = 0, tai stat gauname f(b) = 0, ko negali bti. Taigi ji nra
surjektyvi.
9. sistatykime x = y, y = x, gausime f(x + f(y)) = f(y + f(x)). Kadangi f yra
grietai didjanti, tai ji injektyvi, tai x +f(y) = y +f(x) f(x) = x +c. stat
randame c = 2005.
10. sistatykime x = y, gausime (y + y)(f(y)y) = y
2
f(f(y) + f(y)). Jei f(x) =
f(y), tai i abiej lygybi gauname
x
2
x+y
=
y
2
y+y
x = y. Gavome kad funkcija
injektyvi. statykime y = 1 ir x =
1+

5
2
= , t.y. lygties x + 1 = x
2
sprendin.
Tuomet gausime, kad f(f()) = f(f(1) +f()) f(1) = 0, o taip bti negali.
11. Nesunku pastebti, kad funkcija yra injektyvi. sistat x = 1, y = 1 gauname
f(f(1)) = f(1) = f(1) = 1. sistat x = 1 gauname (y+1)f(y) = f(y)+1 =
f(y) =
1
y
.
12. Kadangi g yra surjektyvi ir f(y) +x gyja visas realisias reikmes, tai i lygybs
gauname, kad ir f surjektyvi. Tegu a toks, kad g(a) = 0. stat x = a gauname
f(y) = g(f(y) +a). Kadangi f surjektyvi, tai g(x) = x a su visas x R. stat
g iraik pradin lygyb gauname, kad f(x+y a) = f(y) +xa. stat y = a
gauname f(x) = x +b. Vadinasi, sprendiniai yra g(x) = x +a, f(x) = x +b, kur
a, b R.
13. rodykime, kad f injektyvi. Naudodami keitin x +y = a, xy = b gauname lygt
f(a +f(b)) = f(f(a)) +b. Taiau ji galioja ne visiems a ir b, o tik tenkinantiems
slyg 4b a
2
, nes kitaip sistema x +y = a, xy = b neturi sprendini. Bet tai ne
bda - kiekvieniem b
1
ir b
2
galime paimti a tok, kad 4b
1
a
2
ir 4b
2
a
2
. Tuomet
190
Sprendimai
galime naudotis lygtimi ir i f(b
1
) = f(b
2
) gauname b
1
= b
2
- injektyvumas
rodytas. statykime pradin lygt y = 0, gausime f(x + f(0)) = f(f(x)), i
injektyvumo f(x) = x +c.
14. I lygybs g(f(x)) = x
3
seka, kad f yra injektyvi ir kad f(g(f(x))) = f(x
3
)
f
2
(x) = f(x
3
). sistat x = 1, 0, 1 gauname, kad f(1), f(0) ir f(1) gali gyti
tik reikmes 0 arba 1, kas prietarauja injektyvumui.
15. Pastebkime, kad f injektyvi. statykime x = 0, gausime f(f(0)) =
f(0)
2
. staty-
kime x = f(0), gausime 4f(f(f(0))) = 2f(f(0)) +f(0) = f(0) +f(0) = 2f(0), i
kur f(f(f(0))) =
f(0)
2
= f(f(0)). Naudodamiesi injektyvumu gauname
f(f(f(0))) = f(f(0)) f(f(0)) = f(0) f(0) = 0.
Kadangi funkcija injektyvi, tai ities f(x) = 0 x = 0.
16. Patyrinkime keitin y =
x
f(x)1
. I pradi gali pasirodyti, kad jis yra nuleistas i
dangaus, bet viskas daug paparasiau - jis tiesiog kyla i natralaus noro sulyginti
f(yf(x)) ir f(x +y) (yf(x) = x +y = y =
x
f(x)1
). Taiau prisiminkime, kad
funkcijos apibrimo sritis yra teigiami skaiiai. Tuomet tenka samprotauti taip:
jei egzistuoja toks x, kad f(x) > 1, tai galime sistatyti y =
x
f(x)1
ir gausime
f(x)f(
xf(x)
f(x)1
) = f(
xf(x)
f(x)1
) f(x) = 1 - prietara!
Vadinasi gavome, kad f(x) 1 ir, i pradins lygybs, f yra nedidjanti (f(x +
y) = f(x)f(yf(x)) f(x)).
Nagrinkime injektyvum: Jei egzistuoja tokie a < b, kad f(a) = f(b), tai gau-
name, kad f(a + y) = f(b + y) su visais y, todl f(y) = f(b a + y) su visais
y > a, vadinasi, funkcija yra monotonika ir periodin f(x) = c su visais x > a.
sistat pradin lygt pakankamai didelius x ir y gauname c = 1, o sistat x
pakankamai didel gauname f(y) = 1 su visais y. Lieka atvejis, kai funkcija yra
injektyvi. Pakeit y =
z
f(x)
gausime f(x)f(z) = f(x +
z
f(x)
) su visais z, x > 0.
Sukeit x ir z vietomis bei pasinaudoj injektyvumu gauname x+
z
f(x)
= z +
x
f(z)
,
i kur lengvai randame f(x) =
1
1+cx
, kur c R
+
.
17. Tegu f(x
0
) = 1, tada sistat x = x
0
gauname f(x
0
+ y) = f(y), vadinasi,
funkcija yra periodin ir vienet gis be galo daug kart, o to bti negali, vadinasi,
f(x) ,= 1, x R
+
.
Tegu f(a) = f(a + b), tuomet sistatykime x = a, y =
b
f(a)
, gausime 1 = f(
b
f(a)
),
prietara, vadinasi funkcija injektyvi.
Pradinje lygtyje statykime x = y, y = x, gausime f(x + yf(x)) = f(x)f(y) =
f(y + xf(y)). Kadangi f injektyvi, tai x + yf(x) = y + xf(y) f(x) = kx + 1.
Patikrin matome, kad tinka.
18. Pastebkime, kad f injektyvi. stat x = 0, y = 0 ir paymj f(0) +f(f(0)) = u
gauname f(u) = u. stat y = u gauname f(x +u) = f(f(x) +u) f(x) +u =
x +u f(x) = x.
191
Sprendimai
19. Funkcija akivaizdiai bijektyvi, todl egzistuoja toks x
0
, kad f(x
0
) = 0. sista-
t x = x
0
gauname f(f(y)) = y. sistat x = f(x) gauname f(x
2
+ f(y)) =
xf(x) + y = f(f
2
(x) + f(y)). Kadangi f injektyvi, tai f
2
(x) = f(x)
2
. Vadinasi,
kiekviename take x funkcija lygi arba x, arba x. Tegu egzistuoja du nenuliniai
takai, kuriuose f(x) = x ir f(y) = y. Tuomet gauname f(x
2
+y) = x
2
y, kas
yra nemanoma (x
2
+ y = x
2
y = y = 0, x
2
y = x
2
y = x = 0 ).
Vadinasi, tinka tik f(x) = x ir f(x) = x.
20. Funkcija bijektyvi. statykime x = 0 ir x = a, kur a toks, kad f(a) = 0. Gausime
lygybes f(f(y)) = f
2
(0) +y ir f(f(y)) = y, i kur f(0) = 0 ir a = 0.
statykime x = f(x) ir pasinaudokime lygybe f(f(x)) = x. Gausime f(f(x)x +
f(y)) = x
2
+y, vadinasi f
2
(x) = x
2
su visais x.
Tegu x ir y tokie, kad f(x) = x ir f(y) = y ir x,y ,= 0. Tada i pradins lygties
gauname f(x
2
y) = x
2
+ y. Kadangi f(x
2
y) gali bti lygus tik x
2
y arba
y x
2
, tai gauname, kad arba y = 0 arba x = 0 - prietara. Vadinasi, sprendiniai
yra f(x) = x ir f(x) = x.
21. stat y = z = 0 gauname f(h(g(x))) = x + h(0). stat y = 0 gauname
g(z + f(0)) = g(f(z)). Kadangi g injektyvi (atkreipkite dmes kintamj x
pradinje lygtyje), tai f(x) = x +a.
sistat gauname lygt h(g(x))+y+a = h(y)+x, i kurios akivaizdiai h(x) = x+b,
ir g(x) = x a.
22. Funkcija injektyvi. Raskime f(0): x = 0 f(f(y)) = y + f
2
(0) f(f(0)) =
f
2
(0). Paymkime f(0) = a, tuomet paskutinioji lygyb pavirsta f(a) = a
2
.
statykime x = 0, y = a ir x = a, y = 0. Gausime f(a
2
) = a
2
+ a ir f(a
2
+ a) =
a
4
+a
2
. I ia f(f(a
2
)) = f(a
2
+a) = 2a
2
= a
4
+a
2
= a = 1,0 arba 1.
Jei f(0) = 1, tai tuomet i f(f(y)) = y + f
2
(0) gauname f(1) = 1 - prietara
injektyvumui.
Jei f(0) = 1, tai i f(f(y)) = y + f
2
(0) gauname f(1) = 1 f(1) = 0
f(0) = 2 - prietara.
Vadinasi, f(0) = 0. Tuomet f(f(x)) = x ir f(x
2
) = f
2
(x). stat x = y
gauname f(f(y)) = f
2
(y) yf(y) + y y = f((y)
2
) yf(y) + y f
(
y) =
yf(y). Kadangi funkcija injektyvi, tai f(y) = 0 tik kai y = 0 f(y) = y.
23. f(x) = 0 yra sprendinys, iekosime likusi. rodykime, kad f turi bti lygin.
Pastebkime, kad f(f(x) f(y)) = f(f(y) f(x)), todl utenka rodyti, kad
f(x) f(y) gyja visas reikmes. Ities, tegu a toks, kad f(a) ,= 0. statykime
y = a x = f(f(x) f(a x)) = (2x a)
2
f(a). I ia matome, kad f gyja
visas teigiamas arba visas neigiamas reikmes (priklausomai nuo f(a)), vadinasi,
f(x) f(y) tikrai gyja visas realisias reikmes.
statykime y = y. Gausime f(f(x)f(y)) = (x+y)
2
f(xy) = (xy)
2
f(x+
y) = (x +y)
2
f(x y). Kadangi visiems realiesiems a, b egzistuoja tokie x,y, kad
x +y = a ir x y = b, tai lygt galime urayti b
2
f(a) = a
2
f(b) = f(x) = cx
2
.
Patikrin gauname c = 1, vadinasi, sprendiniai yra f(x) = x ir f(x) = 0.
192
Sprendimai
24. statykime x = 0, gausime f(0) +y = f(g(y)), vadinasi f surjektyvi, g injektyvi.
rodykime, kad g(1) = 1. statykime y = 0, gausime f(xg(1)) = xf(0) + f(x +
g(0)). Jei g(1) ,= 1, tai galime sulyginti xg(1) = x + g(0) pam x =
g(0)
g(1)1
.
Tuomet gauname
g(0)f(0)
g(1)1
= 0 f(0) = g(0) = 0 (pasinaudojus antrja slyga).
sistat y = 1 gauname f(x) = ax ir g(x) =
x
a
. Patikrin gauname, kad a = 1,
taigi f(x) = g(x) = x - prietara prielaidai g(1) ,= 1.
I f surjektyvumo inome, kad egzistuoja toks u, kad g(u) = 0. rodykime, kad
u = 0. Tegu u ,= 0, tada g(u + 1) ,= g(1) = 1 (i g injektyvumo). statykime
x =
g(u)
g(u+1)1
ir y = u, gausime u = 0 - prietara. Taigi gavome, kad f(0) = 0 ir
g(0) = 0, ir i ia jau inome, kad gaunasi f(x) = g(x) = x.
25. statykime x = 0. Gausime f(f(y)) = y. statykime x = f(x), gausime
f(f
2
(x) + f(y)) = y + f(x)x = f(x
2
+ f(y)). Kadangi funkcija tenkinanti lygt
yra akivaizdiai bijektyvi, tai gauname f
2
(x) = x
2
f(x) = x.
Tegu x ir y tokie, kad f(x) = x ir f(y) = y bei x,y ,= 0. Tada pradin lygtis
tampa f(x
2
y) = y +x
2
. Kadangi f(x
2
y) = x
2
y arba f(x
2
y) = y x
2
,
tai gauname y = 0 arba x = 0 - prietara. Vadinasi, sprendiniai yra tik f(x) = x
ir f(x) = x.
26. Funkcija bijektyvi, todl egzistuoja toks a, kad f(a) = 0. sistat x = y = a
gauname f(a
2
) = a f(f(a
2
)) = 0. Taiau kadangi f(f(y)) = y + f
2
(0), tai
a
2
+f
2
(0) = 0 a = 0 ir f(0) = 0.
Tuomet i pradins lygties gauname, kad f(x
2
) = f
2
(x) = f(x)
2
. Dl injekty-
vumo f(x) ,= f(x), todl f(x) = f(x). I ia ir i f(x
2
) = f
2
(x) gauname,
kad f(x) > 0, kai x > 0 ir f(x) < 0, kai x < 0.
Galiausiai stat y = x
2
gauname, kad f(x
2
f
2
(x)) = (x
2
f
2
(x)) f
2
(x) =
x
2
f(x) = x.
27. f(x) = 0 yra sprendinys, nagrinkime galimus likusius. Tegu f(a) = 0, tuomet
stat x = a gausime 0 = af(y) = a = 0, vadinasi, jei 0 yra gyjamas, tai tik
take 0. stat x = y = 1 gausime f(f(1) 1) = 0 = f(0) = 0, f(1) = 1.
stat x = 1 gauname f(f(y) + 1) = f(y) + 1() i kur f(n) = n visiems n N.
rodysime, kad f-injektyvi. Paymj xy = a gauname f(x+f(a)) = f(x)+xf(
a
x
).
Jei f(a) = f(b), tai visiems x teisinga f(
a
x
) = f(
b
x
). Pakeit x =
b
y
ir paymj
a
b
= m, gausime, kad su visomis y reikmmis f(ym) = f(y). I ia randame
f(m) = 1. stat x = m gauname f(m+f(y)) = 1+mf(y), i kur f(m+1) = m+1
(y = 1) ir f(m + 2) = 1 + 2m (y = 2). Taiau pagal (*) f(m + 2) = m + 2
(y = m+ 1), todl 1 + 2m = m+ 2 = m = 1 = a = b = f injektyvi.
statykime x = y = 2 = f(2) = 2 = f(1) = 1. statykime
x = 1 = f(1 +f(y)) = 1 f(y).
Naudodamiesi f(1 + f(y)) = 1 f(y) ir f(f(y) + 1) = f(y) + 1 gausime,
kad kiekvienam x egzistuoja toks y, kad f(x) = f(y) + 1. Ities: jei a priklauso
f vaizdui = 1 a priklauso vaizdui = a priklauso vaizdui = 1 +a
priklauso vaizdui. Kadangi kiekvienam x f(x) priklauso vaizdui, tai f(x) 1
193
Sprendimai
priklauso vaizdui, todl egzistuoja toks y, kad f(y) = f(x) 1. stat f(f(y) +
1) = f(y) + 1 gauname, kad kiekvienam x f(f(x)) = f(x). Kadangi f injektyvi,
tai f(x) = x.
28. Pastebkime, kad f(0) = 0 ir f(xf(x)) = x
2
(*). stat x = 1 gauname f(f(1)) =
1, stat x = f(1) gauname 1 = f(1)
2
. Jei f(1) = 1, tai f(x) + f(f(x)) = 2x -
f injektyvi. Jei f(1) = 1, tai stat x = y = 1 gauname f(1) = 1 ir stat
y = 1 gauname f(x) +f(f(x)) = 2x - f injektyvi.
rodysime, kad f(
1
x
) =
1
f(x)
. statykime y = f(
1
x
)
1
x
:
f(xf(f(
1
x
)
1
x
)) +f(f(
1
x
)
1
x
f(x)) = 2f(
1
x
).
I (*) gauname, kad
f(f(
1
x
)
1
x
) =
1
x
2
,
todl lygyb galime perrayti
f(f(
1
x
)
1
x
f(x)) = f(
1
x
).
Lieka pasinaudoti injektyvumu ir gauname f(x) =
1
x
.
Jei f(1) = 1, tai stat x =
1
x
f(x) +f(f(x)) = 2x gauname
1
f(x)
+
1
f(f(x))
=
2
x

1
f(x)
+
1
2x f(x)
=
2
x
(f(x) x)
2
= 0 f(x) = x.
Jei f(1) = 1, tai x =
1
x
statome f(x) + f(f(x)) = 2x ir analogikai
gauname f(x) = x.
Cauchy funkcin lygtis
1. Pasiymkime f(x) = g(x)+x
2
. Gausime g(x+y) = g(x)+g(y). Tada g(x) = kx,
ir f(x) = kx +x
2
. Nesunku patikrinti, kad sprendinys tiks.
2. Pasiymkime f(x) = g(x) + 1. Gausime g(x + 1 + g(y)) = g(x + 1) + y, arba
g(t + g(y)) = g(t) + y. I ia nesunku sitikinti, kad funkcija bijektyvi. stat
t = y = 0, gausime g(0) = 0, o paskui stat t = 0 - g(g(y)) = y. Tada prie tai
gautoje lygtyje pakeit y = g(y), gausime Cauchy funkcin lygt, i kur g(x) = kx.
Nesunku patikrinti, kad tiks tik k = 1 arba -1. Randame sprendinius f(x) = x+1
arba f(x) = 1 x.
3. Statykime x = y = 0. Gausime h(0) = f(0) g(0). Paimkime pradinje lygtyje
y = 0. Tada tursime g(x) = f(x) h(0) = f(x) + g(0) f(0). Paimkime
pradinje lygtyje x = 0. Gausime h(y) = f(y) g(0). stat gautas g(x) ir
h(y) iraikas pradin lygt gausime: f(x + y) = f(x) + f(y) f(0). sived
keitin i(x) = f(x) f(0), gausime, kad i tenkina Cauchy funkcin lygt ir yra
tolydi vadinasi i(x) = kx. Tada, jei paymsime f(0) = a ir g(0) = b, gausime
f(x) = kx +a, g(x) = kx +b a, h(x) = kx b
stat pradin lygt, gausime, kad a = 0, o k ir b - bet kokios realiosios konstantos.
194
Sprendimai
4. Pasiymkime f(x) = g(x) + 1. Tada pradin lygtis virs g(xy) + g(x + y) =
g(x)g(y) + g(x) + g(y). sistat x = y = 0 gausime g(0) = 0. Tada, paymj
g(1) = k, po nesudtingos indukcijos gausime
g(n) = k
n
+k
n1
+... +k, n N.
Jei g(1) = 1, tai gausime g(n) = n, kitu atveju g(n) =
k
n+1
1
k1
1. stat prie
tai turt lygt ir iprastin gausime
k
xy+2
k
xy+1
k
x+y+1
= k
2
k
x+1
k
y+1
.
ia galime statyti bet kokius naturaliuosius x ir y. T darydami, nesunkiai gau-
sime k = 1, 0, 1. Kai k = 0 gausime sprendin f(x) = 1. Kai k = 1, nesunkiai
gausime prietar. Kai k = 1, pradinje lygtyje stat x = 1, y = 1 gausime
g(1) = 1. Tada pradinje lygtyje pam y = 1, o paskui x = x, y = 1
ir sudj abi gautas lygybes gausime g(x) = g(x) visiems x R. Tada pra-
dinje lygybje pam x = x, y = y ir pritaik paskutinij lygyb gausime:
g(xy) g(x + y) = g(x)g(y) g(x) g(y). Sudj su pradine lygyb gausime
g(x + y) = g(x) + g(y) ir g(xy) = g(x)g(y), i ko, kaip jau matme pavyzdyje,
gausime g(x) = x. Taigi, ios lygties sprendiniai yra f(x) = x + 1 ir f(x) = 1.
5. Nesunku atspti, kad f(x) = x
2
yra lygties sprendinys. I ia kyla idja sivesti
keitin f(x) = g(x
2
), visiems x 0. Gausime g((x
2
y
2
)
2
+ (2xy)
2
) = g((x
2

y
2
)
2
)+g((2xy)
2
). Kita vertus, jei paymsime a = x
2
y
2
ir b = 2xy, tai nesunku
sitikinti, kad lygi sistema
_
a = x
2
y
2
b = 2xy
visados turs sprendini, kad ir kokius a ir b pasirinktume (tiesiog isireiktume
i antros lygties x, statytume pirm ir gautume kvadratin lygt y
2
atvilgiu,
kurios diskriminantas tikrai teigiamas). Tada gaut funkcin lygt galime pasi-
keisti g(a
2
+ b
2
) = g(a
2
) + g(b
2
) , arba g(z + t) = g(z) + g(t), kur z ir t
bet kokie neneigiami realieji. Kadangi turime f : R [0, +), funkcija g bus
aprta i apaios ir galime teigti, kad g(x) = kx visiems neneigiamiems x (nors
funkcija Cauchy lygt tenkina tik neneigiamiems skaiiams, nesunku sitikinti, kad
aprtumo vistiek uteks). Tada f(x) = kx
2
visiems teigiamiems x, bet pradinje
lygtyje pam y = 0, gausim f(0) = 0, o tada vl pradinje lygtyje pam x = 0
gautume f(y) = f(y) visiems y, taigi f(x) = kx
2
ir neigiamiems x.
6. Nesunku sitikinti, kad funkcija bijektyvi. staius x = 0, y = x
n
, gausime:
f(f(x
n
)) = x
n
+f(0)
n
. I bijektyvumo aiku, kad egzistuoja toks t, kad f(t) = 0 ir
z, kad f(z) = t. Tada sistat pradinje lygtyje y = t gausime: f(x
n
) = f(x)
n
+t.
Panaudoj tai ankiau gautoje lygtyje gauname f(f(x)
n
+t) = x
n
+f(0)
n
. Dabar
pradinje lygtyje pakeit x f(x) ir y z, gausime, kad f(f(x)
n
+t) = f(f(x))
n
+z
ir, sulygin tai su prie tai gauta lygtimi, gausime f(f(x))
n
+ z = x
n
+ f(0)
n
.
Galiausiai pagrindinje lygtyje pam x = 0 ir y = x, gausime f(f(x)) = x+f(0)
n
.
i f(f(x)) iraik stat prie tai gaut lygt gauname: (x + f(0)
n
)
n
+ z =
195
Sprendimai
x
n
+ f(0)
n
visiems x, i kur lengvai gauname f(0) = t = z = 0. Tai stat
prie tai turtas lygtis gausime f(f(x)) = x ir f(x
n
) = f(x)
n
. Tada pradinje
lygtyje pakeit y f(y) tursime f(x
n
+y) = f(x
n
) +f(y), kas jau labai panau
Cauchy funkcin lygt. Lyginiams n f(x +y) = f(x) +f(y), kur x teigiamas, o
y - betkoks. Tada pam y = x gauname, kad f(x) = f(x) x 0 ir taip
f(x + y) = f(x) + f(y) bet kokiems realiesiems x, y. Taiau ankiau turjome
f(x
n
) = f(x)
n
, taigi f(x) 0 visiems x 0 ir funkcija yra aprta intervale,
vadinasi, - pavidalo kx. Patikrin pradinje lygtyje gauname, kad tiks tik k = 1,
taigi kai n - lyginis gauname sprendin f(x) = x.
Kai n - nelyginis, tai ikart gauname, kad f(x + y) = f(x) + f(y) bet kokiems
realiesiems x, y. Be to, turjome, kad f(x
n
) = f(x)
n
, tada f(1) = f(1)
n
ir
f(1) = 1, 1 (0 netiks, nes f - injektyvi). Tada gauname du atvejus: f(p) = p
arba p p Z ir abiem atvejais galios f(px) = pf(x). Paymkime b
k
= f(x
k
),
k = 2, 3, ..., n ir q = f(x). I ankiau gauto rezultato galios f((x + p)
n
) =
(f(x + p))
n
= (f(x) + f(p))
n
. ia galime statyti bet kok sveik p ir tai yra
tiesin lygtis bet kurio b
k
atvilgiu. Tada keisdami vairias p reikmes galime
gauti n 1 neekvivaleni lygi su n 1 kintamj b
k
. Tada aiku, kad tokia
tiesini lygi sistema turs daugiausiai tik vien sprendin. Nesunku patikrinti,
kad pirmam atvejui tiks sprendinys b
k
= q
k
, o antram b
k
= q
k
, lyginiams k
ir b
k
= q
k
nelyginiams k. Tada pirmu atveju gausime f(x
2
) = f(x)
2
, o antru
f(x
2
) = f(x)
2
. I ia funkcija ir vl aprta ir gausime, kad kai n - nelyginis,
tiks tik tiesiniai sprendiniai f(x) = x ir f(x) = x.
7. stat duotojoje lygtyje x = 0, gausime f(0) ,= 0, nes kitaip f(y) = 0 visiems y,
bet f - nekonstanta. Taigi g(y) = 1
f(y)
f(0)
. stat pradinj lygtyje x = y = 1
ir panaudoj a), gausime f(1) = 0 ir tada galime paymti f(0) = k, kur
k - kakoks teigiamas skaiius. Tada stat g iraik pradin lygt gausime
f(xy) = f(x) +f(y) +
f(x)f(y)
k
, arba: k +f(xy) = (

k +
f(x)

k
)(

k +
f(y)

k
). Pakeit
h(x) =

k +
f(x)

k
, gausime

kh(xy) = h(x)h(y), o tada pakeit h(x) =

ki(x):
i(xy) = i(x)i(y). Monotonikumas niekur nedingo ir i lygt jau esam sprend,
tad nesunku gauti atsakym:
f(x) = k +k sgn(x) [x[
a
ir g(y) = sgn(y) [y[
a
,
kur sgn(x) - x enklo funkcija.
8. stat pradin lygt x = y = 0, gausime, kad f(0) = 0 (jei u(0) = 0, f -
konstanta). f - grietai monotonin, taigi 0 ji negys su jokia kita argumento
reikme. I pradins lygties u(y)f(x) + f(y) = f(x + y) = u(x)f(y) + f(x). ia
ksav y, gausime: u(x) =
u(y)1
f(y)
f(x) + 1 = Af(x) + 1. Jei u(z) = 1, visiems
realiesiems z, tai egzistuos f(x) = x, tenkinanti pradines slygas. Kitu atveju:
f(x +y) = Af(x)f(y) +f(x) +f(y). Tada pakeit h(x) = Af(x) + 1 gausime
h(x +y) = h(x)h(y).
Tai viena i Cauchy tipo lygi, kurias sutikome ankiau. Kadangi f monotonin,
h irgi monotonin ir h(x) = b
x
, kur b > 0. Tada f(x) = A
1
(b
x
1) ir u(y) = b
y
196
Sprendimai
bus sprendiniai. Visk apibendrinus, u(x) = b
x
, kur b > 0 (skaitant ir b = 1),
bus vienintels slygas tenkinanios funkcijos.
9. Pirmiausiai darykime keitin f(x) = g(x)[1 +x[. Pradin lygtis taps: g(
x+y
1+xy
) =
g(x)g(y). Pastebkime, kad reikinys
x+y
1+xy
primena tangent sumos formul, ta-
iau tangentas nepaprastas, o - hiperbolinis. Hiperbolinis tangentas - tai funkcija
tanh(x) =
e
2x
1
e
2x
+1
. Nesunku sitikinti, kad irgi galios panai tangent sumos for-
mul, t.y. tanh(x + y) =
tanh(x)+tanh(y)
1+tanh(x) tanh(y)
. Taigi keiiame lygtyje x = tanh(x),
y = tanh(y). Gausime g(tanh(x +y)) = g(tanh(x))g(tanh(y)). siveskime keitin
h(x) = g(tanh(x)). Gausime lygt h(x+y) = h(x)h(y). Galime nesunkiai sitikin-
ti, kad tolydumas niekur nedingo, tai viena i Cauchy tipo lygi, kurios spren-
diniai bus h(x) = a
x
, kur a 0. Tada a
x
= g(tanh(x)) = g(x) = a
arctanh(x)
,
f(x) = a
arctanh(x)
[1 +x[.
Kombinatorika
Matematiniai aidimai
Strategija
1. Vienu jimu galime sumainti tik vien i parametr (ilg arba plot). Nagrin-
dami paprastesnius atvejus pastebime, kad atvejais 0 0, 1 1 ir 2 2 laimi B.
Natralu galvoti, kad atveju n n visada laims B. A atlieka jim su kvadratu
ir B gauna ne kvadratin plytel i kurios visada gali padaryti kvadrat ir taip
isaugoti savo laiminij pozicij. Atveju m = n laimi B, kitais atvejais laimi A.
2. Purpurinsiui tereikia dti irg langel, kuris yra simetrikas aliasio uimtam
lentos horizantaliosios (arba vertikaliosios) aies atvilgiu.
3. Pirmu jimu A prideda 1 ir gauna n = 3. Dabar A visada gals paeiti taip, kad B
gaut nelygin skaii, o po io jimo A atitekt lyginis. B gals pridti nedaugiau
negu vien treij turimo skaiiaus, o A visada gals pridti bent pus. Taigi A
ramiai stebi prieininko agonij tol, kol gauna n 1328. Jis, priddamas pus io
skaiiaus, pasieks skaii nemaesn u 1990
4. Pirmasis turi laiminij strategij. Jis daugina 1 i 4. Tada antrasis gali
padaugins duoti skaii nuo 8 iki 36. Tada pirmasis daugina skaii i to-
kio skaiiaus, kad gautsi skaiius, didesnis arba lygus 56 (tai visada manoma
(8 7 = 56, 36 2 = 72). Antrasis skaii turi dauginti bent jau i 2, tad ma-
iausiai sudaro skii 112, o j pirmasis daugina i 9 ir gauna 1008. Pirmasis
laimjo.
5. Jokiais. Pirmasis aidjas visada gali pateikti antrajam nelygin skaii, o antrasis
j turs atsakyti lyginiu. Tad pirmajam tereikia visada k keisti k + 1. Taip
jis tikrai neparays skaiiaus didesnio u n, nes antrasis aidjas negali parayti
jokio nelyginio skaiiaus, tuo tarpu ir n.
197
Sprendimai
6. Antrasis. Kiekvienu jimu jis spalvina langel per du langelius aukiau arba
emiau pirmojo nuspalvintam. Nesunku pastebti, kad tai veda pergal.
7. Laimi Pirmasis (P). Suporuojame kortas poras (k, 1000+k), kur k = 1, , 1000
ir (2001, 2002). Vis por, iskyrus paskutinij, paskutinis skaitmuo abiejuose
skaiiuose lygus. Pirmu jimu P renkasi 2002. Tada atsakinja paimdamas kort
i tos poros, i kurios paima antrasis. Kakada A bus priverstas paimti 2001, jei
ant stalo dar yra kort, tai P ima bet kuri ir elgiasi taip pat, kaip prie tai.
aidimo pabaigoje P turi sum, kuri lygsta 2 moduliu 10, o A 1 moduliu 10.
8. Tebnie p ir l yra P ir L sugalvoti natriniai skaiiai. p[2002, kitaip P inot l.
Taip pat l[2002, kitaip L inot p. Be to (2002 l)[2002, kitaip L inot P. ia
jau nesunku sitikinti, kad 1001 yra vienintelis tinkamas 2002 daliklis tenkinantis
sry. Tad l = 1001
9. 1) Lent galima padalinti staiakampius 2 1. A tereikia pereiti gretim to
paio staiakampio langel. B tada turs pereiti kit staiakamp ir A visada
gals atlikti dar vien jim.
2) Lent galima padalinti staiakampius 2 1 netraukiant apatinio kairiojo
kampo. Tada analogikai aisdamas laimi B.
3) ia B jau bejgis. Lyginiams n strategija analogika (1). Kitu atveju lent
padaliname staiakampius 2 1, bet netraukiame apatinio kairiojo kampo.
Lent nuspalviname prastiniu bdu. Pastebime, kad apatinis kairys langelis B
yra nepasiekiamas, tad A laimi pajuddamas gretim staiakampio langel.
10. Suskirstome lent staiakampius 2 4. Pastebime, kad i bet kurio staia-
kampio langelio irgo jimu galime patekti tik vien to staiakampio langel. A
padeda irg vien i staiakampi, B tereikia paeiti langel esant tame pa-
iame staiakampyje. Kitu jimu A btinai turs pereiti kit staiakamp, taip
sudarydamas galimyb B judti to staiakampio viduje. aidim visada laims
B.
11. Jei nors vienoje i krveli yra nelyginis akmenuk skaiius, laimi A. Jam tereikia
pirmu jimu akmenuk skaiius paversti lyginiais abejose krvelse. Tada po B
jimo nors vienoje krvelje tikrai bus nelyginis akmenuk skaiius ir A gals tsti
savo spektakl. Kitu atveju analogikai aisdamas laims B.
12. 1) A tereikia atlauti kvadrat m1m1 ir tada lauti simetrikai striainei.
2) A tereikia visada lauti kampin langel.
13. A renkasi pusiaukratini susikirtimo tak, o B bria per j ties, lygiagrei
vienai kratini, ir gauna
5
9
pyrago. Brdamas kit ties per X jis gaut ma-
iau, o jei X nebt is takas, tai B tikrai galt gauti daugiau (rodykite tai
geometrikai).
14. A pirmu jimu rao 1 prie x. B rao a, o A atsako a. x
3
ax
2
1x +a = 0
turi aknis 1, 1 ir a. Tai sveikieji skaiiai.
198
Sprendimai
15. Taip, Arkliui pergals aki aidime tiktis neverta. Asiliukui pakanka padalinti
likusi lentos dal domino staiakampiukus ir mgdioti Dominyk eiti tam
paiam domino, kaip ir Arklio aplankytam, priklausant langel.
16. rodysime, kad visiems N > 1, antrasis aidjas laimi tada ir tik tada, jei N =
2
m
. Tokiu atveju pirmasis aidjas paima 2
a
(2b + 1) akmenuk, kur a 0 ir
b 0. Tada antrasis aidjas paima 2
a
, o kitais jimais kopijuoja pirmojo aidjo
veiksmus (sitikinkite, kad tai garantuoja pergal). Jei N = 2
a
(2b +1), kur a 0
ir b 1 tada laimi pirmasis aidjas pirmu jimu paimdamas 2
a
akmenuk ir
kitais jimais kopijuodamas antrojo aidjo veiksmus.
17. 1994 vektori suma yra a. Pirmasis aidjas aidia tokioje kordinai sistemoje,
kur x ais sutampa su a kryptimi. Jei a = 0, tada kryptis gali bti bet kokia.
Kiekvienu jimu aidjas renkasi vektori, kurio projekcija x a didiausia. Gal
gale pirmojo aidjo vektoriaus projekcija x a bus nemaesn u antrojo, o
abiej aidju vektori projekcijos y a bus lygios (j suma lygi nuliui) Taigi
pirmasis aidjas niekada nepralaims.
18. Taip gali. P(x) yra daugianaris aidimo pabaigoje. Prie paskutin B jim
turime daugianar F(x). aidjas B gali usitikrinti, kad A paskutiniu jimu keis
tritak prie nelyginio laipsnio. Tada P(x) = F(x) + ax
m
+ bx
2p+1
. P(2) =
F(2) + a(2)
m
b2
2p+1
, cP(1) = cF(1) + ca + cb. Jei c = 2
2p+1
, gauname
2
2p+1
P(1) + P(2) = 2
2p+1
F(1) + F(2) + 2
2p+1
a + a(2)
m
. Jei 2
2p+1
P(1) +
P(2) = 0, tai P(x) tikrai turs reali akn tarp 1 ir 2. 2
2p+1
F(1) +F(2) +
2
2p+1
a+a(2)
m
= 0, tada a =
cF(1)f(2)
c+(2)
m
. Paskutiniu jimu B tereikia parayti
a taip, kad A reikt rayti koecient prie nelyginio laipsnio. Tada P(x) turs
akn tarp 1 ir 2.
19. Pirmas sprendimas Imame dvi virutines eilutes ir sunumeruojame langelius i
kairs dein. Briame rodykl i apatinio treio langelio virutin pirm, i
apatinio 5 virutin 3 ir t.t. Imame dvi emensnes eilutes ir briame rodykles i
virutinio antro langelio apatin ketvirt, i virutinio ketvirto apatin 6 ir t.t.
Dar dvi emesnes eilutes paymime kaip pirmas dvi ir t.t. Matome, kad rodykl
atitinka horizontal irgo jim, o vertikaliu irgo jimu i rodykls smaigalio
visada atsiduriama rodykls pradioje. aidjui A tereikia irg pastatyti rodykls
pradioje ir paeiti smaigal. Tada B btinai paeis kitos rodykls pradi ir A
gals paeiti rodykls smaigal.
Antras sprendimas Susiymkime lentels langelius kaip kordinates (x, y), kur x, y
yra teigiami sveikieji. Tarkime, kad irgo pastatymas (1, 1) langelyje ir pajimas
langel (3, 2) stumia A pralaimini pozicij (kitu atveju rodymas jau yra
baigtas). Tada B savo jimu peina langel (X, Y ) taip, kad A vl atsidurt
pralaiminioje pozicijoje. Pastebime, kad jei A pirmu jimu pastato irg (2, 3),
tada jimas (Y, X) garantuoja A pergal. Dabartin situacija nuo pirmosios
skiriasi tik tuo, kad irgas nepabuvojo langelyje (1, 1). Taiau is langelis yra
nepasiekiamas B, tad tai nedaro takos baigiai.
20. Atveju N = 2 antrajam aidjui pakanka nuspalvinti tak simetrik raudona-
jam centro atvilgiu. Kad ir kok didel lank atsiriekt pirmasis aidjas antrojo
199
Sprendimai
jimo metu, antrasis visada gals atriekti didesn (tak ant pasirinkto apskritimo
lanko yra be galo daug). Nagrindami atvej N = 3 vl bandome spalvinti takus
simetrikai centro atvilgiu, bet pastebime, kad tai nieko gero neduoda. Galim
strategij skaiius nra jau toks didelis ir gudusi akis greit pastebs, kad atve-
ju N = 2 pasiteisino strategija spalvinti taisyklingojo dvikampio virnes. Tai
praktikai ir yra visas udavinio sprendimas.
Antrasis aidjas tol spalvina taisyklingojo N-kampio, kurio virn yra pirmasis
raudonas takas, virnes, kol gali. Jis nuspalvina a virni. N-kampis yra
suskirstytas bent N lank, vadinsime iuos lankus pagrindiniais. Yra nedaugiau
negu Na1 pagrindini lank, kuri abu galai yra raudoni ir pirmasis aidjas
gali visuose juose nuspalvinti po tak ir jam dar lieka vienas jimas. Jei jam
lieka daugiau jim, tai jis spalvina takus lankuose, kuri abu galai raudoni, kol
lieka vienintelis. Taip ilgiausias antrojo aidjo lankas bus tikrai trumpesnis u
pagrindin. Kai visos N-kampio virns nuspalvinamos, yra bent a + 1 lank,
kuri nors vienas galas yra mlynas; vadinsime iuos lankus melsvais. Pirmasis
aidjas jau atliko bent N a jim (nuspalvino N a taisyklingojo N-kampio
virni), tad jam liko ne daugiau a jim ir jis negali sudarkyti vis melsv
lank. Prie paskutin jim tikrai nra n vieno pagrindinio lanko, kurio abu
galai raudoni ir yra nors vienas melsvas lankas. Antrasis aidjas gali usitikrinti
lank mlynais galais, kurio ilgis kaip norima artimas pagrindinio lanko ilgiui.
is lankas bus tikrai ilgesnis u ilgiausi raudon lank. Antrasis aidjas turi
laiminij strategij.
21. Tegu a ir b yra A ir B skaiiai, o x < y teisjo skaiiai. Tarkime, kad aidimas
begalinis. A ino, kad y b 0 ir sako ne". Kitu ingsniu B suvokia, kad A
suprato, jog y b 0, taiau, jei a > x, tada A inot, kad a+b = y ir pasakyt
taip", taigi B supranta, kad x a 0 ir aidimas tsiasi.
Tarkime, kad n-tuoju ingsniu A ino, jog B suvok, kad s
n1
a r
n1
. Jei
b > x r
n1
, B inot, kad a +b > x, t.y. a +b = y. Jei b < y s
n1
, B inot,
kad a +b < y, t.y. a +b = x. Abiem atvejais B galt aptspti A, bet jis pasako
ne", taigi x r
n1
b y s
n1
. Dabar r
n
= y s
n1
ir s
n
= x r
n1
. Kitu
ingsniu B analogikai suvokia, kad r
n+1
= y s
n
ir s
n+1
= x r
n
. Pastebime,
jog abiem atvejais s
i+1
r
i+1
= s
i
r
i
(y x). Kadangi y x > 0, tai egzistuoja
m, kuriam galioja s
m
r
m
< 0. Prietara.
22. S
n
vadinsime aidim, kuriame duotas skaiius yra n. S
2l+1
laims pirmasis
aidjas visada keisdamas k k + 1, tad nagrinsime tik atvej, kai n yra lyginis.
Pirmasis mogus paras lygin skaii didesn u
n
2
laims, nes niekas nebenors
dauginti i dviej. Jei n yra dalus i keturi, tai pralaimi pirmasis mogus paras
didesn skaii u
n
4
, nes tas skaiius bus maesnis u
n
2
, o prieininkas gals j
padauginti i dviej ir taip garantuoti sau pergal. Vadinasi aidim S
4l
laims
tas pats aidjas kaip ir S
l
. Analogikai samprotaudami atveju, kai n nra dalus
i keturi, gauname, kad ir aidim S
4l+2
ir S
l
laimtojai sutaps.
Taigi, turimam lyginiam n nordami isiaikinti, kuris aidjas laims, mes at-
liekame algoritm jei skaiius dalus i keturi, tai padaliname, o jei ne, tai
daliname i dviej, atimame vienet ir dar kartel padaliname i dviej. Jei taip
200
Sprendimai
tsdami gausime nelygin skaii, tai antrasis aidjas pralaims. O jei ne, tai al-
goritmas sustos ties skaiiumi 2, o tai reikia, kad aidim laimi antrasis aidjas.
Kaip tai urayti tvarkingai? Jei jau dauginame skaii i keturi tai ketvirtainje
sistemoje prie jo uodegos priraome 0, jei dauginame i 4 ir pridedame 2, tai prie
uodegos priduriame 2. Startuojame su 2, vadinasi antrasis aidjas laims tuos ir
tik tuos aidimus, kuriuose n ketvirtainje iraikoje bus ireikiamas vien tik 2 ir
0.
23. Tai yra plaiai inomas ir magikas Wythofo aidimas (angl. Wythos game).
Jis gana isamiai nagrinjamas A. Engel knygoje "Problem solving strategies".
Tikrai nesunku rasti pirmsias pralaiminias pozicijas:
0. (0,0) 4. (6,10) 8. (12,20)
1. (1,2) 5. (8,13) 9. (14,23)
2. (3,5) 6. (9,15) 10. (16,26)
3. (4,7) 7. (11,18) 11. (17,28)
Pairkime lentel atidiau kiekvienas skaiius (kogero!) pasirodo tiksliai po
vien kart ir skaii poros skirtumas n-tojoje pozicijoje (kogero!) lygus n. Po
i pastebjim udavinys jau beveik isprstas. Keliame hipotez, kad pirmasis
pralaiminios poros skaiius yra maiausias dar niekada nepasirods lentelje x
n
,
o antrasis lygus x
n
+n. rodysime, kad taip gauname visas pralaimiias pozicijas
beigi tik pralaiminias. Pralaiminias pozicijas mes vertinsime pagal maiausij
poros element. Taip vertinant (0, 0) < (1, 2) < (3, 5) ir t.t. Nesunku suprasti,
kad skaiius x gali bti tik vienos poros maiausias elementas (Nesunku?).
Pirmoji pora (0, 0) yra tikrai pralaiminti ir nra maensi pralaimini pozicij.
Tarkime, kad visos poros iki (x
n
, x
n
+n) yra pralaiminios ir tarp j nra maesni
pralaimini pozicij, kurios nra trauktos pagal i taisykl. rodysim, kad ir
(x
n+1
, x
n+1
+n+1) yra pralaiminti, bei tarp jos ir (x
n
, x
n
+n) nra pralaimini
pozicij.
1) Jei tarp j atsirast pralaiminti pozicija (a, b), tai jos maiausias elementas
x
n
< a < x
n+1
, taiau pagal x
k
apibrim, jau yra pralaiminti pora (c, a)<(x
n
, x
n
+
n), kuri turi didesnj element a. b > a > c, tad i poros (a, b) vienu jimu galime
gauti por (c, a), tad (a, b) nra pralaiminti. Prietara.
2) Tarkime, kad pora (x
n+1
, x
n+1
+ n + 1) yra laiminti. I jos vieno jimo metu
turime galti pasiekti pralaimiij pozicij. Tikrai nra pralaiminios pozicijos
su nari skirtumu n+1, tad mums teks nuimti akmenukus i kurios nors vienos
krvels. Pagal apibrim skaiiaus x
n+1
tikrai nra n vienoje pralaiminio-
je poroje, gal ten yra skaiius x
n+1
+ n + 1? x
n+1
yra didesnis u visus kitus
maiausius pralaimini por elementus, o n+1 daugiau u visus pasitaikanius
skirtumus tarp poros element, tad x
n+1
+n+1 yra aikiai didesnis u visus skai-
ius pasitaikanius pralaiminiose pozicijose. Tad nei num po lyg akmenuk
skaii i abiej krveli, nei paaid su viena krvele mes niekaip nepasieksime
pralaiminios pozicijos. Prietara
24. Kai k = 1, aidjui A tereikia nuspalvinti tris takus esanius vienoje tiesj ly-
giagreioje aims taip, kad vienas gult lygiai per vidur tarp kit dviej, nutols
201
Sprendimai
nuo j atstumu X ir, trys takai, nutol nuo pirmj trij atstumu X vertikaliai
vir arb apai, bt laisvi. Pabandius nesunku sitikinti, kad tai manoma
ir tai pasiekus A lengvai gali laimti.
Bandydami atvej k = 2 pastebime, kad ploktumos begalinumas sprendiant
udavin yra kertinis faktorius. Kuo daugiau A nuspalvina tak, tuo daugiau ga-
lim kvadrat turi ublokuoti B. ia ir atsiranda nuojauta, kad pirmasis aidjas
gali laimti su bet kokiu k.
rodindami udavin pasinaudosime keletu paprast fakt:
(1) A gali nuspalvinti kaip nori daug tak vienoje tiesje, nes tak skaiius
begalinis.
(2) A visada suras tui ties lygiagrei aims, kurioje nra nuspalvintas dar n
vienas takas, nes tiesi skaiius begalinis.
Pirmasis aidjas nuspalvina Z tak x ayje ir bria per kiekvien tak ties
lygiagrei aiai y (vadinsime ias tieses statiniais). Tada susiranda tui ties
lygiagrei x aiai ir spalvina ios tiess sankirtas su statinmis. A naujojoje
tiesje nuspalvins ne maiau negu
N
Z+1
sankirt. Kitu ingsniu A nutrina visus
statinius, kuri sankirt ioje tiesje nenuspalvino. A tsia aidim isirinkdamas
tui ties, nuspalvindamas sankirtas ir nutrindamas nepanaudotus statinius.
Pastebime, kad statiniai su pasirinktomis tiesmis sudaro staiakamp gardel,
kurios visos sankirtos nuspalvintos raudonai. Pasirinkdamas pakankamai didel
Z, A gali gauti toki gardel a b, kokios tik usigeidia.
Nuspalvins pakankamai didel gardel (pakankamumo slygos radim paliksime
skaitytojui), aidjas A spalvina x ayje tak Q ir bria per j ties d sudarani
45

kamp su x aimi taip, kad visi nuspalvintieji takai gult kairiau ios tiess.
Prasitsiame a gardels horizontali tiesi ir spalviname i tiesi sankirtas su
d. A gals nuspalvinti bent
a
k+1
sankirt (1). Po i
a
k+1
jim liks bent b
a nenuspalvint sankirt tarp b pratst gardels vertikali ir tiess d, A gali
nuspalvinti bent jau
ba
k+1
i sankirt (2).
Dabar nagrinsime r =
a
k+1
horizontali tiesi, kurios kerta d raudonuose takuose
(1) ir s =
ba
k+1
vertikali tiesi, kurios kerta d raudonuose takuose (2). Paste-
bime, kad bet kuriems 2 raudoniems takams i (1) ir (2) gardelje atsiras jau
nuspalvintas raudonai takas, kuris su jais sudaro tris kvadrato, kurio kratins
lygiagreios aims, virnes. A gali pasirinkti r s skirting kvadrat, kuri tris
virnes jau yra nuspalvins. Jam lieka nuspalvinti vien i r s tak deiniau
linijos d ir taip laimti aidim. Parodysime, kad jis visada gals tai padaryti.
Nuo d linijos nubrimo B nuspalvino nedaugiau nei b tak i nagrinjam r s.
Taigi A tereikia pasirinkti tokius a ir b, kad a b bei b bt pakankamai dideli,
r s r s > b. (r, s ireikkiamii per a, b, k ir nesunku apskaiiuoti kiek b turi
bti didesnis u a). Kadangi aidimo pradioje A gali spalvinti tiek tak, kiek tik
irdis geidia, tad tikrai gals pasirinkti pakankamus a ir b. Patariame skaitytojui
paiam isiaikinti, kokie gi a ir b yra pakankami.
Udavinys gracingai neigia nusistovjusias normas. Vienas begaliniame lauke
puikiausiais karys.
202
Sprendimai
aidimas NIM
1. Matekaralius tikrai perskait vis skyrel, sisavino mediag ir nori okolado.
Jei Matekaralius nepersitempt ir nupiet staiakampius, kuri viena kratini
yra vienetin, tai gautume NIM aidim su N krveli, kuri dydius pasirinko
Matekaralius. Jis bus antrasis aidjas, tad turi pasirinkti tokias krveles, kuri
NIM suma bt lygi 0. Ar jis gali taip padaryti? Jei N lyginis, tai jam tereikia
nupieti daug lygi krveli (j dydiai priklauso nuo okolado poreikio), jei N
nelyginis, tai jis yra nemaesnis u 3, o N(1, 2, 3) = 0, tad Matekaralius visada
gals nupieti tokius staiakampius ir lygin skaii lygi krveli. Merlinkas
neturi laiminiosios strategijos.
2. aidimo pabaigos pozicija yra 1 akmenukas. Akmenuk skaiiams 1, 2, 3, 4, 5,
6, 7 priskiriame NIM vertes lygias 0, 1, 0, 2, 1, 3, 0. Priskyrus vertes didesniems
skaiiams nesunku pastebti ir rodyti, kad nulines vertes turs skaiiai, kuri
forma 2
k
1, kur k sveikasis neneigiamas. Taigi jei n = 2
k
1, tada laimi B,
kitais atvejais pergal venia A.
3. Tai yra Kryiukai-nuliukai. Pamginkite susikonstruoti magikj kvadrat arba
panagrinti visas galimas trij skaii sumas lygias 15.
4. rodysime taikydami indukcij. Skyrelio metu rodme, kad tai teisinga su n = 2
Tarkime tai teisinga su n, rodysime, kad tai teisinga su n + 1. Paimame bet
kuriuos du aidimus ir bet kurias dvi pozicijas. Tarkime, kad j NIM verts
atitinkamai lygios a ir b. Tada j suminio aidimo pozicijos NIM vert lygi a b
ir sumin aidim galim traktuoti kaip vien aidim su atitinkama NIM verte.
Tad dabar turime n aidim, o tarme, kad jiems galioja slyga. rodyta.
5. Tarkime, kad visiems ilgiams maesniems u n jau priskyrme NIM vertes. Paste-
bime, kad jei juost kur nors nuspalviname tai taip j padaliname du regionus.
Jei juosta yra ilgio n, tai galime j padalinti [n/2] skirting bd. Padalin
gauname dviej aidim sum, kuri abiej NIM vertes jau inome, tad galime
apskaiiuoti ir suminio aidimo NIM vert. Inagrinj visus galimus padalinimus
rasime visas NIM vertes, kurias galime pasiekti vieno jimo metu, tad inosime ir
n ilgio juostos NIM vert. Suprantant algoritm visai nesudtinga isiaikinti,
kurios pozicijos laiminios, o kurios pralaiminios.
6. aidimas identikas NIM aidimui su trimis krvelmis, kuri dydiai yra a, b
ir c (jei tariame, kad kampinio langelio kordinats lygios (1,1,1) ir a, b ir c, jei
kampinio langelio kordinats lygios (0, 0, 0). Tad sryis, kur turi tenkinti a, b ir
c yra a b c ,= 0, arba a 1 b 1 c 1 ,= 0.
7. Jei turime vien krvel ir joje yra vienas akmenukas, tai pirmasis aidjas tu-
rs j nuimti ir pralaims. Jei akmenuk daugiau, tai jis gals laimti aidim
nuimdamas visus iskyrus vien akmenuk.
aidiame su dviem krvelmis. (0, 0) yra laiminti pozicija, (0, 1) pralaiminti,
(0, n + 1) - laiminti, (1, 1) - laiminti, (2, 2) - pralaiminti. Utenka perrinkti dar
kelet variant ir pastebime, kad visos pozicijos (n, n), kur n daugiau u 1, bus
pralaiminios. I ties, tarkime aidimas prasideda tokioje pozicijoje. Tada antrasis
203
Sprendimai
aidjas gali atlikinti simetrikus aidimus tol, kol pirmasis vienoje i krveli
padarys 0 arba 1 akmenuk. Abiem atvejais antrasis gals ant stalo palikti tik
vien akmenuk ir taip laimti aidim.
Bendru atveju aidiama labai panaiai. Matome, kad ia svarbiausios yra viene-
tins krvels (turinios vien akmenuk). Jei ant stalo yra ne vienetini krveli,
tai aidimas aidiamas lygiai taip pat kaip normalus NIM iki to momento, kai
laiminiojo aidjo jimas gali ant stalo palikti tik nelygin skaii vienetini kr-
veli (joki kit), tada jis atlieka jim ir laimi. Akylesnis i js paklaus,
kodl "pralaimintysis" aidjas negals pirmas atlikti tokio jimo. Bet atsiminki-
me, kokioj baloj jis tupi, ogi tokioj, kurios NIM vert visada yra lygi 0. Jei jis
gauna nelygin vienetini krveli skaii sumaindamas nevienetin krvel, tai
jis per klaid buvo atsidrs nenulinje pozicijoje ir visgi yra laimintysis aidjas.
Prietara. Jei jis tai atlieka nuimdamas vienetin krvel, tai laimintysis aidjas
pats galjo nuimti t krvel arba palikti viena daugiau ir taip pateikti nelygin
vienetini krveli skaii.
Geometrija
Udaviniai apilimui
1. Keturkampio, sudaryto i keturi kito keturkampio kratini vidurio tak, prie-
ingos kratins lygiagreios viena kitai i trikampio vidurio linijos savybs.
2. Tarkime, kad E yra tarp D ir A. Tada trikampyje BAD auktin sutampa su pu-
siaukampine, taigi ED = AE. I ia ED =
DC
2
. Tada i pusiaukampins savybs
trikampiui BEC,
BE
BC
=
ED
DC
=
1
2
, taigi staiajame trikampyje BEC ambin dvi-
gubai ilgesn u statin. Todl BCE = 30

, CBE = 60

. Nesunkiai randame,
kad ABC = 90

, BAC = 60

.
3. Tegu BAC = 2x = ADC = BEA. Suskaiiuojame, kad
CBA = 2ABE = 2(180

2x 2x) = 360

8x.
I priekampio savybs
2x = CDA = DAB +DBA = 360

7x.
Taigi x = 40

, ir i ia trikampio kampai yra A = 80

, B = 40

, C = 60

.
4. Paymkime ACB = 2x. Kadangi AC = DC, tai ADC = 90

x. Kadangi
AC = AB, tai ABD = ACB = 2x. Kadangi AD = BD, tai ADB =
180 4x. Tada 180

= ADC + ADB = 270

5x. I ia x = 18

, ir todl
A = 108

, B = C = 36

.
5. Pagal staiojo trikampio pusiaukratins savyb ED =
CB
2
= DF, tad trikampiai
EDC ir BDF lygiaoniai. Tuomet CDE = 180

2C ir BDF = 180

2B.
Kadangi EDF = 60

, tai 180

2C + 180

2B = 120

ir i ia A =
180

B C = 60

.
204
Sprendimai
6. Tegu MO kerta CD take N. rodysime, kad trikampiai MBC ir MND yra
lygs. I ties MB = ND (nes ios atkarpos simetrikos O atvilgiu), kratin
MN bendra abiems, o DNM = 180

AMO = 180

MAD = MBC.
7. I Pitagoro teoremos AD
2
= AO
2
+OD
2
= BC
2
+OD
2
= BO
2
+OC
2
+OD
2
=
CF
2
+CD
2
= DF
2
, taigi AD = DF.
8. Pastebkime, kad ABM yra lygiaonis. Bet A = 60

, todl ABM lygiakratis.


Panaiai ir ACN lygiakratis. Todl CMBN yra lygiaon trapecija ir todl
MN = BC.
9. Tegu kampo A pusiaukampin ir kratins AB vidurio statmuo kertasi take E,
o BH ir CF yra auktins. Tada AEB yra lygiaonis. Taigi ABE = BAE =
EAC, ir i ia A = 60

. Tegu kampo A pusiaukampin ir CF kertasi take K.


Tada KAC = KCA = 30

, taigi AC vidurio statmuo eina per tak K.


10. Tegu AC

= b, CB

= a, BA

= c. Sudj trikampi AB

, A

BC

,
A

C kampus, mes gauname 540

= 180

+ 180

+ 180

= (A + b + c) + (c +
a + B) + (C + a + b) = 2(a + b + c) + 180

, todl a + b + c = 180

. Taigi
A = a, B = b, C = c. Todl AB | A

, BC | B

, AC | A

. Tada i
Talio teoremos
CA

B
=
AC

B
=
B

A
B

C
=
A

B
CA

, taigi A

yra BC vidurio takas. Panaiai


su B

ir C

.
11. Tegu pradinio trikampio kampai bna lygs 2a, 2b, 2c, a b c. Pastebkime,
kad visi 3 gautieji trikampiai yra bukieji (rodykite tai!), o 2 i j turi kamp, lyg
c. Taiau pradinio trikampio visi kampai didesni u c, taigi trikampis su kampais
a, b, 180

a b yra panaus pradin. Todl 180

a b = 2a, a = 2b, b = 2c.


Isprend gauname, kad trikampio kampai yra lygs
180

7
,
360

7
,
720

7
.
12. Ne, negali. AA
1
, BB
1
, CC
1
vidurio takai yra ant trikampio ABC vidurio lini-
j, lygiagrei atitinkamai kratinms BC, AC, AB. ios vidurio linijos sudaro
trikamp, o bet kuri linija kerta trikamp daugiausia dviejuose takuose.
13. Tegu kvadratas bna ABCD, o kratini vidurio takai E, F, G, H, bei trikampi
ABM, BCM, CDM, DAM pusiaukratini susikirtimo takai P, O, N, L - taip,
kaip parodyta paveiksllyje emiau. Tada EFGH yra taip pat kvadratas. I
pusiaukratini sankirtos tako savybs,
MP
MH
=
MO
MG
=
2
3
, todl i Talio teoremos
arba panaij trikampi, PO | HG ir
PO
HG
=
2
3
. Panaiai su kitomis kratinmis.
Todl PONL turi gretimas kratines statmenas, o taip pat visos kratins lygios.
Taigi PONL yra kvadratas.
205
Sprendimai
A
B C
D E
H
G
F
L
N
O
P
M
14. Bet kokiame staiajame trikampyje statinis trumpesnis u ambin. Jei ms
trikampis yra ABC, su auktinmis AA

ir BB

, bei AA

BC ir BB

AC,
tai tada AA

BC BB

AC AA

, su lygybmis tada ir tik tada jei ABC


lygiaonis statusis. Todl kampai yra lygs 90

, 45

, 45

.
15. Tegu P yra atkarpos BN vidurio takas. Tada MP | AN | NC, taigi MPCN
yra trapecija. Todl trikampiai PKM ir CKN yra panas. Tada 6 =
CK
KM
=
NC
PM
, taigi
AC
AB
=
AN+NC
AB
=
AN
AB
+
NC
AB
=
1
2
+
NC
MP
MP
AN
AN
AB
=
1
2
+ 6
1
2

1
2
= 2. Todl
trikampio ABC kampai yra lygs 30

, 60

, 90

.
16. Tegu AB ilgis bna 6x, o M kratins AB vidurio takas. Tada BM = 3x,
DB = 2x, taigi
MB
DB
=
3
2
=
CB
EB
. Todl trikampiai CMB ir EDB yra panas ir
dl to BCM = BED =
ACB
2
, todl CM yra trikampio ABC pusiaukampin
ir pusiaukratin tuo pat metu, taigi ABC lygiaonis(AC = CB).
Panaieji trikampiai ir brinio papildymai
1. Tegu E yra lygiagretainio striaini sankirtos takas, o F yra ED vidurio takas.
Tada BDK = BDA = DBK, tad trikampis BDK lygiaonis ir todl KE
BD. Be to, CD =
CA
2
= CE, tagi DCE taip pat lygiaonis, todl CF BD. I
Talio teoremos,
BK
KC
=
BE
EF
= 2.
2. Paymkime ant AC tak E taip, kad AE = AD ir CE = CB. Tegu trapecijos
striains kertasi take F. Tada ADF ir CBF yra panas trikampiai, todl
AF
CF
=
AD
BC
=
AE
CE
. Taigi takai E ir F sutampa, ir todl 60

= AFD = AED.
Dl to AED yra lygiaonis su kampu prie pagrindo lygiu 60

, todl yra lygiakra-


tis. Panaiai ir CEB lygiakratis. Tada i simetrijos trapecija yra lygiaon.
3. Paimkime tak D ant AC taip, kad KD | BC. Tada DKBC ir DKLC yra tra-
pecijos, o AKD lygiaonis. I trapecijos vidurio linijos formuls MN =
KD+LC
2
=
KA+LC
2
=
KL
2
= MK = ML. Taigi LNK = 90

.
206
Sprendimai
4. Tegu CK ir AD kertasi take E, KD ir BC take F. Tegu linija, lygiagreti
KC ir einanti per tak A kerta CD take P, o linija, lygiagreti DK ir einanti
per tak B kerta CD take Q. Reikia rodyti, kad P = Q. Tai visai nesunku:
DP
PC
=
DA
AE
=
FB
BC
=
DQ
QC
, taigi P = Q. (
DA
AE
=
FB
BC
, nes
DA
FB
=
AK
BK
=
EA
BC
).
5. Tegu AM kerta ties CB take N. Tada trikampiai ADM ir CMN yra vienodi,
todl BC = AD = CN. Tada HC yra staiojo trikampio BHN pusiaukratin
i staiojo kampo, ir todl HC = CB.
6. Paimkime tak K ant BM tok, kad BK = CD ir KM = DM. Tada DMC
ir AMK vienodi pagal dvi kratines ir kamp, taigi KA = DC = BK. Todl
BKA lygiaonis, ir dl to BAC = KAM + KAB = DCM + KBA =
MBA+BCA.
7. Tegu M yra BC vidurio takas. Tada BKL ir BKM yra vienodi pagal dvi
kratines ir kamp. Taigi, BLA = 180

BLK = 180

BMK = CMK.
Bet CM = BL ir AL = KM, taigi KMC ir BLA yra vienodi. Taigi KC = BA.
8. Trikampiai KMC ir KOA yra lygiaoniai. Todl CMA = 180

MOC
MCO = 180

AKOMKC = MKB. Taigi trikampiai MCA ir MKB


yra vienodi pagal 2 kampus ir kratin, todl AM = BK.
9. Tegu AB ir CD keratsi take E. I trikampio priekampio savybs, ABD =
ADC +CAD = ACE. Tada trikampiai ACE ir ADB yra vienodi pagal du
kampus ir kratin, taigi AE = AD. Bet EAD = DEA, taigi AD = DE.
Todl ADE yra lygiaonis ir BAD = 60

.
10. Trikampiai ADB ir DFC yra vienodi pagal 2 kratines ir kamp, nes DFC =
180

BFD = 180

BDF = BDA, taigi DAE = CDF = EDA,


todl ADE yra lygiaonis.
11. Tegu keturkampio striains kertasi take E. Tada trikampiai ADE ir ADC yra
panas pagal du kampus. Taigi ADC = AED. Bet trikampiai CEB ir CAB
taip pat panas pagal du kampus, taigi AED = CEB = ABC.
12. Tegu tiess AB ir KN kertasi take E, o AC ir NM take F. Akivaizdiai
NF = FM, LE | NM, todl i trikampi KNM ir KEL panaumo LA = AE.
Tada AN yra LE vidurio statmuo ir taigi KNA = LNA.
13. Tegu E yra B
1
C
1
vidurio takas. I panaij trikampi, AA
1
eina per E. Be to,
KE = KB
1
+B
1
E =
BC
4
+
BC
4
=
BC
2
= CA
1
, taigi CKEA
1
yra lygiagretainis ir
tada CK = A
1
E = BA
1
. Bet AA
1
B = KCB, taigi trikampiai KCB ir AA
1
B
yra vienodi pagal 2 kratines ir kamp. Tad AB = BK.
14. Tegu ties, lygiagreti DE eina per virn A ir kerta kratin BC take M. Tada
ED yra trikampio AMC vidurio linija bei AMC = DEC = AEB, todl
AME yra lygiaonis. Taigi
AE
DE
=
AM
DE
= 2.
15. Tereikia rodyti, kad i atkarp, kuri ilgiai yra GH, HD, BG galima suformuoti
statuj trikamp. Tam imame tak X ant spindulio EB u B taip, kad XB =
207
Sprendimai
FD. Tada trikampiai AFD ir ABX yra vienodi. Imame tak P ant AX taip,
kad
XP
PA
=
FH
HA
. Tada ADH = ABP, PB = DH, PA = HA, GAH = 45

=
PAG, GBP = 45

+45

= 90

. Trikampiai GAH ir GAP yra vienodi pagal du


kampus ir dvi kratines, taigi BGP yra ms iekomas statusis trikampis.
16. Tegu kampo B pusiaukampin kerta AF take P, o ties, lygiagreti BC ir einanti
per tak H, kerta AF take Q. Reikia rodyti, kad P = Q. Pastebkime, kad
CHB ir AFB yra panas. Tada i pusiaukampins savybs ir Talio teoremos
FP
PA
=
FB
BA
=
HB
CB
=
HB
AH
=
FQ
QA
. Taigi P = Q.
17. I pusiaukampins savybs
C
1
A
C
1
C
=
C
1
A
BC
1
BC
1
C
1
C
=
AC
BC
BA
AC
=
BA
BC
=
AB
1
B
1
C
, taigi B
1
C
1

yra kampo AC
1
C pusiaukampin.
18. Tegu ties per tak A, lygiagreti BC, kerta CD take E. Tada ABCE yra
lygiaon trapecija, taigi CE = AB =
CD
2
= ED. Be to, AED = BCD =
ABC, taigi trikampiai ABX ir AED yra vienodi pagal kratin ir du kampus.
Taigi AD = AX.
19. Tegu ties, lygiagreti AB ir einanti per tak Z, kerta AC take V , o N tebnie
XZ vidurio takas. Tada ZV C yra lygiakratis, o XZV A trapecija su vidurio
linija NY . Be to, NY taip pat yra staiojo trikampio XZY pusiaukratin i
staiojo kampo. Taigi, AX +ZC = AX +ZV = 2NY = XZ.
20. Tegu takas F yra simetrikas takui E tako A atvilgiu. Tada i trikampio
priekampio savybs, CAD = AEB + ABE = BAF bei EA = AE = AD.
Tada trikampiai BAF ir CAD yra vienodi pagal dvi kratines ir kamp, taigi
CD = BF. Bet i vidurio linijos savybs BF yra dvigubai ilgesn u trikampio
ABE pusiaukratin AM.
21. Pastebkime, kad PA = PC (nes P guli ant kampo B pusiaukampins), taigi
APC yra lygiaonis. Bet BCA = CAP, taigi ABC ir ACP yra vienodi, ir
todl BCPA yra rombas. Jeigu M yra BA vidurio takas, tai tada i simetrijos
QP = MP. Bet MP =
BD
2
i trikampio vidurio linijos savybs.
22. Tegu keturkampio striains kertasi take E. Tada trikampiai ECB ir CBA
yra panas pagal tris kampus, taigi
EC
CB
=
CB
AC
. Panaiai
AE
AD
=
AD
AC
. I i
lygybi CB
2
= AC EC ir AD
2
= AE AC. Sudj abi lygybes, gauname
CB
2
+AD
2
= AC (EC +AE) = AC
2
, ko ir reikjo.
23. I pusiaukampins savybs
AC
AK
=
AC
CL
=
AB
BL
=
AB
AL
. Bet CAK = LAB, taigi
trikampiai ACK ir LAB yra panas. Kadangi BAL lygiaonis, tai ACK taip
pat lygiaonis, todl AK = CK.
24. Tegu CE kerta ties AD take F. Tada trikampiai EAF ir EBC yra panas,
ir todl
AF
BC
=
AE
BE
=
AD
BC
, taigi AF = AD. Bet trikampis DFH yra status, o AH
yra pusiaukratin i staiojo kampo, todl AH = AD.
25. Trikampiai ADB ir ADC yra panas. I Talio teoremos
AF
DE
=
AC
CD
=
AB
AD
=
AB
DE
,
taigi AF = AB. Gauname, kad BAF yra statusis lygiaonis, todl ABF = 90

.
208
Sprendimai
26. Trikampiai XY B ir BAC yra panas pagal du kampus. Taigi,
AB
1
B
1
C
=
AB
BC
=
Y B
BX
=
AX
BX
, todl trikampiai AB
1
X ir ACB yra panas, taigi B
1
X | BC.
Apskritimai
1. Kadangi BAC+KLC = BLK+KLC = 180

, tai keturkampis AKLC yra


brtinis. Panaiai ir AMLB yra brtinis. Tada i brtini kamp savybs,
AMP + AKP = AMB + AKC = ALB + ALC = 180

, taigi AKPM
yra brtinis.
2. Tegu H bna trikampio auktini susiskirtimo takas. Pastebkime, kad tri-
kampis MBC

yra lygiaonis, o keturkampiai CBC

, ABA

ir AB

HC

yra
brtiniai (nes AA

B = AB

B = CC

B = CB

B). Taigi, XC

Y =
MC

B = MBC

= CB

= XB

Y , todl XB

Y yra brtinis. Tada


XY C = 180

XC

= BCA, todl XY | BC.


3. Tegu G yra CP ir AB sankirtos takas. Kadangi AMC yra lygiaonis, tai PM
yra ne tik AMC auktin, bet ir pusiaukampin. Tada APCM yra brtinis
deltoidas, taigi PAM = PCM = 90

. Be to, P yra GC vidurio takas, nes P


yra stataus trikampio ACG ambins ir statinio AC vidurio statmens susikirtimo
takas. Taigi PB dalija GC pusiau. Bet trikampiai BGC ir BAH yra panas,
taigi BP taip pat dalina AH pusiau.
4. Tegu H bna kvadrato ABDE centras, o CF kerta AB take K. Tada AHB =
ACB = 90

, taigi CBHA yra brtinis. Be to, HA = HB, todl ACH =


HCB, t.y kampo C pusiaukampin eina per tak H. Tada i simetrijos BK =
EF ir todl
EF
FD
=
BK
KA
=
BC
CA
= 3.
5. Tegu apskritimai lieia kampo kratines takuose D ir I, o patys lieiasi take G.
Tegu bendra vidin abiej apskritim liestin kerta kampo kratin take H. Tada
DHGA ir HIBG yra deltoidai, ir be to, DHA =
DHG
2
=
180

IHG
2
= 90

BHG = HBG bei AHB = AHG+GHB =


DHG
2
+
GHI
2
=
180

2
= 90

.
Taigi apie AHB apibrtas apskritimas turi skersmen AB (ABH status) bei
lieia CH (i vienos mintj savybi).
A B
C
D
I
G
H
6. Tegu X yra BC vidurio takas. Tada i simetrijos ir lygi brtini kamp
KC = KM = KX. Tegu O yra apie ABC apibrto apskritimo centras, o Q
209
Sprendimai
yra XC vidurio takas. Tada OX | KQ (abi linijos statmenos BC), ir i Talio
teoremos
BK
BO
=
BQ
BX
=
3
2
.
7. Tegu apskritimo e centras yra takas O. Tada i kampo tarp stygos ir liestins
savybs, OBA = OAB = OBC, taigi A ir C yra simetriki OB atvilgiu,
taigi BA = BC.
8. Vl i kampo tarp stygos ir liestins savybs, NAB = AMB bei MAB =
ANB. Tada i paveiksllio emiau matyti, kad MQA = MQB + BQA =
APB + BPN = APN bei QAM = NAP. Todl ANP ir QMA yra
panas, taigi AQM = ABP = ANP = QMA, taigi QA = MA. Panaiai
AN = AP. Tada trikampiai AQN ir AMP yra vienodi pagal kratin ir 2
kampus, todl MP = NQ.
A
B
M
N
P
Q
9. Tegu tas statusis trikampis bna ACH su staiu kampu A. brto apskritimo
centras tebnie D. Visi lik takai paymti paveiksllyje. Reikia rasti kamp
KAL. Pastebkime, kad AFDE, DPKG, DJLG yra visi vienodi kvadratai.
Todl DK = DL = DA, t.y D yra apie AKL apibrto apskritimo centras.
Taiau KDL = 90

, taigi KAL =
KDL
2
= 45

.
A
H C
E
F
J P
D
K G L
10. Pritaik vien i mintj savybi, gauname, kad keturkampiai CAPK ir QABK
yra arba brtiniai, arba deltoidai. Taiau deltoidais n vienas j bti negali,
nes kitaip kakurios dvi trikampio kratins bus lygiagreios. Todl jie abu yra
brtiniai, ir todl PAQ = PAK + KAQ = PBK + QCK =
BCA
2
+
ABC
2
= 90

BAC
2
.
11. Pasinaudoj kampo tarp stygos ir liestins savybe bei priekampio savybe, mes
gauname, kad EAD = EBD = BDC + BCD = BAD + BAC =
DAC, ko ir reikjo.
210
Sprendimai
12. Tegu H ir G yra pagrindai statmen, nuleist i O atitinkamai BC ir AD.
Tada GOA =
DOA
2
= DBA = 90

BAC = 90

BOH = OBH. Taigi


trikampiai BOH ir AOG vienodi pagal kratin ir tris kampus. Tad OG =
BC
2
.
13. Kadangi AO = DO, tai OCD = OAD = ADO = OCB. Taigi takai D ir
B yra simetriki OC atvilgiu, ko ir reikjo.
14. I kampo tarp stygos ir liestins mes turime ACD = CAB = OBC, ir pritai-
k vien i mint nauding fakt turime, kad apie BCO apibrtas apskritimas
lieia CD.
15. AKB = CKL = 180

ANL = ABL. Tada i aukiau mint nauding


fakt, AB
2
= AK AL. Panaiai ir AC
2
= AM AN. Bet i t pai fakt,
AM AN = AK AL. Taigi AC
2
= AB
2
, ko ir reikjo.
16. Tegu M yra BB

vidurio takas. Trikampiai OAB ir OA

yra vienodi, to-


dl OB = OB

, ir todl OMB

= 90

= OAB = OA

. Gavome, kad
keturkampiai MOAB ir MOA

yra brtiniai, ir i ia AMO = ABO =


A

O = A

MO. Todl takai M, A, A

yra vienoje tiesje.


17. Pastebkime, kad trikampiai PAB ir PCD yra panas, o PK ir PM yra
j abiej pusiaukratins i atitinkamo kampo. Tada i vieno anksiau min-
t nauding fakt (Udavini apilimui skyrius), KPB = CPM. Taigi,
NMP = MPC = KPB = PKN. Apie trikampius PNM ir PNK api-
brt apskritim spinduliai vienodi, nes juose prie lygius kampus yra lygios
kratins ( tie apskritimai simetriki NP atvilgiu). Panaiai ir su kitomis tri-
kampi poromis.
18. Apskritimas S
2
lieia kampo ACB kratines, taigi CO
2
yra kampo ACB
pusiaukampin. Pritaik vien i io skyriaus nauding fakt, turime, kad AO
2
=
BO
2
, t.y O
1
AO
2
B yra deltoidas. Taigi AB O
1
O
2
.
19. NAM +NCM = 60

+120

= 180

, taigi apie NAM apibrtas apskritimas


eina per tak C. Taigi jo centras guli AC vidurio statmens, t.y ant striains
BD.
20. Paimkime tak D ant CY tok, kad Y Z = Y D. Tada i simetrijos AD = CZ =
AB. Taigi, XBY = ABD = ADY = CZY = 180

XZY , taigi XBY Z


yra brtinis.
21. Kadangi yra manomos kelios skirtingos kongracijos ir udavinys labai leng-
vas, tai pateiksime tik nepiln sprendim: abiem atvejais keturkampiai BKCP
ir DKAB yra brtiniai; vienu atveju tada nesunkiai rodome, kad PKB +
AKB = 180

, kitu atveju APC +KPC = 180

, i ko ir seka rezultatas.
22. Keturkampis CEHD yra brtinis, nes HEC +HDC = 180

. Be to, X yra
apie keturkamp apibrto apskritimo centras. Panaiai ir BDEA yra brtinis,
o apie j apibrto apskritimo centras yra Y . ie du apskritimai kertasi takuose
D ir E, taigi DXEY yra deltoidas, ir todl XY DE.
211
Sprendimai
23. Tegu AP, BP, CP kerta trikampio kratines takuose A

, B

, C

. Tada keturkam-
piai ABA

ir BCB

yra brtiniai, nes ABP = ACP ir CBP = CAP.


Bet tada B

C = B

BC = A

AC, todl keturkampis AB

PC

yra brti-
nis. Taigi, AB

P = BC

P = BB

C = 180

AB

P, taigi AB

P = 90

=
AC

P, t.y CC

yra auktin. Panaiai ir AA

yra auktin, todl H yra auktini


sankirtos takas.
24. Trikampis ACE yra lygiaonis, taigi EAC = DAC = DEC = BDC =
BAC, taigi BC = CD. Be to, DEC = BAC ir EDC = ABC, taigi
trikampiai ABC ir EDC yra vienodi pagal kratin ir 2 kampus. Todl AB = DE.
25. I duot kamp mes turime C
1
A
1
| AC, taigi CAC
1
A
1
yra lygiaon trapecija.
Be to, keturkampis ABA
1
B
1
yra brtinis, taigi AB
1
B = AA
1
B = CC
1
B =
180

AC
1
C, taigi keturkampis AB
1
PC
1
yra brtinis.
26. Jei LK | BM, tai trikampiai ALK ir ABM yra panas. Taiau ALK yra
lygiaonis su AL = AK, o ABM lygiaonis su MB = MA, taigi AB = AM =
MA. Todl ABM yra lygiakratis, ir i ia BCA = 30

.
27. Tegu H yra auktini susikirtimo takas, o M yra FL vidurio takas. Tada tri-
kampiai CA
1
F ir CB
1
L yra panas pagal du kampus, todl CFA
1
= CLB
1
,
todl HFL yra lygiaonis pagal du kampus. Tada HMC = 90

= HA
1
C =
HB
1
C, tad penkiakampis CA
1
HMB
1
yra brtinis. Kadangi CM yra kampo
A
1
HC
1
pusiaukampin, mes turime MB
1
= MA
1
.
28. Pastebkime, kad trikampiai AKB ir ABC yra panas. Tada i brtini
kamp ir priekampio savybs 180

DBC BCD = BDC = BKC =


BAK + ABK = 2 BCA = 2 (90

DBC) = 180

DBC DBC,
todl DBC = BCD, tad BD = DC.
29. Tegu apie trikamp MLC apibrtas apskritimas kerta AC take F. Tada
MFC = MLC = AKM, todl keturkampis AKMF yra brtinis, ko ir
reikjo.
A B
C D
K
L
M
F
212
Sprendimai
30. Tegu takas Q yra simetrikas takui A atkarpos BE atvilgiu. is takas yra ant
atkarpos BC bei QEA = 90

= QHAtodl keturkampis AHQE yra brtinis.


Tad EHC = EHQ = EAQ = 45

(nes AQE yra status lygiaonis).


31. Tegu trikampi ACL ir BCM apibrtiniai apskritimai kertasi ant atkarpos
AB, take X. Tada BCA = BCX +ACX = LCX +MCX = LAX +
MBX =
BAC
2
+
CBA
2
=
BAC+CBA
2
=
180

BCA
2
= 90


BCA
2
, taigi
BCA = 60

.
32. Pritaikome vien i mintj nauding fakt keturkampiui ADOE, ir gauname,
kad ADOE yra arba brtinis, arba deltoidas. Jeigu jis deltoidas, tai i simetrijos
ABC yra lygiaonis. Jeigu jis brtinis, tai tada paimkime tak H ant BC tok,
kad ODA = OHC = OEB. Tada keturkampiai ABHD ir AEHC yra
brtiniai ir toliau mes galime tsti kaip prie tai buvusiame udavinyje.
33. I duotos slygos
A+C
2
= 60

. Tada i priekampio savybs LDA = LIA =


A
2
+
C
2
= 60

= LBC, taigi keturkampis BCDL brtinis.


34. I duot slyg keturkampiai CDFA, DEAB ir EFBC yra lygiaons trapeci-
jos. Tada kratini CD ir AF vidurio statmenys sutampa, ir jie taip pat sutampa
su kampo QPR pusiaukampine. Panaiai ir su kitomis kratinmis ir j vidurio
statmenimis. Tada vis kratini AB, BC, CD, DE, EF, FA vidurio statme-
nys kertasi viename take, nes trikampio PQR pusiaukampins kertasi viename
take. Todl visos eiakampio virns vienodai nuotolusios nuo io tako, tad
eiakampis yra brtinis.
35. Tarkime, kad takas K yra ant maesniojo lanko AB (Kiti atvejai sprendiami
panaiai). Tada KNB = 180

NKB NBK = 180

90

KAC =
AMD = KMB, taigi KNMB brtinis. Tada ACN = BKM = BNM,
taigi AC | NM.
36. Tegu visi takai yra tokie, kokie paymti brinyje apaioje. Tada keturkampiai
KPC
1
B ir FPC
1
L yra brtiniai. Taigi, KPB = KC
1
B = 180

90

B =
BCC
1
= CC
1
L = FPL, taigi KP | PL ir todl takai K, P, L yra vienoje
tiesje. Panaiai ir takai P, L, H guli vienoje tiesje. Todl takai K, P, L, H guli
vienoje tiesje.
A B
C
F
A
1
B
1
C
1
L
K
P
H
213
Sprendimai
37. Kadangi ADF = AFC, tai AF
2
= AD AC. Panaiai ir AG
2
= AE AB. Bet
BEDC brtinis, taigi AE AB = AD AF. Taigi AG = AF. Panaiai gauname,
kad CF
2
= CDCA = CGCH. Tada FHG+FGA = CFG+AFG = 90

Plotai
1. Tegu R - apskritimo spindulys, a, b - keturkampio striaini ilgiai, - kampas
tarp striaini, S - keturkampio plotas. Tada S =
absin
2

2R2R1
2
= 2R
2
, kas
yra kvadrato, brto apskritim su spinduliu R, plotas.
2. Tegu ties per tak A kerta BC take A

, ties per B kerta AC take B

, o
AA

ir BB

kertasi take M. Tarkime, kad tarp trij gr nra keturkampio.


Tada vis trij trikampi plotai yra lygs. Tada trikampiai ABM ir AMB

turi
bendr auktin i tako A ir vienod plot. Todl MB = MB

. Panaiai ir AM
= MA

. Todl ABA

yra lygiagretainis. Tai nemanoma, nes tada AC | CB.


3. Tegu P yra O projekcija BD. Tarkime, kad B ir O yra skirtingose AC pusse.
Tada ABCO plotas lygus ABCP plotui, kuris yra
BPAC
2
=
BDAC
4
, t.y pus
ABCD ploto.
4. Paveikslliuose emiau abu eiakampiai iskaidyti 7 dalis, ir lygiaplots nu-
spalvintos vienoda spalva. (Naudojams tuo, kad jei dviej trikampi pagrindai
ir auktins lygios, tai ir plotai lygs, pavyzdiui, abiej mlyn dali plotai yra
lygs trikampio AFX plotui).
A
B
C
D
E
F
X
Y
Z
214
Sprendimai
A
B
C
D
E
F
X
Y
Z
5. Tegu KB = a, LB = b. Tada 1 = a+b+

a
2
+b
2
=

a
2
+b
2
= 1ab =
a
2
+b
2
= 1+a
2
+b
2
2a2b+2ab = 12a2b+2ab = 0 =2(1a)(1b) =
1 =
1
4
=
(1a)(1b)
2
. Taigi DMN plotas yra
1
4
.
6. Tarkime prieingai. Tegu kampas tarp striaini yra , o striains dalija viena
kit atkarpas ilgio a
1
, a
2
, b
1
, b
2
. Tarkime, kad a
1
> a
2
, b
1
> b
2
. Tada i slygos
a
1
b
1
sin
2
+
a
2
b
2
sin
2
=
a
1
b
2
sin
2
+
a
2
b
1
sin
2
. (Prisiminkite, kad sin = sin(180

)).
Bet tai ekvivalentu (a
1
a
2
)(b
1
b
2
) sin = 0, kas yra nemanoma, nes kair
lygybs pus yra grietai teigiama.
7. Pastebkime, kad i slygos A

+ B

+ C

= 360

. Nuspalvinkime AB


ir AC

mlynai, BC

ir BA

aliai, CA

ir CB

raudonai. Tada ikirpkime


B

AC, C

AB, A

BC i popieriaus ir i j sudkime trikamp (dedame taip, kad


vienodos spalvos briaunos sutapt). Tada sudtas trikampis yra toks pat kaip ir
ABC, nes j kratins vienodo ilgio. I ia ir seka rezultatas.
8. S
ABCD
= S
ABC
+ S
ADC
= S
ABD
+ S
ADC
= 2(S
ABM
+ S
DCM
) = 2(S
ABCD

S
BCM
), i ko gauname rezultat.
9. S
EFGH
= S
EFG
+S
EGH
= S
BEG
+S
GDE
= S
BEDG
= S
BGD
+S
BED
=
S
ABD
3
+
S
CDB
3
=
S
ABCD
3
.
10. Vl naudosime trikampio pribrim: pratesiame AD iki tako E taip, kad
DE = AB. Trikampiai ABC ir EDC tada yra vienodi pagal dvi kratines ir
kamp. Taigi ABCD plotas yra lygus ACE plotui, o ACE plotas yra lygus
ACAEsin ACE
2
=
ACAC sin BCD
2
=
AC
2
sin A
2
(Galite palyginti su priepaskutiniu
pavyzdiu i geometrini nelygybi skyrelio).
11. Utenka rodyti, kad eiakampio kamp B, D, F suma yra 360

ir tsti kaip 6
udavinyje. O tai yra beveik akivaizdu: B + D + F = (180

AEC) +
(180

CAE) + (180

ACE) = 540

180

= 360

.
12. Tegu trikampyje ABC yra auktin AH, o P yra D projekcija AH. Tada ABC
plotas yra
BCAH
2
, o BDCE plotas yra
BCDC+BCHE
2
. Taigi utenka rodyti,
215
Sprendimai
kad DC + HE = AH. Tai akivaizdu, nes CD = HP ir HE = AP (AECD yra
lygiaon trapecija).
13. Tegu ABCD yra lygiagretainis, o takai K, L, M, N yra atitinkamai ant krati-
ni AB, BC, CD, DA. Tarkime, kad KM ir LN nra lygiagreios lygiagretainio
kratinms. Paimkime tak Z ant AB tok, kad MZ | BC. Tada abiej ketur-
kampi KLMN ir ZLMN yra lygs pusei lygiagretainio ploto, ir todl trikampi
ZLN ir KLN plotai lygs. Bet tada ZK | LN, prietara.
14. Paimkime tak M ant spindulio DE taip, kad DM = 1. Trikampiai ABC
ir AEM yra vienodi pagal 2 kratines ir kamp. Tada ABCDE plotas lygus
ACDM plotui. Bet MD = 1 = CD ir AM = AC, taigi ACDM yra deltoidas.
Bet AMD plotas yra
MDAE
2
= 0.5. Todl penkiakampio plotas yra 1.
Apibrtins gros
1.
I liestini savybi AL = AN. Tada
AL =
AL +AN
2
=
LB +BA+AC +CN
2
=
BM +BA+AC +CM
2
=
a +b +c
2
= s.
Skaiiuojame:
GC =
FC +GC
2
=
FC +GC +BF BE +AGAE
2
=
=
CB +AC AB
2
=
a +b c
2
=
a +b +c
2
c
= s c.
Taip pat BM = BL = ALAB = sc, taigi GC = BM. Panaiai gauname
ir kad NC = BE = s b ir AE = AG = s a.
Pastebkime, kad
I
A
CN =
BCN
2
=
CAB +CBA
2
=
CBA
2
+
CAB
2
= BII
A
,
taigi BIA = I
A
CA ir todl trikampiai I
A
CA ir BIA yra panas pagal
2 kampus. Tada
AB
AI
=
AI
A
AC
, i kur gauname AI AI
A
= AB AC.
Paymkime IG = r ir pastebkime, kad AI
A
N ir AIG yra panas. I ia
S = s r = AN IG = AG I
A
N = (s a) r
A
.
Reikia rodyti, kad S
2
= (s c)(s a)(s b)s, bet mes inome, kad S =
(sa)r
A
ir S = rs, taigi S
2
= sr(sa)r
A
. Utenka rodyti, kad I
A
M IG =
rr
A
= (s b)(s c) = MC CG, arba kad
IA
M
MC
=
CG
IG
. Tai akivaizdu i
trikampi I
A
MC ir IGC panaumo: jie abu stats ir
MI
A
C = 90

MCI
A
= 90

BAC +CBA
2
=
BCA
2
= ICG.
216
Sprendimai
2. Mes inome i mintj fakt, kad I, A, I
A
yra vienoje tiesje ir IA I
B
I
C
(nes
I
C
I
B
yra iorin kampo A pusiaukampin). Taigi I
A
A yra trikampio I
A
I
B
I
C
auktin. Panaiai su BI
B
ir CI
C
. Kadangi CI
C
, BI
B
, AI
A
kertasi take I, tai
tas takas ir yra trikampio I
A
I
B
I
C
auktini susikirtimo takas.
3. Tarkime, kad keturkampis yra ABCD, nubrta striain AC, ABC brtas
apskritimas lieia AC take K, o ADC brtas apskritimas lieia AC take L.
Pastebkime, kad i slygos AB BC = AD DC. Tada KC =
AC+BCAB
2
=
AC+DCAD
2
= CL, ir i ia K = L.
4. Kadangi keturkampis apibrtinis, tai jo prieing kratini sumos lygios. Bet
iuo atveju sandaugos taip pat lygios. I paprastos algebros, keturkampis yra
deltoidas (rodykite tai). Tada nesunkiai iekomas kampas yra lygus 65

.
5. Tegu trikampio MCK pribrtinis apskritimas prie kamp C lieia spindul
CM take B

. I pirmojo udavinio, AB

=
CM+MK+KC
2
= AB, taigi B

=
B. Todl tas pribrtinis apskritimas eina per B ir D, o jo centras yra takas
A. Tada MAK = 180

KMA MKA = 180


KMB
2

MKD
2
=
360

(180

MKC)(180

KMC)
2
= 45

.
6. I kampo tarp stygos ir liestins savybs, KSB = KLB = KML ir todl
ML | BS. Panaiai ir NK | BS. Taigi KLMN yra lygiaon trapecija, ir todl
i simetrijos NSDM yra brtinis.
7. Tarkime, kad takai K ir B yra skirtingose AC pusse (atvejis, kai jie vienoje pu-
sje sprendiamas labai panaiai). Tiess BA ir BC yra iorins trikampio ACK
pusiaukampins, taigi B yra trikampio ACK pribrtinio apskritimo centras. To-
dl B yra ant kampo AKC pusiaukampins. Tegu I yra trikampio ACK brto
apskritimo centras. Tada BA AI ir BC CI, ir todl BAIC yra brtinis su
centru ant tiess IB, kuri sutampa su tiese BK.
8. Kadangi DAC = 60

=
180

BAD
2
, tai AC yra iorin kampo BAD pusi-
aukampin. Tada E yra trikampio BAD pribrto apskritimo centras (nes jis
yra sankirta pusiaukampins ir iorins pusiaukampins), ir todl DE yra kampo
ADC pusiaukampin. Panaiai ir FD yra kampo BDA pusiaukampin. Tada
FDE = FDA+ADE =
BDA
2
+
CDA
2
= 90

.
9. AMK = ACM + CAM = KAB + MAB = MAK. Taigi AMK
lygiaonis. Bet NAM yra status, ir todl K yra MN ir AM vidurio statmens
sankirta. Todl MK = KN.
10. Tegu trapecija bna ABCD, AD | CB. Du apskritimai lieia vienas kit jeigu
atstumas tarp j centr yra lygus j spinduli sumai. Tegu K yra AB, o L yra
CD vidurio takai. Tada KL =
AD+BC
2
=
AB+CD
2
= KA+LD, ko ir reikjo.
11. DOB =
180

BDO
2
=
180

(2(180

BAO))
2
= BAO 90

= BAC +
CBA+BCA
2
90

=
BAC
2
= COB (r. 1 udavin). Taigi C, O, D yra vienoje
tiesje. Tada BDC+BAC = BDO+BAC = 2(180

BAO)+BAC =
(180

BAC) +BAC = 180

. Taigi B, A, C, D yra ant vieno apskritimo.


217
Sprendimai
12. Tegu kratins AB, BC, CD, DE, EA lieia S atitinkamai takuose H, K, L,
M, N. Tada KC = BC BK = AB BH = AH = AN ir BK = BC KC =
CD CL = LD = MD. Taigi KLMH ir HKLN yra lygiaons trapecijos,
ir todl KN = HL = KM. Tada KMEN yra deltoidas ir todl KE eina per
apskritimo centr. Taigi EK BC.
13. Atsakymas yra 120

. Sprendimas beveik toks pats kaip ir 8 udavinio.


14. BM yra kampo DBA pusiaukampin, taigi taip pat yra DA vidurio statmuo.
Todl DMB = BMA. Tada DMA+DCA = 2BMA+BCA = 2(180

MBA MAB) + BCA = 360

DBA (180

BAC) + BCA =
180

(DBABCABAC) = 180

, taigi MDCA brtinis.


15. Tegu kratins AB, BC, CD, DE, EA lieia apskritim atitinkamai takuose H,
K, L, M, N. Kadangi AD eina per apskritimo centr, tai HN AD. Panaiai
ir LM AD. Taigi LM | HN ir todl HNML yra lygiaon trapecija. Panaiai
ir KLMN yra lygiaon trapecija. Galime ubaigti kaip ir 12 udavinyje.
16. Tegu visi takai bna tokie, kokie yra paveikllyje apaioje. Tada GZOA
1
yra
kvadratas, nes jo visi kampai stats ir GZ = GA
1
. Lygiai taip pat kvadratai
yra TMB
1
N, UWPV , AFOC, ACIP, ABNH. Be to, didysis apskritimas yra
trikampi EQO, RLP, KNS pribrtinis apskritimas. Tada i pirmo udavinio
ZO = QF, NB
1
= HS, RI = WP. Tada GZ+UW +MT = ZO+NB
1
+WP =
QF +HS +RI =
QF+QB+HS+DS+DR+RI
2
=
Q6R
2
.
A
B
C
D
E
F
G
H
I
K
L
M
N
O P
R
S
T
U
V
W
Z
Q
A
1
B
1
17. Tegu I
A
yra apskritimo, brto AEH, centras (o kiti centrai yra I
B
, I
C
, I
D
).
Kadangi AEH lygiaonis, tai tada HI
A
E = 180

I
A
EH I
A
HE =
180

AEH = 180

HFE, todl I
A
yra ant ABCD brto apskritimo. Pana-
iai visi kit trikampi brtini apskritim cenrai irgi yra ant to paties apskriti-
mo. Pastebkime, kad I
A
, I
B
, I
C
, I
D
atitinkamai dalina lankus HE, EF, FG, GH
pusiau. Tada kampas tarp I
A
I
C
ir I
B
I
D
yra lygus I
A
I
C
I
B
+ I
C
I
B
I
D
=
HGF
2
+
HEF
2
= 90

, ko ir reikjo.
18. Tegu kamp brtas apskritimas lieia AB ir AC atitinkamai takuose P ir
J, ir lieia kampo kratines takuose E ir D (D yra AO vidurio takas), taip
kaip paveiksllyje apaioje. Tada OAJ = 30

, nes staiajame trikampyje AOJ


218
Sprendimai
ambin lygi pusei statinio. Tegu takas F yra simetrikas takui O tako E at-
vilgiu. Tada BFC = BOC = 180

OBCOCB = 180

ABC+ACB
2
=
180

120

2
= 120

= 180

BAC, taigi F yra ant apie ABC apibrto apskri-


timo. Tada AF vidurio statmuo sutampa su DE vidurio statmeniu, kuris yra
DQE pusiaukampin (nes QDOE yra deltoidas). Bet AF vidurio statmuo eina
ir per apie ABC apibrto apskritimo centr, ko ir reikjo.
A
B
C
D
E
F
J
K
L
O
P
Q
19. Tegu takai bna tokie, kokie paymti paveiksllyje. Tegu KI ir DC kerta-
si take U . Tada apskritimas su centru F yra trikampio KDU brtinis ap-
skritimas, o apskritimas su centru E- pribrtinis. Tada i pirmojo udavinio
rezultat, KL = MD = DG ir KM = LD = DH. Taip pat i pirmojo u-
davinio rezultat, MD =
AD+BDAB
2
ir MK = DH =
DC+DACA
2
. Taigi,
AK = AD DM MK = AD
AD+BDAB
2

DC+DACA
2
=
AB+CABC
2
, kas
tikrai nepriklauso nuo D.
A
B C
D
E
F
G H
I
L
K
M
N
P
O
U
Vienareikmiki udaviniai
1. Aikiai M yra trikampio ACD viduje. AMC = 180

MAC MCA =
180

MAC(45

MCD) = 135

. Todl MABC tenkina pirmj mintj


savyb, taigi MB = BC = BA. Tada MBA = 180

2MBA = 180

2(45

+
u) = 90

2u.
2. Tegu takas N yra simetrikas takui A tako C atvilgiu, o P yra A projekcija
BM. Tada
NA
AB
= 2 =
AB
AM
, taigi trikampiai ANB ir ABM panas. Kadangi
PAN = MBA = ANB, tai ties PA yra trikampio BAN pusiaukratin.
Bet BC taip pat yra pusiaukratin, o pusiaukratins dalija viena kit santykiu
2:1.
219
Sprendimai
3. Trikampis ACE yra lygiaonis, nes turi du kampus po 50

. Be to, ADC =
40

. Paimkime tak F ant BC tok, kad AFC = 80

. Tada AF = AC bei
EAF = 60

, taigi EAF lygiakratis. Be to, DAF yra lygiaonis, nes jo kampai


lygs 100

, 40

, 40

. Tada DF = FA = FE, taigi F yra apie DAE apibrto


apskritimo centras. I ia nesunkiai gauname, kad iekomas kampas lygus 30

.
A
B
E
D
F
C
4. Tegu linija, lygiagreti AB ir einanti per tak K, kerta linij, lygiagrei AL ir
einani per tak C take P. Tada kadangi BKA = 50

, tai AKP = 40

.
Panaiai BCP = 80

, tai ACP = 40

. Taigi AKCP brtinis. Kadangi CA


yra KCP pusiaukampin, tai AK = AP. Tada KP = 2BA ir i trikampi
CKP ir BLA panaumo
KC
BL
= 2.
5. Tegu ACB = x. Pastebkime, kad keturkampis ADCE tenkina slyg, mint
pirmame naudingame fakte, nes DA = DC bei AEC +
ADC
2
= (180

x
x
2
) + 1.5x = 180

. Taigi DE = DC = DA. Kadangi D yra apie AEC apibrto


apskritimo centras, EDC = 2EAC = x = ECF. Tada EDC ir EFC
panas pagal du kampus, taigi EFC lygiaonis.
6. Tegu takas F yra simetrikas takui D tiess BC atvilgiu.Kadangi FCB +
BCA = 180

, tai A, C, F yra vienoje tiesje. Tada BAC = BFC = BDC,


taigi ABCD yra brtinis ir i ia ABD = 50

.
7. Tegu takai B

, C

dalina pusapskritim tris lygias dalis, o AB

ir AC

atitinkamai
kerta BC takuose F ir E. Tegu D yra BC vidurio takas. Tada DB | AB, taigi
BF
FD
=
AB
DB

=
AB
DB
= 2. Bet
EF
FD
= 2, taigi BF = EF. Panaiai ir EF = CE.
220
Sprendimai
A
B
C
D
E
F
B

8. Paimame tak F trikampio viduje taip, kad FBC bt lygiakratis. Tada


trikampiai FBA ir ADC vienodi pagal dvi kratines ir kamp. Taigi BCD =
80

10

= 70

.
9. Paimkime kvadrato viduje tak P taip, kad PBC = PCB = 15

. Tada
PCM = 60

, PC = MC, taigi PCM lygiakratis. Tada MPB lygiaonis,


i kur MBP = 15

. Tada MBC = 30

, taigi ABM lygiakratis. Todl


AMB = 60

.
10. Nesunkiai randame BDA = 30

. Tada apie ABD apibrto apskritimo centras


O tenkina BOA = 60

, t.y. O = C. Tada CA = CD, = ADC = DAC =


45

.
11. Tegu BN yra auktin. Tada BN =
BA
2
=
DC
2
, taigi BN = NC = ND. I ia
BDC = 45

, ABD = 105

90

= 15

.
12. Imame tak E ant AD tok, kad CE = CD (E ,= D). Paskaiiav kampus,
gauname, kad CEA lygiaonis, o CEB lygiakratis. Tada EA = EB = EC, taigi
E yra apie ABC apibrto apskritimo centras. I ia BAC =
BEC
2
= 30

.
13. Atidiai inagrinje visus variantus, gauname, kad yra tik du skirtingi atvejai,
pavaizduoti apaioje:
55

55

55

55

16

16

19

19

A
B
C
D
A

Atvejis kairje tenkina io skyrelio naudingj savyb, nes ABC +


ADC
2
=
145

+ 35

= 180

ir AD = DC. Taigi D yra apie apskritim ABC apibrto


apskritimo centras ir todl BDC = 2BAC = 34

, ir kampas tarp keturkampio


striaini yra 55

+ 32

= 87

. O atvejui deinje reikia atskiro sprendimo - mes


rodysime, kad kampas tarp striaini lieko toks pats. Paimkime keturkamp
221
Sprendimai
ABCD kaip paveiksllyje viruje kairje. Tegu striains kertasi take E, linija,
lygiagreti DC ir einanti per tak A, kerta BD take F, o linija per D, lygiagreti
AB, kerta AC take G. Tada trikampiai DEG ir AEB yra panas, taip pat
kaip ir trikampiai DEC ir AEF. Tada
EB
EC
=
EB
DE

DE
EC
=
AE
EG

EF
AE
=
EF
EG
, taigi
EBC ir EFG panas ir todl EGF = ECB = 19

. Belieka pastebti, kad


keturkampis ADGF yra btent tas kurio mums reikia, nes kampai tarp striaini
ir kratini yra 55

, 55

, 16

, 19

.
55

55

16

16

19

A
B
C
D
F
E
G
14. Pirmas bdas: Tegu AP kerta BC take D, o CP kerta AB take E. Tada nesun-
kiai paskaiiuodami kampus gauname, kad BDA yra statusis, o BCE lygiaonis.
Todl BE = 2BD. Paimkime ant tiess AD tak F tok, kad BE = BF (F
nra ABC viduje). Tada BFD statusis, ir BF = 2BD, taigi FBD = 60

. Tada
keturkampyje BEPF BE = BF ir EPF +
EBF
2
= 110

+
60

+80

2
= 180

,
taigi BEPF tenkina mintj savyb, ir todl BP = BE. Tada nesunkiai
BPD = 30

, taigi BPC = 100

.
A
B
C
D
F
E
P
Antras bdas: Tegu CP kerta kampo B pusiaukampin take M. Nesunkiai
skaiiuodami kampus gauname, kad P guli ant kamp BMA ir BAM pusiau-
kampini, ir todl yra trikampio ABM brto apskritimo centras. Todl BP yra
ABM pusiaukampin ir vl paskaiiav kampus gauname t pat atsakym.
15. Sukonstruojame lygiakrat trikamp ABT trikampio ABC iorje. Tada trikam-
piai ART ir AQC panas pagal 3 kampus (abu turi kampus lygius 30

, 60

x,
90

+ x). Tada
AT
AC
=
AR
AQ
, TAC = RAQ. Taigi trikampiai TAC ir RAQ
panas pagal 2 kratines ir kamp, o panaumo koecientas yra
AT
AR
=
AB
AR
. Pa-
naiai RBP ir CBT irgi panas su tuo paiu panaumo koecientu
AB
BR
. Taigi
QR =
CTAR
AB
=
CTRB
AB
= PR.
222
Sprendimai
A
B
C
Q
T
R
P
30

30

16. Paimkime tak E tok, kad EAD bt lygiakratis, o B ir E bt skirtingose AD


pusse. Tegu BE kerta AC take F. Suskaiiav kampus nesunkiai gauname, kad
EAF yra lygiaonis, ir tada EFD taip pat lygiaonis. Toliau suskaiiav kampus
gauname, kad BDF = 44

= BCF, taigi BCDF brtinis. I ia nesunkiai


gauname, kad DCF = DBF = 30

.
A
B
C
D
F
E
16

48

30

17. (Pirmas bdas) Trigonometrinis bdas: Nubriame kuo tikslesn brin ir sp-
jame, kad atsakymas yra 60

. Tegu striains kertasi take O. Tada


tan BDC =
CE
DE
=
CE BE AE
BE AE DE
= tan 20

tan 40

tan 80

.
Belieka parodyti, kad tan 20

tan 40

tan 80

= tan 60

, kas yra vidutinio sunkumo


trigonometrijos udavinys, paliekamas skaitytojui.
(Antras bdas) Geometrinis bdas: Tegu striains vl kertasi take O. Imame
tak H ant OC tok, kad DHA = 40

( 11 udavinio idja). Tada i trikampi


223
Sprendimai
HDO ir OCB panaumo
HO
OB
=
OD
OC
, taigi trikampiai COD ir HOB panas ir
mums tada tereikia rasti kamp BHO. Imame tak K ant spindulio HD taip,
kad DKA = 80

. Tegu W yra tiesi DH ir AB sankirta, o takas Z simetrikas


takui D kampo AWK pusiaukampins atvilgiu (Tada AWK yra lygiaonis
su kampais 80

prie pagrindo). Taikome treiojo udavinio sprendim trikampiui


AWK ir gauname, kad ZHD = 70

= DBZ, tai yra ABHK yra lygiaon


trapecija. I ia nesunkiai iekomas kampas yra 60

.
A
B
C
D
H
Z
O
K
W
Ivada: kartais trigonometrinis sprendimas yra geriau.
18. Mes pirmiau iekosime BDE. Tam imame tak F, simetrik takui D ta-
ko B atvilgiu, ir tak G, simetrik takui E tako B atvilgiu. Tereikia rasti
BFG. Dabar pastebime, kad udavinys pasidar netiktinai panaus prie tai
buvus. Ties sakant, sprendimas nuo ios vietos irgi yra kone identikas - j palie-
kame skaitytojui. (Atsakymas yra 40

). Abiej udavini graumas slypi tame,


kad egzistuoja ei keturkampiai, kuri striains dalina juos keturis staiuosius
trikampius su kampais (30

, 60

), (20

, 70

), (40

, 50

), (10

, 80

), o keturkampiai
vienas su kitu susij 11 udavinio konstrukcijomis. Kaip matme prie tai bu-
vusiame udavinyje, su jais susidrus geriau naudotis trigonometrija (taip pat ir
iuo atveju).
Geometrins nelygybs
1. Tegu takas B

yra simetrikas takui B tiess a atvilgiu. Tegu AB

kerta a
take G. is takas ir yra reikiamas takas: jeigu H yra koks nors kitas takas ant
a, tai tada AH+BH = AH+B

H AB

= AG+GB

= AG+GB i trikampio
nelygybs.
a
A
B
B

H G
F
224
Sprendimai
2. Tegu ties, lygiagreti BC ir einanti per O, kerta kratines AB ir AC atitinkamai
takuose X ir Y . Tada AB +BC +CA > AB +CA+XY = AX +XB +CY +
Y A +XO +OY = (AX +AY ) + (XO +OB) + (Y O +OC) > AO +OB +OC
i trikampio nelygybs. Kitai nelygybs pusei vl taikome trikampio nelygyb:
AO +BO +CO = (
AO+BO
2
) + (
BO+CO
2
) + (
CO+AO
2
)> (
AB
2
) + (
BC
2
) + (
AC
2
).
3. Tegu takas P yra simetrikas takui A tako M atvilgiu. Tada ABPC yra
lygiagretainis, ir i trikampio nelygybs AB +AC = AB +BP > AP = 2AM.
4. Pirma dalis seka i 2 udavinio, pritaikyto visoms pusiaukratinms i eils ir
sudjus tris gautas nelygybes. Kitai daliai galime pritaikyti 2 udavinio nelygyb
pusiaukratini susikirtimo takui ir padauginti rezultat i
3
2
.
5. Paimkime ant kratins BC tak K tok, kad AKC = 80

. Tada trikampiai
AMB ir AKB yra vienodi pagal du kampus ir kratin. Taigi BM = AK, bet
AK < AC i trikampio nelygybs pritaikytos trikampiui AKC, ko ir reikjo.
6. Akivaizdu, kad kvadrat brto apskritimo spindulys maesnis nei trikamp
brto apskritimo spindulys, o apibrto didesnis. I ia viskas akivaizdiai seka
(palyginti galite su rodymu kad R > 2r i pavyzdi).
7. Sprendimo idja tokia pati, kaip ir 2 udavinio pirmosios dalies - briame per
tak O ties, lygiagrei AB, kuri kerta CA ir CB atitinkamai takuose X ir Y .
Tada 2 = CA+CB = (AX +XO) + (OY +Y B) +CY > AO +BO +CO.
8. Tarkime, kad keturkampis yra ABCD, o striains kertasi take O, takas viduje
yra P. Neprarasdami bendrumo, galime teigti, kad P yra trikampyje AOB. Tada
i antro udavinio AB+BC +CA > PA+PB+PC ir AD+DB > PD. Sudj
nelygybes ir pridj DC prie kairs puss, gauname tai, ko reikia. (Gali pasiroyti,
kad i nelygyb visai negrieta, bet taip nra, pavyzdiui, jei takai A, B, C beveik
sutampa, o D yra labai toli nuo j ir P yra arti D, tai gauname visai ger vert).
9. Padalin keturkamp striaine du trikampius kuri dvi kratins yra a ir d, b ir c
gauname 2S < ad+bc. Tada imame keturkamp kurio kratins yra a, b, d, c (tokia
tvarka), kuris gaunamas pradin keturkamp perkirpus pusiau kita striaine du
trikampius, viena j apvertus ir trikampius suklijavus atgal ta paia striaine.
io keturkapio plotas irgi yra S, ir kaip anksiau gauname 2S < ac +bd. Sudj
dvi nelygybes, gauname k reikia.
10. Tegu M yra kvadrato kratins prie virns A vidurio takas, o O-kvadrato
centras. Tada AO OM + MA =
1
2
+
1
2
, taigi ABCD telpa apskritm su
centru O ir spinduliu 1. Mes inome i pirmojo udavinio, kad jei keturkampis
telpa apskritim spindulio R, tai jo plotas nevirija 2R
2
. I ia ir seka rezulta-
tas. Antrajai daliai pastebkime, kad takai O
1
, O
2
, O
3
, O
4
guli ant apskritimo,
apibrto apie kvadrat (suskaiiuokite kampus). Vl pritaikome t pat fakt:
syk apskritimo spindulys yra
1

2
, o tai ir yra tai, ko reikia.
11. AC yra kampo BAD pusiaukampin, nes kampai BAD ir BAC remiasi
lygius lankus. Tegu takas F yra simetrikas takui B AC atvilgiu. Tada takai
225
Sprendimai
A, D, F yra vienoje tiesje, ir trikampis CDF yra lygiaonis, nes CD = BC = FC.
Tegu M yra DF vidurio takas. Tada EM =
CA
2
, nes CMA statusis. Pritaik
3 udavinio nelygyb trikampiui EFD, mes gauname ED + EF 2EM, kas
ekvivalentu BE +DE AC.
12. Tegu brto septynkamp apskritimo skersmuo yra r, apibrto - R, o sep-
tynkampio kratin a. Tada iedo plotas yra R
2
r
2
= (R
2
r
2
). Bet
(R
2
r
2
) =
a
2
4
i Pitagoro teoremos, taigi iedo plotas yra
a
2
4
. Taigi iedo plotas
tiesiogiai priklauso nuo kratins ilgio ir nepriklauso nuo kratini skaiiaus, todl
septynkampio ir septyniolikakampio kratins vienodo ilgio.
13. Utenka rodyti, kad persiklojanios dalies (lygiaonio trikampio) plotas yra dau-
giau negu
1
4
. Tai yra beveik akivaizdu: onin jo kratin yra daugiau nei pus
pradinio staiakampio kratins, o auktins, nuleistos t kratin, ilgis sutampa
su kitos staiakampio kratins ilgiu.
14. Tegu AB = c, AC = b, BC = a, AM = m, AMB brto apskritimo skersmuo
r, o AMC 2r. Tada AMB plotas yra
r(c+
a
2
+m)
2
, o AMC plotas r(
a
2
+ m + b).
Bet ie plotai lygs, taigi c + m +
a
2
= a + 2m + 2b, arba c =
a
2
+ m + b, kas
prietarauja trikampio nelygybei pritaikytai trikampiui AMB.
15. Kadangi CC
1
B yra statusis, tai A
2
C
1
=
CB
2
. Panaiai ir A
1
B
2
=
AC
2
ir B
1
C
2
=
AB
2
. Taigi i i atkarp galima sudti trikamp, dvigubai maesn u ABC.
16. Tegu apskritimo centras yra O. Jeigu paymsime lanko A
1
A
2
vidurio tak
raide B
1
, kitus takus panaiai, tai eiakampio A
1
B
1
A
2
B
2
A
3
B
3
ploto skaitin
vert bus
RA
1
A
2
2
+
RA
1
A
3
2
+
RA
3
A
2
2
= A
1
A
2
+ A
2
A
3
+ A
3
A
1
. (Iskaidius tris
keturkampius A
1
B
1
A
2
O, A
2
B
2
A
3
O, A
3
B
3
A
1
O).
17. Kadangi styga ne ilgesn u skersmen, tai 2R a. Be to, i AM-GM nelygybs
b +c 2

bc. Taigi
R
a

1
2

bc
b+c
. Pagal slyg, abi nelygybs yra lygybs. Taigi
a = 2R (todl trikampis statusis) bei b = c (trikampis lygiaonis). Taigi kratini
ilgiai yra 2R,

2R,

2R.
18. Tegu liestin pirmajam apskritimui, lygiagreti AC ir esanti ariau jos, kerta BC
take E, o liestin antrajam, lygiagreti BA ir esanti ariau jos, kerta BC take
F. Tada atkarpos BE ir CF turi turti bendr tak, arba kitaip du apskritimai
negalt liestis.Todl
r
1
r
ABC
+
r
2
r
ABC
=
BE
BC
+
CF
BC
>
BC
BC
= 1, ko ir reikjo.
19. Kadangi MCA < MCB + MBC = AMC, tai AM < AC ir panaiai
KC < AC. Tegu MC kerta apie AKC apibrt apskritim take X. rody-
sime, kad AM > MK. Tam parodysime, kad M ir C yra toje paioje AK
vidurio statmens pusje.Kadangi X yra ant AK vidurio statmens vidurio stat-
mens, tai pakanka parodyti, kad M yra ant atkarpos XC(ne ant tsinio), arba
kad KAM < KAX. Tas akivaizdu, nes KAM =
BAC
2
<
BCA
2
= KAX.
Panaiai rodome kit nelygyb.
20. Pastebkime, kad MBK brto apskritimo spindulys maesnis nei ABC
brto apskritimo spindulys. Taigi 2P
MBK
=
4S
MBK
r
MBK
>
2S
ABC
r
ABC
= P
ABC
. Todl
226
Sprendimai
4(MB + BK) > 2(MB + BK) + 2MK = 2P
MBK
> P
ABC
= (MB + BK) +
(MA+AC +CK), i ko seka rezultatas.
227
Literatra
Bendra
http://www.mathlinks.ro (olimpiadins matematikos forumas)
http://www.math.ca/crux/ (olimpiadins matematikos urnalas)
http://www.math.ust.hk/excalibur/ (olimpiadins matematikos urnalas)
http://www.math.toronto.edu/oz/turgor/archives.php (Tournament of Towns
at Toronto)
Arthur Engel, Problem Solving Strategies, Springer, 1998.
Paul Zeitz, Art and Craft of Problem Solving, Wiley, 2007.
D. Djukic, V. Jankovic, I. Matic, N.Petrovic, The IMO Compendium, Springer,
2006.
http://www.pdmi.ras.ru/~olymp/index.html (Peterburgo miesto olimpiad
udaviniai (rusikai))
http://www.turgor.ru/ (Miest turnyras (rusikai))
Skaii teorija
http://www.mathlinks.ro/index.php?f=456 (Problems in Elementary Num-
ber theory (PEN))
T. Andreescu, D. Andrica, Z. Feng, 104 Number Theory Problems, Birkhauser,
2007.
T. Andreescu, D. Andrica, An Introduction to Diophantine Equations, GIL, 2002.
K. Ireland, M. Rosen, A Classical Introduction to Modern Number Theory, Sprin-
ger, 1990.
Algebra
Pham Kim Hung, Secrets in Inequalities (Volume 1), GIL Publishing House, 2007.
Samin Riasat, Basics of Olympiad Inequalities, 2008.
Ivan Matic, Classical Inequalities, The IMO Compendium Group, 2007.
Hojoo Lee, Topics in Inequalities - Theorems and Techniques, 2007.
Tran Phuong, Diamonds in Mathematical Inequalities, Hanoi Publishing House,
2007.
Thomas J. Mildorf, Olympiad Inequalities, 2006.
(http://www.artofproblemsolving.com/Resources/Papers/MildorfInequalities.
pdf)
T. Andreescu, V. Cartoaje, G. Dospinescu, M. Lascu, Old and New Inequalities,
GIL Publishing House, 2003.
J. Michael Steele, The Cauchy-Schwarz Master Class, Cambridge University Press,
2004.
Kombinatorika
Ted Alper, Two Player Games in Olympiads and Real Life, Berkeley Math Circle,
2000.
Elwyn R. Berlekamp, Jonh H. Conway, Richard K. Guy, Winning Ways for Your
Mathematical Plays, A K Peters/CRC Press, 2001.
John H. Conway, On Numbers and Games, A K Peters/CRC Press, 2000.
Geometrija
http://www.gogeometry.com/

You might also like